222
300 Aufgaben zur Geometrie und zu Ungleichungen insbesondere zur Vorbereitung auf Mathematik-Olympiaden Version 2.5 (Dezember 2000) von Dr. Eckard Specht Otto-von-Guericke-Universit¨ at Magdeburg Fakult¨ at f¨ ur Naturwissenschaften

300 Aufgaben - Mathematikrätsel · 300 Aufgaben zur Geometrie und zu Ungleichungen insbesondere zur Vorbereitung auf Mathematik-Olympiaden Version 2.5 (Dezember 2000) von Dr.Eckard

Embed Size (px)

Citation preview

300 Aufgaben

zur Geometrie und zu Ungleichungen

insbesondere zur Vorbereitung auf Mathematik-Olympiaden

Version 2.5 (Dezember 2000)

von

Dr. Eckard Specht

Otto-von-Guericke-Universitat MagdeburgFakultat fur Naturwissenschaften

c© Kopieren und Vervielfaltigung ausdrucklich erlaubt.

Dieses Dokument kann jederzeit als PostScript-Datei mit anonymous ftp vomServer hydra.nat.uni-magdeburg.de/pub/geometry heruntergeladen werden.Hier die entsprechenden Linux-Kommandos (Tastatureingaben sind unterstrichen):

$ ftp 141.44.46.52

Connected to 141.44.46.52.

220 hydra FTP server (Version wu-2.4(1) Tue Dec 5 20:51:15 CST 1995) ready.

Name (141.44.46.52:blah): anonymous

331 Guest login ok, send your complete e-mail address as password.

Password:<your complete email address>

230 Guest login ok, access restrictions apply.

ftp> cd pub/geometry

250 CWD command successful.

ftp> bin

200 Type set to I.

ftp> get geom 2.5.ps

200 PORT command successful.

150 Opening BINARY mode data connection for geom 2.5.ps (2408182 bytes).

226 Transfer complete.

2108634 bytes received in 0.314 secs (1.0e+33 Kbytes/sec)

ftp> bye

221 Goodbye.

$

Online kann man sich die Aufgaben unter

http://www.math4u.de

ansehen.

Vorwort

Zu den Gebieten der Mathematik, die von SchulerInnen anscheinend nicht besondersgeliebt werden, gehort die Elementargeometrie. Obwohl sie zu den altesten wissenschaftli-chen Disziplinen uberhaupt zahlt, scheint es so, daß eine Vielzahl ihrer Erkenntnisse undbemerkenswerten Entdeckungen seit Euklid (365–300 v. Chr.) im Laufe der Jahrhun-derte verloren gegangen sind. Selbst ein Aufbluhen im 19. Jahrhundert, in dem zahlreicheund bis dahin unbekannte oder inzwischen wieder vergessene Beziehungen zwischen Punk-ten, Geraden, Kreisen im Dreieck gefunden wurden, vermochte die Elementargeometrieoffenbar nicht als attraktive Wissenschaft in die Neuzeit hinuberzuretten. So spielen inder heutigen Zeit diese, mit Namen großer Geometer belegter Entdeckungen im Allge-meinwissen eigentlich keine Rolle mehr.

Mitunter sieht man Mathematiker etwas geringschatzig auf die Elementargeometrie her-abblicken. Sicherlich ist sie nicht so abstrakt wie andere Gebiete, die zudem haufig eineneigenen Kalkul entwickelt haben und dessen Beherrschung die Mathematiker zu Spezia-listen macht. Doch gerade das Fehlen dieser systematischen Losungsmethoden macht siezu einer hochgradig intuitiven Angelegenheit und fur jeden verstandlich — auch ohne ab-solviertes Studium! Sie ist also bestens geeignet, logisches Denkvermogen, Ausdauer undKonzentration zu trainieren, ganz gleich fur welchen Wissenschaftszweig oder Altersgrup-pe auch immer (sogar rustige Rentner entdecken sie oft fur sich neu). Leider Gottes istsie streng axiomatisch aufgebaut, so daß ihr Hauptinhalt aus Satzen und den notwendigzugehorigen Beweisen besteht. Und daß das abschreckend ist, kann man dann auch wiederverstehen :-)

Die vorliegende Aufgabensammlung entstand ursprunglich als Zusammenstellung vonThemen, die ich anlaßlich der alljahrlich im April in Gardelegen stattfindenden Vorberei-tungskurse der nominierten Teilnehmer Sachsen-Anhalts an der Deutschen Mathematik-Olympiade in den Klassenstufen 9 und 10 behandelte; spater kam auch Material fur dieAbiturstufe hinzu. In diesen dreieinhalbtagigen Kursen stand vor allem das Trainingdes Losens von Aufgaben und die Diskussion von Losungsansatzen im Vorder-grund. Die systematische Vermittlung von Stoffgebieten, wie z. B. Trigonometrie, ana-lytische Geometrie oder Vektorrechnung, war dabei nicht vorrangiges Ziel dieser Kurse,obwohl die Erfahrung zeigt, daß auch hier ein Interesse besteht, uber den Schulunter-richt hinausgehendes Wissen zu erwerben. Aus diesem Grunde wurden auch einige altereOlympiade-Aufgaben und vor allem historisch interessante geometrische Probleme aufge-nommen. Großes Gewicht wurde auf die ausfuhrlichen Losungen gelegt, wobei eher dieLosungsideen und nicht die strenge Beweisfuhrung im Vordergrund standen.

Obwohl Ungleichungen augenscheinlich nicht viel mit Geometrie zu tun haben außer derTatsache, daß es ein ziemlich eigenstandiges Gebiet geometrischer Ungleichungen gibt(s. Kapitel G), sind zwei (noch sehr unvollstandige) Kapitel hinzugekommen, die diesesThema naher beleuchten. Gerade in hoher angesiedelten Wettbewerben wie der Interna-tionalen Mathematik-Olympiade sind Ungleichungen eine durchaus beliebte Rubrik.

4

Die vorliegende Broschure bemuht sich daher, einerseits eine (immer unvollstandig blei-bende) Faktensammlung zu sein und andererseits auch einige Methoden, Tips und Tricksvorzustellen, die benotigt werden um geometrische Puzzles erfolgreich zu losen.

Fur inhaltliche Anregungen und Verbesserungshinweise bin ich jederzeit dankbar.

Ich wunsche viel Spaß und Erfolg beim Losen der Aufgaben!

Magdeburg, 23. Dezember 2000 E. Specht

e-mail: [email protected]

Hinweise zur Schreibweise 5

Hinweise zur Schreibweise

Es wird eine teilweise vom Schulunterricht abweichende Bezeichnung verwendet:

• AB kann sowohl eine zwischen zwei Punkten A und B gelegene Strecke bezeichnenals auch deren Lange (sonst haufig |AB|, AB oder sogar |AB|). Aus dem Kontextgeht jedoch i. a. immer hervor, was gerade gemeint ist.

• Eine durch A, B gehende Gerade bezeichnen wir mitunter auch mit g(A,B).

• Wird eine Große durch eine andere definiert (um sie z. B. als Abkurzung zu verwen-den), schreiben wir l ≡ AB.

• Kreise k sind durch Angabe des Mittelpunktes O und eines Punktes A auf dessenPeripherie eindeutig bestimmt; wir verwenden daher die Schreibweise OA. Ist da-gegen anstelle eines Punktes auf der Peripherie der Radius r = XY als Streckegegeben, schreiben wir Or oder OXY .

• Den Flacheninhalt eines Polygons ABC . . . Z bezeichnen wir kurz mit [ABC . . . Z];bei Dreiecken notieren wir stets ∆.

• Vektoren erscheinen im Text fett gedruckt und werden in Bildern mit einem Pfeiluber dem Symbol gekennzeichnet.

Aufgaben werden einheitlich mit dem Kapitelbuchstaben und einer ein- oder zweistelligenNummer bezeichnet (z. B. M.41). Gleichungen stehen in runden Klammern und werdenkapitelweise bei den Aufgaben beginnend mit 1, bei den Losungen beginnend mit 101,durchnumeriert. Literaturhinweise werden von eckigen Klammern eingeschlossen.

Besteht eine Aufgabe nur aus einer Aussage”Blah, blah, blah . . .“, so ist als Losung

immer der Beweis des Satzes gefordert. Gleiches gilt auch fur eine einzelne Gleichungoder Ungleichung. Dieses kleine Quadrat

”�“ gibt stets das Ende eines Beweises an; es

steht also fur”w. z. b. w.“ (was zu beweisen war) oder

”q. e. d.“ (quod erat demonstran-

dum). Eine weitere gebrauchliche Abkurzung ist”o. B. d. A.“ (ohne Beschrankung der

Allgemeinheit).

Kursiv gesetzte Landernamen mit Jahreszahlen (z. B. Peru, 1857 ) geben an, daß diebetreffende Aufgabe in jenem Jahr eine Olympiadeaufgabe war.

6 Abkurzungen

Abkurzungen

AIME — American Invitational Mathematics Competition

APMO — Asian Pacific Mathematical Olympiad

IMO — Internationale Mathematik-Olympiade

IMTS — International Mathematical Talent Search

NMC — Nordic Mathematical Contest

Inhaltsverzeichnis

A KONSTRUKTIONEN . . . . . . . . . . . . . . . . . . . . . . . . . . . . . . . 7A.1 Euklidische Konstruktionen . . . . . . . . . . . . . . . . . . . . . . . . . . . . . . . 8A.2 Geometrische Orter . . . . . . . . . . . . . . . . . . . . . . . . . . . . . . . . . . . 9A.3 Kreiskonstruktionen . . . . . . . . . . . . . . . . . . . . . . . . . . . . . . . . . . . 11A.4 Mohr-Mascheronische Konstruktionen . . . . . . . . . . . . . . . . . . . . . . . . . 12A.5 Verschiedene Konstruktionen . . . . . . . . . . . . . . . . . . . . . . . . . . . . . . 13

B DREIECKSKONSTRUKTIONEN . . . . . . . . . . . . . . . . . . . . . . . 15B.1 Die Grundaufgaben . . . . . . . . . . . . . . . . . . . . . . . . . . . . . . . . . . . 16B.2 Auffinden von Hilfsdreiecken . . . . . . . . . . . . . . . . . . . . . . . . . . . . . . 16B.3 Berechnung fehlender Stucke . . . . . . . . . . . . . . . . . . . . . . . . . . . . . . 18B.4 Rekonstruktion aus gegebenen Punkten . . . . . . . . . . . . . . . . . . . . . . . . 18

K KREISE . . . . . . . . . . . . . . . . . . . . . . . . . . . . . . . . . . . . . . . . 19K.1 Winkel und Langen . . . . . . . . . . . . . . . . . . . . . . . . . . . . . . . . . . . 19K.2 Inversion am Kreis . . . . . . . . . . . . . . . . . . . . . . . . . . . . . . . . . . . . 20K.2.1 Eigenschaften der Inversion . . . . . . . . . . . . . . . . . . . . . . . . . . . . . . . 21K.2.2 Anwendungen zur Inversion . . . . . . . . . . . . . . . . . . . . . . . . . . . . . . . 23K.3 Anwendungen . . . . . . . . . . . . . . . . . . . . . . . . . . . . . . . . . . . . . . 23

D DREIECKE . . . . . . . . . . . . . . . . . . . . . . . . . . . . . . . . . . . . . . 25D.1 Klassische Transversalen . . . . . . . . . . . . . . . . . . . . . . . . . . . . . . . . 25D.2 Ceva & Menelaus . . . . . . . . . . . . . . . . . . . . . . . . . . . . . . . . . . . . 28D.3 Extremalaufgaben . . . . . . . . . . . . . . . . . . . . . . . . . . . . . . . . . . . . 31D.4 Einige Formeln . . . . . . . . . . . . . . . . . . . . . . . . . . . . . . . . . . . . . . 32D.5 Lotfußpunktdreiecke . . . . . . . . . . . . . . . . . . . . . . . . . . . . . . . . . . . 34D.6 Noch mehr uber Dreiecke . . . . . . . . . . . . . . . . . . . . . . . . . . . . . . . . 35

V VIERECKE . . . . . . . . . . . . . . . . . . . . . . . . . . . . . . . . . . . . . . 37V.1 Allgemeine Vierecke . . . . . . . . . . . . . . . . . . . . . . . . . . . . . . . . . . . 38V.2 Trapeze, Parallelogramme, Rhomben etc. . . . . . . . . . . . . . . . . . . . . . . . 38V.3 Sehnenvierecke . . . . . . . . . . . . . . . . . . . . . . . . . . . . . . . . . . . . . . 39V.4 Tangentenvierecke . . . . . . . . . . . . . . . . . . . . . . . . . . . . . . . . . . . . 39V.5 Sehnentangentenvierecke . . . . . . . . . . . . . . . . . . . . . . . . . . . . . . . . 40

M METHODEN . . . . . . . . . . . . . . . . . . . . . . . . . . . . . . . . . . . . . 41M.1 Vektorrechnung . . . . . . . . . . . . . . . . . . . . . . . . . . . . . . . . . . . . . 41M.2 ”Winkel jagen“ . . . . . . . . . . . . . . . . . . . . . . . . . . . . . . . . . . . . . . 42M.3 Verwandlung von Figuren . . . . . . . . . . . . . . . . . . . . . . . . . . . . . . . . 43M.4 Das Flachenprinzip . . . . . . . . . . . . . . . . . . . . . . . . . . . . . . . . . . . 44

8 Inhaltsverzeichnis

W WETTBEWERBSAUFGABEN . . . . . . . . . . . . . . . . . . . . . . . . . 49W.1 Deutsche Mathematik-Olympiade . . . . . . . . . . . . . . . . . . . . . . . . . . . 50W.2 Nationale Wettbewerbe . . . . . . . . . . . . . . . . . . . . . . . . . . . . . . . . . 53W.3 Internationale Wettbewerbe . . . . . . . . . . . . . . . . . . . . . . . . . . . . . . 54W.4 Crux Mathematicorum . . . . . . . . . . . . . . . . . . . . . . . . . . . . . . . . . 55

U UNGLEICHUNGEN . . . . . . . . . . . . . . . . . . . . . . . . . . . . . . . . 57U.1 Fundamentale Ungleichungen . . . . . . . . . . . . . . . . . . . . . . . . . . . . . . 58U.2 Einfache Tips und Tricks . . . . . . . . . . . . . . . . . . . . . . . . . . . . . . . . 61U.2.1 ”Teile und (be)herrsche“ . . . . . . . . . . . . . . . . . . . . . . . . . . . . . . . . 62U.2.2 Die Arbeitspferde: AM-GM und Cauchy-Schwarz . . . . . . . . . . . . . . . . . . . 63U.2.3 AM-HM Kandidaten . . . . . . . . . . . . . . . . . . . . . . . . . . . . . . . . . . 65U.2.4 Ungleichungen unter Nebenbedingungen . . . . . . . . . . . . . . . . . . . . . . . . 65U.3 Elementare symmetrische Funktionen . . . . . . . . . . . . . . . . . . . . . . . . . 66U.4 Weitere Ungleichungen . . . . . . . . . . . . . . . . . . . . . . . . . . . . . . . . . 68

G GEOMETRISCHE UNGLEICHUNGEN . . . . . . . . . . . . . . . . . . . 71G.1 Ungleichungen im Dreieck . . . . . . . . . . . . . . . . . . . . . . . . . . . . . . . 71G.1.1 Ungleichungen fur die Seitenlangen . . . . . . . . . . . . . . . . . . . . . . . . . . 72G.1.2 Ungleichungen fur die Winkel . . . . . . . . . . . . . . . . . . . . . . . . . . . . . 73G.1.3 Ungleichungen fur die Radien . . . . . . . . . . . . . . . . . . . . . . . . . . . . . 73G.1.4 Ungleichungen fur die Seiten- und Winkelhalbierenden . . . . . . . . . . . . . . . 73G.2 Ungleichungen in Vierecken . . . . . . . . . . . . . . . . . . . . . . . . . . . . . . . 73

X HINWEISE . . . . . . . . . . . . . . . . . . . . . . . . . . . . . . . . . . . . . . 75

Y LITERATUR . . . . . . . . . . . . . . . . . . . . . . . . . . . . . . . . . . . . . 79Y.1 Bucher . . . . . . . . . . . . . . . . . . . . . . . . . . . . . . . . . . . . . . . . . . 79Y.2 Zeitschriften-Artikel . . . . . . . . . . . . . . . . . . . . . . . . . . . . . . . . . . . 82Y.3 Zeitschriften . . . . . . . . . . . . . . . . . . . . . . . . . . . . . . . . . . . . . . . 82Y.4 WWW-Adressen . . . . . . . . . . . . . . . . . . . . . . . . . . . . . . . . . . . . . 82

A

KONSTRUKTIONEN

Eines der altesten Spiele der Welt ist das Spiel mit Zirkel und Lineal. Die zugehorigenSpielregeln wurden vor langer Zeit von Plato aufgestellt und legen fest, daß das Line-al nur dazu benutzt werden darf, um gerade Linien durch gegebene Punkte zu ziehen.Mit dem Zirkel durfen nur Kreise gezeichnet werden, die einen vorgegebenen Punkt alsMittelpunkt haben und durch einen weiteren Punkt gehen. Manchen Spielern ist das zueinfach, sie erlegen sich daher Einschrankungen auf, wie z. B. ein fest eingestellter Zirkel.Etwas Verwegene dagegen benutzen allein das Lineal, nachdem sie einen einzigen Kreisgezeichnet haben. Oder sie verzichten ganz auf das Lineal und benutzen allein einen Zir-kel. Andere Spieler wiederum verwenden andere Hilfsmittel, etwa das markierte Lineal(ein Lineal mit zwei Marken an seiner Kante) oder Streichholzer. Wer mehr dazu wissenwill, dem sei [Bie52], [Mar98] oder [Sma98] empfohlen.

Das Ziel all dieser Spiele ist es, geometrische Konstruktionen auszufuhren, in deren Ergeb-nis Figuren mit bestimmten Eigenschaften vorliegen. Um die geforderten Eigenschaftennachzuweisen, mussen wir oft die Algebra, Analytische Geometrie oder die Vektorrechnungbemuhen. Es gibt umfangreiche mathematische Theorien, aus denen folgt, welche Figurenmit welchen Hilfsmitteln konstruiert werden konnen und welche nicht. Die beruhmtestendrei antiken Probleme, die sich nicht mit Zirkel und Lineal in einer endlichen Anzahl vonSchritten losen lassen, kennt jeder:

1. Die Dreiteilung eines beliebigen Winkels. Gegeben sei ein beliebiger ebener Win-kel. Man teile ihn mit Zirkel und Lineal in drei gleiche Teile.

2. Die Verdopplung des Wurfels (Delianisches Problem). Gegeben sei eine Streckeder Lange 1. Man konstruiere daraus mit Zirkel und Lineal eine Strecke der Lange3√

2.

3. Die Quadratur des Kreises. Gegeben sei eine Strecke der Lange 1. Man konstruieredaraus mit Zirkel und Lineal ein Quadrat mit dem Flacheninhalt π.

Es gab in der Vergangenheit zahlreiche Versuche (und es gibt sie noch heute), die behaup-ten, eine Losung dieser klassischen Probleme gefunden zu haben, obwohl bewiesen ist, daßes nicht gehen kann. Zu diesem Thema gibt es eine recht unterhaltsame Literatur [Cou62],[Dun94, p. 237–247], [Dud92] und viele interessante Links [WWW.1–3].

10 KONSTRUKTIONEN

A.1 Euklidische Konstruktionen

Um ebene Geometrie oder Planimetrie erfolgreich betreiben zu konnen, benotigen wirdas dazu notwendige grundlegende Handwerkszeug. Darunter verstehen wir all diejenigenKonstruktionsaufgaben, die den meisten von uns aus der Schule bekannt sind und auf diebei der Losung schwierigerer Aufgaben immer wieder zuruckgegriffen wird. Man nennt sieauch die Grundaufgaben der Planimetrie. Dabei setzen wir nachPlato voraus, daß uns alsHilfsmittel lediglich ein Lineal (ohne Maßeinteilung) und ein Zirkel zur Verfugung stehen.Daruber hinaus gibt es auch geometrische Konstruktionen, die mit eingeschrankterenHilfsmitteln zu bewerkstelligen sind, wie etwa allein mit einem Zirkel; wir verweisen hierauf Abschnitt A.4.

Die folgenden Aufgaben, die also gewissermaßen das Fundament aller Fertigkeiten beigeometrischen Konstruktionen darstellen, konnen daher von geubten Lesern ruhig uber-sprungen werden.

A.1 Errichten der Senkrechten. In einem Punkt P auf einer Geraden g ist die Senk-rechte zu errichten.

A.2 Mittelsenkrechte. Man konstruiere die Mittelsenkrechte einer gegebenen StreckeAB.

A.3 Fallen des Lotes. Gegeben sei eine Gerade g und ein Punkt P , der nicht auf gliegt. Man falle das Lot von P auf g.

A.4 Parallele zu einer Geraden. Gegeben sei eine Gerade g und ein Punkt P , dernicht auf g liegt. Man konstruiere diejenige Gerade, die durch P geht und parallelzu g verlauft.

A.5 Winkelhalbierende. Gesucht ist die Winkelhalbierende eines beliebigen ebenenWinkels.

A.6 Abtragen eines Winkels. Man trage an eine gegebene Gerade g in einem be-stimmten Punkt einen gegebenen Winkel ab.

Es gibt noch weitere Aufgaben, die ebenfalls als”Grundbausteine“ geometrischer Kon-

struktionen betrachten werden konnen, so wie sie in komplexeren Losungen immer wiederauftreten. Wir betrachten hier nur eine Auswahl:

A.7 Drehen einer Strecke. Eine gegebene Strecke AB ist um einen Punkt P und umeinen bestimmten Winkel α zu drehen.

A.8 Kreis durch drei Punkte. Es ist ein Kreis zu beschreiben, der durch drei PunkteA, B, C einer Ebene geht, die nicht samtlich auf einer Geraden liegen.

A.9 Anlegen der Tangenten. Von einem außerhalb eines Kreises liegenden Punkt sinddie Tangenten an den Kreis zu legen.

A.10 Gemeinsame Tangenten zweier Kreise. Von zwei gegebenen Kreisen sind diegemeinsamen Tangenten zu bestimmen.

Geometrische Orter 11

A.11 Vierte Proportionale. Gesucht ist die vierte Proportionale q dreier gegebenerStrecken m, n und p; das heißt, es soll gelten:

m

n=p

q.

A.12 Teilen einer Strecke.Wie laßt sich eine gegebene Strecke a) innerlich, b) außerlichin einem rationalen Verhaltnis m : n (m,n > 0) teilen?

A.13 Stetige Teilung (Goldener Schnitt). Eine Strecke AB ist durch einen auf ihrliegenden Punkt C so zu teilen, daß der großere Abschnitt AC die mittlere Pro-portionale aus dem kleineren Abschnitt CB und der gesamten Strecke AB wird:AC : CB = AB : AC.

A.14 Mittelwerte. Mittlere Proportionale. Man konstruiere a) das arithmetische, b)das geometrische und c) das harmonische Mittel zweier Strecken l1 und l2.

A.15 Quadratische Gleichung. Wie lassen sich mit Zirkel und Lineal die Wurzeln derquadratischen Gleichung

x2 ± px± q = 0 mit x > 0

konstruieren, wenn p > 0 eine gegebene Lange und q > 0 ein gegebener Flachen-inhalt (etwa durch die Kantenlange a ≡ √

q eines Quadrates) ist?

A.2 Geometrische Orter

Oft werden bei der Losung geometrischer Aufgaben Punkte in der Ebene gesucht, die aufeiner gewissen (geraden oder

”krummen“) Linie liegen und gleichzeitig eine bestimmte

Bedingung erfullen. Gibt es außerhalb dieser Linie keinen weiteren Punkt, der die gefor-derte Eigenschaft besitzt, so nennt man diese Linie den geometrischen Ort aller Punkte,die gerade jene Bedingung erfullen. Einfache Beispiele fur geometrische Orter sind:

1. Der geometrische Ort aller Punkte einer Ebene, die von einem festen Punkt dieserEbene einen bestimmten Abstand haben, ist der Kreis um den gegebenen Punktmit dem Abstand als Radius.

2. Der geometrische Ort aller Punkte einer Ebene, die von zwei gegebenen Punktengleich weit entfernt sind, ist die Mittelsenkrechte auf deren Verbindungsstrecke.

3. Der geometrische Ort aller Punkte einer Ebene, die von einer gegebenen Geradeneinen vorgegebenen Abstand haben, ist das Parallelenpaar zur gegebenen Geradenin diesem Abstand.

4. Der geometrische Ort aller Punkte einer Ebene, die von zwei festen Parallelen glei-chen Abstand haben, ist die Mittelparallele zu den beiden gegebenen Parallelen.

5. Der geometrische Ort aller Punkte einer Ebene, die von zwei sich schneidenden Gera-den den gleichen Abstand haben, sind die beiden (senkrecht aufeinanderstehenden)Winkelhalbierenden der durch die Geraden gebildeten Winkel.

12 KONSTRUKTIONEN

6. Thales-Kreis. Der geometrische Ort der Scheitelpunkte aller rechten Winkel, derenSchenkel durch zwei feste Punkte A und B gehen, ist der Kreis uber der Strecke ABals Durchmesser.

Ein geometrischer Ort kann aber auch ein einzelner Punkt oder ein flachenhaftes Gebietder Ebene sein, je nachdem, wieviel Bedingungen gestellt werden. Allgemein ist festzustel-len: Je mehr Bedingungen zu erfullen sind, desto niedriger ist die Zahl der Freiheitsgradeund damit die Dimension des geometrischen Ortes.

Der Vorteil, den der Begriff geometrischer Ort bietet, besteht in der einfachen Umsetzungvon Bedingungen, die bestimmte Punkte erfullen sollen, in eine Konstruktionsvorschrift.Dabei konnen durchaus mehrere Bedingungen miteinander verknupft sein. Der zweifellosam haufigsten auftretende Fall ist der, daß Bedingung X und Bedingung Y zu erfullensind und somit der gesuchte geometrische Ort der Schnittpunkt X ∩ Y (im Sinne einesmengentheoretischen Durchschnitts) beider einzelner Orte ist. Wird z. B. ein Punkt Pgesucht, der von zwei gegebenen Punkten A und B den Abstand r ≡ AB hat, so kann mitHilfe obiger Aussagen sofort gefolgert werden, daß P der Schnittpunkt der beiden Kreisek1 ≡ Ar und k2 ≡ Br sein muß. Weiterhin ist damit auch klar, daß stets zwei Punkte Pund P ′ existieren, die diese Bedingung erfullen, namlich diejenigen, fur die die DreieckeABP und ABP ′ gleichseitig sind.

Weitere geometrische Orter werden wir in den folgenden Aufgaben betrachten:

A.21 Konstanter Sehwinkel. Gesucht ist der geometrische Ort aller Punkte P einerEbene, von denen aus eine gegebene Strecke AB unter einem konstanten, gegebe-nen Winkel erscheint, d. h., es sei �APB ≡ α = const.

A.22 Kreis des Apollonius. Gesucht ist derjenige geometrische Ort aller Punkte P einerEbene, fur den der Quotient der Abstande zu zwei fest vorgegebenen Punkten A

und B einen konstanten Wert PA/PB ≡ q = const annimmt.

A.23 Gesucht ist derjenige geometrische Ort aller Punkte P einer Ebene, fur den dieSumme der Quadrate der Abstande zu zwei fest vorgegebenen Punkten A und B

einen konstanten Wert PA2 + PB2 ≡ e2 = const annimmt.

A.24 Isoscelizer.† Gegeben seien zwei nichtparallele Geraden sowie ein beliebiger PunktP in der Ebene, der nicht mit dem Schnittpunkt beider Geraden zusammenfallt.Welches ist der geometrische Ort aller derjenigen Punkte, fur die die Summe derAbstande zu den Geraden gleich der Abstandssumme von P zu den Geraden ist?

†Leider gibt es hierfur kein vernunftiges deutsches Wort: ”Gleichschenkligmacher“ ware einemogliche Ubersetzung. Wir belassen es daher bei dem englischsprachigen Begriff.

A.25 Welches ist der geometrische Ort der Mittelpunkte aller von A ausgehenden Sehneneines Kreises MA?

A.26 Ein Boot mit Schmugglern bewegt sich in Kustennahe senkrecht auf das Ufer zu.Zu einem bestimmten Zeitpunkt wird es von einem Patrouille-Schnellboot† geortet.Welchen Kurs muß letzteres zum Aufbringen steuern, wenn die Positionen und die(als konstant vorausgesetzten) Geschwindigkeiten beider Boote bekannt sind?

†Nehmen wir spaßenshalber einmal an, der Zoll habe das schnellere Boot.

Kreiskonstruktionen 13

A.27 Gesucht ist derjenige geometrische Ort aller Punkte P einer Ebene, fur den derQuotient der Abstande zu zwei gegebenen Geraden g und h einen konstanten Wertq hat.

A.3 Kreiskonstruktionen

Die wohl bekanntesten Kreiskonstruktionen gehen auf antike Zeiten zuruck; man faßt sieauch unter dem Namen

”Beruhrungsproblem des Apollonius“ zusammen. Die Aufgabe be-

steht darin, Kreise zu finden, die drei gegebene Kreise beruhren. Dabei ist zugelassen, daßdie gegebenen Kreise auch in Punkte oder Geraden entartet sein konnen. Insgesamt erge-ben sich zehn verschiedene Falle der Kombination von vorgegebenen Stucken P (Punkt),G (Gerade) und K (Kreis):

1. drei Punkte (PPP),

2. zwei Punkte und eine Gerade (PPG),

3. ein Punkt und zwei Geraden (PGG),

4. drei Geraden (GGG),

5. zwei Punkte und ein Kreis (PPK),

6. ein Punkt, eine Gerade und ein Kreis (PGK),

7. ein Punkt und zwei Kreise (PKK),

8. zwei Geraden und ein Kreis (GGK),

9. eine Gerade und zwei Kreise (GKK) und

10. drei Kreise (KKK).

Wir beginnen mit den ersten vier Fallen:

A.31 Beruhrungsproblem des Apollonius (PPP). Gesucht sind Kreise, die durch dreigegebene Punkte P1, P2 und P3 gehen.

A.32 Beruhrungsproblem des Apollonius (PPG). Gesucht sind Kreise, die durch zweigegebene Punkte P1 und P2 gehen und die Gerade g beruhren.

A.33 Beruhrungsproblem des Apollonius (PGG). Gesucht sind Kreise, die durcheinen gegebenen Punkt P gehen und zwei Geraden g1 und g2 beruhren.

A.34 Beruhrungsproblem des Apollonius (GGG). Gesucht sind Kreise, die drei ge-gebene Geraden g1, g2 und g3 beruhren.

Die nachfolgende Tabelle A.1 enthalt eine Zusammenfassung der Losungsmoglichkeitendes Beruhrungsproblems des Apollonius (w(g1, g2) bedeutet fur g1 ∦ g2 die Winkelhal-bierende und fur g1 ‖ g2 die Mittelparallele beider Geraden).

14 KONSTRUKTIONEN

Tabelle A.1. Anzahl der Losungen beim Beruhrungsproblem des Apollonius

Gegeben Bedingung Anzahl

P1, P2, P3 Punkte nicht kollinear 1Punkte kollinear 0

P1, P2, g P1, P2 /∈ g und h(P1, P2) ∦ g 2P1, P2 /∈ g und h(P1, P2) ‖ g 1genau ein Punkt auf g 1beide Punkte auf g 0beide Punkte in unterschiedlichen Halbebenen 0

P , g1, g2 g1 ∦ g2 und P nicht auf einer Geraden 2g1 ‖ g2 und P zwischen beiden Geraden 2P auf genau einer Geraden 1P = (g1 ∩ g2) 0g1 ‖ g2 und P nicht zwischen beiden Geraden 0

g1, g2, g3 3 Schnittpunkte der drei Geraden 4genau zwei Geraden parallel 2alle drei Geraden parallel 0nur ein Schnittpunkt der drei Geraden 0

A.4 Mohr-Mascheronische Konstruktionen

In gewohnter Weise verstehen wir unter der Aufgabe, eine geometrische Konstruktiondurchzufuhren, diese mit den Hilfsmitteln Zirkel und Lineal zu bewerkstelligen. Es er-scheint aber durchaus angebracht danach zu fragen, ob dieselben Aufgaben nicht ebensomit nur einem dieser Gerate gelost werden konnen.

Dieser Abschnitt beschaftigt sich damit, diejenigen geometrischen Konstruktionen zuerortern, fur die ausschließlich der Zirkel zugelassen ist. Als erster nahm sich Georg

Mohr dieser Problemstellung an. Sein 1672 erschienenes Buch”Euclidis Danicus“ war

jedoch lange Zeit verschollen, bis es 1927 von J. Hjelmslev in einer KopenhagenerBuchhandlung wieder aufgefunden wurde. Aus diesem Grunde wird haufig der italieni-sche Mathematiker Lorenzo Mascheroni als Wegbereiter der

”Geometrie des Zirkels“

angesehen, der ohne Kenntnis von Mohrs Ideen im Jahre 1797 sein Werk”La geometria

del compasso“ veroffentlichte. Als Literatur zu diesem Thema sei [Bie52], [Enr07], [Mar98]

empfohlen.

Analysieren wir einmal die einzelnen Schritte, die bei einer herkommlichen Konstruktionmit Zirkel und Lineal auszufuhren sind, so erkennen wir, daß jeder Schritt in eine der dreifolgenden Grundkonstruktionen zerfallt:

1. die Schnittpunkte zweier Kreise zu bestimmen,

2. die Schnittpunkte einer Geraden und eines Kreises zu finden und

3. den Schnittpunkt zweier Geraden zu ermitteln.

Verschiedene Konstruktionen 15

Demgemaß haben wir lediglich zu zeigen, daß sich die beiden Grundkonstruktionen 2und 3 mit dem Zirkel allein bewerkstelligen lassen. Da selbstverstandlich mit dem Zirkelkeine Gerade gezogen werden kann, gilt in diesem Abschnitt eine solche als gegeben oderbestimmt, wenn zwei ihrer Punkte bekannt sind.

Bevor wir zu den beiden Grundkonstruktionen kommen, mussen noch einige Vorarbeitenerledigt werden:

A.41 Durch einen Punkt A ist die Parallele zu einer Geraden BC zu ziehen.

A.42 Es ist eine gegebene Strecke OA zu verdoppeln, zu verdreifachen usw.

A.43 Ein gegebener Punkt C ist bezuglich einer Geraden AB zu spiegeln.

A.44 Es ist ein Kreisbogen zu halbieren.

A.45 Gesucht ist die vierte Proportionale q zu drei gegebenen Strecken m, n und p.

Nach diesen”aufwarmenden“ Ubungen sollte es gelingen, auch Schnittpunkte von Ge-

raden und Kreisen und von Geraden untereinander zu finden. Wir unterscheiden dabeizweckmaßigerweise bei der obigen Grundaufgabe 2 zwischen nichtzentralen und zentralenGeraden sowie bei der Grundaufgabe 3 , ob die Geraden senkrecht aufeinander stehenoder nicht.

A.46 Es sind die gemeinsamen Punkte eines Kreises und einer nicht durch seinen Mit-telpunkt gehenden (nichtzentralen) Gerade zu finden.

A.47 Es sind die gemeinsamen Punkte eines Kreises und einer durch seinen Mittelpunktgehenden (zentralen) Gerade zu finden.

A.48 Es ist der gemeinsame Punkt zweier nicht zueinander senkrechter Geraden zubestimmen.

A.49 Es ist der gemeinsame Punkt zweier senkrecht aufeinander stehender Geraden zubestimmen, oder gleichwertig damit: Es ist der Fußpunkt des von einem Punkt aufeine Gerade gefallten Lotes zu bestimmen.

A.50 Man konstruiere den Mittelpunkt einer Strecke.

A.5 Verschiedene Konstruktionen

Abschließend zum Thema Konstruktionen geben wir noch einige”Klassiker“ an.

A.61 Von einem Winkel sind Teile beider Schenkel gegeben; sein Scheitel sei nichtzuganglich. Gesucht ist die Winkelhalbierende.

A.62 In einen Kreissektor ist ein Kreis einzubeschreiben.

A.63 C sei die Mitte der Strecke AB. Uber AB, AC und BC als Durchmesser werdenHalbkreise nach derselben Seite errichtet. Es soll der Kreis konstruiert werden,welcher die drei Halbkreise beruhrt.

16 KONSTRUKTIONEN

A.64 In einem Trapez ABCD ist eine Gerade parallel zu den Grundseiten AB und CD

zu konstruieren, die das Trapez halbiert.

A.65 Es ist ein Quadrat zu zeichnen, dessen (ggf. verlangerte) Seiten durch vier gegebenePunkte gehen.

A.66 Es ist ein Kreis zu beschreiben, der durch die Punkte A, B geht und aus derGeraden g eine Sehne der Lange s herausschneidet.

A.67 Es ist ein gleichseitiges Dreieck zu beschreiben, dessen Ecken auf drei Parallelenliegen.

A.68 Gegeben sei eine Gerade g sowie zwei nicht auf ihr, aber in derselben Halbebeneliegende Punkte A und B. Gesucht sind diejenigen Punkte C auf g, fur die CBdie Winkelhalbierende von CA und g ist.

A.69 Es ist ein gleichseitiges Dreieck zu beschreiben, dessen Ecken auf den Umfangendreier konzentrischer Kreise liegen.

A.70 Zauberspiegel†. Gegeben seien zwei feste Punkte A und B sowie eine Geradeg. Gesucht ist derjenige Punkt X auf g, fur den der Strahlenverlauf bei einerReflexion A → X → B gerade so ist, daß der Einfallswinkel doppelt so groß wieder Ausfallswinkel ist.

†(nach einer Idee inkl. Losung von Johannes Wutscher)

A.71 Gegeben sind zwei Kreise k1, k2 und eine Gerade g. Ein Quadrat ist so zu zeichnen,daß zwei Gegenecken auf die Peripherien von k1 und k2, die beiden anderen auf gfallen.

A.72 Gegeben seien zwei feste Punkte A und B sowie eine Gerade g; auf ihr ist S so zubestimmen, daß AS + SB gleich einer gegebenen Strecke s wird.

B

DREIECKSKONSTRUKTIONEN

Die drei Seiten und die drei Winkel eines Dreiecks nennt man ublicherweise die sechsBestimmungsstucke desselben. Daneben gibt es eine Vielzahl weiterer Stucke, wie z. B.die Langen der Hohen, Seiten- und Winkelhalbierenden, In- und Umkreisradius usw., dieebenfalls charakteristische Großen eines Dreiecks darstellen. Die Abhangigkeit der Drei-ecksstucke voneinander macht es nun unmoglich, die Große der einzelnen Stucke beliebig,also unabhangig voneinander festzusetzen und daraus ein Dreieck herstellen zu wollen. Esstellt sich vielmehr heraus, daß schon drei von den sechs Bestimmungsstucken das ganzeDreieck festlegen und somit alle ubrigen Stucke mitbestimmen. Als einzige Ausnahme giltdie Vorgabe von drei Winkeln, da durch zwei Winkel der dritte bereits mitbestimmt ist,in Wirklichkeit also nur zwei Stucke festgelegt sind.

Wir wiederholen im Abschnitt B.1 zunachst die allseits bekannten Kongruenzsatze, umanschließend im Abschnitt B.2 auf die gebrauchlichste Methode des Auffindens von Hilfs-dreiecken einzugehen. Die Abschnitte B.3 und B.4 sind noch sehr unvollstandig, werdenaber in Zukunft noch kraftig erweitert. Ein geplanter Abschnitt uber nicht mit Zirkel undLineal durchfuhrbare Dreieckskonstruktionen wird das Kapitel spater abrunden.

B.1 Die Grundaufgaben

Die einfachsten Falle von Dreieckskonstruktionen liegen vor, wenn nur Bestimmungsstuckevorgegeben werden. Diese werden auch die vier Grundaufgaben fur das Dreieck genannt,und man faßt deren Inhalt in den Kongruenzsatzen zusammen. Wir wiederholen sie andieser Stelle:

B.1 Kongruenzsatz SWS. Es ist ein Dreieck zu zeichnen, von dem zwei Seiten undder von ihnen eingeschlossene Winkel gegeben sind.

B.2 Kongruenzsatz WSW. Es ist ein Dreieck zu zeichnen, von dem eine Seite undzwei Winkel gegeben sind.

B.3 Kongruenzsatz SSS. Es ist ein Dreieck zu zeichnen, dessen drei Seiten gegebensind.

18 DREIECKSKONSTRUKTIONEN

B.4 Kongruenzsatz SSW. Es ist ein Dreieck zu zeichnen, von dem zwei Seiten undein gegenuberliegender Winkel gegeben sind.

Alle anderen Dreieckskonstruktionen sind weniger gelaufig und werden daher gern als

”Knobelaufgaben“ gestellt oder zum Argern von Schulern (oder Lehrern!) verwendet. Es

wird in den folgenden Abschnitten der Versuch unternommen, etwas Systematik in diesesGebiet zu bringen.

B.2 Auffinden von Hilfsdreiecken

Kommen unter den drei Stucken, die zur Konstruktion eines Dreiecks gegeben sind, auchandere Stucke als Seiten und Winkel vor, so kann man zur Losung der Aufgabe einenWeg einschlagen, den man als die Losung durch Hilfsdreiecke bezeichnet. Man zeichnetzunachst ein beliebiges Dreieck und hebt darin die jeweils gegebenen Stucke kraftig her-vor. Hierbei gelangt man im allgemeinen zu weiteren Dreiecken und untersucht, ob unterdiesen neuen Dreiecken eines ist, das man mit Hilfe der gegebenen Stucke nach einer dersoeben behandelten Grundaufgaben konstruieren kann. Schließlich uberlegt man, wie nachKonstruktion dieses Hilfsdreiecks das verlangte Dreieck hergestellt werden kann. DiesenTeil der Losung nennt man die Analysis. Darauf folgen dann: die Konstruktion, die aufGrund der in der Analysis angestellten Uberlegungen ausgefuhrt wird, die Behauptung,daß die Konstruktion wirklich das verlangte Dreieck geliefert hat, und der Beweis furdie Richtigkeit der Behauptung. Als letzter Teil folgt die Determination (nahere Bestim-mung). In dieser wird untersucht, in welchen Fallen die Aufgabe nicht losbar ist, undermittelt, ob die moglichen Losungen das Dreieck eindeutig oder eventuell mehrdeutigbestimmen.

Als Beispiele fur ausfuhrliche Losungen mit allen ihren Schritten Analysis, Konstrukti-on, Behauptung, Beweis und Determination mogen folgende Aufgaben dienen, wobei diezweite eine altere Olympiadeaufgabe ist, deren Aufgaben- und Losungstext weitestgehendoriginal ubernommen wurde.

B.11 Gesucht ist ein Dreieck, von dem zwei Seiten b, c und die Hohe hc auf die eine vonihnen gegeben sind.

B.12 Konstruieren Sie ein rechtwinkliges Dreieck ABC mit dem rechten Winkel bei Caus ma = 6 cm, mb = 8 cm. Dabei seien ma die Lange der Seitenhalbierendenvon BC und mb die der Seitenhalbierenden von AC. Beschreiben und begrundenSie Ihre Konstruktion. Untersuchen Sie, ob ein derartiges Dreieck ABC mit dengegebenen Langen ma, mb existiert und bis auf Kongruenz eindeutig bestimmt ist.(13. Mathematik-Olympiade 1973/74, Klasse 10, Stufe 4 )

Es gibt naturlich eine Unmenge moglicher Kombinationen von drei Stucken, durch derenVorgabe ein Dreieck konstruiert werden kann. Eine umfassende Zusammenstellung findetder interessierte Leser in [Her86]. Aus Platzgrunden wird in den weiteren Losungen aufdie Darlegung des vollstandigen Losungsweges von Analysis bis Determination verzichtetund meist nur die Losungsidee mit der Konstruktionsbeschreibung angegeben. Es sei

Berechnung fehlender Stucke 19

jedoch noch einmal ausdrucklich darauf hingewiesen, daß die obigen funf Schritte bei derschriftlichen Losung einer Wettbewerbsaufgabe moglichst vollstandig abzuarbeiten sind.

Daruber hinaus ist es angebracht, fur haufig auftretende Großen in einem Dreieck einheit-liche Symbole einzufuhren. Diese sind in Tabelle X.2 aufgefuhrt. Die gewahlten Bezeich-nungen stimmen zwar nicht in jedem Fall mit den im Unterricht gebrauchlichen uberein,wir wollen sie dafur aber uberall konsequent verwenden.

Bei den folgenden Konstruktionsaufgaben besteht der erste Schritt immer im Auffindeneines geeigneten Hilfsdreiecks. Dieses garantiert jedoch nicht in allen Fallen einen Wegzur Losung. Oft sind dazu weitere Kenntnisse notwendig, so daß es sich ggf. empfielt,zunachst im Kapitel D nachzusehen. Es ist klar, daß es in vielen Fallen auch vollig andere(und teilweise sicherlich kurzere) Losungsmoglichkeiten gibt.

B.13 Es ist ein Dreieck aus γ, wc und r zu konstruieren.

B.14 Es ist ein Dreieck aus γ, mb und R zu konstruieren.

B.15 Es ist ein Dreieck aus hc, wc und mc zu konstruieren.

B.16 Es ist ein Dreieck aus γ, hb und mc zu konstruieren.

B.17 Es ist ein Dreieck aus a, b und mc zu konstruieren.

B.18 Es ist ein Dreieck aus ma, mb und hc zu konstruieren.

B.19 Es ist ein Dreieck aus ma, mb und mc zu konstruieren.

B.20 Es ist ein Dreieck aus γ, hc und s zu konstruieren.

B.21 Es ist ein Dreieck zu konstruieren, von dem die Lange der Seite c, die Lange derHohe hc und die Differenz der Innenwinkel 0 < α− β < 90◦ gegeben sind.(35. Mathematik-Olympiade 1995/96, Klasse 10, Stufe 4 )

B.22 Es ist ein Dreieck aus α− β, hc und R zu konstruieren.

B.23 Es ist ein rechtwinkliges Dreieck aus c und a+ b zu konstruieren.

B.3 Berechnung fehlender Stucke

In einigen Fallen gelingt es nicht ohne weiteres, mit den vorgegebenen Stucken an ein kon-struierbares Hilfsdreieck zu gelangen. Oft kann man sich dennoch helfen, indem fehlendeStucke durch Anwendung bekannter elementarer Satze berechnet werden, die ihrerseitskonstruierbar sind. Meist genugt es, z. B. die Strahlensatze oder den Satz des Pythago-ras geschickt auszunutzen, wie folgendes Beispiel zeigt.

B.41 Es ist ein Dreieck aus a, b und wc zu konstruieren.

20 DREIECKSKONSTRUKTIONEN

B.4 Rekonstruktion aus gegebenen Punkten

Allgemein schwerer scheint es, Dreiecke aus vorgegebenen Lagen besonderer Punkte zu(re)konstruieren. Diese Aufgaben verlangen mitunter die Kenntnis spezieller (aber nichtunbedingt komplizierter) Sachverhalte aus der Geometrie des Dreiecks. Sie sollten dahererst nach Durcharbeiten des Kapitels D in Angriff genommen werden.

B.51 Es ist ein Dreieck aus γ = 90◦, c und der Lage des Beruhrungspunktes F desInkreises auf der Hypotenuse zu konstruieren.(Crux Mathematicorum 2415, Marz 1999)

B.52 Uber jede Seite eines Dreiecks ABC, dessen Innenwinkel kleiner als 120◦ sind, wer-den gleichseitige Dreiecke BCA′, CAB′ und ABC ′ errichtet. Man (re)konstruiere�ABC, wenn die Punkte A′, B′ und C ′ gegeben sind.

K

KREISE

Der Kreis ist eine der einfachsten und zugleich schonsten Figuren der Geometrie. Wer hatnicht in den ersten Tagen, als er den Umgang mit einem Zirkel lernte, entlang der Peri-pherie eines Kreises ein regelmaßiges Sechseck oder blumenartige Ornamente konstruiertund sich an deren Asthetik erfreut. Wollen wir jedoch mehr uber die Wechselbeziehungenzwischen Kreisen und Punkten, Geraden, Winkeln oder Flachen erfahren, kommen wirnicht an den folgenden Satzen vorbei, die ausnahmslos aus dem Unterricht bekannt seindurften. Wir beginnen mit Satzen uber Winkel- und Langenbeziehungen am Kreis.

K.1 Winkel und Langen

In diesem Abschnitt wiederholen wir die wichtigsten Satze uber Winkel- und Langenbe-ziehungen am Kreis, die uns noch aus dem Schulunterricht vertraut sind.

K.1 Peripheriewinkelsatz. Alle Peripheriewinkel uber demselben Bogen sind einandergleich.

K.2 Peripherie-Zentriwinkel-Satz. Die uber einem Bogen und einer Sehne liegendenPeripheriewinkel eines Kreises sind untereinander gleich und halb so groß wie derzugehorige Zentriwinkel; Peripheriewinkel, die auf verschiedenen Seiten derselbenSehne liegen, erganzen sich zu 180◦.

K.3 Satz des Thales. Verbindet man einen Punkt der Peripherie eines Kreises mitden Endpunkten eines beliebigen Durchmessers, so stehen die Verbindungsliniensenkrecht aufeinander.

K.4 Sehnen-Tangentenwinkel-Satz. Ein Sehnen-Tangentenwinkel hat stets die gleicheGroße wie jeder Peripheriewinkel uber dem Kreisbogen, der zwischen den Schen-keln des Sehnen-Tangentenwinkels liegt.

Daruber hinaus gibt es einige Satze, die Aussagen uber Langenrelationen zwischen Sehnen,Sekanten bzw. Tangenten am Kreis machen. Dabei taucht mitunter folgender Begriff auf,der auf Jacob Steiner zuruckgeht:

22 KREISE

Potenz. Als Potenz P eines Punktes A bezuglich ei-nes Kreises k ≡ Or (Mittelpunkt O, Radius r) bezeich-net man die Große P(A) ≡ OA2 − r2. Klar, daß allePunkte auf der Peripherie von k die Potenz null haben;innerhalb des Kreises ist sie negativ und außerhalb po-sitiv. Der geometrische Ort aller Punkte gleicher Potenzbezuglich eines gegebenen Kreises k sind konzentrischeKreise zu k.

A

Or

k

d

K.11 Sehnensatz. Schneiden sich in einem Kreis zwei Sehnen, so ist das Produkt derAbschnittslangen der einen Sehne gleich dem Produkt der Abschnittslangen deranderen.

K.12 Sekantensatz. Schneiden sich zwei Sekanten eines Kreises außerhalb des Kreises,so ist das Produkt der Abschnittslangen vom Sekantenschnittpunkt bis zu denSchnittpunkten von Kreis und Sekante auf beiden Sekanten gleich groß.

K.13 Sekanten-Tangentensatz. Fur jeden Punkt außerhalb eines Kreises ist die Langedes Abschnitts bis zum Beruhrungspunkt auf einer vom Punkt an den Kreis ge-legten Tangente die mittlere Proportionale zu den Langen der Abschnitte, die derKreis auf einer Sekante durch den Punkt abschneidet.

K.14 Netzhaut-Satz. (Bild) Der Abstand derMittelpunkte zweier Kreise mit den Radiena und b betrage s > a + b. Zieht man dieTangenten von einem Mittelpunkt an denjeweils anderen Kreis, dann sind die durchdie ruckwartigen Verlangerungen herausge-

a bsx y

schnittenen Sehnen x und y untereinander gleich.

K.15 In einem Kreis halbiere der Durchmesser AB die Sehne CD. Eine weitere SehneAQ schneide CD in P . Dann hat der Ausdruck AP ·AQ unabhangig von der Lagevon P stets denselben Wert.

K.16 PT und PU seien Tangenten von einem Punkt P an zwei konzentrische Kreise,wobei PT den kleineren Kreis beruhrt. Weiterhin schneide PT den großeren Kreisin Q. Dann gilt: PT 2 − PU2 = QT 2.

D

DREIECKE

Das Dreieck ist neben dem Kreis die am haufigsten betrachtete Figur der Planimetrie.Zahlreiche Beziehungen und Satze uber Großen im Dreieck wurden im Laufe der Zeit ge-funden. Geschichtlich gesehen kann man zwei Epochen ausmachen, in denen wesentlichesentstand bzw. hinzukam: die Antike und das 19. Jahrhundert, das mit solchen Namenwie Feuerbach, Gergonne und Nagel verbunden ist (

”Neuere Dreiecksgeometrie“, s.

[Bap92]).

Dieses Kapitel listet eine Vielzahl von Satzen uber das Dreieck auf, wobei klar ist, daßVollstandigkeit keinesfalls erreicht werden kann. Je mehr Sachverhalte wir aber kennen,desto leichter wird uns das Problemlosen fallen. Bezuglich der Schreibweise halten wir unsan Tabelle X.2.

Wir beginnen mit denjenigen bemerkenswerten Punkten und Linien im Dreieck, die schonim Altertum bekannt waren.

D.1 Klassische Transversalen

Jeder kennt aus dem Geometrieunterricht jene Paare von Begriffen, die stets zusammen-gehoren:

1. der Umkreismittelpunkt O als Schnittpunkt der Mittelsenkrechten der Seiten,

2. der Inkreismittelpunkt I als Schnittpunkt der Winkelhalbierenden wa, wb, wc,

3. der Schwerpunkt G als Schnittpunkt der Seitenhalbierenden ma, mb, mc und

4. der Hohenschnittpunkt H als gemeinsamer Punkt der Hohen ha, hb, hc.

Alle auftretenden Geraden haben gemeinsam, daß sie Transversalen eines Dreiecks sind,d. h. dieses schneiden. 2 bis 4 sind sogar Ecktransversalen, im Englischen auch cevian ge-nannt. Naturlich sollten wir zunachst zeigen, daß die genannten Schnittpunkte tatsachlichdie Schnittpunkte aller drei Transversalen sind. Wir wollen den Nachweis an dieser Stellejedoch nur fur den Umkreismittelpunkt fuhren; die anderen Beweise lassen sich elegantermit der Umkehrung des Satzes von Ceva erledigen (s. Abschnitt D.2).

D.1 Die Mittelsenkrechten der Seiten eines Dreiecks schneiden sich in einem Punkt,dem Umkreismittelpunkt O des Dreiecks.

24 DREIECKE

Bleiben wir bei unserer Tour quer durch das”klassische“ Dreieck zunachst beim Umkreis,

zu dem es weitere bemerkenswerte Dinge festzustellen gibt.

D.2 (Bild) In einem �ABC schneide die Mittelsenkrechte derSeite AB den Umkreis in den beiden Punkten F und F ′.Dabei sollen C und F ′ auf derselben Seite bezuglich AB, Cund F auf unterschiedlichen Seiten liegen. Man zeige, daßdann die Strecke CF den Innenwinkel �ACB sowie CF ′ A B

CF ′

Fden zugehorigen Außenwinkel halbiert.

Fur zahlreiche Satze gilt — wie wir noch sehen werden — auch die Umkehrung, d. h., dieBehauptung wird zur Voraussetzung und umgekehrt. So ist es auch bei dem vorangegan-genen Satz. Wenn wir ihn nicht beachten, gelangen wir mitunter zu kuriosen Resultaten.

D.3 In einem Dreieck schneiden sich die Mittelsenkrechte einer Seite und die Winkel-halbierende des gegenuberliegenden Winkels stets auf dem Umkreis.

D.4 Paradoxon: Jedes Dreieck ist gleichschenklig.(Pseudo)Beweis: (Bild) Im Dreieck ABC halbiere CDden Innenwinkel bei C undMD sei Mittelsenkrechte aufAB. Dann ist �CED ∼= �CFD nach KongruenzsatzWSW; die rechtwinkligen Dreiecke AMD und BMD

sind nach Kongruenzsatz SWS ebenfalls kongruent. We-gen ED = FD, AD = BD, �AED = �BFD = 90◦ A B

C

D

E F

M

folgt daraus �AED ∼= �BFD. Also ist AE = BF und somit auch AC = BC,d. h., das Dreieck ABC ist gleichschenklig. — Wo steckt der Fehler?

Auch bei dem bekannten Sinussatz gibt es ein Detail, das gern ubersehen wird. Es findetsich aber leicht, wenn der Umkreis hinzugenommen wird.

D.5 Erweiterter Sinussatz. In jedem Dreieck gilt

a

sinα=

b

sin β=

c

sin γ= 2R. (D.1)

D.6 Auf dem Umkreis eines gleichseitigen Dreiecks ABC liege ein Punkt P , der nichtmit einem Eckpunkt zusammenfallt. Die Sehne AP schneide die Seite BC in Q.Man zeige, daß gilt:

a)1

PB+

1

PC=

1

PQ, b) PB + PC = PA.

Da es ohnehin nicht moglich ist, die Satze der Elementargeometrie streng nach beteiligtenGebilden zu ordnen, beziehen wir im weiteren die Winkelhalbierenden eines Dreiecks inunsere Betrachtungen mit ein. Folgendes Resultat ist so einfach, daß es eigentlich nichtin diese Reihe gehort. Da wir es aber spater haufig zitieren, sei es hier angegeben.

D.7 Die Winkelhalbierenden von Innen- und zugehorigem Außenwinkel eines Dreiecksstehen senkrecht aufeinander.

Klassische Transversalen 25

Der wohl am haufigsten benutzte Satz, der im Zusammenhang mit Winkelhalbierendenim Dreieck steht, ist der folgende:

D.8 In einem Dreieck teilt jede Halbierende eines Innenwinkels (Außenwinkels) diegegenuberliegende Seite innerlich (außerlich) im Verhaltnis der anliegenden Seiten.

D.9 Der Winkel, unter dem eine Dreieckseite vom Inkreismittelpunkt aus gesehen wird,ist gleich dem um 90◦ vermehrten halben Gegenwinkel der Seite.

Wenden wir uns nun dem Schwerpunkt eines Dreiecks zu. Seinen Namen verdankt erfolgender Beobachtung: Es wird ein beliebiges Dreieck auf ein Stuck Sperrholz gezeichnetund mit der Laubsage ausgesagt. Versuchen wir nun die Dreiecksflache von unten mit einerNadel zu balancieren, gelingt dies nur, wenn die Nadel das Dreieck genau im Schwerpunktunterstutzt.

D.10 Die drei Seitenhalbierenden eines Dreiecks schneiden sich im Schwerpunkt G desDreiecks. — In welchem Verhaltnis teilt der Schwerpunkt die Seitenhalbierenden?

D.11 Ein Dreieck wird durch seine Seitenhalbierenden in sechs kleinere Teildreiecke zer-legt, die untereinander gleichen Flacheninhalt haben.

D.12 Eine Gerade durch den Schwerpunkt G eines Dreiecks schneide dessen Seiten inden Punkten X, Y , Z. Man beweise, daß dann fur die gerichteten Strecken GX,GY , GZ (s. dazu Abschnitt D.2) gilt:

1

GX+

1

GY+

1

GZ= 0.

D.13 Gegeben sei ein Dreieck ABC mit einer beliebigen Gerade g durch dessen Schwer-punkt. Liegen zwei Eckpunkte des Dreiecks auf der gleichen Seite von g, so ist dieSumme ihrer Abstande von g gleich dem Abstand des dritten Eckpunktes von g.

Den Abschluß unserer Tour bildet der Hohenschnittpunkt. Er ist der Ausgangspunkt furein spezielles

”Dreieck im Dreieck“, das Hohenfußpunktdreieck. Die zahlreich vorhandenen

rechten Winkel pradestinieren es geradezu fur Anwendungen der Ahnlichkeitssatze undder besonderen Eigenschaften von Sehnenvierecken.

D.21 Die Hohen in einem spitzwinkligen Dreieck halbieren die Innenwinkel des Hohen-fußpunktdreiecks. Oder anders ausgedruckt: Der Hohenschnittpunkt ist der In-kreismittelpunkt des Hohenfußpunktdreiecks.

D.22 In einem Dreieck teilt der Hohenschnittpunkt jede Hohe so in zwei Abschnitte,daß die Produkte ihrer Langen untereinander gleich sind.(14. Mathematik-Olympiade 1974/75, Klasse 9, Stufe 2 )

D.23 Spiegelt man in einem spitzwinkligen Dreieck den Hohenschnittpunkt an den Sei-ten, so liegen die Bildpunkte auf dem Umkreis des Dreiecks.

D.24 Im �ABC seien H der Hohenschnittpunkt und O der Mittelpunkt des Umkreises.Dann sind die Winkel �HAB und �OAC gleich groß.

26 DREIECKE

Der Hohenschnittpunkt fuhrt in Verbindung mit dem Umkreismittelpunkt auf eine Diffe-renz von Innenwinkeln im Dreieck, welche mitunter vorgegeben ist, aber ohne Kenntnisdieses Zusammenhangs nicht so einfach anzuwenden ist.

D.25 Im Dreieck ABC betragt der Winkel �HCO = |α− β|.

D.2 Ceva & Menelaus

Es ist schade, daß zwei so uberaus nutzliche Satze, wie die von Ceva und Menelaus

sowie deren Umkehrung, nicht zum Standardprogramm im Geometrieunterricht gehoren.Sie leisten oft gute Dienste, wenn es zu zeigen gilt, daß sich drei Geraden in einem Punktschneiden (kopunktal sind) bzw. drei Punkte auf einer Geraden liegen (kollinear sind).Ohne diese Satze lassen sich derartige Beweise ungleich aufwendiger fuhren. Wir demon-strieren dies an den bisher offenen Problemen aus Abschnitt D.1.

Geschichtlich interessant ist die Tatsache, daß 15 Jahrhunderte zwischen beiden Ent-deckungen liegen, obwohl sie eigentlich sehr ahnliche Aussagen machen. Wahrend Mene-

laus seinen Satz bereits ca. 80 n. Chr. in seinem Werk”Spharik“ niederschrieb, wurde

sein”dualer“ Partner erstmals 1678 von dem Italiener Giovanni Ceva in seiner Schrift

”De lineis rectis se invicem secantibus, statica constructio“ erwahnt.

Wir beschaftigen uns im ersten Teil dieses Abschnitts mit der Kopunktualitat, d. h. derBedingung fur das Zusammentreffen dreier Geraden in einem Punkt.

D.31 Satz von Ceva. (Bild) Wenn sich in einem �ABC diedrei Ecktransversalen AX, BY und CZ in einem Punktschneiden, dann hat das Produkt der Teilverhaltnisse,das ihre Schnittpunkte mit den Gegenseiten auf diesenbilden, den Wert 1:

AZ

ZB· BXXC

· CYYA

= 1. (D.2)

A B

C

XY

Z

D.32 Wie lautet die trigonometrische Form des Satzes von Ceva, wenn anstelle derLangen AZ, . . . , YA die sechs gegenuberliegenden Winkel verwendet werden?

D.33 Umkehrung des Satzes von Ceva. Wenn das Produkt der Teilverhaltnisse, dasdie Schnittpunkte der Ecktransversalen eines Dreiecks mit den Gegenseiten aufdiesen bilden, den Wert 1 hat, dann schneiden sich die Ecktransversalen in einemPunkt (vgl. Aufgabe D.31).

Mitunter wird nicht so streng zwischen diesem Satz und seiner Umkehrung unterschiedenund bei beiden vom Satz von Ceva gesprochen. Jetzt sind wir in der Lage, sehr bequemnachzuweisen, daß sich auch die anderen Ecktransversalen aus Abschnitt D.1 stets ineinem Punkt schneiden.

Ceva & Menelaus 27

D.34 Unter Verwendung der Umkehrung des Satzes von Ceva ist zu beweisen, daßsich a) die Seitenhalbierenden, b) die Winkelhalbierenden und c) die Hohen einesDreiecks stets in einem Punkt schneiden.

Damit sind die Anwendungsmoglichkeiten des Satzes vonCeva noch lange nicht erschopft,wie einige, aus dem 19. Jahrhundert stammende Beispiele zeigen. Dazu benotigen wirallerdings noch folgende Definitionen:

Halbumfangspunkte. Fahren wir ausgehend von Eck-punkt A entlang der Seiten einmal den Umfang einesDreiecks ab, so gibt es auf der gegenuberliegenden SeiteBC einen Punkt A′, fur den der Weg AB + BA′ uberEckpunkt B gerade gleich dem Weg AC + CA′ uber Cist. A′ heißt dann Halbumfangspunkt zu A; ebenso defi-nieren wir B′ bzw. C ′. A B

C

A′B ′

C ′

D.35 Die Beruhrungspunkte der drei Ankreise mit den Dreieckseiten fallen mit denHalbumfangspunkten zusammen.

Isotomische Geraden. (Bild) Angenommen, eine Eck-transversale g durch C schneide die gegenuberliegendeDreieckseite AB im Punkt D. Dann gibt es auf dieserSeite immer einen Punkt D′ (den zu D isotomisch gele-genen Punkt), fur den AD′ = BD bzw. BD′ = AD gilt.Die durch C und D′ gehende Gerade g′ heißt dann isoto-misch zu g. Wir erkennen sofort, daß D und D′ auch alsspiegelbildliche Punkte aufgefaßt werden konnen, wobei A B

C

D D ′

g g ′

die Mitte der Seite AB als Spiegelungszentrum auftritt. Oder anders ausgedruckt:Wenn wir das Teilungsverhaltnis AD/DB mit x bezeichnen, dann teilt der isoto-mische Punkt AB im Verhaltnis AD′/D′B = 1/x.

D.36 Isotomisch konjugierter Punkt. (Bild) P sei ein imInnern des �ABC liegender Punkt. Die drei Eck-transversalen AP , BP , CP schneiden die jeweils ge-genuberliegenden Seiten in den PunktenX, Y bzw. Z.Man beweise, daß sich die drei isotomischen GeradenAX ′, BY ′ und CZ ′ ebenfalls in einem gemeinsamenPunkt, dem zu P isotomisch konjugierten Punkt P ′,schneiden. A B

C

P

P′

X

X ′

Y

Y ′

ZZ ′

Was bei den isotomischen Geraden die besondere Langenrelation AD = BD′ ist, ist auchfur die von zwei Ecktransversalen eingeschlossenen Winkel mit den Dreieckseiten denkbar.Dies fuhrt auf den Begriff

28 DREIECKE

Isogonale Geraden. (Bild) Die Ecktransversale g durchC schneide die gegenuberliegende Dreieckseite AB imPunkt D. Dann gibt es auf dieser Seite immer einenPunkt D′ (den zu D isogonal gelegenen Punkt), fur den�ACD′ = �BCD bzw. �BCD′ = �ACD gilt. Diedurch C und D′ gehende Gerade g′ heißt dann isogo-nal zu g. Offensichtlich gehen g und g′ ineinander uber,wenn wir sie an der Winkelhalbierenden w spiegeln. Ins-besondere hat sich der Name Symmediane fur die an der A B

C

D D ′

g g ′w

Winkelhalbierenden gespiegelten zugehorigen Seitenhalbierenden (Mediane) ein-gepragt.

D.37 Isogonal konjugierter Punkt. (Bild) P sei ein imInnern des �ABC liegender Punkt. Die drei Eck-transversalen AP , BP , CP schneiden die jeweils ge-genuberliegenden Seiten in den Punkten X, Y bzw.Z. Man beweise, daß sich die drei isogonalen Gera-den AX ′, BY ′ und CZ ′ ebenfalls in einem gemeinsa-men Punkt, dem zu P isogonal konjugierten Punkt P ′,schneiden. A B

C

P

P′

X

X ′

Y

Y ′

ZZ ′

Nunmehr sind wir in der Lage, auf einige weitere merkwurdige Punkte in einem Dreieckeinzugehen, von denen die antiken Geometer anscheinend noch nichts wußten.

D.38 Gergonnes Punkt. In einem �ABC schneiden sich die Strecken AX, BY undCZ stets in einem Punkt Ge, wobei X, Y , Z die Beruhrungspunkte des Inkreisesmit den Seiten BC, CA, AB sind.

D.39 Nagels Punkt. In einem �ABC schneiden sich die Strecken AX, BY und CZ

stets in einem Punkt Na, wobei X, Y , Z die Beruhrungspunkte der drei Ankreisemit den Seiten BC, CA, AB sind.

D.40 Gergonnes Punkt und Nagels Punkt sind isotomisch konjugierte Punkte.

D.41 Lemoine-Punkt. Die Symmedianen eines Dreiecks schneiden sich in einem Punkt.

D.42 Welche beiden merkwurdigen Punkte in einem Dreieck sind ebenfalls isogonal kon-jugiert?

Im zweiten Teil dieses Abschnitts geht es um die Kollinearitat, d. h. die Bedingung furdie gleichzeitige Lage dreier Punkte auf einer Geraden.

Eines ist noch vorauszuschicken: Beim Satz von Menelaus wird vom Begriff der gerich-teten Strecke Gebrauch gemacht. Befinden sich z. B. die Punkte A, B, Z so auf einerGeraden, daß B zwischen A und Z liegt, so haben AB und BZ gleichen Richtungssinnund AB/BZ ist positiv. Dagegen ist in diesem Beispiel wegen ZB = −BZ der Quoti-ent AZ/ZB negativ. Diese Unterscheidung, die auf Newton zuruckgeht, laßt bereits dieAnfangsgrunde der Vektorrechnung erkennen.

Extremalaufgaben 29

D.43 Satz von Menelaus. (Bild) Eine Gerade schneidet die(ggf. verlangerten) Seiten eines Dreiecks so, daß das Pro-dukt der Teilverhaltnisse, das ihre Schnittpunkte mitden drei Seiten bilden, den Wert −1 hat:

AZ

ZB· BXXC

· CYYA

= −1. (D.3)

A B

C

X

Y

Z

D.44 Umkehrung des Satzes von Menelaus. Wenn fur drei Punkte X, Y , Z auf denSeiten eines Dreiecks die Gleichung (D.3) gilt, dann sind X, Y , Z kollinear.

Im weiteren finden sich Problemstellungen, in denen ebenfalls die Kopunktualitat vonGeraden bzw. Kollinearitat von Punkten nachzuweisen ist.

D.45 Simson-Gerade. Es sei P ein Punkt auf dem Umkreis des �ABC und X, Y , Z dieFußpunkte der von P auf die Dreieckseiten bzw. deren Verlangerungen gefalltenLote. Man zeige, daß dann X, Y , Z auf einer gemeinsamen Geraden liegen.

D.46 Die Tangenten an den Umkreis eines Dreiecks in dessen Eckpunkten schneiden diejeweils gegenuberliegenden (verlangerten) Seiten in drei kollinearen Punkten.Singapore Mathematical Society Interschool Competition, 1989

D.47 In einem �ABC werden in den Eckpunkten A,B und C jeweils gleich große Winkelδ, ε bzw. η nach außen abgetragen. Die freien Schenkel treffen sich in den PunktenD, E F , die in dieser Reihenfolge den Eckpunkten A, B, C gegenuberliegen. Manbeweise, daß AD, BE und CF sich in einem Punkt schneiden.

Abschließend sei bemerkt, daß wir in Abschnitt M.4 noch andere Beweise der Satze vonCeva und Menelaus unter Verwendung des Flachenprinzips angeben.

D.3 Extremalaufgaben

Fur Billardspieler ist folgende Frage nichts besonderes: Wie ist eine Kugel zu stoßen, damitsie von Punkt A nach einer Reflexion an der Bande zum Punkt B gelangt? Viele erledigendie Aufgabe vielleicht intuitiv richtig, aber konnen sie ihr Vorgehen auch mathematischerklaren? Klar, wir haben diesen Anspruch, deswegen formulieren wir das folgende

D.51 Problem von Heron. Fur zwei Punkte auf derselben Seite einer Geraden ist derkurzeste Weg gesucht, der vom ersten Punkt zur Geraden und anschließend zumzweiten Punkt fuhrt.

Bemerkenswert sind ebenso Extremaleigenschaften bestimmter Punkte oder Linien imDreieck, die schon seit jeher das Interesse der Mathematiker auf sich zogen. Die erstederartige Extremalaufgabe stammt aus der Mitte des 17. Jahrhunderts von dem franzosi-schen Mathematiker Pierre Fermat, der sie u. a. Evangelista Torricelli bekanntmachte und die daher nach beiden benannt wurde.

30 DREIECKE

D.53 Problem von Fermat. Man bestimme denjenigen Punkt P innerhalb eines spitz-winkligen �ABC, fur den die Summe seiner Abstande zu den Eckpunkten A, Bund C moglichst klein wird.

D.54 Uber den Seiten eines spitzwinkligen �ABC werden nach außen drei gleichseitigeDreiecke ABW , BCU und CAV errichtet. Man zeige:a) AU = BV = CW ;b) �ABC und �UVW haben denselben Fermat-Punkt F .

D.55 Auf eine waagrechte Tischplatte wird ein Dreieck gezeichnet, und in den drei Eck-punkten werden Locher durch die Platte gebohrt. Durch jedes Loch wird ein dunnerFaden gezogen und eine Masse daran befestigt. Uber dem Tisch werden die Fadenverknotet und losgelassen. Wo kommt der Knoten zur Ruhe, wenn alle Gewichts-krafte gleich groß sind und Reibungskrafte vernachlassigt werden?

In der nachsten Extremalaufgabe geht es auch um einen minimalen Weg in einem Dreieck.Sie wurde 1775 durch J. F. de Toschi di Fagnano gestellt.

D.56 Problem von Fagnano. Schreibe einem spitzwinkligen Dreieck ABC ein DreieckUVW von moglichst kleinem Umfang ein.

D.57 Fur welchen Punkt im Innern eines Dreiecks ABC ist die Summe der Quadrateder Abstande zu den Eckpunkten minimal?

D.58 Welcher Punkt im Innern eines Dreiecks ABC minimiert die Summe der Quadrateder Abstande zu den Seiten?

D.59 Gegeben sei eine Gerade g mit einem Punkt P ∈ g sowie einem Punkt Q /∈ g. Furwelchen Punkt R ∈ g wird der Ausdruck (QP + PR)/QR maximal?

Weitere Aufgaben, die bestimmte Extremaleigenschaften in einem Dreieck beinhalten,sind W.22, W.52 und W.82.

D.4 Einige Formeln

Die Elementargeometrie ware kein solches faszinierendes Gebiet der Mathematik, wennsie nicht noch genug Spielraum fur andere Gebiete wie Algebra oder Trigonometrie bote.Deshalb versuchen wir in diesem Abschnitt eine Reihe von Formeln zusammenzutragen,die Winkel, Langen von Strecken, Flacheninhalte u. v. a. m. untereinander auf vielfaltigeWeise verknupfen. Davon ausgenommen sind Ungleichungen in diesen Großen; hierfurhaben wir ein gesondertes Kapitel G.

Beginnen wir mit einigen Gleichungen fur den Flacheninhalt eines Dreiecks:

D.61 Der Flacheninhalt eines Dreiecks ist gleich dem Produkt aus halber Seitenlangeund zugehoriger Hohe:

∆ =aha2

=bhb2

=chc2. (D.4)

Einige Formeln 31

D.62 Der Flacheninhalt eines Dreiecks ist gleich

∆ =1

2bc sinα =

1

2ca sin β =

1

2ab sin γ. (D.5)

D.63 Der Flacheninhalt eines Dreiecks ist gleich dem Produkt aus Inkreisradius r undhalben Umfang s = 1

2(a+ b+ c):

∆ = rs. (D.6)

D.64 Der Flacheninhalt eines Dreiecks ist gleich dem Produkt der Seitenlangen a, b, cdividiert durch das Vierfache des Umkreisradius R:

∆ =abc

4R. (D.7)

D.65 Der Flacheninhalt eines Dreiecks ist gleich

∆ = (s− a)ra = (s− b)rb = (s− c)rc. (D.8)

Der nachste Ausdruck fur den Flacheninhalt eines Dreiecks wird allgemein Heron von

Alexandria zugeschrieben, stammt aber tatsachlich von Archimedes [Cox63, S. 27].Alle diese Gleichungen konnen weiter in vielfaltiger Weise verknupft werden und lieferndabei einige bemerkenswerte Zusammenhange.

D.66 Herons Formel. Der Flacheninhalt eines Dreiecks ist gleich

∆ =√s(s− a)(s− b)(s− c). (D.9)

D.67 Das Produkt der Seitenlangen a, b, c eines Dreiecks ist gleich dem doppelten Pro-dukt aus Inkreisradius r, Umkreisradius R und Umfang 2s:

abc = 2rR(2s) = 4rRs. (D.10)

D.68 Fur die Ankreisradien ra, rb, rc sowie den Inkreisradius r eines Dreiecks gilt:

1

ra+

1

rb+

1

rc=

1

r. (D.11)

Auf Matthew Stewart (”Some general theorems of considerable use in the higher parts

of mathematics“, Edinburgh, 1746) geht ein Satz zuruck, der die Lange einer Ecktransver-salen angibt und eine nette Anwendung des Kosinussatzes ist. Mit seiner Hilfe berechnenwir die mitunter benotigten Langen der klassischen Transversalen aus Abschnitt D.1.

D.69 Satz von Stewart. (Bild) In einem �ABC teile die Eck-transversale CZ die gegenuberliegende Seite c in die Ab-schnitte AZ ≡ m bzw. ZB ≡ n. Man zeige, daß fur dieLange CZ ≡ t dann gilt:

c(t2 +mn) = ma2 + nb2. (D.12)A B

C

Z

abt

m n

32 DREIECKE

D.70 Welche Lange hat a) eine Seitenhalbierende, b) eine Winkelhalbierende und c) eineHohe in einem beliebigen �ABC, ausgedruckt durch die Langen der Seiten a, bund c?

Das letzte Resultat erlaubt einen kurzen, direkten Beweis des folgenden bekannten Satzes:

D.71 Satz von Steiner-Lehmus. Jedes Dreieck mit zwei gleich langen Winkelhalbieren-den ist gleichschenklig.

D.72 Welche Lange (ausgedruckt durch a, b, c) haben die Abschnitte AF und BF aufder Seite AB eines beliebigen Dreiecks, wenn CF die Hohe auf dieser Seite ist?

D.73 In jedem Dreieck mit den Winkeln α, β, γ gilt:

cotα+ cotβ + cot γ =a2 + b2 + c2

4∆. (D.13)

Leonhard Euler, einer der großten Mathematiker aller Zeiten, berechnete 1765 denAbstand zwischen Umkreismittelpunkt O und Inkreismittelpunkt I in einem Dreieck.Daraus folgt unmittelbar eine bekannte Ungleichung zwischen dem Umkreisradius R unddem Inkreisradius r, die in Abschnitt G.1.3 zu finden ist.

D.79 Eulers Abstand OI. Fur den Abstand d ≡ OI zwischen den Mittelpunkten vonUm- und Inkreis eines Dreiecks gilt:

OI2 = d2 = R2 − 2rR. (D.14)

D.5 Lotfußpunktdreiecke

Greifen wir uns einen beliebigen Punkt im Innern eines Dreiecks heraus und fallen vondiesem die Lote auf die Seiten des Dreiecks, so erhalten wir die Lotfußpunkte von P .Verbinden wir diese Lotfußpunkte noch untereinander, entsteht das zu P gehorige Lot-fußpunktdreieck. Derartige Dreiecke haben besondere Eigenschaften, weshalb wir ihneneinen eigenen Abschnitt widmen.

D.81 P sei ein Punkt im Innern eines �ABC. Das Lotfußpunktdreieck von P hat danndie Seitenlangen

ax

2R,

by

2R,

cz

2R,

wobei x, y, z die Abstande von P zu den Eckpunkten A, B, C und R der Um-kreisradius von �ABC ist.

D.82 Gegeben sei ein spitzwinkliges Dreieck ABC. Wie ist ein Punkt P im Innern zuwahlen, damit dessen Lotfußpunktdreieck im �ABC gleichschenklig und recht-winklig wird?

Noch mehr uber Dreiecke 33

Die folgende Aufgabe wurde 1935 von Paul Erdos im Amer. Math. Monthly gestellt,zwei Jahre spater erschien in der gleichen Zeitschrift die Losung [2,3]. Unsere Losungstammt von Leon Bankoff [4].

D.83 Satz von Erdos-Mordell. Ist P ein beliebiger Punkt im Dreieck ABC und sindD, E, F die Fußpunkte der Lote von P auf die Seiten BC, CA und AB, so gilt:

PA+ PB + PC ≥ 2(PD + PE + PF ). (D.15)

D.84 Wann gilt in der Ungleichung von Erdos-Mordell (D.15) das Gleichheitszei-chen?

D.6 Noch mehr uber Dreiecke

Das Dreieck scheint schier unerschopflich in seinen Eigenschaften zu sein. Im folgendendeshalb wiederum nur eine kleine Auswahl von Aufgaben.

D.91 (Bild) Fur welchen Punkt P im Innern eines DreiecksABC gilt die Gleichung

PD

BC=PE

CA=PF

AB,

wobei D, E, F die Fußpunkte der Lote von P auf dieSeiten BC, CA, AB sind? A B

C

DE

F

P

34 DREIECKE

V

VIERECKE

Vierecke nennt man bekanntlich alle Polygone mit n = 4 Ecken und Seiten. Im Gegensatzzu Dreiecken treten hierbei Diagonalen auf, die gegenuberliegende Eckpunkte verbinden.Dementsprechend spricht man von benachbarten oder gegenuberliegenden Seiten des Vier-ecks, je nachdem, ob sie einen Eckpunkt gemeinsam haben oder nicht. Ebenso sind zweiEckpunkte benachbart oder gegenuberliegend, wenn sie zu einer Seite gehoren oder nicht.

A

B

C

D

A

B

C

D

A

B

C

D

Bild V.1 Arten von Vierecken

In Bild V.1 hat das Viereck ABCD die Seiten AB, BC, CD und DA sowie die DiagonalenAC und BD. Offenbar gibt es mehrere Arten von Vierecken: Im Bild links ist ein konvexesViereck zu sehen, dessen Diagonalen beide im Innern des Vierecks liegen, in der Mitte einkonkaves oder re-entrantes Viereck, bei dem eine Diagonale innen und eine außen liegtund rechts ein uberschlagenes Viereck, welches beide Diagonalen außerhalb hat.

Den Flacheninhalt eines konvexen Vierecks konnen wir als Summe der Flacheninhalte derbeiden Dreiecke berechnen, in welche das Viereck durch eine seiner Diagonalen zerfallt:

[ABCD] = [ABC] + [CDA] = [BCD] + [DAB].

Damit diese Formel auch fur konkave Vierecke gultig bleibt, betrachten wir den Flachen-inhalt als positiv oder negativ, je nachdem, ob die Eckpunkte entgegen des Uhrzeigersinnsoder im Uhrzeigersinn benannt wurden. Deshalb ist

[ABC] = [BCA] = [CAB] = −[CBA].

Das mittlere Viereck in Bild V.1 hat daher den Flacheninhalt

[ABCD] = [BCD] + [DAB] = [CDA] − [CBA] = [CDA] + [ABC].

36 VIERECKE

Schließlich erlaubt uns die Formel, den Flacheninhalt eines uberschlagenen Vierecks alsdie Differenz der Flacheninhalte der beiden kleinen Dreiecke anzusehen, aus denen eszusammengesetzt ist.

V.1 Allgemeine Vierecke

Kommen wir zum ersten Satz, der auf Pierre Varignon 1731 zuruckgeht.

V.1 Varignon-Parallelogramm. Verbindet man die Mittelpunkte der Seiten eines be-liebigen ebenen Vierecks, so entsteht ein Parallelogramm, dessen Flacheninhalthalb so groß wie der des Vierecks ist.

Daraus ergibt sich ein weiterer Satz, den wir hier allerdings nicht anfuhren, da er genaumit der Olympiadeaufgabe W.1 identisch ist.

V.2 Wenn eine Diagonale ein Viereck in zwei flachengleiche Halften teilt, halbiert sieauch die andere Diagonale. Und umgekehrt: Wenn eine Diagonale die andere hal-biert, halbiert sie auch die Flache des Vierecks.

V.3 In einem Viereck ABCD treffen sich die (verlangerten) gegenuberliegenden SeitenAB und CD im Punkt W . Die Mittelpunkte der Diagonalen AC und BD seien Xbzw. Y . Dann gilt

[XWY ] =1

4[ABCD].

(Kanada, 1978 )

V.8 Wie groß ist der Winkel, der von zwei gegenuberliegenden Seiten eines Viereckseingeschlossen wird?

V.9 AB sei eine der kurzeren Seiten des Rechtecks ABCD. Fallt man von C aus dasLot auf die Diagonale BD, so schneidet dieses Lot g(A,B) in E. Der Schnittpunktder Seite AD mit dem Kreis um B vom Radius BC sei F . Man beweise, daß EFauf FB senkrecht steht.

V.10 In einem Viereck ABCD sind die Winkel �DAB = 60◦ und �DCB = 150◦

gegeben. Weiterhin sei AB = AD. Man zeige, daß dann auch AC = AB gilt.

V.2 Trapeze, Parallelogramme, Rhomben etc.

Auch die uns allseits vertrauten speziellen Vierecke, wie Trapez, Parallelogramm, Dra-chenviereck, Rhombus, Rechteck, Quadrat konnen mit einer Menge nicht so bekannterEigenschaften aufwarten. Es lohnt sich also, auch ihnen einen eigenen Abschnitt zu wid-men.

V.11 Im Trapez gilt mit den Grundseiten a, c, Schenkeln b, d und Diagonalen e, f

e2 + f 2 = b2 + d2 + 2 ac.

Sehnenvierecke 37

V.19 In einem Parallelogramm ABCD bezeichnen E, F Punkte auf BC bzw. CD. DieParallele durch D zu FB schneide AB im Punkt G. Der Schnittpunkt von DEmit BF sei H. Man zeige, daß AH und GE parallel sind.(Elem. Math. 31 (1976), Aufgabe 738 )

Man beachte auch die Aufgabe M.12.

V.3 Sehnenvierecke

Da ein Kreis durch drei Punkte auf seiner Peripherie vollstandig bestimmt ist, kann manim allgemeinen keinen Kreis zeichnen, der durch die Ecken eines beliebigen Vierecks geht.Wohl aber laßt sich in einen Kreis ein Viereck einzeichnen, dessen Seiten Sehnen desKreises sind. Um ein solches Sehnenviereck muß sich dann auch ein Kreis beschreibenlassen, den wir analog zum Dreieck Umkreis nennen.

Sehnenvierecke haben offenbar besondere Eigenschaften, die sie von anderen Viereckenunterscheiden; wir wollen einige naher beleuchten.

V.21 In einem Sehnenviereck ist die Summe der Großen gegenuberliegender Innenwinkelstets 180◦.

V.22 Die Mittelsenkrechten gegenuberliegender Seiten eines Sehnenvierecks schneidensich im Mittelpunkt des umschriebenen Kreises.

V.23 In einem Sehnenviereck ABCD zerlegen die Diagonalen AC und BD das Viereckin vier Dreiecke, die paarweise keinen Punkt gemeinsam haben. Man zeige, daßdie jeweils gegenuberliegenden Dreiecke zueinander ahnlich sind.

V.24 O sei der Mittelpunkt des Umkreises in einem Sehnenviereck ABCD. Stehen inihm die Diagonalen AC und BD senkrecht aufeinander, sind die Winkel �AOBund �COD supplementar.

V.25 Stehen in einem Sehnenviereck die Diagonalen senkrecht aufeinander, so halbiertjede Gerade, die senkrecht auf einer Seite steht und durch den Diagonalenschnitt-punkt geht, die jeweils gegenuberliegende Seite.(35. Mathematik-Olympiade 1995/96, Klasse 9, Stufe 3 )

Zu Sehnenvierecken mit senkrecht aufeinander stehenden Diagonalen beachte man auchdie Aufgabe W.16.

Auf den AstronomenClaudius Ptolemaus (ca. 85 – ca. 165) geht der folgende nutzlicheSatz zuruck, wobei anzumerken ist, daß bis ins 17. Jahrhundert der Satz des Menela-

os irrtumlich als Satz des Ptolemaus bezeichnet wurde. Er hat daruber hinaus eineUmkehrung, die als Ungleichung formuliert wird, s. Aufgabe G.81.

V.26 Satz des Ptolemaus. (Bild) In einem konvexen Sehnen-viereck ist die Summe der Produkte gegenuberliegenderSeitenlangen gleich dem Produkt der Diagonalen:

AB · CD +BC · AD = AC ·BD. (V.1) A B

CD

38 VIERECKE

V.27 ABCD sei ein Sehnenviereck. Die Gerade durch die Inkreismittelpunkte der Drei-ecke ABC und ABD verlauft dann stets parallel zu der Winkelhalbierenden zwi-schen den Diagonalen AC und BD.

V.4 Tangentenvierecke

Im allgemeinen ist es nicht moglich, in ein beliebiges Viereck einen Kreis zu zeichnen,der die vier Seiten desselben beruhrt. Wohl aber laßt sich um einen Kreis ein Viereckzeichnen, dessen Seiten Tangenten des Kreises sind. Ein diesem Inkreise umschriebenesViereck nennt man daher Tangentenviereck.

Auch Tangentenvierecke haben besondere, erwahnenswerte Eigenschaften, die es im fol-genden festzuhalten gilt.

V.31 Im Tangentenviereck ist die Summe der Langen zweier gegenuberliegender Seitengleich der Summe der Langen der anderen beiden Gegenseiten.

V.32 Die Winkel, unter denen gegenuberliegende Seiten eines Tangentenvierecks vomInkreismittelpunkt aus gesehen werden, sind supplementar.

V.33 In einem Tangentenviereck teilt jede Beruhrungssehne, die gegenuberliegende Be-ruhrungspunkte verbindet, dieses in zwei Vierecke mit jeweils paarweise gleichenWinkeln.

V.34 Die Diagonalen eines Tangentenvierecks schneiden sich im gleichen Punkt wie diebeiden Sehnen gegenuberliegender Beruhrungspunkte des Vierecks mit dem In-kreis.

Weiteres zu Tangentenvierecken ist in Aufgabe M.43 zu finden.

V.5 Sehnentangentenvierecke

Nachdem wir uns in den beiden vorangegangenen Abschnitten mit Sehnen- und Tangen-tenvierecken beschaftigt haben, gehen wir die Leiter der Spezialisierung noch eine Sprossehoch und befassen uns mit Vierecken, die Eigenschaften von beiden in sich vereinen. Un-ter einem Sehnentangentenviereck oder bizentrischen Viereck versteht man ein Viereck,das gleichzeitig einem Kreis einbeschrieben und einem anderen Kreis umbeschrieben ist.Beginnen wir mit der bemerkenswertesten Eigenschaft derartiger Vierecke:

V.41 In einem Sehnentangentenviereck stehen die Beruhrungssehnen zwischen gegenuber-liegenden Beruhrungspunkten senkrecht aufeinander.

M

METHODEN

Allgemeingultige Methoden zur Behandlung beliebiger geometrischer Fragestellungen kannes sicherlich nicht geben. Es laßt sich jedoch eine kleine Anzahl von Rezepten aus derumfangreichen Literatur herausfiltern, die fur bestimmte Problemstellungen zu besonderskurzen und eleganten (oder aber auch zu langeren, aber von vornherein erfolgversprechen-den) Losungen fuhrt. Oder anders ausgedruckt: Manchen Aufgaben sieht man regelrechtan, mit welchem Ansatz und Hilfsmitteln sie zu

”knacken“ sind. Das dazu trotzdem noch

eine gehorige Portion Intuition notwendig ist, muß sicher nicht naher begrundet werden.

Dieses Kapitel stellt daher den bescheidenen und unvollstandigen Versuch dar, einigehaufig wiederkehrende Losungsideen und -methoden vorzustellen. Das Konzept des geo-metrischen Ortes, welches wir schon aus dem Abschnitt A.2 kennen, gehort auch dazu.

M.1 Vektorrechnung

Eine bestimmte Klasse von Aufgaben laßt sich besonders elegant mit den Hilfsmitteln derVektorrechnung losen. Ein Vektor kann als Translation eines Punktes A zum Punkt B be-trachtet werden; wir bringen diese Verschiebung mit der Schreibweise

−→AB zum Ausdruck.

Ein Punkt der Ebene oder des Raumes spielt eine Sonderrolle: O (fur origin); er dientals Ausgangs- oder Bezugspunkt. Eine Verschiebung von O nach A wurden wir also mit−→OA oder kurz

−→A bezeichnen, verwenden aber ublicherweise a. Da mehrere Translationen

auch nacheinander ausgefuhrt werden konnen, ist z. B.−→OA+

−→AB =

−−→OB bzw.

−→AB = b−a.

Von dieser Beziehung werden wir haufig Gebrauch machen.

Etwas verwirrend ist mitunter die benutzte Schreibweise. Z. B. ist mit BC2 das Skalar-produkt

−−→BC · −−→BC = |BC|2 gemeint und nicht etwa die Ausdrucke

−→B

−→C

2= BC2 (welcher

ja ein Vektor ware) oder (−→B

−→C )2 = (BC)2 (welcher zwar ein Skalar ist, der aber nur

im Falle B ‖ C gleich BC2 ware). Wir mussen also genau zwischen (normal gesetzten)Skalaren und (fett gesetzten oder mit dem Vektorpfeil kenntlich gemachten) Vektorenunterscheiden (außer der bekannten Ausnahme a · a = a2 = a2).

Der Vorteil der Einfuhrung und konsequenten Verwendung von Vektoren zum Losen vonAufgaben wird augenscheinlich, wenn wir lediglich die formalen Rechenregeln ausnutzenund uns nicht auf eine Darstellung der Vektoren in Komponenten (etwa in einem kartesi-schen Koordinatensystem) einlassen. Dadurch unterscheidet sich diese Herangehensweise

40 METHODEN

von der analytischen Geometrie. Dazu ist naturlich die Kenntnis dieser Rechenregelnnotig (etwa daß die Vektoraddition assoziativ oder das Skalarprodukt vertauschbar ist);aus Platzgrunden ist es aber hier nicht moglich alles anzugeben. Die folgenden Aufgabenmogen als Demonstration dieser Methode dienen.

M.1 Von einem Dreieck sind die Ortsvektoren der Eckpunkte gegeben. Man finde einenAusdruck fur die Lage des Schwerpunkts.

M.2 (Bild) In einem beliebigen, unregelmaßigen Sechseckwerden die durch jeweils drei aufeinanderfolgendeEckpunkte gebildeten Dreiecke gezeichnet und derenSchwerpunkte untereinander verbunden. Man zeige,daß in dem neu entstandenen Sechseck die drei Paa-re jeweils gegenuberliegender Seiten gleich lang undparallel zueinander sind.

M.3 In einem Viereck stehen die Diagonalen genau dann senkrecht aufeinander, wenna) die Summe der Quadrate gegenuberliegender Seiten gleich ist, oderb) die Mittellinien (d. h. die Verbindungsstrecken der Mittelpunkte gegenuberlie-gender Seiten) gleich lang sind.

M.4 Fur vier beliebige Punkte A, B, C, D im Raum gilt:

BC2 + AD2 − AB2 − CD2 = 2−→AC · −−→BD. (M.1)

M.8 Die Mittelpunkte der vier Quadrate, die uber den Seiten eines Parallelogrammsnach außen errichtet werden, bilden die Eckpunkte eines Quadrats.(10. Mathematik-Olympiade 1970/71, Klasse 10, Stufe 1 )

M.9 Varignon-Parallelogramm. Gegeben seien vier beliebige Punkte A, B, C und Dim Raum. Man zeige, daß die Mittelpunkte der Strecken AB, BC, CD und DA

ein ebenes Parallelogramm bilden.

M.10 Wigner-Seitz-Zelle. (Bild) In einem schiefwink-ligen, ebenen Gitter bilden jeweils vier Punkteein Parallelogramm — das kleinste von ihnenist die Elementarzelle des Gitters. Greift mansich einen beliebigen Gitterpunkt heraus undzeichnet die Mittelsenkrechten der Verbindungs-strecken zu den sechs nachsten Nachbarn, so beranden diese Linien ein Gebiet, dasman Wigner-Seitz-Zelle (WSZ) nennt. Man zeige, daß die Elementarzelle unddie WSZ den gleichen Flacheninhalt haben.

M.2”Winkel jagen“

Unter dieser Uberschrift konnen wir alle Bemuhungen zusammenfassen, bestimmte Rela-tionen zwischen Winkeln nachweisen zu wollen. Dazu bietet die Elementargeometrie auch

Verwandlung von Figuren 41

eine Reihe von speziellen Beziehungen zwischen Winkeln: Neben-, Stufen- und Wechsel-winkel, Satze uber die (Innen- und Außen-)Winkelsumme in Polygonen, den Peripherie-winkelsatz, den Peripherie-Zentriwinkel-Satz, den Sehnen-Tangentenwinkel-Satz usw. Eingeschicktes Ausnutzen dieser Hilfsmittel fuhrt gelegentlich zu langwierigen, aber sichererreichbaren Losungen. Was hiermit gemeint ist, laßt sich am besten an der nachstenAufgabe verdeutlichen:

M.11 (Bild) Ein Gliedermaßstab, bestehend aus 7 gleichlangen Staben, dessen Enden drehbar miteinanderverbunden sind, wird so geformt, daß die Verbin-dungspunkte A, D, E, B bzw. A, F , G, C jeweilsauf einer Geraden liegen. Wie groß ist der Winkel

A

B

C

DE

FG

α

α bei Punkt A?

M.12 In einem Trapez ABCD sei AB = BC +CD. Welche Beziehung besteht zwischenden Innenwinkeln bei A und B?

M.13 In einem rechtwinkligen �ABC wird das Lot von einem Punkt D der Kathete ACauf die Hypotenuse AB gefallt. Der Fußpunkt sei E. Die Transversalen BD undCE schneiden sich in S derart, daß gerade BC = BS gilt. Man zeige, daß BS denWinkel �ABC drittelt.

M.14 Zwei Kreise beruhren sich von innen in einem Punkt T . Eine Sehne AB des außerenKreises beruhre den inneren im Punkt P . Man beweise, daß TP den Winkel �ATBhalbiert.

M.15 Das Dreieck ABC sei einem Kreis einbeschrieben. Zwei Sehnen — ausgehend vomPunkt A — schneiden die Seite BC in den Punkten K und L sowie den Bogen BCin M und N . Man beweise: Wenn KLNM ein Sehnenviereck ist, ist das DreieckABC gleichschenklig.(22. Tournament of Towns, Autumn 2000, O-level)

M.3 Verwandlung von Figuren

Eine gegebene Figur in eine andere verwandeln heißt, eine zweite Figur zeichnen, die mitder gegebenen Figur gleichen Inhalt hat. Der Zweck einer solchen Verwandlung bestehtdarin, z. B. aus Polygonen mit n > 3 Seiten unter Beibehaltung des Flacheninhalts solchemit weniger Seiten zu machen, um ggf. schließlich zu einem einfachen Dreieck zu gelangen.Auch der umgekehrte Fall, aus einem beliebigen Dreieck z. B. ein Quadrat zu konstruieren,ist moglich. Es ist klar, daß wir Verwandlungen bei einer Aufgabe nur in Betracht ziehenbrauchen, wenn der Flacheninhalt uberhaupt eine Rolle spielt.

Das grundlegende Prinzip dabei ist die Ausnutzung der bekannten Tatsache, daß sichdie Flache eines Dreiecks nicht andert, wenn wir einen Eckpunkt auf derjenigen Geradenverschieben, die parallel zu der Geraden durch die beiden anderen verlauft. Oder in einerGleichung ausgedruckt: 2∆ = chc mit hc als Abstand beider Parallelen.

42 METHODEN

Die Methode besteht also nur in einer geschickten und zweckmaßigen Verschiebung vonEckpunkten auf Verlangerungen benachbarter Seiten eines Polygons. Wie das im einzelnenablauft, demonstrieren wir an folgenden Aufgaben.

M.21 Man verwandle ein gegebenes regelmaßiges Funfeck in ein flachengleiches Dreieck.

M.22 (Bild) Im Innern eines Funfecks ABCDE sollen alle diejeni-gen Punkte P gefunden werden, fur die das Viereck ABCPeinen ebenso großen Flacheninhalt wie das Funfeck CDEAPhat. a) Man beschreibe eine von den gegebenen Eckpunk-ten des Funfecks ausgehende Konstruktion, mit der sich eineMenge von Punkten P ergibt. b) Beweisen Sie, daß jederPunkt P , der sich aus dieser Konstruktion ergibt, die Bedin-gung der Flachengleichheit erfullt. A B

C

D

E

(36. Mathematik-Olympiade 1996/97, Klasse 10, Stufe 3 )

Eine weitere Aufgabe hierzu ist W.31.

M.4 Das Flachenprinzip

In diesem Abschnitt wird eine Methode vorgestellt, die den Flacheninhalt als das grundle-gende Werkzeug zum Losen von Geometrie-Aufgaben verwendet. Es wird sich bald zeigen,daß dieses sog.

”area principle“ (oder auch

”K method“, [Zha92], [Wei99]) ein ebenso ein-

faches wie universelles Verfahren ist. Die abschließenden Beispiele verdeutlichen, daß essich auf alle Falle lohnt, das Flachenprinzip mit in die

”Trickkiste“ aufzunehmen.

Das Ruckgrat der Methode bilden folgende drei Satze, die jeweils Aussagen uber dasVerhaltnis von Flacheninhalten zweier Dreiecke machen. Wir verwenden fur die Großedes Flacheninhalts einer Figur F wie bisher die Abkurzung [F ].

M.31 Satz der gemeinsamen Hohen. (Bild) Wenn vier Punk-te A, B, C und D auf einer Geraden liegen, die nichtdurch P geht, dann gilt:

[PAB]

[PCD]=AB

CD. (M.2)

A B CD

P

M.32 Satz der gemeinsamen Seite. (Bild) Die Dreiecke PAB undQAB mogen die Seite AB gemeinsam haben. Wenn PQ undAB sich in M schneiden, dann gilt:

[PAB]

[QAB]=PM

QM. (M.3)

A

B

M

P

Q

Das Flachenprinzip 43

M.33 Satz des gemeinsamen Winkels. Wenn die Winkel �ABC und �A′B′C ′ gleichsind (Bild a) oder sich zu 180◦ erganzen (Bild b), dann gilt:

[ABC]

[A′B′C ′]=

AB ·BCA′B′ ·B′C ′ . (M.4)

a)

A

A′

B = B ′

C

C ′b)

A

A′

B = B ′

C

C ′

Diese grundlegenden Satze lassen bereits erkennen, worauf es hier stets ankommt: DasVerhaltnis von Strecken wird auf das Verhaltnis von Flacheninhalten ausgedehnt. Die-ses ist von großem Vorteil, sind doch Flacheninhalte fur das

”geometrisch sehende Auge“

leichter zu erfassen und zu kombinieren als Strecken. Gerade bei komplizierteren Proble-men, in denen eine Figur aus mehreren Teilfiguren zusammengesetzt ist, lassen sich dieeinzelnen Teile anschaulich leicht als Flachenstucke uberblicken.

Bevor die Eleganz dieser Methode an einigen Beispielen demonstriert werden soll, seian folgende algebraische Identitat erinnert, die uns schon beim Beweis des Satzes dergemeinsamen Seite begegnet ist, und auf die im folgenden haufig zuruckgegriffen wird:

Genau dann, wenna

b=c

derfullt ist, gilt auch

a

b=a+ c

b+ d.

Dies zu zeigen, durfte dem Leser keine Muhe bereiten.

Wir beginnen mit den Satzen von Ceva und Menelaus, die bereits im Abschnitt Dbehandelt wurden. Die sich daran anschließenden Satze, wie der von Euler-Gergonne,stellen weitere nutzliche Beziehungen zwischen den Streckenabschnitten auf.

M.34 Satz von Ceva. (Bild) Wenn sich in einem �ABC diedrei Ecktransversalen AX, BY und CZ in einem Punktschneiden, dann hat das Produkt der Teilverhaltnisse,das ihre Schnittpunkte mit den Gegenseiten auf diesenbilden, den Wert 1:

AZ

ZB· BXXC

· CYYA

= 1.

A B

C

XY

Z

M.35 Satz von Menelaus. (Bild) Eine Transversale schneidetdie Seiten eines Dreiecks so, daß das Produkt der Teil-verhaltnisse, das ihre Schnittpunkte mit den drei Seitenbilden, den Wert −1 hat:

AZ

ZB· BXXC

· CYYA

= −1.

A B

C

X

Y

Z

44 METHODEN

M.36 Satz von Euler-Gergonne. (Bild) Fur die Teilungs-verhaltnisse u ≡ AK/KX, v ≡ BK/KY , w ≡ CK/KZdreier sich in K schneidender Ecktransversalen eines�ABC gilt:

1

1 + u+

1

1 + v+

1

1 + w= 1. (M.5)

A B

C

K

X

Y

Z

M.37 Mit den weiteren Abkurzungen x ≡ BX/XC, y ≡ CY/YA und z ≡ AZ/ZB sowiedenen aus Aufgabe M.36 gilt:

u =1

y+ z, v =

1

z+ x, w =

1

x+ y.

Beim genauen Betrachten dieser Gleichungen kommt die Vermutung auf, daß in der vor-liegenden

”Ceva-Konfiguration“ jedes der sechs Verhaltnisse x, y, z, u, v, w bereits durch

zwei andere bestimmt ist. Dies ist auch tatsachlich der Fall [1]. Wir geben diese Bezie-hungen im folgenden ohne Beweis tabellarisch an.

Tabelle M.1: Beziehungen zwischen den Streckenverhaltnissen x, y, z, u, v, w in derCeva-Konfiguration

u, v v, w w, u

x = uv − 11 + u

1 + v1 + w

1 + uwu− 1

y = 1 + vuv − 1

vw − 11 + v

1 + w1 + u

z = 1 + u1 + v

1 + wvw − 1

wu− 11 + w

x, y y, z z, x

u = 1y

(1 + 1

x

)1y

+ z z(1 + x)

v = x(1 + y) 1z

(1 + 1

y

)1z

+ x

w = 1x

+ y y(1 + z) 1x

(1 + 1

z

)

Neben dem (hier Ceva-Konfiguration genannten) Fall, daß sich drei Ecktransversaleneines Dreiecks in einem Punkt treffen, ist auch noch der Fall moglich, daß sie letzteres nichttun und somit im Innern ein kleines Dreieck umschließen. Der folgende Satz behandeltdiesen:

M.38 Satz von Routh. (Bild) Wenn drei Ecktransversalen ei-nes Dreiecks ABC sich nicht in einem Punkt, sondernin drei verschiedenen Punkten schneiden, schließen sieein Dreieck RST im Innern von �ABC ein. Bezeich-net man die Abschnittsverhaltnisse, die die Ecktransver-salen auf ihren Gegenseiten bilden, mit x ≡ BX/XC,y ≡ CY/YA bzw. z ≡ AZ/ZB, so gilt fur das Verhaltnisder Flacheninhalte: A B

C

RS

T

X

Y

Z

[RST ]

[ABC]=

(1 − xyz)2

(xy + x+ 1)(yz + y + 1)(zx+ z + 1). (M.6)

Das Flachenprinzip 45

Abschließend werden noch einige Beispiele angefuhrt, die sich besonders elegant mit demFlachenprinzip behandeln lassen.

M.39 In einem Trapez ABCD schneiden sich die Diagonalen AC und BD im Punkt E.Die Flache des Dreiecks ABE sei 72, die des Dreiecks CDE sei 50. Welche Flachehat das Trapez?(IMTS 7 )

M.40 In einem Dreieck ABC schneiden sich die Ecktransversalen AD und BE in P .Man beweise, daß dann folgende Gleichung gilt:

[ABC] · [DPE] = [APB] · [CDE].

(Crux Mathematicorum 2367, Oktober 1998)

M.41 P sei ein beliebiger Punkt im Innern eines Dreiecks ABC. Die Verlangerungen vonAP , BP und CP schneiden die jeweils gegenuberliegenden Dreieckseiten in denPunkten X, Y bzw. Z. Man beweise folgende Ungleichung:

PA

PX· PBPY

+PB

PY· PCPZ

+PC

PZ· PAPX

≥ 12.

M.42 Im spitzwinkligen Dreieck ABC seien D, E, F die Fußpunkte der Hohen auf denSeiten BC, CA und AB sowie H der Hohenschnittpunkt. Man beweise

AH

AD+BH

BE+CH

CF= 2.

(Australien, 1993 )

M.43 Ist ABCD ein Tangentenviereck mit dem Inkreismittelpunkt I, so gilt

AB

CD=AI ·BICI ·DI .

(14. Mathematik-Olympiade 1974/75, Klasse 10, Stufe 4 )

46 METHODEN

W

WETTBEWERBSAUFGABEN

Um dem Charakter eines Olympiade-Trainings gerecht zu werden, sind in diesem Kapiteleine Vielzahl von Aufgaben aus den unterschiedlichsten Wettbewerben zusammengestellt.Mit diesem Ubungsmaterial soll in erster Linie der Ehrgeiz des Lesenden herausgefordertwerden, diese Aufgaben wirklich selbstandig zu losen. Es wird angeraten, insbesondere beiden schwierigen Problemen nicht gleich in der Losung nachzusehen und uber die Aufgabeerst noch einige Zeit nachzudenken, wenn nicht gleich die Erleuchtung kommt.

Die Anfertigung einer Skizze, die alle wesentlichen Sachverhalte der Aufgabenstellungenthalt, ist ein unbedingtes Muß und zumeist ein guter Start. Sie muß die Voraussetzungender Aufgabenstellung nicht exakt widergeben; es genugt, wenn z. B. gleiche Strecken oderWinkel als solche besonders gekennzeichnet werden.

Es bietet sich an dieser Stelle an, auch einmal auf haufige Fehler hinzuweisen, die bei derBearbeitung von Olympiade-Aufgaben gemacht werden und oft wertvolle Punkte kosten.Zunachst sollte man sich die Aufgabenstellung mehrmals genau durchlesen. So ist manz. B. schon im Hintertreffen, wenn man durch unachtsames Lesen von falschen Voraus-setzungen ausgeht oder bei Beweisen Formulierungen wie

”genau dann, wenn“ uberliest

und damit den Beweis nur in einer Richtung fuhrt. Ebenso sollten verlangte allgemeineBeweise nicht durch Angabe von (einzelnen) Beispielen erfolgen.

Haufig wird auch ubersehen, daß mitunter nicht nur eine Losung existiert. Hier muß dannunbedingt eine Fallunterscheidung erfolgen und jeder Fall gesondert betrachtet werden.

Grundsatzlich erfolglos bleiben Versuche, Relationen von geometrischen Großen (Glei-chungen, Ungleichungen von Langen oder Winkeln) anhand von Zeichnungen, die mitdem Lineal oder Winkelmesser ausgemessen wurden, nachweisen zu wollen.

Und: Eine knapp und prazise formulierte Losung, in der zu Beginn alle vorkommendenSymbole erklart sind, die alle verwendeten Hilfssatze benennt und daruber hinaus noch inlogisch und grammatikalisch einwandfreien Satzen vorliegt, kommt beim Korrektor immergut an!

48 WETTBEWERBSAUFGABEN

W.1 Deutsche Mathematik-Olympiade

Dieser Abschnitt stellt eine unvollstandige Auswahl der Geometrie-Olympiadeaufgabender letzten Jahre dar. Die Aufgaben sind dabei nach aufsteigender Klasse/Stufe geordnet.Zum Zweck einer besseren Lesbarkeit wurde der Text der Aufgaben und Losungen leichtmodifiziert und stimmt somit vielfach nicht mit der offiziellen Version uberein, derenLosungen uberdies in der Vergangenheit zahlreiche Fehler enthielten.

W.1 Man beweise, daß fur jedes konvexe Viereck ABCD die folgende Aussage gilt: SindM1, M2, M3, M4 die Mittelpunkte der Seiten AB, BC, CD, DA und M5, M6 dieMittelpunkte der Diagonalen AC, BD, so gehen die drei Strecken M1M3, M2M4

und M5M6 durch einen gemeinsamen Punkt.(33. Mathematik-Olympiade 1993/94, Klasse 9, Stufe 3 )

W.2 Beweisen Sie: In jedem konvexen Funfeck gelten die Ungleichungen

u < s < 2u.

Dabei bezeichne u den Umfang des Funfecks und s die Summe der Diagonalen-langen.(39. Mathematik-Olympiade 1999/2000, Klasse 9, Stufe 3 )

W.3 Gegeben sei ein gleichschenklig-rechtwinkliges Dreieck ABC mit dem rechten Win-kel bei C und der Kathetenlange 2. Uber den Seiten des Dreiecks seien nach außendie Quadrate ABED, BCGF und CAJH gezeichnet. Beweisen Sie, daß es eineKreislinie gibt, auf der die Punkte D, E, F , G, H und J liegen und berechnen Sieden Radius dieses Kreises.(39. Mathematik-Olympiade 1999/2000, Klasse 9, Stufe 3 )

W.4 Beweisen Sie, daß fur jedes spitzwinklige �ABC die folgende Aussage gilt:Sind D, E und F die Fußpunkte der auf BC, CA bzw. AB senkrechten Hohen, soist

AF 2 +BD2 + CE2 = FB2 +DC2 + EA2.

Untersuchen Sie ferner, ob die Gleichung auch fur jedes rechtwinklige �ABC gilt.(35. Mathematik-Olympiade 1995/96, Klasse 9, Stufe 4 )

W.5 In einem beliebig gegebenen Kreisausschnitt AMB, dessen Zentriwinkel �AMB

eine Große kleiner als 90◦ hat, werden von einem beliebigen Punkt P des BogensAB die Lote auf die Radien MA und MB gefallt; die Fußpunkte seien mit C bzw.D bezeichnet. Beweisen Sie, daß die Lange der Strecke CD unabhangig von derLage des Punktes P auf dem Bogen AB ist.(36. Mathematik-Olympiade 1996/97, Klasse 9, Stufe 4 )

W.6 Beweisen Sie, daß in jedem spitzwinkligen Dreieck ABC der HohenschnittpunktH von allen drei Seiten des Dreiecks DEF gleichgroße Abstande hat, wobei D, Eund F die Fußpunkte der Hohen sind.(35. Mathematik-Olympiade 1995/96, Klasse 10, Stufe 3 )

Deutsche Mathematik-Olympiade 49

W.7 a) In einem beliebigen Dreieck seien ha, hb und hc die Hohen und r der Inkreisra-dius. Man beweise, daß stets ha + hb + hc ≥ 9r gilt.b) Gibt es ein Dreieck, fur welches ha + hb + hc = 9r gilt?(38. Mathematik-Olympiade 1998/99, Klasse 10, Stufe 3 )

W.8 Auf einem Halbkreis uber einer gegebenen Strecke AB als Durchmesser seien zweiPunkte C und D gelegen. Ferner sei P ein beliebiger Punkt der Strecke CD, undQ sei der Fußpunkt des von P auf AB gefallten Lotes. Man beweise, daß unterdiesen Voraussetzungen stets die Gleichung

AQ ·QB − CP · PD = PQ2

gilt.(38. Mathematik-Olympiade 1998/99, Klasse 10, Stufe 3 )

W.9 Gegeben sei ein Kreis mit dem Durchmesser d ≡ AB. Eine zu AB senkrechteGerade schneidet AB in P und den Kreis in C und D. Die Umfange der DreieckeAPC und BPD verhalten sich zueinander wie 2 : 1. Wie groß ist unter diesenVoraussetzungen das Verhaltnis AP : PB?(39. Mathematik-Olympiade 1999/2000, Klasse 10, Stufe 3 )

Achtung, Klasse 10! Die letzten beiden Aufgaben aus den Jahren 1999/2000 waren doch sehr ahnlich,oder? Ob es 2001 wieder etwas mit ”AB als Durchmesser eines Kreises“ gibt? Wahrscheinlich jetzt nichtmehr, schade eigentlich.

W.10 (Bild) Gegeben sei ein aus drei kongruenten Quadra-ten zusammengesetztes Rechteck. Man zeige ohne Be-nutzung trigonometrischer Formeln, daß die Summeder Winkel α und β gleich 45◦ ist. A B C D

EFGHαβ

(3. Mathematik-Olympiade 1963/64, Klasse 10, Stufe 4 )

W.11 Man beweise: Sind a, b, c die Seitenlangen und ist ∆ der Flacheninhalt einesDreiecks, so hat die Summe der Langen der drei Lote, die von je einer Seitenmitteauf die in der Gegenecke an den Umkreis gelegte Tangente gefallt werden, denWert

2∆ · a2 + b2 + c2

abc.

(32. Mathematik-Olympiade 1992/93, Klasse 10, Stufe 4 )

W.12 (Bild) In ein Quadrat mit gegebener Seitenlange a sollen n kongruente, moglichstgroße Kreise so einbeschrieben werden, daß keine zwei Kreise einen inneren Punktgemeinsam haben und daß kein Punkt eines Kreises außerhalb des Quadrates liegt.Das Bild zeigt ein Beispiel fur n = 6. Sind die Kreise in Teilbild a) oder b) großer?(35. Mathematik-Olympiade 1995/96, Klasse 10, Stufe 4 )

50 WETTBEWERBSAUFGABEN

a) b)

W.13 Einer Halbkugel mit dem Radius R werde ein Tetraeder ABCD so einbeschrieben,daß die Eckpunkte A, B, C auf der Peripherie der Grundflache der Halbkugel liegenund D im Scheitelpunkt der Halbkugel liegt. Das Volumen des Tetraeders sei mitV , der Umfang des Dreiecks ABC mit u bezeichnet. Weisen Sie nach, daß dannstets

V ≤ u3

324

gilt. Unter welchen Voraussetzungen gilt das Gleichheitszeichen?(37. Mathematik-Olympiade 1997/98, Klasse 10, Stufe 4 )

W.14 Es sei ABCD ein beliebig gegebenes konvexes Viereck. Ein Punkt P durchlaufealle Punkte der Seite AB, ein Punkt Q durchlaufe unabhangig hiervon alle Punkteder Seite CD. Man ermittle die Menge der Mittelpunkte aller so entstehendenStrecken PQ.(35. Mathematik-Olympiade 1996/97, Klasse 11–13, Stufe 3 )

W.15 Es sei k der Umkreis eines gegebenen regelmaßigen Sechsecks P1P2P3P4P5P6. IstX ein Punkt auf k, so seien A, B und C die Fußpunkte der Lote von X aufdie Diagonalen P1P4, P2P5 bzw. P3P6. Man beweise, daß der Flacheninhalt desDreiecks ABC nicht von der Wahl des Punktes X auf k abhangt.(35. Mathematik-Olympiade 1996/97, Klasse 11–13, Stufe 3 )

W.16 Die Punkte A, B, C und D liegen so auf einem Kreis k mit dem MittelpunktO, daß sie Eckpunkte eines Sehnenvierecks sind, dessen Diagonalen AC und BDsenkrecht aufeinander stehen. Man beweise, daß die Seite CD doppelt so lang wieder Abstand des Punktes O von der Seite AB ist.(38. Mathematik-Olympiade 1998/99, Klasse 11–13, Stufe 3 )

W.17 Fur ein konvexes Viereck ABCD mit den Diagonalen AC und BD seien folgen-de Winkelgroßen vorausgesetzt: �CBD = 10◦, �CAD = 20◦, �ABD = 40◦,�BAC = 50◦. Man berechne die Großen der beiden Innenwinkel �BCD und�ADC des Vierecks ABCD.(36. Mathematik-Olympiade 1996/97, Klasse 11–13, Stufe 4 )

Weitere Olympiadeaufgaben sind B.12, B.21, D.22, V.25, M.8, M.22, M.43, U.44 undU.81.

Nationale Wettbewerbe 51

W.2 Nationale Wettbewerbe

Mathematik-Olympiaden gibt es in fast allen Landern der Erde. Hier schauen wir mal,was woanders so gefordert wird:

W.21 (Bild) Im �ABC treffen sich AD, BE, CF ineinem Punkt P derart, daß AP = PD = 6, EP =3, PB = 9 und CF = 20 ist. Wie groß ist derFlacheninhalt von �ABC?(AIME, 1989 )

A

B CD

EF

P

W.22 Man bestimme die Punkte P im Innern eines Dreiecks ABC, fur die das ProduktPD · PE · PF maximal wird, wobei D, E und F die Lotfußpunkte der Lote vonP auf die Dreieckseiten BC, CA bzw. AB sind.(Großbritannien, 1978 )

W.23 Die Punkte Q und R liegen auf einem Kreis k. Von P werden die Tangenten PQund PR an k gelegt. A liegt auf der Verlangerung von PQ; der Umkreis von �PAR

sei k′. Der Kreis k′ schneide k wieder in B, AR den Kreis k im Punkt C. Manbeweise, daß �PAR = �ABC gilt.(Großbritannien, 1994 )

W.24 �ABC sei ein spitzwinkliges Dreieck mit dem Umkreismittelpunkt O. Der Kreisk gehe durch C, O und B. Die Seiten AB und AC schneiden k ein weiteres Malin P bzw. Q. Man beweise, daß die Geraden AO und PQ senkrecht aufeinanderstehen.(Großbritannien, 1996 )

W.25 Von einem Punkt P im Innern eines gleichseitigen �ABC werden die Lote auf dieSeiten BC, CA, AB gefallt; die Lotfußpunkte seien D, E, F . Es ist der geometri-sche Ort aller Punkte P zu bestimmen, fur die �FDE ein Rechter ist.(Irland, 1997 )

W.26 Seien D, E, F Punkte auf den Seiten BC, CA, AB eines Dreiecks ABC derart,daß AD ⊥ BC, BE die Winkelhalbierende von �ABC und F der Mittelpunkt vonAB ist. Beweise, daß sich AD, BE und CF dann und nur dann in einem Punktschneiden, wenn folgende Gleichung gilt:

a2(a− c) = (b2 − c2)(a+ c).

(Irland, 1999 )

W.27 Es seien O der Mittelpunkt eines Kreises k und A ein fester Punkt im Innern vonk, der nicht mit O zusammenfallt. Man bestimme alle Punkte P auf der Peripheriedes Kreises, fur die der Winkel �OPA maximal wird.(Kanada, 1977 )

W.28 In der im Bild gezeigten Figur haben die Strecken AB

und CD die Lange 1; die Winkel �ABC und �CBDbetragen 90◦ bzw. 30◦. Wie lang ist AC?(Kanada, 1986 )

A

B

C

D

52 WETTBEWERBSAUFGABEN

W.29 Eine Gerade t habe keine gemeinsamen Punkte mit einem Kreis k mit dem Mittel-punkt O. Punkt E liegt auf t mit OE ⊥ t. M sei ein anderer Punkt auf t; MA undMB seien Tangenten an k mit den Beruhrungspunkten A und B; AB schneidetOE in X. Man beweise, daß X nicht von der Lage von M abhangt.(Lettland, 1997 )

W.30 Gegeben sei ein Kreis k mit dem Mittelpunkt M und dem Radius r, AB sei einfester Durchmesser von k und K ein fester Punkt auf der Strecke AM . t sei dieTangente an k in A. Fur eine beliebige Sehne CD, die durch K geht und von ABverschieden ist, werden die Punkte P und Q als Schnittpunkte von BC bzw. BDmit t konstruiert. Man beweise, daß das Produkt AP · AQ konstant bleibt, wenndie Sehne CD variiert wird.(Osterreichisch-Polnischer Mathematik-Wettbewerb, 1992 )

W.31 Im �ABC sei P ein beliebiger Punkt auf der Seite BC. Durch den MittelpunktM derselben Seite wird eine Gerade gezeichnet, die parallel zu AP ist und eine derbeiden anderen Dreieckseiten im Punkt D schneidet. Man beweise, daß die StreckeDP die Dreiecksflache halbiert.(Schweden, 1987 )

W.32 Vom Inkreismittelpunkt I eines �ABC werden Strecken zu den Eckpunkten ge-zeichnet; diese zerlegen das Dreieck in drei kleinere Dreiecke. Die Umkreismittel-punkte dieser drei kleineren Dreiecke seien O1, O2 und O3. Man zeige, daß danndie Umkreise von �ABC und �O1O2O3 konzentrisch sind.(USA, 1988 )

W.33 ABCD sei ein konvexes Viereck, dessen Diagonalen AC und BD sich rechtwinkligin S schneiden. Man beweise, daß dann die zu den Seiten des Vierecks AB, BC,CD, DA spiegelbildlichen Punkte von S ein Sehnenviereck bilden.(USA, 1993 )

Weitere Aufgaben in dieser Kategorie sind K.35, V.3, M.42, U.22, U.23, U.41, U.45, G.4.

W.3 Internationale Wettbewerbe

Die Teilnahme an der Internationalen Mathematik-Olympiade (IMO) ist wohl das Er-strebenswerteste fur einen mathematikbegeisterten Schuler. Die dort gestellten Aufgabenwerden zuvor in einem Auswahlverfahren aus den Vorschlagen vieler einzelner Landerermittelt, und sind meist nicht ohne Kenntnis spezieller Sachverhalte (und damit auchintensiver Forderung) zu losen. Wir geben an dieser Stelle einen kleinen Einblick.

W.51 Im �ABC sei AC = BC. Ein Kreis k liegt so, daß er den Umkreis von �ABC voninnen beruhrt, und die Seiten AC, BC Tangenten an k mit den BeruhrungspunktenP , Q sind. Man beweise, daß der Mittelpunkt von PQ der Inkreismittelpunkt von�ABC ist.(20. IMO, Rumanien, Bukarest, 1978 )

Crux Mathematicorum 53

W.52 P ist ein Punkt im Innern eines gegebenen �ABC. Die Lotfußpunkte von P aufdie Seiten BC, CA und AB des Dreiecks seien D, E bzw. F . Man bestimme alleP , fur die folgender Ausdruck minimal wird:

BC

PD+CA

PE+AB

PF.

(22. IMO, USA, Washington D. C., 1981 )

W.53 Die Diagonalen AC und CE eines regularen Sechsecks ABCDEF werden durchdie auf ihnen liegenden Punkte M und N so geteilt, daß

AM

AC=CN

CE= r

gilt. Man bestimme r, wenn B, M und N kollinear sind.(23. IMO, Ungarn, Budapest, 1982 )

W.54 Der Mittelpunkt eines Kreises liege auf der Seite AB eines Sehnenvierecks ABCD.Die drei anderen Seiten liegen tangential an diesem Kreis. Man beweise, daß dannAD +BC = AB gilt.(26. IMO, Finnland, Joutsa/Helsinki, 1985 )

Doch bis man dahin gelangt, mussen einige Hurden genommen werden. Haufig finden zurintensiven Vorbereitung Trainingslager oder spezielle Kurse statt, in denen naturlich auchAuswahlklausuren geschrieben werden (siehe z. B. [WWW.11–12]). Hier einige Aufgabenaus dieser Kategorie:

W.61 Gegeben sei ein nicht-gleichschenkliges Dreieck ABC. Seine Winkelhalbierende wc

soll gleichzeitig den Winkel zwischen der Hohe hc und der Seitenhalbierenden mc

halbieren. Man beweise, daß dann das Dreieck ABC rechtwinklig ist.(Auswahlwettbewerb Deutschland, 1999 )

Eine weitere IMO-Aufgabe ist G.3. Aufgaben aus uberregionalen Wettbewerben wie derAPMO sind U.34, U.84 und G.2.

W.4 Crux Mathematicorum

Fur mathematisch interessierte Schuler ist die Zeitschrift”Crux Mathematicorum with

Mathematical Mayhem“, herausgegeben von der Canadian Mathematical Society, warm-stens zu empfehlen. Sie berichtet ausfuhrlich uber mathematische Wettbewerbe in allerWelt und erscheint in Englisch. Dies sollte jedoch fur gute Schuler keine ernsthafte Hurdedarstellen. Außerdem ist das zum Lesen und Verstehen der Aufgaben notige Vokabularnicht sehr umfangreich und schnell erlernt. Jeder kann dort seine Losungen hinschicken,die bei etwas Gluck auch abgedruckt werden. Fragt Euren Mathematik-Lehrer, wie es miteinem Abonnement der Zeitschrift an Eurer Schule aussieht!

54 WETTBEWERBSAUFGABEN

W.81 (Bild) Im �ABC werden diejenigen Tangenten anden Inkreis gezeichnet, die parallel zu den Seiten desDreiecks sind. Dadurch entsteht das Sechseck H ≡PQRSTU . Man beweise, daß der Umfang von H nie-mals großer als 2

3des Umfangs des �ABC ist.

(Crux Mathematicorum 189, 1988) A B

C

P Q

R

ST

U

W.82 a) Von einem Punkt D der Hypotenuse AB eines rechtwinkligen �ABC werdendie Lote DE und DF auf die Seiten BC und AC gefallt. Man bestimme diejenigePosition von D, fur die die Strecke EF minimale Lange hat.b) Welche Lage ergibt sich fur D, wenn �ABC spitzwinklig, aber nicht rechtwink-lig ist?(Crux Mathematicorum 246, 1987)

W.83 In der Ebene sei ein Kreis k mit dem Mittelpunkt O und dem Radius r sowiezwei weitere Punkte A und B außerhalb von k gegeben. Es ist eine Sehne PQ

zu konstruieren, die von A aus unter einem rechten Winkel erscheint und derenVerlangerung durch B geht.(Crux Mathematicorum 1188, 1988)

W.84 P sei ein Punkt im Innern von �ABC mit den Seiten a, b und c. AP schneide denKreis durch B, P , C ein zweites Mal in A′. Definiere B′ und C ′ analog. Beweise,daß fur den Umfang p des Sechsecks AB′CA′BC ′ gilt:

p ≥ 2(√ab+

√bc+

√ca).

(Crux Mathematicorum 2301, Februar 1998)

W.85 Im Dreieck ABC schneide die Winkelhalbierende von �CAB die Seite BC imPunkt D. Weiterhin sei AB + AD = CD und AC + AD = BC. Man bestimmedie Winkel �ABC und �BCA.(Crux Mathematicorum 2302, Februar 1998)

W.86 Angenommen, im �ABC erfullen die Winkel β und γ die Bedingung γ = 90◦+ 12β,

die außere Winkelhalbierende von α schneide die Verlangerung von BC in D,und die Seite AB beruhre den Inkreis von �ABC in E. Man beweise, daß dannCD = 2AE gilt.(Crux Mathematicorum 2303, Februar 1998)

W.87 Uber den Seiten AB und AC eines spitzwinkligen Dreiecks ABC werden nachaußen zwei gleichseitige Dreiecke �ABD und �ACE errichtet. Die Strecke CDschneide AB in F ; G sei der Schnittpunkt von BE und AC sowie P der von CD

und BE. Unter der Voraussetzung, daß das Viereck AFPG und das Dreieck PBCflachengleich sind, bestimme man den Winkel �BAC.(Crux Mathematicorum 2304, Februar 1998)

Crux Mathematicorum 55

W.88 Gegeben sei ein rechtwinkliges Dreieck ABC mit �BAC = 90◦. I sei der Inkreis-mittelpunkt sowie D und E die Schnittpunkte von BI bzw. CI mit den SeitenAC bzw. AB. Man zeige:

BI2 + ID2

CI2 + IE2=AB2

AC2.

(Crux Mathematicorum 2397, Dezember 1998)

Weitere Crux-Aufgaben sind B.51, K.62 und M.40.

56 WETTBEWERBSAUFGABEN

U

UNGLEICHUNGEN

Mit Gleichungen umzugehen ist uns bestens vertraut. Wir mussen lediglich darauf ach-ten, daß nur aquivalente Umformungen durchgefuhrt werden und kommen so von einerGleichung zu einer anderen und mit Gluck zur gewunschten. Etwas aufpassen mussenwir bei Manipulationen, daß nicht versehentlich durch

”versteckte Nullen“ dividiert wird

(darauf beruhen viele Beweise, die von einer richtigen Voraussetzung zu einem offensicht-lich falschen Ergebnis fuhren) oder etwa beim Wurzelziehen, bei dem mitunter mehrereLosungen entstehen. Dies wird im Schulunterricht ausreichend geubt.

Interessant wird es dagegen, wenn wir mit Ungleichungen rechnen, bei denen es einige Re-geln mehr zu beachten gilt. Zunachst sollte das Relationszeichen in die richtige Richtungzeigen. Wir erinnern uns daran, daß sich das Relationszeichen umdreht, wenn eine Un-gleichung mit einer negativen Zahl multipliziert wird. Bei Ungleichungsketten (mehrerenhintereinander geschriebenen Ungleichungen) mussen alle Relationszeichen gleichsinnigausgerichtet sein, wenn wir auseinander liegende Terme vergleichen wollen (so kann z. B.bei a < b > c keine Aussage uber a und c getroffen werden, wohl aber bei a < b < c).Ebenso ist bekannt, daß zwei gleichgerichtete Ungleichungen zwar addiert und multipli-ziert, aber nicht subtrahiert oder dividiert werden durfen: Aus 5 < 7 und 2 < 6 folgtrichtig 7 < 13 und 10 < 42, aber nicht 3 < 1 oder 5

2< 7

6. Derartige Besonderheiten

machen Ungleichungen ebenfalls zu beliebten Olympiadeaufgaben.

Da in vielen Mathematikbuchern Ungleichungen sehr stiefmutterlich behandelt werden,wollen wir hier etwas tiefer in dieses Gebiet eindringen. So wird in diesem Kapitel zunachstdas Standard-Repertoir an Ungleichungen mit reellen Zahlen vorgestellt (Abschnitt U.1),welches großtenteils mit Namen fruherer großer Mathematiker verknupft ist. Diese fun-damentalen Ungleichungen, die eher selten im Mathematik-Unterricht behandelt werden,sind unerlaßlich zur Losung von Olympiadeaufgaben. Es ist nicht leicht, allgemeingultigeLosungsstrategien aufzustellen. Dennoch laufen viele Beweise nach einem gewissen Sche-ma ab; wir widmen uns im Abschnitt U.2 den einfachen

”Tips und Tricks“, mit denen sich

eine Vielzahl von Ungleichungen beweisen lassen. In spateren Abschnitten behandeln wirUngleichungen in naturlichen bzw. ganzen Zahlen. Den Abschluß bildet eine

”Querbeet“-

Sammlung von Ubungsaufgaben. Ausgenommen sind hier geometrische Ungleichungen,die in Kapitel G zu finden sind.

Klassische Literatur zum Thema Ungleichungen sind die Bucher [Har52], [Mit64], [Mit70],auch neuere, wie [Clo98], sind empfehlenswert.

58 UNGLEICHUNGEN

U.1 Fundamentale Ungleichungen

In diesem Abschnitt stellen wir die wichtigsten allgemein bekannten Ungleichungen vor,von denen einige außerst leistungsfahig sind. Aus ihnen lassen sich eine Vielzahl vonFolgerungen gewinnen. Wir beginnen mit der einfachen Bernoullischen Ungleichung,an die sich weitere gelaufige Ungleichungen, wie die Dreiecksungleichung, die AM-GM-HM Ungleichungen oder die Cauchy-Schwarzsche Ungleichung anschließen. Gerade diezuletzt genannten werden zur Losung von Aufgaben oft benotigt.

Bei den Beweisen, die oft auf recht unterschiedliche Weise gefuhrt werden konnen, begeg-net uns der Anspruch, diese auch mit den Hilfsmitteln des jeweiligen Gebietes der Mathe-matik in Angriff zu nehmen, in welches die Ungleichung fallt. Wir sind daher bestrebt,moglichst

”elementare“ Beweise anzugeben. Nur gelingt es uns nicht in allen Fallen, ganz

auf grundlegende Satze aus der Differential- und Integralrechnung oder auf das Rechnenmit komplexen Zahlen zu verzichten. Der Leser moge dies verzeihen und es eventuell alsAnsporn nehmen, sich mit diesen Dingen weiter auseinanderzusetzen.

U.1 Bernoullische Ungleichung. Es sei n ∈ N und x ≥ −1. Dann gilt:

(1 + x)n ≥ 1 + nx. (U.1)

Gleichheit liegt genau dann vor, wenn n = 1 oder x = 0 ist.

U.2 (Verallgemeinerte) Bernoullische Ungleichungen. Es sei −1 < x �= 0. Danngilt:

(1 + x)a > 1 + ax fur a > 1 oder a < 0, (U.2)

(1 + x)a < 1 + ax fur 0 < a < 1. (U.3)

U.3 Dreiecksungleichung. Es seien z1, . . . , zn von null verschiedene komplexe Zahlen.Dann gilt:

n∑i=1

|zi| ≥∣∣∣∣∣

n∑i=1

zi

∣∣∣∣∣ . (U.4)

Das Gleichheitszeichen gilt, wenn die Argumente aller zi untereinander gleich sind.

U.4 Gewichtete AM-GM Ungleichung. Es seien a1, . . . , an positive reelle Zahlen;δ1, . . . , δn ebenfalls positive reelle Zahlen (Gewichte) mit δ1 + · · · + δn = 1. Danngilt:

δ1a1 + · · · + δnan ≥ aδ11 · · · aδnn , (U.5)

wobei Gleichheit genau dann vorliegt, wenn alle ai untereinander gleich sind.

U.5 AM-GM Ungleichung. Es seien a1, . . . , an positive reelle Zahlen. Dann gilt furdas arithmetische und geometrische Mittel dieser Zahlen:

a1 + · · · + ann

≥ (a1 · · · an) 1n . (U.6)

Gleichheit liegt genau dann vor, wenn alle ai untereinander gleich sind.

Fundamentale Ungleichungen 59

U.6 GM-HM Ungleichung. Es seien a1, . . . , an positive reelle Zahlen. Dann ist dasgeometrische Mittel dieser Zahlen stets großer oder gleich dem harmonischen Mit-tel :

(a1 · · · an) 1n ≥ n

1a1

+ · · · + 1an

. (U.7)

Gleichheit liegt genau dann vor, wenn alle ai untereinander gleich sind.

U.7 AM-HM Ungleichung. Es seien a1, . . . , an positive reelle Zahlen. Dann gilt furdas arithmetische und harmonische Mittel dieser Zahlen:

a1 + · · · + ann

≥ n1a1

+ · · · + 1an

. (U.8)

Gleichheit liegt genau dann vor, wenn alle ai untereinander gleich sind.

U.8 (Verallgemeinerte) Holdersche Ungleichungen. Angenommen, aij seien von nullverschiedene reelle Zahlen fur i = 1, . . . ,m und j = 1, . . . , n. Weiterhin seienp1, . . . , pm bzw. q1, . . . , qn positive reelle Zahlen (Gewichte) mit p1+· · ·+pm = 1 undq1+· · ·+qn = 1. Fur jedes i bzw. j seien die Zeilensummen Ri bzw. SpaltensummenCj definiert:

Ri ≡n∑

j=1

|aij|, Cj ≡m∑i=1

|aij|.

Dann gilt:

R p11 · · ·R pm

m ≥n∑

j=1

|a1j|p1 · · · |amj|pm und (U.9)

C q11 · · ·C qn

n ≥m∑i=1

|ai1|q1 · · · |ain|qn . (U.10)

Gleichheit liegt dann und nur dann in (U.9) vor, wenn alle Paare von Spaltenvek-toren (|a1j|, . . . , |amj|)T untereinander proportional sind; dagegen in (U.10), wennpaarweise alle Zeilenvektoren (|ai1|, . . . , |ain|) proportional sind.

U.9 Holdersche Ungleichung. Fur alle nicht verschwindenden reellen Zahlen a1, . . . , anund b1, . . . , bn sowie positiven p, q > 1 mit 1

p+ 1

q= 1 gilt:

(n∑i=1

|ai|p)1

p(

n∑i=1

|bi|q)1

q

≥n∑i=1

|aibi|. (U.11)

Gleichheit ist genau dann erfullt, wenn |bi| = λ|ai|p−1, λ ∈ R, fur alle i gilt.

60 UNGLEICHUNGEN

U.10 Minkowskische Ungleichung. Fur alle reellen Zahlen a1, . . . , an und b1, . . . , bnsowie p ≥ 1 gilt:

(n∑i=1

|ai|p)1

p

+

(n∑i=1

|bi|p)1

p

≥(

n∑i=1

|ai + bi|p)1

p

. (U.12)

Gleichheit ist genau dann erfullt, wenn die |ai| und |bi| proportional sind, d. h.|bi| = λ|ai|, λ ∈ R, fur alle i gilt.

U.11 Cauchy-Schwarzsche Ungleichung.Angenommen, die Vektoren uT ≡ (a1, . . . , an)und vT ≡ (b1, . . . , bn) bestehen aus nichtnegativen reellen Zahlen. Dann gilt:(

n∑i=1

a2i

) (n∑i=1

b2i

)≥

(n∑i=1

aibi

)2

bzw. u2v2 ≥ (u · v)2. (U.13)

Gleichheit gilt genau dann, wenn entweder alle ai verschwinden (u = 0), alle biverschwinden (v = 0), oder bi = λai, λ ∈ R, fur alle i (v = λu) ist.

U.12 Tschebyscheffsche Ungleichung. Angenommen, a1, . . . , an und b1, . . . , bn seiengleichsinnig geordnete reelle Zahlen:

{a1 ≤ · · · ≤ an,b1 ≤ · · · ≤ bn,

oder

{a1 ≥ · · · ≥ an,b1 ≥ · · · ≥ bn.

Dann gilt:

1

n

n∑i=1

aibi ≥(

1

n

n∑i=1

ai

) (1

n

n∑i=1

bi

). (U.14)

Gleichheit gilt genau fur a1 = · · · = an oder b1 = · · · = bn.

U.13 Schursche Ungleichung. Fur alle positiven reellen Zahlen x, y, z und λ > 0 gilt

xλ(x− y)(x− z) + yλ(y − z)(y − x) + zλ(z − x)(z − y) ≥ 0. (U.15)

Gleichheit gilt genau fur x = y = z.

Bevor wir zur nachsten Ungleichung kommen, mussen wir noch klaren, was unter derEigenschaft Konvexitat bzw. Konkavitat einer Funktion verstanden wird.

Konvexitat (Konkavitat). Eine Funktion f(x) heißt in einem Intervall (a, b) genaudann konvex, wenn die Ungleichung

f(θx1 + (1 − θ)x2) ≤ θf(x1) + (1 − θ)f(x2) (U.16)

fur alle x1, x2 ∈ (a, b) und jedes θ ∈ (0, 1) erfullt ist. Gilt in (U.16) sogar dasKleiner-Zeichen fur x1 �= x2, heißt die Funktion f streng konvex. Im Fall, daß −fkonvex ist, heißt f konkav bzw. (im Fall des Kleiner-Zeichens) streng konkav.

Einfache Tips und Tricks 61

x

y

f x

x1 x2x

( )

θ

Bild U.1. Konvexitat einer Funktion

Jedes xθ ∈ (x1, x2) laßt sich dabei als xθ ≡ x1 + (1 − θ)(x2 − x1) = θx1 + (1 − θ)x2 mitirgendeinem θ ∈ (0, 1) schreiben. In Bild U.1 hat die Gerade durch die Punkte (x1, f(x1))und (x2, f(x2)) die Gleichung

f(x) ≡ f(x1) +

[f(x2) − f(x1)

x2 − x1

](x− x1),

so daß der Funktionswert an der Stelle xθ gleich f(xθ) = θf(x1) + (1 − θ)f(x2) ist. DieBedingung (U.16) laßt sich somit kurz als f(xθ) ≤ f(xθ) schreiben. Geometrisch bedeutetKonvexitat (Konkavitat) also, daß der Graph der Funktion f(x) niemals oberhalb (unter-halb) irgendeiner Sekante liegt, die zwei auf dem Graphen liegenden Punkte verbindet.

U.14 Jensensche Ungleichung. Es sei f(x) eine konvexe Funktion im Intervall (a, b)und x1, . . . , xn beliebige Punkte darin. Weiterhin seien c1, . . . , cn nichtnegativeKonstanten mit c1 + · · · + cn = 1. Dann gilt:

n∑i=1

cif(xi) ≥ f

(n∑i=1

cixi

). (U.17)

Ist f streng konvex und außerdem jedes ci > 0, dann liegt Gleichheit genau furx1 = · · · = xn vor.

U.2 Einfache Tips und Tricks

Nachdem im vorigen Abschnitt die grundlegenden Werkzeuge bereitgelegt wurden, mitdenen wir die kommenden Aufgaben angehen konnen, betrachten wir einleitend einigeeinfache Ungleichungen. Als ersten allgemeingultigen Tip halten wir fest, daß es stets einguter Losungsansatz ist, die Behauptung auf einen Ausdruck zuruckzufuhren, der offen-sichtlich großer oder gleich null ist. Besonders eignen sich hierzu Quadrate oder Summenbzw. Produkte von solchen Ausdrucken. Gelingt es z. B., wie in der Ungleichung

(1 + a+ a2)2 < 3(1 + a2 + a4) ∀ a �= 1,

welche nach kurzer aquivalenter Umformung in a4 − a3 − a + 1 > 0 ubergeht, diesenAusdruck zu faktorisieren (was haufig Intuition erfordert):

a4 − a3 − a+ 1 = (a− 1)(a3 − 1) > 0,

62 UNGLEICHUNGEN

ist das Wichtigste schon geschafft. Sowohl fur a < 1 als auch fur a > 1 sind beideFaktoren offenbar positiv, und die Ungleichung ist bewiesen. Nebenbei bemerkt: Daßeins eine Nullstelle dieses Polynoms ist und somit der Faktor a − 1 auftreten konnte, istmanchmal schon der Aufgabenstellung zu entnehmen (hier die Einschrankung a �= 1).

U.21 Es seien a, b, c die Langen der Seiten eines rechtwinkligen Dreiecks, wobei dieHypotenuse die Lange c habe. Beweise, daß dann

a+ b ≤√

2c

gilt. Wann gilt das Gleichheitszeichen?

U.22 Zeige, daß folgende Ungleichung fur alle reellen Zahlen x �= 0 gilt:

x8 − x5 − 1

x+

1

x4≥ 0.

(Irland, 1998 )

U.23 Es seien a, b und c beliebige positive reelle Zahlen. Zeige, daß

a) a3 + b3 + c3 ≥ a2b+ b2c+ c2a;

b) abc ≥ (a+ b− c)(b+ c− a)(c+ a− b).

(Großbritannien, 1981 )

U.2.1”Teile und (be)herrsche“

Bei Ungleichungen, die mehrere Terme enthalten und deren Seiten bei einer zyklischenVertauschung der Variablen unverandert bleiben, bietet es sich an, zunachst nur Teileder gesamten Ungleichung zu betrachten. Oft gelingt es, auf beiden Seiten Terme zugruppieren, in denen einige Variablen fehlen. Liegt uns z. B.

a2 + b2 + c2 ≥ ab+ bc+ ca fur alle a, b, c ∈ R

zum Beweis vor, finden wir auf der linken Seite a2 + b2 und auf der rechten ab, zwischendenen diejenige Ungleichung besteht, auf die wohl am haufigsten zuruckgegriffen wird:(a− b)2 ≥ 0, oder daraus abgeleitet

a2 + b2 ≥ 2ab odera

b+b

a≥ 2 oder x+

1

x≥ 2. (U.18)

Ebenso erkennen wir b2+c2 ≥ 2bc und c2+a2 ≥ 2ca. Eine Addition dieser drei einfacherenBeziehungen und anschließende Division durch 2 liefert das gewunschte Ergebnis.

U.24 Man zeige, daß folgende Ungleichung gilt:

a4 + b4 + c4 ≥ a2bc+ b2ca+ c2ab ∀ a, b, c > 0.

Einfache Tips und Tricks 63

U.25 Fur alle a, b, c, d > 0 gilt:

a3 + b3 + c3

a+ b+ c+a3 + b3 + d3

a+ b+ d+a3 + c3 + d3

a+ c+ d+b3 + c3 + d3

b+ c+ d≥ a2 + b2 + c2 + d2.

U.26 Man beweise

(1

b+

1

c

)(1

c+

1

a

)(1

a+

1

b

)≥ 8

abc∀ a, b, c > 0.

U.27 Fur alle x, y, z > 0 gilt:

x2

y2+y2

z2+z2

x2≥ y

x+z

y+x

z.

U.2.2 Die Arbeitspferde: AM-GM und Cauchy-Schwarz

Die Struktur der AM-GM Ungleichung (U.6) sowie der Cauchy-Schwarzschen Unglei-chung (U.13) geben vor, welche Ungleichungen sich auf diese zuruckfuhren lassen: Be-zeichnen wir einmal Summen von irgendwelchen Termen mit S und Produkte mit P, soschreibt sich die AM-GM Ungleichung grob als

S ≥ P (Typ a)

und die Cauchy-Schwarzsche Ungleichung als

S1 · S2 ≥ S23 (Typ b), oder

S1 ≥ S23 (Typ c), oder auch

S1 ≥ S3 (Typ d).

Typ c) geht dabei aus dem”Ur“typ b) hervor, wenn eine Summe auf der linken Seite zu

einer Zahl entartet; Typ d) liegt hingegen bei S2 = S3 > 0 vor und geht aus diesem durchDivision von S2 hervor bzw. bei S1 = S2, so daß auf beiden Seiten die Wurzel gezogenwerden kann. Hier ein Beispiel zu jedem Typ:

U.31 Fur alle x, y, z ∈ R gilt:

x4(1 + y4) + y4(1 + z4) + z4(1 + x4) ≥ 6x2y2z2.

U.32 a1, a2, . . . , an und b1, b2, . . . , bn seien beliebige reelle Zahlen. Dann gilt

(a2

1 + 2a22 + 3a2

3 + · · · + na2n

) (b21 +

1

2b22 +

1

3b23 + · · · + 1

nb2n

)

≥ (a1b1 + a2b2 + a3b3 + · · · + anbn)2.

64 UNGLEICHUNGEN

U.33 Man bestimme alle reelle Zahlen e, die mit gegebenen a, b, c, d die Gleichungen

a+ b+ c+ d+ e = 7,

a2 + b2 + c2 + d2 + e2 = 13

erfullen.

U.34 Es seien a1, a2, . . . , an, b1, b2, . . . , bn positive reelle Zahlen mit a1 + a2 + · · · + an =b1 + b2 + · · · + bn. Zeige, daß

a21

a1 + b1+

a22

a2 + b2+ · · · + a2

n

an + bn≥ a1 + a2 + · · · + an

2.

(APMO, 1991 )

U.35 Fur alle ai ∈ R, i = 1, . . . , n gilt:

a21 + a2

2 + · · · + a2n ≥ a1a2 + a2a3 + · · · + ana1.

U.2.3 AM-HM Kandidaten

Mitunter begegnen uns Ungleichungen, die Summen von Bruchen enthalten. Naturlichkonnen wir dann den Versuch unternehmen, die Bruche gleichnamig zu machen. Diesfuhrt aber oft auf umfangreiche und unubersichtliche Ausdrucke, insbesondere wenn dieNenner alle verschieden sind. Sind jedoch in unserer Ungleichung gerade alle Zahler gleich,ist sie ein guter Kandidat fur die AM-HM Ungleichung (U.8).

U.36 Es sei S ≡ x1 + x2 + · · · + xn mit xi > 0 (i = 1, 2, . . . , n). Man zeige

S

S − x1

+S

S − x2

+ · · · + S

S − xn≥ n2

n− 1,

wobei das Gleichheitszeichen genau bei x1 = x2 = · · · = xn gilt.

U.37 Fur alle reellen Zahlen a, b, c gilt folgende Ungleichung:

a2

b2 + c2+

b2

c2 + a2+

c2

a2 + b2≥ 3

2.

U.2.4 Ungleichungen unter Nebenbedingungen

Haufig sind Ausdrucke nach oben bzw. unten abzuschatzen, d. h., es ist das Maximumbzw. Minimum des Ausdrucks in einem bestimmten Definitionsbereich oder unter zusatz-lich einschrankenden Bedingungen gesucht. Angenommen, wir sollen zeigen, daß fur nicht-negative Zahlen a und b mit der Nebenbedingung a2 + b2 = 4 der Ausdruck

ab

a+ b+ 2

Einfache Tips und Tricks 65

niemals großer als√

2−1 wird (Osterreich, 1989 ). Die Losungsidee besteht nun darin, denin der Nebenbedingung enthaltenen Ausdruck (hier also a2 + b2) so in dem Zielausdruckoder der zu beweisenden Ungleichung unterzubringen, daß letzteres sich vereinfacht. DieNebenbedingung ist damit

”verkocht“ und wir konnen uns voll auf den neuen Term bzw.

die neue Ungleichung konzentrieren. In unserem obigen Beispiel lassen die Terme a2 + b2,ab und a+ b vermuten, daß wir es zunachst mit

(a+ b)2 = a2 + b2 + 2ab = 4 + 2ab, oder

2ab = (a+ b)2 − 22 = (a+ b+ 2)(a+ b− 2)

versuchen konnen, was uns obigen Ausdruck tatsachlich auf

ab

a+ b+ 2=a+ b− 2

2=a+ b

2− 1

vereinfacht. Somit reduziert sich die Aufgabe auf den Nachweis von a + b ≤ 2√

2. Nunkommt der entscheidende Schritt:

(a+ b)2 = (a2 + b2) + 2ab

{ ≤ 2(a2 + b2),≥ 4ab,

beides wegen a2 + b2 ≥ 2ab. Die obere Moglichkeit liefert eine Abschatzung nach oben(Maximum), die andere nach unten (Minimum). Wir sind hier an einem Maximalwert vona + b interessiert und finden nach Wurzelziehen und Einsetzen der Nebenbedingung dieUngleichung a+ b ≤ 2

√2 bestatigt.

U.41 Man finde den minimalen Wert des Ausdrucks (x+ y)(y+ z) mit positiven reellenZahlen x, y und z, die die Bedingung xyz(x+ y + z) = 1 erfullen.(Großbritannien, 1991 )

U.42 a, b, c seien positive reelle Zahlen, die die Gleichung (1+a)(1+b)(1+c) = 8 erfullen.Man zeige abc ≤ 1.

U.43 Beweise, daß der Wurfel von allen Quadern a) das großte Volumen bei konstanterOberflache und b) den kleinsten Oberflacheninhalt bei konstantem Volumen hat.

U.44 Man beweise: Fur alle positiven reellen Zahlen a und b mit a+ b = 1 gilt

(a+

1

a

)2

+

(b+

1

b

)2

≥ 25

2.

(10. Mathematik-Olympiade 1970/71, Klasse 11–12, Stufe 1 )

U.45 Es seien a, b, c, d positive reelle Zahlen, deren Summe 1 betragt. Zeige, daß

a2

a+ b+

b2

b+ c+

c2

c+ d+

d2

d+ a≥ 1

2,

mit Gleichheit nur fur a = b = c = d = 14.

(Irland, 1999 )

66 UNGLEICHUNGEN

U.3 Elementare symmetrische Funktionen

Vielleicht ist es uns schon an den bisherigen Aufgaben aufgefallen: Viele Ungleichungenbeinhalten symmetrische Ausdrucke in den betreffenden Variablen. Einen Ausdruck nen-nen wir symmetrisch in seinen Variablen, wenn jede Permutation der Veranderlichen denAusdruck unverandert laßt. Dagegen heißt ein Ausdruck zyklisch (vertauschbar), wenn erlediglich invariant gegenuber einer zyklischen Vertauschung der Variablen x1, x2, . . . , xn

(x1, x2, . . . , xn) → (x2, x3, . . . , xn, x1) → · · · → (xn, x1, . . . , xn−1), oder auch

(x1, x2, . . . , xn) → (xn, x1, . . . , xn−1) → · · · → (x2, x3, . . . , xn, x1)

ist, also nicht”wild“ permutiert wird, sondern die Reihenfolge eingehalten wird. Betrach-

ten wir beispielsweise die symmetrische Summe

Q1(a, b, c) ≡ a

b+ c+

b

c+ a+

c

a+ b,

so konnen wir uns leicht uberzeugen, daß

Q1(a, b, c) = Q1(b, c, a) = Q1(c, a, b) = Q1(a, c, b) = Q1(b, a, c) = Q1(c, b, a)

erfullt ist. Andererseits hat das zyklische Polynom

Q2(x, y, z) ≡ x2y + y2z + z2x

nur drei Vertauschungmoglichkeiten: Q2(x, y, z) = Q2(y, z, x) = Q2(z, x, y).

Im folgenden beschaftigen wir uns zunachst mit den einfachsten symmetrischen Termen.Sind x1, . . . , xn ∈ R die Nullstellen des Polynoms

P(x) ≡ (x− x1)(x− x2) · · · (x− xn)

= xn − σ1xn−1 + σ2x

n−2 − · · · + (−1)n−1σn−1x+ (−1)nσn = 0, (U.19)

so werden dessen Koeffizienten σ1, σ2, . . . , σn die elementaren symmetrischen Funktionender Variablen xi genannt. Formal konnen wir dafur auch

σk =∑sym

(n∏

j=1

xπj

j

)(U.20)

schreiben, wobei π ≡ (π1, . . . , πn) eine Permutation von πj = {0, 1} mit π1 + · · ·+ πn = kdarstellt, und die Summation

∑sym

bedeutet, daß uber alle Permutationen zu summieren

ist (insgesamt also(nk

)Summanden). Vereinfacht gesagt, ist σk somit die Summe aller

Produkte der xi, wobei immer k Faktoren genommen werden.

Fur n = 2, x1 = a, x2 = b ergibt sich somit

σ1 ≡ a+ b, σ2 = ab, (U.21)

fur n = 3, x1 = a, x2 = b, x3 = c

σ1 ≡ a+ b+ c, σ2 ≡ bc+ ca+ ab, σ3 ≡ abc, (U.22)

Elementare symmetrische Funktionen 67

und fur n = 4, x1 = a, x2 = b, x3 = c, x4 = d

σ1 ≡ a+ b+ c+ d, σ2 ≡ ab+ ac+ ad+ bc+ bd+ cd,

σ3 ≡ bcd+ cda+ dab+ abc, σ4 ≡ abcd. (U.23)

Weiterhin sei angenommen, daß σ0 = 1 und σk = 0 fur k > n gilt.

Der wohl wichtigste Satz im Zusammenhang mit den elementaren symmetrischen Funk-tionen ist der sog.

U.51 Fundamentalsatz (Hauptsatz) uber symmetrische Polynome. Jedes symmetri-sche Polynom P (x1, . . . , xn) (mit ganzzahligen Koeffizienten) laßt sich als Polynom(mit ganzzahligen Koeffizienten) in den elementaren symmetrischen Funktionenσ1, . . . , σn schreiben.

Wie sieht diese Zerlegung in die Bausteine σk in Praxis aus? Wir geben im folgenden einigeBeispiele fur n = 3 Variablen an, die wohl in Aufgaben auch am haufigsten anzutreffensind. Dabei treten mitunter die Potenzsummen

sk ≡ xk1 + xk2 + · · · + xkn (U.24)

auf, die ublicherweise mit sk abkurzt werden.

Polynome 2. Grades

s2 ≡ a2 + b2 + c2 = σ21 − 2σ2, (U.25)

Polynome 3. Grades

s3 ≡ a3 + b3 + c3 = σ31 − 3σ1σ2 + 3σ3, (U.26)

bc(b+ c) + ca(c+ a) + ab(a+ b) = σ1σ2 − 3σ3, (U.27)

(b+ c)(c+ a)(a+ b) = σ1σ2 − σ3, (U.28)

Polynome 4. Grades

s4 ≡ a4 + b4 + c4 = σ41 − 4σ2

1σ2 + 4σ1σ3 + 2σ22, (U.29)

bc(b2 + c2) + ca(c2 + a2) + ab(a2 + b2) = σ21σ2 − σ1σ3 − 2σ2

2, (U.30)

b2c2 + c2a2 + a2b2 = −2σ1σ3 + σ22. (U.31)

Eine umfangreiche Ubersicht findet sich in den Tabellen T.1 bis T.3.

Aus diesen Identitaten lassen sich nun sehr leicht Ungleichungen in den elementaren sym-metrischen Funktionen gewinnen: Auf den linken Seiten stehen klar positive Großen, alsomussen auch die rechten Seiten positiv sein. Dort sind jedoch stets negative Summandenvorhanden, so daß diese auf die andere Seite der Ungleichung gebracht werden konnen.Aus (U.28) folgt z. B. σ1σ2 − σ3 ≥ 0 oder σ1σ2 ≥ σ3.

Es zeigt sich jedoch, daß diese Ungleichungen nicht besonders”scharf“ sind. Man sagt,

eine Ungleichung x ≥ y kann fur x verscharft werden, wenn sie auch fur ein großeresy + ε mit ε > 0 gilt (ebenso fur y, wenn sie fur ein kleineres x − ε gilt). Z. B. ist (U.27)σ1σ2 ≥ 3σ3 scharfer als (U.28).

68 UNGLEICHUNGEN

Wie gelangen wir nun zu den interessanten scharfen Ungleichungen? Ganz einfach: Wirsetzen in quadratische Ausdrucke moglichst viel Differenzen ein! Auf diese Weise lassensich mit etwas Aufwand folgende Ungleichungen gewinnen:

U.52 Es gelten folgende Ungleichungen in den elementaren symmetrischen Funktionenσ1 ≡ a+ b+ c, σ2 ≡ bc+ ca+ ab und σ3 = abc:

σ21 ≥ 3σ2, (U.32)

σ1σ2 ≥ 9σ3, (U.33)

2σ31 + 9σ3 ≥ 7σ1σ2, (U.34)

σ41 + 9σ2

2 ≥ 6σ21σ2, (U.35)

σ22 ≥ 3σ1σ3, (U.36)

σ1(σ1σ2 + 3σ3) ≥ 4σ22, (U.37)

σ21σ2 ≥ 3σ1σ3 + 2σ2

2, (U.38)

σ41 + 3σ1σ3 + 5σ2

2 ≥ 5σ21σ2, (U.39)

σ2(σ31 + 27σ3) ≥ 3σ1(3σ1σ3 + σ2

2), (U.40)

σ2(σ1σ2 + 3σ3) ≥ 4σ21σ3, (U.41)

σ1(2σ41 + 9σ1σ3 + 21σ2

2) ≥ σ2(13σ31 + 27σ3), (U.42)

σ1σ22 ≥ σ3(2σ

21 + 3σ2), (U.43)

σ31σ2 + σ2

1σ3 + 6σ2σ3 ≥ 4σ1σ22, (U.44)

σ2(σ31 + 15σ3) ≥ σ1(5σ1σ3 + 3σ2

2), (U.45)

σ1(2σ41 + 10σ1σ3 + 17σ2

2) ≥ 3σ2(4σ31 + 7σ3), (U.46)

σ1(2σ41 + 11σ1σ3 + 13σ2

2) ≥ σ2(11σ31 + 15σ3), (U.47)

σ1(σ41 + 9σ1σ3 + 12σ2

2) ≥ σ2(7σ31 + 27σ3), (U.48)

σ1(2σ51 + 57σ1σ

22 + 54σ2σ3) ≥ 3(6σ4

1σ2 + 4σ31σ3 + 22σ3

2 + 27σ23), (U.49)

σ21σ

22 + 27σ2

3 ≥ 2(σ31σ3 + σ3

2), (U.50)

σ41σ2 + 7σ3

1σ3 + 16σ32 + 27σ2

3 ≥ σ1σ2(8σ1σ2 + 27σ3), (U.51)

σ2(σ41 + 27σ1σ3 + 2σ2

2) ≥ 7σ31σ3 + 4σ2

1σ22 + 27σ2

3, (U.52)

σ1σ2(σ1σ2 + 10σ3) ≥ 4σ31σ3 + 2σ3

2 + 9σ23, (U.53)

2σ32 + 9σ2

3 ≥ 7σ1σ2σ3, (U.54)

σ21 (2σ

41 + 2σ1σ3 + 41σ2

2) ≥ 16σ41σ2 + 34σ3

2 + 27σ23, (U.55)

2σ61 + 12σ3

1σ3 + 51σ21σ

22 + 81σ2

3 ≥ 6σ2(3σ41 + 9σ1σ3 + 7σ2

2), (U.56)

σ1σ2(σ1σ2 + 18σ3) ≥ 4σ31σ3 + 4σ3

2 + 27σ23, (U.57)

σ61 + 10σ3

1σ3 + 18σ21σ

22 + 27σ2

3 ≥ 4σ2(2σ41 + 9σ1σ3 + 2σ2

2), (U.58)

3σ1σ2(σ1σ2 + 12σ3) ≥ 10σ31σ3 + 10σ3

2 + 27σ23. (U.59)

Weitere Ungleichungen 69

U.53 Zeige, daß folgende Bedingungen einander aquivalent sind:

{σ1 > 0, σ2 > 0, . . . , σn > 0} ⇐⇒ {x1 > 0, x2 > 0, . . . , xn > 0}. (U.60)

Beispiel: Fur alle a, b, c ∈ R gilt:

{a+ b+ c > 0, bc+ ca+ ab > 0, abc > 0} ⇐⇒ {a > 0, b > 0, c > 0}.

U.54 Folgt aus σ1 = a+ b+ c > 0 und σ3 = abc > 0 auch a > 0, b > 0, c > 0?

U.4 Weitere Ungleichungen

Abschließend eine Sammlung von Ungleichungen, die sich sowohl als Ubungsmaterial alsauch zum Nachschlagen eignet.

U.81 Es seien a, b, c und d positive Zahlen. Beweisen Sie, daß unter dieser Voraussetzungstets folgende Ungleichung gilt:

1

a+ b+ c+

1

a+ b+ d+

1

a+ c+ d+

1

b+ c+ d>

4

a+ b+ c+ d.

(36. Mathematik-Olympiade 1996/97, Klasse 9, Stufe 2 )

U.82 Fur a, b, c, d, e ≥ 0 gilt

(a2 + b2 + c2 + d2 + e2)(a3 + b3 + c3 + d3 + e3) ≥ 25abcde.

U.83 Es sei b1, b2, . . . , bn eine beliebige Permutation der positiven Zahlen a1, a2, . . . , an.Dann gilt:

a1

b1+a2

b2+ · · · + an

bn≥ n.

U.84 Ist S ≡ x1 + x2 + · · · + xn die Summe positiver reeller Zahlen xi, i = 1, 2, . . . , n,dann gilt

(1 + x1)(1 + x2) · · · (1 + xn) ≤ 1 + S +S2

2!+S3

3!+ · · · + Sn

n!.

(APMO, 1989 )

U.85 a ≡ (a1, a2, . . . , an) und b ≡ (b1, b2, . . . , bn) seien Folgen positiver reeller Zahlen.Dann gilt

min1≤k≤n

akbk

n∑k=1

ak

n∑k=1

bk

≤ max1≤k≤n

akbk

mit Gleichheit in beiden Ungleichungen genau dann, wenn beide Folgen a und b

zueinander proportional sind.

70 UNGLEICHUNGEN

U.86bc

b+ c+

ca

c+ a+

ab

a+ b≤ 1

2(a+ b+ c) (a, b, c > 0).

U.871

b+ c− a+

1

c+ a− b+

1

a+ b− c≥ 9

a+ b+ c(a, b, c > 0).

U.88 Fur positive Zahlen x, y, z, die die Gleichung x+ y + z = 1 erfullen, zeige man

a)

(1

x+ 1

) (1

y+ 1

) (1

z+ 1

)≥ 64,

b)

(1

x− 1

) (1

y− 1

) (1

z− 1

)≥ 8.

U.89 Fur alle reelle Zahlen a, b, c > −1, die

1

1 + a+

1

1 + b+

1

1 + c= 1

erfullen, gilt abc ≥ 8.

U.90 Man zeige, daß aus c+ d ≤ min(a, b) mit a, b, c, d ≥ 0 folgt:

ad+ bc ≤ ab und ac+ bd ≤ ab.

U.91 Die nichtnegativen reellen Zahlen a, b, c, d, e, f erfullen die Bedingungen a+ b ≤ eund c+ d ≤ f . Dann gilt

√ac+

√bd ≤

√ef bzw.

√ad+

√bc ≤

√ef.

U.92 Fur x, y, z > 0 ist

x4

yz3+

y4

zx3+

z4

xy3≥ 3.

G

GEOMETRISCHEUNGLEICHUNGEN

Die Uberschrift dieses Kapitels deutet darauf hin, daß es eine Kombination aus den Ge-bieten Geometrie und Ungleichungen gibt, die eine gewisse Eigenstandigkeit erlangt hat.Bestes Beispiel dafur sind die Titel der Bucher [Bot69] und [Kaz61].

In erster Linie ist die Eigenstandigkeit gegenuber den”gewohnlichen“ Ungleichungen, so

wir sie im Kapitel U betrachtet haben, dadurch begrundet, daß bei geometrischen Un-gleichungen mitunter zusatzliche Voraussetzungen erfullt sein mussen. Nehmen wir z. B.Ungleichungen in den Seitenlangen a, b, c eines Dreiecks (Abschnitt G.1), so ist klar, daßnicht nur wie bisher haufig a, b, c ≥ 0 vorausgesetzt wird, sondern außerdem die Dreiecks-ungleichungen (s. Aufgabe U.3) erfullt sein mussen, wenn es sich bei a, b, c tatsachlich umdie Seitenlangen eines Dreiecks handeln soll. Daruber hinaus gibt es in einigen Fallen auch

”anschauliche“ geometrische Interpretationen der ansonsten abstrakten Ungleichungen.

Die in diesem Kapitel verwendeten Bezeichnungen der Stucke geometrischer Figuren sindin Tabelle X.2 zusammengestellt.

G.1 Ungleichungen im Dreieck

Dieser Abschnitt enthalt eine Vielzahl bekannter Ungleichungen, die zwischen gewissenStucken eines Dreiecks gelten. Gerade fur mathematische Puzzles in Problemzeitschriftenwerden diese haufig benotigt, und sei es auch nur als Teil der angestrebten Losung. Deshalbist diese Ubersicht bewußt im Stil eines Kompendiums angelegt. In den Losungen wirddennoch ein Beweis der Relation erbracht. Naturlich ist das Material nicht neu, es stutztsich im wesentlichen auf [Bot69]. Auch die Gliederung wurde von dort ubernommen, wobeizahlreiche neuere Ungleichungen eingearbeitet wurden.

Um Schreibarbeit zu sparen, verabreden wir, das Symbol � hinter der betreffenden Unglei-chung anzugeben, wenn Gleichheit im Fall eines gleichseitigen Dreiecks vorliegt, dagegendas Symbol � fur Gleichheit im Fall eines gleichschenkligen Dreiecks.

72 GEOMETRISCHE UNGLEICHUNGEN

G.1.1 Ungleichungen fur die Seitenlangen

Wenn nicht anders angegeben, setzen wir nachfolgend stets a, b, c > 0 voraus, d. h., zuStrecken entartete Dreiecke werden nicht betrachtet. Außerdem gelten die Dreiecksunglei-chungen (s. Aufgabe U.3)

|b− c| < a < b+ c, |c− a| < b < c+ a, |a− b| < c < a+ b. (G.1)

Um nun diese Voraussetzungen unter einen Hut zu bringen, wird haufig folgende Varia-blensubstitution durchgefuhrt, die wir bereits aus Aufgabe D.63 kennen:

x ≡ b+ c− a

2= s− a, y ≡ c+ a− b

2= s− b, z ≡ a+ b− c

2= s− c, (G.2)

wobei s ≡ 12(a+ b+ c) der halbe Umfang des Dreiecks ist. Wegen (G.1) gilt dann stets

x, y, z > 0. (G.3)

Dies hat den Vorteil, daß eine in die Großen x, y, z umgeschriebene Ungleichung bereitsgarantiert, daß tatsachlich ein Dreieck vorliegt, und wir mit (G.3) den Anschluß an dieStandard-Ungleichungen des Kapitels U haben. Naturlich laßt sich die Transformation(G.2) auch umkehren:

a = y + z, b = z + x, c = x+ y. (G.4)

Hieraus sehen wir, daß der bei AM-GM-HM oder Cauchy-Schwarz eintretende Falldes Gleichheitszeichens in der betreffenden Ungleichung fur x = y = z gerade bei einemgleichseitigen Dreieck auftritt.

G.1 Es gilt

abc ≥ (a+ b− c)(b+ c− a)(c+ a− b). � (G.5)

G.2 Man beweise

√a+ b− c+

√b+ c− a+

√c+ a− b ≤ √

a+√b+

√c (G.6)

und bestimme, wann Gleichheit auftritt.(APMO, 1996 )

G.3 Es seien a, b, c die Seitenlangen eines Dreiecks. Beweise

a2b(a− b) + b2c(b− c) + c2a(c− a) ≥ 0 (G.7)

und bestimme, in welchem Fall Gleichheit auftritt.(24. IMO, Frankreich, Paris, 1983 )

G.4 Beweise, daß mit a, b, c als Seitenlangen eines Dreiecks folgende Ungleichung gilt:

1

a+

1

b+

1

c≤ 1

a+ b− c+

1

c+ a− b+

1

b+ c− a. (G.8)

(Polen, 1993 )

Ungleichungen in Vierecken 73

G.1.2 Ungleichungen fur die Winkel

Hier haben wir als Nebenbedingung die Winkelsumme im Dreieck zu beachten:

α+ β + γ = π. (G.41)

G.42 Fur die Winkel α, β, γ eines Dreiecks gilt stets

cosα+ cos β + cos γ ≤ 3

2, (G.42)

sinα+ sin β + sin γ ≤√

33

2. (G.43)

G.1.3 Ungleichungen fur die Radien

G.51 Der Umkreisradius R eines Dreiecks ist mindestens doppelt so groß wie der In-kreisradius I:

R ≥ 2 r. (G.51)

G.1.4 Ungleichungen fur die Seiten- und Winkelhalbierenden

G.61 Fur die Summe der Langen der Seitenhalbierenden m ≡ ma + mb + mc einesDreiecks gilt stets

3

2s < m < 2s, (G.61)

wobei s ≡ 12(a+ b+ c) dessen halber Umfang ist.

G.2 Ungleichungen in Vierecken

Die wohl am haufigsten anzutreffende Ungleichung in den Seiten eines Vierecks ist

G.81 Ptolemaus’ Ungleichung. In jedem konvexen Viereck ABCD gilt:

AB · CD +BC · AD ≥ AC ·BD, (G.81)

mit Gleichheit genau bei einem Sehnenviereck.

G.82 In jedem konvexen Viereck ABCD gilt mit dem halben Umfang s ≡ 12(a+b+c+d)

sowie den Diagonalenlangen p ≡ AC, q ≡ BD:

s < p+ q < 2s. (G.82)

74 GEOMETRISCHE UNGLEICHUNGEN

T

TABELLEN

Hier finden sich einige mitunter nutzliche Tabellen, die aus dem laufenden Text heraus-genommen wurden, damit dieser kompakt bleibt.

76 TABELLEN

Tabelle T.1. Symmetrische Polynome S{p, q}, p ≥ q fur n = 2 ausgedrucktdurch die elementaren symmetrischen Funktionen σ1 ≡ a+ b und σ2 ≡ ab

Polynome 1. Grades σ1

S{1, 0} = a+ b ≡ σ1 1

Polynome 2. Grades σ21 σ2

S{2, 0} = a2 + b2 ≡ s2 1 −2S{1, 1} = 2ab ≡ 2σ2 0 2

Polynome 3. Grades σ31 σ1σ2

S{3, 0} = a3 + b3 ≡ s3 1 −3S{2, 1} = ab(a+ b) 0 1

Polynome 4. Grades σ41 σ2

1σ2 σ22

S{4, 0} = a4 + b4 ≡ s4 1 −4 2S{3, 1} = ab(a2 + b2) 0 1 −2S{2, 2} = 2a2b2 0 0 2

Polynome 5. Grades σ51 σ3

1σ2 σ1σ22

S{5, 0} = a5 + b5 ≡ s5 1 −5 5S{4, 1} = ab(a3 + b3) 0 1 −3S{3, 2} = a2b2(a+ b) 0 0 1

Polynome 6. Grades σ61 σ4

1σ2 σ21σ

22 σ3

2

S{6, 0} = a6 + b6 ≡ s6 1 −6 9 −2S{5, 1} = ab(a4 + b4) 0 1 −4 2S{4, 2} = a2b2(a2 + b2) 0 0 1 −2S{3, 3} = 2a3b3 0 0 0 2

Polynome 7. Grades σ71 σ5

1σ2 σ31σ

22 σ1σ

32

S{7, 0} = a7 + b7 ≡ s7 1 −7 14 −7S{6, 1} = ab(a5 + b5) 0 1 −5 5S{5, 2} = a2b2(a3 + b3) 0 0 1 −3S{4, 3} = a3b3(a+ b) 0 0 0 1

TABELLEN 77

Tabelle T.1. Symmetrische Polynome S{p, q}, p ≥ q fur n = 2 ausgedrucktdurch die elementaren symmetrischen Funktionen σ1 ≡ a+ b und σ2 ≡ ab

(Fortsetzung)

Polynome 8. Grades σ81 σ6

1σ2 σ41σ

22 σ2

1σ32 σ4

2

S{8, 0} = a8 + b8 ≡ s8 1 −8 20 −16 2S{7, 1} = ab(a6 + b6) 0 1 −6 9 −2S{6, 2} = a2b2(a4 + b4) 0 0 1 −4 2S{5, 3} = a3b3(a2 + b2) 0 0 0 1 −2S{4, 4} = 2a4b4 0 0 0 0 2

Polynome 9. Grades σ91 σ7

1σ2 σ51σ

22 σ3

1σ32 σ1σ

42

S{9, 0} = a9 + b9 ≡ s9 1 −9 27 −30 9S{8, 1} = ab(a7 + b7) 0 1 −7 14 −7S{7, 2} = a2b2(a5 + b5) 0 0 1 −5 5S{6, 3} = a3b3(a3 + b3) 0 0 0 1 −3S{5, 4} = a4b4(a+ b) 0 0 0 0 1

Polynome 10. Grades σ101 σ8

1σ2 σ61σ

22 σ4

1σ32 σ2

1σ42 σ5

2

S{10, 0} = a10 + b10 ≡ s10 1 −10 35 −50 25 −2S{9, 1} = ab(a8 + b8) 0 1 −8 20 −16 2S{8, 2} = a2b2(a6 + b6) 0 0 1 −6 9 −2S{7, 3} = a3b3(a4 + b4) 0 0 0 1 −4 2S{6, 4} = a4b4(a2 + b2) 0 0 0 0 1 −2S{5, 5} = 2a5b5 0 0 0 0 0 2

78TABELLEN

Tabelle T.2. Symmetrische Polynome S{p, q, r}, p ≥ q ≥ r fur n = 3 ausgedrucktdurch die elementaren symmetrischen Funktionen σ1 ≡ a+ b+ c, σ2 ≡ bc+ ca+ abund σ3 ≡ abc

Polynome 1. Grades σ1

S{1, 0, 0} = 2(a+ b+ c) ≡ 2σ1 2

Polynome 2. Grades σ21 σ2

S{2, 0, 0} = 2(a2 + b2 + c2) ≡ 2s2 2 −4S{1, 1, 0} = 2(bc+ ca+ ab) ≡ 2σ2 0 2

Polynome 3. Grades σ31 σ1σ2 σ3

S{3, 0, 0} = 2(a3 + b3 + c3) ≡ 2s3 2 −6 6S{2, 1, 0} = bc(b+ c) + ca(c+ a) + ab(a+ b) 0 1 −3S{1, 1, 1} = 6abc ≡ 6σ3 0 0 6

Polynome 4. Grades σ41 σ2

1σ2 σ1σ3 σ22

S{4, 0, 0} = 2(a4 + b4 + c4) ≡ 2s4 2 −8 8 4S{3, 1, 0} = bc(b2 + c2) + ca(c2 + a2) + ab(a2 + b2) 0 1 −1 −2S{2, 2, 0} = 2(b2c2 + c2a2 + a2b2) 0 0 −4 2S{2, 1, 1} = 2(a2bc+ b2ca+ c2ab) 0 0 2 0

TABELLEN

79

Tabelle T.2. Symmetrische Polynome S{p, q, r}, p ≥ q ≥ r fur n = 3 ausgedrucktdurch die elementaren symmetrischen Funktionen σ1 ≡ a+ b+ c, σ2 ≡ bc+ ca+ ab

und σ3 ≡ abc (Fortsetzung)

Polynome 5. Grades σ51 σ3

1σ2 σ21σ3 σ1σ

22 σ2σ3

S{5, 0, 0} = 2(a5 + b5 + c5) ≡ 2s5 2 −10 10 10 −10S{4, 1, 0} = bc(b3 + c3) + ca(c3 + a3) + ab(a3 + b3) 0 1 −1 −3 5S{3, 2, 0} = b2c2(b+ c) + c2a2(c+ a) + a2b2(a+ b) 0 0 −2 1 −1S{3, 1, 1} = 2(a3bc+ b3ca+ c3ab) 0 0 2 0 −4S{2, 2, 1} = 2(a2b2c+ b2c2a+ c2a2b) 0 0 0 0 2

Polynome 6. Grades σ61 σ4

1σ2 σ31σ3 σ2

1σ22 σ1σ2σ3 σ3

2 σ23

S{6, 0, 0} = 2(a6 + b6 + c6) ≡ 2s6 2 −12 12 18 −24 −4 6S{5, 1, 0} = bc(b4 + c4) + ca(c4 + a4) + ab(a4 + b4) 0 1 −1 −4 7 2 −3S{4, 2, 0} = b2c2(b2 + c2) + c2a2(c2 + a2) + a2b2(a2 + b2) 0 0 −2 1 4 −2 −3S{4, 1, 1} = 2(a4bc+ b4ca+ c4ab) 0 0 2 0 −6 0 6S{3, 3, 0} = 2(b3c3 + c3a3 + a3b3) 0 0 0 0 −6 2 6S{3, 2, 1} = ab2c2(b+ c) + bc2a2(c+ a) + ca2b2(a+ b) 0 0 0 0 1 0 −3S{2, 2, 2} = 6a2b2c2 0 0 0 0 0 0 6

Polynome 7. Grades σ71 σ5

1σ2 σ41σ3 σ3

1σ22 σ2

1σ2σ3 σ1σ32 σ1σ

23 σ2

2σ3

S{7, 0, 0} = 2(a7 + b7 + c7) ≡ 2s7 2 −14 14 28 −42 −14 14 14S{6, 1, 0} = bc(b5 + c5) + ca(c5 + a5) + ab(a5 + b5) 0 1 −1 −5 9 5 −4 −7S{5, 2, 0} = b2c2(b3 + c3) + c2a2(c3 + a3) + a2b2(a3 + b3) 0 0 −2 1 6 −3 −7 3S{5, 1, 1} = 2(a5bc+ b5ca+ c5ab) 0 0 2 0 −8 0 8 4S{4, 3, 0} = b3c3(b+ c) + c3a3(c+ a) + a3b3(a+ b) 0 0 0 0 −3 1 5 −1S{4, 2, 1} = ab2c2(b2 + c2) + bc2a2(c2 + a2) + ca2b2(a2 + b2) 0 0 0 0 1 0 −1 −2S{3, 3, 1} = 2(a3b3c+ b3c3a+ c3a3b) 0 0 0 0 0 0 −4 2S{3, 2, 2} = 2(a3b2c2 + b3c2a2 + c3a2b2) 0 0 0 0 0 0 2 0

80TABELLEN

Tabelle T.2. Symmetrische Polynome S{p, q, r}, p ≥ q ≥ r fur n = 3 ausgedrucktdurch die elementaren symmetrischen Funktionen σ1 ≡ a+ b+ c, σ2 ≡ bc+ ca+ ab

und σ3 ≡ abc (Fortsetzung)

Polynome 8. Grades σ81 σ6

1σ2 σ51σ3 σ4

1σ22 σ3

1σ2σ3 σ21σ

32 σ2

1σ23 σ1σ

22σ3 σ4

2 σ2σ23

S{8, 0, 0} = 2(a8 + b8 + c8) = 2s8 2 −16 16 40 −64 −32 24 48 4 −16S{7, 1, 0} = bc(b6 + c6) + ca(c6 + a6) + ab(a6 + b6) 0 1 −1 −6 11 9 −5 −17 −2 8S{6, 2, 0} = b2c2(b4 + c4) + c2a2(c4 + a4) + a2b2(a4 + b4) 0 0 −2 1 8 −4 −9 0 2 2S{6, 1, 1} = 2(a6bc+ b6ca+ c6ab) 0 0 2 0 −10 0 10 10 0 −10S{5, 3, 0} = b3c3(b2 + c2) + c3a3(c2 + a2) + a3b3(a2 + b2) 0 0 0 0 −3 1 3 6 −2 −7S{5, 2, 1} = ab2c2(b3 + c3) + bc2a2(c3 + a3) + ca2b2(a3 + b3) 0 0 0 0 1 0 −1 −3 0 5S{4, 4, 0} = 2(b4c4 + c4a4 + a4b4) 0 0 0 0 0 0 4 −8 2 8S{4, 3, 1} = ab3c3(b+ c) + bc3a3(c+ a) + ca3b3(a+ b) 0 0 0 0 0 0 −2 1 0 −1S{4, 2, 2} = 2(a4b2c2 + b4c2a2 + c4a2b2) 0 0 0 0 0 0 2 0 0 −4S{3, 3, 2} = 2(a3b3c2 + b3c3a2 + c3a3b2) 0 0 0 0 0 0 0 0 0 2

TABELLEN

81

Tabelle T.3. Symmetrische Polynome S{p, q, r, s}, p ≥ q ≥ r ≥ s fur n = 4 ausgedrucktdurch die elementaren symmetrischen Funktionen σ1 ≡ a+ b+ c+ d,σ2 ≡ ab+ ac+ ad+ bc+ bd+ cd, σ3 ≡ abc+ abd+ acd+ bcd und σ4 ≡ abcd

Polynome 1. Grades σ1

S{1, 0, 0, 0} = a+ b+ c+ d ≡ σ1 1

Polynome 2. Grades σ21 σ2

S{2, 0, 0, 0} = a2 + b2 + c2 + d2 ≡ s2 1 −2S{1, 1, 0, 0} = ab+ ac+ ad+ bc+ bd ≡ σ2 0 1

Polynome 3. Grades σ31 σ1σ2 σ3

S{3, 0, 0, 0} = a3 + b3 + c3 + d3 ≡ s3 1 −3 3S{2, 1, 0, 0} = ab(a+ b) + ac(a+ c) + ad(a+ d)

+bc(b+ c) + bd(b+ d) + cd(c+ d) 0 1 −3S{1, 1, 1, 0} = abc+ abd+ acd+ bcd ≡ σ3 0 0 1

Polynome 4. Grades σ41 σ2

1σ2 σ1σ3 σ22 σ4

S{4, 0, 0, 0} = a4 + b4 + c4 + d4 ≡ s4 1 −4 4 2 −4S{3, 1, 0, 0} = a3(b+ c+ d) + b3(c+ d+ a)

+c3(d+ a+ b) + d3(a+ b+ c) 0 1 −1 −2 4S{2, 2, 0, 0} = a2b2 + a2c2 + a2d2 + b2c2 + b2d2 + c2d2 0 0 −2 1 2S{2, 1, 1, 0} = a2(bc+ bd+ cd) + b2(cd+ ca+ da)

+c2(da+ db+ ab) + d2(ab+ ac+ bc) 0 0 1 0 −4S{1, 1, 1, 1} = abcd ≡ σ4 0 0 0 0 1

82TABELLEN

Tabelle T.3. Symmetrische Polynome S{p, q, r, s}, p ≥ q ≥ r ≥ s fur n = 4 ausgedrucktdurch die elementaren symmetrischen Funktionen σ1 ≡ a+ b+ c+ d,σ2 ≡ ab+ ac+ ad+ bc+ bd+ cd, σ3 ≡ abc+ abd+ acd+ bcd und σ4 ≡ abcd

(Fortsetzung)

Polynome 5. Grades σ51 σ3

1σ2 σ21σ3 σ1σ

22 σ2σ3 σ1σ4

S{5, 0, 0, 0} = a5 + b5 + c5 + d5 ≡ s5 1 −5 5 5 −5 −5S{4, 1, 0, 0} = a4(b+ c+ d) + b4(c+ d+ a)

+c4(d+ a+ b) + d4(a+ b+ c) 0 1 −1 −3 5 1S{3, 2, 0, 0} = a2b2(c+ d) + a2c2(b+ d) + a2d2(b+ c)

+b2c2(d+ a) + b2d2(c+ a) + c2d2(a+ b) 0 0 −2 1 −1 5

TABELLEN 83

Tabelle T.4. Minimalpolynome fur n = 3 ausgedruckt durch die elementarensymmetrischen Funktionen σ1 ≡ a+ b+ c, σ2 ≡ bc+ ca+ ab und σ3 ≡ abc

Polynome 2. Grades

Q ≡ (b− c)2 + (c− a)2 + (a− b)2 = 2σ21 − 6σ2, (T.1)

Polynome 3. Grades

a(b− c)2 + b(c− a)2 + c(a− b)2 = σ1σ2 − 9σ3, (T.2)

(b+ c)(b− c)2 + (c+ a)(c− a)2 + (a+ b)(a− b)2 = 2σ31 − 7σ1σ2 + 9σ3, (T.3)

Polynome 4. Grades

(b− c)4 + (c− a)4 + (a− b)4 =

2[(b− c)2(c− a)2 + (c− a)2(a− b)2 + (a− b)2(b− c)2]

= 2σ41 − 12σ2

1σ2 + 18σ22, (T.4)

a2(b− c)2 + b2(c− a)2 + c2(a− b)2 = 2σ22 − 6σ1σ3, (T.5)

bc(b− c)2 + ca(c− a)2 + ab(a− b)2 = σ21σ2 + 3σ1σ3 − 4σ2

2, (T.6)

a(b+ c)(b− c)2 + b(c+ a)(c− a)2 + c(a+ b)(a− b)2

= σ21σ2 − 3σ1σ3 − 2σ2

2, (T.7)

(b2 + c2)(b− c)2 + (c2 + a2)(c− a)2 + (a2 + b2)(a− b)2

= 2σ41 − 10σ2

1σ2 + 6σ1σ3 + 10σ22, (T.8)

Polynome 5. Grades

a(b− c)4 + b(c− a)4 + c(a− b)4 =

(b+ c)(c− a)2(a− b)2 + (c+ a)(a− b)2(b− c)2 + (a+ b)(b− c)2(c− a)2

= σ31σ2 − 9σ2

1σ3 − 3σ1σ22 + 27σ2σ3, (T.9)

a3(b− c)2 + b3(c− a)2 + c3(a− b)2 = −4σ21σ3 + σ1σ

22 + 3σ2σ3, (T.10)

(b+ c)(b− c)4 + (c+ a)(c− a)4 + (a+ b)(a− b)4

= 2σ51 − 13σ3

1σ2 + 9σ21σ3 + 21σ1σ

22 − 27σ2σ3, (T.11)

a2(b+ c)(b− c)2 + b2(c+ a)(c− a)2 + c2(a+ b)(a− b)2

= −2σ21σ3 + σ1σ

22 − 3σ2σ3, (T.12)

bc(b+ c)(b− c)2 + ca(c+ a)(c− a)2 + ab(a+ b)(a− b)2

= σ31σ2 + σ2

1σ3 − 4σ1σ22 + 6σ2σ3, (T.13)

a(b2 + c2)(b− c)2 + b(c2 + a2)(c− a)2 + c(a2 + b2)(a− b)2

= σ31σ2 − 5σ2

1σ3 − 3σ1σ22 + 15σ2σ3, (T.14)

(b3 + c3)(b− c)2 + (c3 + a3)(c− a)2 + (a3 + b3)(a− b)2

= 2σ51 − 12σ3

1σ2 + 10σ21σ3 + 17σ1σ

22 − 21σ2σ3, (T.15)

(b+ c)(b2 + c2)(b− c)2 + (c+ a)(c2 + a2)(c− a)2 + (a+ b)(a2 + b2)(a− b)2

84 TABELLEN

= 2σ51 − 11σ3

1σ2 + 11σ21σ3 + 13σ1σ

22 − 15σ2σ3, (T.16)

c(b− c)2(c− a)2 + a(c− a)2(a− b)2 + b(a− b)2(b− c)2

= σ51 − 7σ3

1σ2 + 9σ21σ3 + 12σ1σ

22 − 27σ2σ3, (T.17)

Tabelle T.4. Minimalpolynome fur n = 3 ausgedruckt durch die elementarensymmetrischen Funktionen σ1 ≡ a+ b+ c, σ2 ≡ bc+ ca+ ab und σ3 ≡ abc

(Fortsetzung)

Polynome 6. Grades

(b− c)6 + (c− a)6 + (a− b)6

= 2σ61 − 18σ4

1σ2 − 12σ31σ3 + 57σ2

1σ22 + 54σ1σ2σ3 − 66σ3

2 − 81σ23, (T.18)

a2(b− c)4 + b2(c− a)4 + c2(a− b)4 = −2σ31σ3 + σ2

1σ22 − 2σ3

2 + 27σ23, (T.19)

bc(b− c)4 + ca(c− a)4 + ab(a− b)4

= σ41σ2 + 7σ3

1σ3 − 8σ21σ

22 − 27σ1σ2σ3 + 16σ3

2 + 27σ23, (T.20)

a(b+ c)(b− c)4 + b(c+ a)(c− a)4 + c(a+ b)(a− b)4

= σ41σ2 − 7σ3

1σ3 − 4σ21σ

22 + 27σ1σ2σ3 + 2σ3

2 − 27σ23, (T.21)

a4(b− c)2 + b4(c− a)2 + c4(a− b)2 =

− 4σ31σ3 + σ2

1σ22 + 10σ1σ2σ3 − 2σ3

2 − 9σ23, (T.22)

a3(b+ c)(b− c)2 + b3(c+ a)(c− a)2 + c3(a+ b)(a− b)2

= −7σ1σ2σ3 + 2σ32 + 9σ2

3, (T.23)

(b2 + c2)(b− c)4 + (c2 + a2)(c− a)4 + (a2 + b2)(a− b)4

= 2σ61 − 16σ4

1σ2 + 2σ31σ3 + 41σ2

1σ22 − 34σ3

2 − 27σ23, (T.24)

(b− c)2 [(c− a)4 + (a− b)4] + (c− a)2 [(a− b)4 + (b− c)4]

+ (a− b)2 [(b− c)4 + (c− a)4]

= 2σ61 − 18σ4

1σ2 + 12σ31σ3 + 51σ2

1σ22 − 54σ1σ2σ3 − 42σ3

2 + 81σ23, (T.25)

(b− c)2(c− a)2(a− b)2 = −4σ31σ3 + σ2

1σ22 + 18σ1σ2σ3 − 4σ3

2 − 27σ23, (T.26)

c2(b− c)2(c− a)2 + a2(c− a)2(a− b)2 + b2(a− b)2(b− c)2

= σ61 − 8σ4

1σ2 + 10σ31σ3 + 18σ2

1σ22 − 36σ1σ2σ3 − 8σ3

2 + 27σ23, (T.27)

(b2 + c2)(c− a)2(a− b)2 + (c2 + a2)(a− b)2(b− c)2

+ (a2 + b2)(b− c)2(c− a)2

= −10σ31σ3 + 3σ2

1σ22 + 36σ1σ2σ3 − 10σ3

2 − 27σ23. (T.28)

TABELLEN 85

Tabelle T.5. Minimalpolynome fur n = 4 ausgedruckt durch die elementarensymmetrischen Funktionen σ1 ≡ a+ b+ c+ d, σ2 ≡ ab+ ac+ ad+ bc+ bd+ cd,σ3 ≡ abc+ abd+ acd+ bcd und σ4 ≡ abcd

Polynome 2. Grades

(a− b)2 + (a− c)2 + (a− d)2 + (b− c)2 + (b− d)2 + (c− d)2

= 3σ21 − 8σ2, (T.51)

Polynome 3. Grades

(a+ b)(a− b)2 + (a+ c)(a− c)2 + (a+ d)(a− d)2 + (b+ c)(b− c)2

+ (b+ d)(b− d)2 + (c+ d)(c− d)2 = 3σ31 − 10σ1σ2 + 12σ3, (T.52)

Polynome 4. Grades

(a− b)4 + (a− c)4 + (a− d)4 + (b− c)4 + (b− d)4 + (c− d)4

= 3σ41 − 16σ2

1σ2 + 4σ1σ3 + 20σ22 − 16σ4, (T.53)

ab(a− b)2 + ac(a− c)2 + ad(a− d)2 + bc(b− c)2

+ bd(b− d)2 + cd(c− d)2 = σ21σ2 + 3σ1σ3 − 4σ2

2, (T.54)

86 TABELLEN

X

HINWEISE

In der Tabelle X.1 sind die am haufigsten benutzten mathematischen Symbole aufgelistet:

Tabelle X.1. Haufig benutzte mathematische Symbole und Abkurzungen

Symbol Bedeutung

∈ Mitgliedschaft in einer Menge/∈ nicht enthalten in einer Menge⊆ Untermenge∪ Vereinigung zweier Mengen∩ Schnitt zweier Mengen⇒ logische Implikation

⇐⇒ logische Aquivalenz (engl.”iff“)

≡ identisch mit (haufig als Definition benutzt)∼ ahnlich zu∼= kongruent zu∝ proportional zu‖ parallel zu∦ nicht parallel zu∀ fur alle∃ existiert� existiert nicht∴ deshalb∵ weiln! sprich:

”n Fakultat“ = 1 · 2 · · ·n

�x� großte ganze Zahl ≤ x

x! kleinste ganze Zahl ≥ xx ∈ (a, b) offenes Intervall a < x < bx ∈ [a, b) halboffenes Intervall a ≤ x < b

x ∈ (a, b] halboffenes Intervall a < x ≤ bx ∈ [a, b] geschlossenes Intervall a ≤ x ≤ b

x ∈ [a, b] geschlossenes Intervall a ≤ x ≤ b

88 HINWEISE

Tabelle X.1. Haufig benutzte mathematische Symbole und Abkurzungen(Fortsetzung)

Symbol Bedeutung

∅ leere Menge∅ leere MengeN Menge aller naturlichen ZahlenZ Menge aller ganzen ZahlenR Menge aller reellen ZahlenC Menge aller komplexen Zahlensgn(x) Vorzeichen von x (

”Signum“)

min(x1, . . . , xn) Minimum der Zahlen x1, . . . , xnmax(x1, . . . , xn) Maximum der Zahlen x1, . . . , xnexp(z) Exponentialfunktion ez

#[z] = x Realteil einer komplexen Zahl z$[z] = y Imaginarteil einer komplexen Zahl z|z| Betrag (Modulus) einer komplexen Zahl zarg z = φ Argument einer komplexen Zahl zz konjugiert komplexe Zahl zu z

uT transponierter Vektor zu u

HINWEISE 89

Fur die Großen in planimetrischen Figuren, insbesondere in Dreiecken und Vierecken, ha-ben sich feste Bezeichnungen eingeburgert, die in Tabelle X.2 aufgefuhrt werden. Dabei istzu beachten, daß den im englischen Sprachraum ublichen Bezeichnungen Vorrang gegebenwurde (s. auch [Bot69]), z. B. ma anstelle von sa fur die Seitenhalbierende (engl.

”medi-

an“) oder G anstelle von S fur den Schwerpunkt (engl.”gravity center“). Fur Winkel

verwenden wir jedoch kleine griechische Buchstaben.

Tabelle X.2. Großen in planimetrischen Figuren

Symbol Bedeutung

A, B, C Eckpunkte eines DreiecksO UmkreismittelpunktI InkreismittelpunktIa, Ib, Ic Mittelpunkte der AnkreiseG SchwerpunktH HohenschnittpunktGe Gergonnes PunktNa Nagels PunktL Lemoines PunktP Punkt im Innern eines Dreiecksα, β, γ Innenwinkel bei A, B, Ca, b, c Seitenlangen, wobei a ≡ BC, b ≡ CA, c ≡ AB

R Umkreisradiusr Inkreisradiusra, rb, rc Ankreisradienwa, wb, wc Langen der Winkelhalbierendenma, mb, mc Langen der Seitenhalbierendenha, hb, hc Langen der Hohens halber Umfang s ≡ 1

2(a+ b+ c)

x, y, z Abstande von P zu den Eckpunkten von �ABCu, v, w Abstande von P zu den Seiten von �ABCw1, w2, w3 Langen der Winkelhalbierenden von �BPC, �CPA, �APBr′1, r

′2, r

′3 Langen von Ecktransversalen PD, PE, PF

∆ FlacheninhaltQ ≡ (a− b)2 + (b− c)2 + (c− a)2

p, q Diagonalen AC, BD

90 HINWEISE

Y

LITERATUR

Y.1 Bucher

[Bap92] Baptist, P.: Die Entwicklung der neueren Dreiecksgeometrie, BI Wissenschaftsver-lag, Mannheim–Leipzig–Wien–Zurich, 1992.

[Bar95] Barbeau, E. J., Klamkin, M. S., Moser, W. O. J.: Five Hundred MathematicalChallenges, The Mathematical Association of America, Washington, D. C., 1995.

[Bie52] Bieberbach, L.: Theorie der geometrischen Konstruktionen, Verlag Birkhauser, Ba-sel, 1952.

[Bir59] Birkhoff, G. D., Beatley, R.: Basic Geometry, Chelsea Publishing Company,New York, 1959.

[Bot69] Bottema, O., Djordjevic, R. Z., Janic, R. R., Mitrinovic, D. S., Vasic, P.

M.: Geometric Inequalities, Wolters-Noordhoff Publishing, Groningen, The Nether-lands, 1969.

[Bre51] Breidenbach, W.: Die Dreiteilung des Winkels, B. G. Teubner Verlagsgesellschaft,Leipzig, 1951.

[Cha97] Chang, G., Sederberg, T. W.: Over and over again, The Mathematical Associa-tion of America, New Mathematical Library, vol. 39, Washington, D. C., 1997.

[Cho88] Chou, Shang-Ching: Mechanical Geometry Theorem Proving, D. Reidel PublishingCompany, Dordrecht, 1988.

[Clo98] Cloud, M. J., Drachman, B. C.: Inequalities – With Applications to Engineering,Springer, New York–Berlin–Heidelberg, 1998.

[Cou62] Courant, R., Robbins, H.: Was ist Mathematik? Springer-Verlag, Berlin–Gottin-gen–Heidelberg, 1962.

[Cox63] Coxeter, H. S. M.: Unvergangliche Geometrie, Birkhauser Verlag, Basel und Stutt-gart, 1963.

[Cox67] Coxeter, H. S. M., Greitzer, S. L.: Geometry revisited, The Mathematical As-sociation of America, New Mathematical Library, vol. 19, Washington, D. C., 1967.

92 LITERATUR

[Cox83] Coxeter, H. S. M., Greitzer, S. L.: Zeitlose Geometrie, Aus d. Amerikan.: Geo-metry revisited, Klett Studienbucher, Klett, Stuttgart, 1983.

[Cra61] Crantz-Hauptmann: Planimetrie, Sammlung Crantz, B. G. Teubner Verlagsgesell-schaft, Leipzig, 1961.

[Cro91] Croft, H. T., Falconer, K. J., Guy, R. K.: Unsolved Problems in Geometry,Springer, New York, 1991.

[Dub58] Dubnow, J. S.: Fehler in geometrischen Beweisen, Deutscher Verlag der Wissen-schaften, Berlin, 1958.

[Dud92] Dudley, U.: Mathematical Cranks, The Mathematical Association of America, Wa-shington, D. C., 1992.

[Dun94] Dunham, W.: The Mathematical Universe – An Alphabetical Journey Through theGreat Proofs, Problems, and Personalities, John Wiley & Sons, Inc., New York, 1994.

[Eng98] Engel, A.: Problem-Solving Strategies, Springer, New York, 1998.

[Eng90] Engel, W., Pirl, U. (Hrsg): Neue Mathematik-Olympiadeaufgaben, Aulis VerlagDeubner & Co KG, Koln, 1990.

[Enr07] Enriques, F.: Fragen der Elementargeometrie, II. Teil, Die geometrischen Aufgaben,ihre Losung und Losbarkeit, Druck und Verlag von B. G. Teubner, Leipzig, 1907.

[Gar97] Gardiner, A.: The Mathematical Olympiad Handbook – An Introduction to ProblemSolving, Oxford University Press, Oxford, New York, Tokyo, 1997.

[Gro69] Grosche, G.: Ubungen fur Junge Mathematiker, Teil 2 Elementargeometrie, BSBB. G. Teubner Verlagsgesellschaft, Leipzig, 1969.

[Har52] Hardy, G., Littlewood, J. E., Polya, G.: Inequalities, Second Edition, Cam-bridge University Press, 1952.

[Hei98] Heilbron, J. L.: Geometry Civilized – History, Culture, and Technique, ClarendonPress, Oxford, 1998.

[Her86] Herterich, K.: Die Konstruktion von Dreiecken, Ernst Klett Verlage GmbH u. Co.KG, Stuttgart, 1986.

[Hon96] Honsberger, R.: From Erdos To Kiev – Problems of Olympiad Caliber, The Ma-thematical Association of America, Washington, D. C., 1996.

[Hon97] Honsberger, R.: In Polya’s Footsteps – Miscellaneous Problems and Essays, TheMathematical Association of America, Washington, D. C., 1997.

[Kaz61] Kazarinoff, N. D.: Geometric Inequalities, Random House & Yale University, NewMathematical Library, No. 4, 1961.

[Ker16] Kerst, B.: Methoden zur Losung geometrischer Aufgaben, Verlag und Druck von B.G. Teubner, Leipzig und Berlin, 1916.

[Kla86] Klamkin, M. S.: International Mathematical Olympiads 1978–1985 and forty sup-plementary problems, The Mathematical Association of America, New MathematicalLibrary, vol. 31, Washington, D. C., 1986.

[Lan97] Lang, S., Murrow, G.: Geometry, Springer, New York, 1997.

Bucher 93

[Lar83] Larson, L. C.: Problem-Solving Through Problems, Problem Books in Mathematics,Springer, New York–Berlin–Heidelberg, 1983.

[Leh96] Lehmann, J.: Mathematik-Olympiaden – 666 Olympiadeaufgaben aus 42 Landern,Ernst Klett Verlag, Stuttgart, 1996.

[Mah10] Mahler, G.: Ebene Geometrie, Sammlung Goschen, G. J. Goschen’sche Verlags-handlung, Leipzig, 1910.

[Mar97] Martin, George E.: Transformation Geometry – An Introduction to Symmetry,Undergraduate Texts in Mathematics, Springer, New York–Berlin–Heidelberg, 1997.

[Mar98] Martin, George E.: Geometric Constructions, Undergraduate Texts in Mathema-tics, Springer, New York–Berlin–Heidelberg, 1998.

[Met00] Mettler, M.: Vom Charme der ”verblassten“ Geometrie , ISBN 973-9441-97-1,Verlag Eurobit, Timisoara, Romania, 2000.

[Mit64] Mitrinovic, D. S.: Elementary Inequalities, P. Noordhoff Ltd, Groningen, The Net-herlands, 1964.

[Mit70] Mitrinovic, D. S.: Analytic Inequalities, Springer, Berlin–Heidelberg–New York,1970.

[Ogi84] Ogilvy, C. S.: Unterhaltsame Geometrie, Vieweg & Sohn, Braunschweig, 1984.

[Ped95] Pedoe, D.: Circles – A Mathematical View, The Mathematical Association of Ame-rica, Washington, D. C., 1995.

[Ped94] Peiffer, J., Dahan-Dalmedico, A.: Wege und Irrwege — Eine Geschichte derMathematik, Birkhauser Verlag, Basel–Boston–Berlin, 1994.

[Pog87] Pogorelov, A. W.: Geometry, Mir Publishers, Moscow, 1987.

[Roe93] Roe, J.: Elementary Geometry, Oxford University Press, Oxford, 1993.

[Sma98] Smart, J. R.:Modern Geometries, Brooks/Cole Publishing Company, Pacific Grove,1998.

[Sve91] Sved, M.: Journey into Geometries, The Mathematical Association of America, Wa-shington, D. C., 1991.

[Tha33a] Thaer, C. (Hrsg.): Die Elemente von Euklid, 1. Teil, Ostwald’s Klassiker der exak-ten Wissenschaften Nr. 235, Akademische Verlagsgesellschaft m.b.H., Leipzig, 1933.

[Tha33b] Thaer, C. (Hrsg.): Die Elemente von Euklid, 2. Teil, Ostwald’s Klassiker der exak-ten Wissenschaften Nr. 236, Akademische Verlagsgesellschaft m.b.H., Leipzig, 1933.

[Wei99] Weisstein, E. W.: CRC Concise Encyclopedia of Mathematics, Chapman & Hall/CRC, Boca Raton, London, New York, Washington, D.C., 1999.

[Wel91] Wells, D.: The Penguin Dictionary of Curious and Interesting Geometry, PenguinBooks, London, 1991.

[Zha??] Zhang, J. Z.: A New Approach to Plane Geometry (in chinesischer Sprache), Sichu-an Educational Publishers, 1992, ISBN 7-5408-1611-2, referiert in: Crux Mathemati-corum 21, p. 232–233, 7 (Sept. 1995).

94 LITERATUR

Y.2 Zeitschriften-Artikel

[1] Paasche, I. ”Aufgabe 714.“ Elem. Math. 30, 41–42, 1975.

[2] Erdos, P. ”Problem 3740.“ Amer. Math. Monthly 42, 396, 1935.

[3] Mordell, L. J., Barrow, D. F. ”Solution to Problem 3740.“ Amer. Math. Monthly 44,252–254, 1937.

[4] Bankoff, L. ”An Elementary Proof of the Erdos-Mordell Theorem“ Amer. Math. Monthly65, 521, 1958.

Y.3 Zeitschriften

1. Crux Mathematicorum with Mathematical Mayhem, Canadian Mathematical Society’s Pro-blem Solving Journal, ISSN 0700-0348,http://www.camel.math.ca/CMS/CRUX.

2. Mathematics Competitions, Journal of the World Federation of National Mathematics Com-petitions, ISSN 1031-7503.

3. Die√Wurzel, Zeitschrift fur Mathematik, ISSN 0232-4539,

http://www.wurzel-ev.de.

Y.4 WWW-Adressen

Hinweis: Die URLs sind tatsachlich anklickbar!

[WWW.1] http://www.cut-the-knot.com/arithmetic/antiquity.html

[WWW.2] http://forum.swarthmore.edu/dr.math/faq/faq.impossible.construct.html

[WWW.3] http://www.treasure-troves.com/math/CubeDuplication.html

[WWW.4] http://www.seanet.com/~ksbrown/kmath487.htm

[WWW.5] http://www.cut-the-knot.com/geometry.html

[WWW.6] http://www.ul.ie/~maths/papers.htm

[WWW.7] http://www.math.princeton.edu/~kkedlaya/competitions.html

[WWW.8] http://www.camel.math.ca/IMO/

[WWW.9] http://www.geocities.com/CapeCanaveral/Lab/4661/

[WWW.10] http://www.ihes.fr/~ilan/mekh-mat.ps

[WWW.11] http://www.bundeswettbewerb-mathematik.de/imo/aufgaben/main.htm

[WWW.12] http://www.unl.edu/amc/

Euklidische Konstruktionen 95

LOSUNGEN: KONSTRUKTIONEN

A.1 (Bild) Mit dem Punkt P als Mittelpunkt beschreiben wir einen Kreis, der von dergegebenen Gerade g in den Punkten A und B geschnitten wird. Nun zeichnen wir zwei wei-

AB

P

Q

g

tere Kreise um A und B mit einem Radius, der großer alsder zuvor gewahlte ist. Es sei Q einer deren Schnittpunkte.Die durch P und Q gehende Gerade steht senkrecht auf g.Bemerkung: Diese einfache Konstruktion stutzt sich auf dieTatsache, daß die in P errichtete Senkrechte zugleich Mit-telsenkrechte und Winkelhalbierende im gleichschenkligen�AQB ist.

A.2 (Bild) Wir schlagen mit dem Zirkel um beide Punkte Aund B jeweils einen Kreisbogen mit beliebigem Radius, so daßsich beide Bogen in zwei Punkten C und D schneiden. Die durchC und D mit einem Lineal gezogene Gerade ist dann die Mit-telsenkrechte der Strecke AB; sie schneidet insbesondere AB inderen Mittelpunkt M .Bemerkung: Die Konstruktion beruht auf der Eigenschaft desRhombusACBD, daß dessen Diagonalen AB und CD stets senk-recht aufeinander stehen.

A B

C

D

M

A.3 (Bild) Wir beschreiben mit dem Zirkel um den Punkt P einen Kreisbogen mitbeliebigem Radius, so daß er die Gerade g in zwei verschiedenen Punkten A und B

AB

F

P

Q

g

schneidet. Dann werden mit einem zweiten Radius (dergleich oder verschieden dem zuerst gewahlten sein kann) je-weils Kreisbogen um die Punkte A und B geschlagen. DieseBogen schneiden sich in einem Punkt Q, der bezuglich g

(bei gleichem Radius) spiegelbildlich zu P liegt. Die miteinem Lineal gezogene Gerade durch P und Q schneidetdie Gerade g im Fußpunkt oder Lotpunkt F . PF ist dasLot von P auf g; es steht immer senkrecht auf g.

Bemerkung: Die Konstruktion nutzt die Tatsache aus, daß im Drachenviereck APBQ dieDiagonalen stets senkrecht aufeinander stehen.

A.4 (Bild) Mit dem Zirkel wird um P ein Kreisbogen mit einem Radius r beschrieben,der die Gerade g in zwei Punkten schneidet; einer davon sei Punkt A. Auf g wird nun vonA die gleiche Lange r abgetragen; wir erhalten so PunktB. Mit derselben Zirkeleinstellung wird anschließend umB ein weiterer Kreisbogen geschlagen. Beide Bogen habendann außer A noch einen Punkt Q gemeinsam. Die Geradeh(P,Q) ist dann die gesuchte Parallele.Bemerkung: Das Viereck APQB ist nach obiger Konstruk-tion ein Rhombus. Die Seiten AB (auf g) und PQ (auf h)

AB

PQ

g

h

verlaufen demzufolge parallel.

96 LOSUNGEN: KONSTRUKTIONEN

A.5 (Bild) Auf beiden Schenkeln des Winkels wird von dessen Scheitelpunkt O eineStrecke beliebiger Lange mit dem Zirkel abgetragen; es entstehen die Punkte A und B,und das �AOB ist somit gleichschenklig. Errichten wir uberder Basis AB ein weiteres gleichschenkliges �ACB, wobei Cnicht mit O zusammenfallt, so liegen die Hohen und Winkel-halbierenden dieser beiden Dreiecke auf der Geraden durch Ound C. Mithin ist w(O,C) die gesuchte Winkelhalbierende. A

B C

O

w

A.6 (Bild) Um den Scheitel O des gegebenen Winkels α wird mit beliebigem Radiusein Kreis beschrieben, der die Schenkel in A und B schneidet. Nun schlagen wir mit demgleichen Radius um den auf der Geraden g gegebenen PunktO1 einen Kreis, der die Gerade

O A

B

αO1 A1

B1

in A1 schneidet; anschließend beschreiben wir mitAB um A1 einen Kreis. Einer der Schnittpunktebeider Kreise sei B1. Ziehen wir nun O1B1, so ist�B1O1A1 der verlangte Winkel α.Bemerkung: Die Richtigkeit der Konstruktion folgtaus dem Kongruenzsatz SSS (s. Aufgabe B.3) furdie beiden Dreiecke O1A1B1 und OAB.

A.7 Wir ziehen die beiden Strahlen PA und PB und tragen an diese den gegebenenWinkel α in der vorgesehenen Richtung ab (Bild a, vgl. Aufgabe A.6). Auf den freienSchenkeln liegen die Endpunkte der gedrehten Strecke A′B′ dann jeweils in den Entfer-nungen PA′ = PA bzw. PB′ = PB.

a)

AA′

B

B ′

P

α

b)

F

F ′

P

g

g ′

α

Bemerkung: Falls wir eine Gerade g zu drehen haben, konnen wir auf ihr willkurlichzwei verschiedene Punkte festlegen und diese um P drehen. Einfacher ist es jedoch, denLotfußpunkt F von P auf g nach F ′ zu drehen und in diesem die Senkrechte zu PF ′ zukonstruieren (Bild b).

A.8 (Bild) Die drei Punkte bilden stets ein �ABC und dergesuchte Kreis ist demzufolge der Umkreis k dieses Dreiecks. DerMittelpunkt O von k zerlegt das �ABC in drei gleichschenkli-ge Teildreiecke AOB, BOC, COA, in denen O jeweils auf denMittelsenkrechten der zugehorigen Basen AB, BC und CA liegt.Wir finden O, indem z. B. Punkt B mit A und C verbunden wirdund die Mittelsenkrechten dieser Strecken zum Schnitt gebrachtwerden. OA ist somit der verlangte Kreis.

A B

C

O

k

Euklidische Konstruktionen 97

A.9 (Bild) Wir beschreiben um O einen Kreis, dessen Radius doppelt so groß wie derdes gegebenen Kreises ist. Danach schlagen wir den Kreisbogen AO, der den konzentri-schen Kreis in D und E schneidet. Die Geraden OD und OE treffen dann den gegebenen

A

B

C

D

E

MO

Kreis in den Beruhrungspunkten B bzw. C. Wir ver-binden A noch mit B und C und haben die gefordertenTangenten konstruiert. — Eine andere Moglichkeit be-steht darin, uber OA den Thales-Kreis zu errichten,indem zunachst der Mittelpunkt M von OA bestimmtwird. Wir beschreiben dann den Kreis MA, der den ge-gebenen Kreis ebenfalls in den Beruhrungspunkten B

und C trifft.

A.10 Die beiden gegebenen Kreise seien k1, k2 mit den Mittelpunkten O1, O2 sowieden Radien r1, r2, wobei o. B. d. A. r1 ≥ r2 angenommen werden kann. Nun hangt esoffensichtlich von der gegenseitigen Lage beider Kreise ab, wieviel gemeinsame Tangentenwir finden. Im Fall 1, bei dem der Mittelpunktsabstand d ≡ O1O2 < r1 + r2 kleiner als dieSumme beider Radien ist (d. h., beide Kreise schneiden sich), gibt es nur zwei gemeinsameaußere Tangenten (Bild a). Angenommen wir haben die Tangenten bereits, und verschie-ben sie so parallel, daß sie durch den Mittelpunkt O2 gehen, dann sind diese offenbar dieTangenten von O2 an einen Kreis k mit dem Radius r = r1 − r2 und dem MittelpunktO1. Letztere sind somit aus den gegebenen Stucken konstruierbar (vgl. Aufgabe A.9). Die

a)

O1 O2k

k1

k2

r1 r2 r2

r2

b)

O1

O2

k

k1

k2

r1

r2

r2

gesuchten Tangenten finden wir wie beschrieben durch eine Parallelverschiebung, die vonder Zentrale O1O2 beider Kreise weggerichtet ist. Im Fall 2 mit d > r1 + r2 (d. h., zweisich nicht schneidende Kreise) gibt es außer den außeren Tangenten noch zwei gemeinsa-me innere Tangenten (Bild b), die wir hier analog zum Fall 1 mittels eines Kreises k mitdem Radius r1 + r2 und dem Mittelpunkt O1 erhalten. Schließlich gibt es noch den Fall3, bei dem sich beide Kreise genau in einem Punkt beruhren, also d = r1 + r2. Dann gibtes zwei gemeinsame außere Tangenten und lediglich eine gemeinsame innere Tangente.

98 LOSUNGEN: KONSTRUKTIONEN

A.11 (Bild) Vom Schnittpunkt S zweier Geraden tragen wirauf drei Strahlen die gegebenen Strecken m, n und p ab, de-ren Endpunkte M , N bzw. P seien. Nun ziehen wir durch zweidieser Endpunkte, etwa M und N , eine Gerade und konstruie-ren die Parallele durch den dritten Endpunkt P . Letztere trifftden vierten Strahl in einem Punkt Q. Die Lange der StreckeSQ ist dann die gesuchte vierte Proportionale.

M

NP

Q

Sm

np

q

Beweis: Da MN ‖ PQ ist, folgt aus dem 1. Strahlensatz unmittelbar

SM

SN=m

n=SP

SQ=p

qoder SQ = q =

np

m. �

A.12 (Bild) Wir zeichnen mit dem Lineal durch A eine Gerade h, die nicht mit derdurch AB gehenden Geraden g zusammenfallt. Anschließend wird eine zu h parallele Ge-rade k konstruiert, die durch B geht (vgl. Aufgabe A.4). Dann tragen wir eine beliebige,

A B

A′

B ′

B ′′

P Qg

h

k

m =n =

53

konstante Strecke von A ausgehend mmal hintereinander mit dem Zirkel aufh ab. Dies liefert den Punkt A′. Von Baus tragen wir auf k dieselbe Strecken mal in beide Richtungen ab und er-halten so die Punkte B′ (in derselbendurch g geteilten Halbebene wie A′ ge-legen) und B′′. a) Die Gerade durch A′

und B′′ schneidet dann die Strecke ABin einem Punkt P , der zwischen A und B liegt. b) Die durch A′ und B′ gehende Geradeschneidet g in einem Punkt Q außerhalb von AB. Diese Punkte erfullen die gefordertenEigenschaften.Beweis: Wegen h ‖ k folgt aus dem 2. Strahlensatz, wenn wir ihn auf die sich bei P bzw.Q schneidenden Geraden anwenden:

AP

PB=

AA′

B′′B=m

nbzw.

AQ

QB=AA′

B′B= −m

n. �

Bemerkung 1: Das in der letzten Gleichung auftretende Minuszeichen geht auf Newtonzuruck und ruhrt vom Begriff der gerichteten Strecke her: Befinden sich z. B. die PunkteA, B, Z so auf einer Geraden, daß B zwischen A und Z liegt, so haben AB und BZgleichen Richtungssinn und AB/BZ ist positiv. Dagegen ist der Quotient AZ/ZB wegenZB = −BZ negativ.Bemerkung 2: Soll etwa eine Strecke AB in einem Verhaltnis a/b (mit gegebenen Langena und b) geteilt werden, fuhrt naturlich die gleiche Konstruktion zum Ziel, nur daß aufden geschnittenen Parallelen jeweils a bzw. b abgetragen werden.

Euklidische Konstruktionen 99

A.13 obiger Gleichung

x2 = 1 − x mit den Losungen x1,2 =−1 ±√

5

2.

Die positive Wurzel ϕ ≡ 12(√

5 − 1) heißt auch Verhaltnis-

zahl des goldenen Schnitts und kann wie folgt konstruiertwerden: Wir beschreiben den Kreis k ≡ AB und bestimmenPunkt D so auf k, daß �DAB ein Rechter ist. E sei der Mit-telpunkt von AD. Der Kreis EB schneide DA in F . Dannist der gesuchte Punkt C der Schnittpunkt von AF mit AB.

Beweis: Nach dem Satz des Pythagoras ist EB =√

52

und

A BC

D

E

F

Gk

somit nach beschriebener Konstruktion AF = AC = EF − EA = EB − 12

= ϕ. �Bemerkung: BF ist zugleich die Seitenlange eines regelmaßigen Funfecks, welches k ein-beschrieben ist. Der Winkel �GAB betragt somit 1

5· 360◦ = 72◦. AF ist hingegen gerade

die Seitenlange eines einbeschriebenen regelmaßigen Zehnecks.

A.14 Das arithmetische Mittel betragt la ≡ 12(l1 + l2), das geometrische Mittel (auch

”dritte“ oder mittlere Proportionale genannt) lg ≡ √

l1l2; fur das harmonische Mittel gilt:

1

lh≡ 1

2

(1

l1+

1

l2

)oder lh =

2 l1l2l1 + l2

. (A.101)

Daraus ergeben sich folgende Moglichkeiten zur Konstruktion: a) Auf einer Geraden wer-den die beiden Strecken mit den Langen l1 ≡ AB und l2 ≡ BC hintereinander abgetragenund der Mittelpunkt von AC konstruiert (Bild a). Die Lange der Strecke AM ist danngleich la. b) Fur das geometrische Mittel gilt nach obiger Gleichung auch: l 2g = l1l2. Wennes also gelingt, ein Rechteck mit dem Flacheninhalt l1l2 in ein flachengleiches Quadrat mit

a)

A B CM

la

b)

A B C

E

l1 l2

lg

c)

A B C D

E F

GO g

hlh

dem Inhalt l 2g umzuwandeln, ist die Aufgabe gelost. Dies laßt sich mit Hilfe des Hohen-

satzes h2 = pq im rechtwinkligen Dreieck erreichen (Bild b): Uber dem Durchmesser derLange l1 + l2 wird der Thales-Kreis gezeichnet. Die Lange der Hohe BE im rechtwink-ligen �AEC ist dann nach dem Hohensatz gleich dem geometrischen Mittel lg. c) Nach(A.101) gilt die Gleichung

l1 · 2 l2 = h2 = (l1 + l2)lh, (A.102)

die auf folgende Konstruktion fuhrt (Bild c): Fur ein rechtwinkliges �AED mit denHypotenusenabschnitten l1 = AB und 2 l2 = BD wird die Kantenlange des flachengleichenQuadrates h ≡ BE ermittelt. Entsprechend der rechten Gleichung von (A.102) muß

100 LOSUNGEN: KONSTRUKTIONEN

dieses in ein flachengleiches Rechteck mit den Kantenlangen l1 + l2 und lh umgewandeltwerden. Dazu wird BE um l2 parallel nach CF verschoben und ein weiterer Thales-Kreiskonstruiert, in dem AC = l1 + l2 ein Hypotenusenabschnitt ist. Der Mittelpunkt O diesesKreises ist dabei der Schnittpunkt der Mittelsenkrechten der Sehne AF mit der Geradeng(A,C). Der zweite Hypotenusenabschnitt CG hat dann die Lange lh.

A.15 folgende drei Falle zu unterscheiden (a ≡ √q):

Fall 1 : x2 + px− q = 0 mit der Losung x = −p

2+

√(p2

)2

+ a2,

Fall 2 : x2 − px+ q = 0 mit den Losungen x1,2 =p

√(p2

)2

− a2,p

2> a,

Fall 3 : x2 − px− q = 0 mit der Losung x =p

2+

√(p2

)2

+ a2.

Durch die Struktur der Wurzeln ist die Konstruktionsidee bereits erkennbar: Wir benutzennaturlich den Satz des Pythagoras. Im Fall 1 zeichnen wir ein rechtwinkliges Dreieckmit den Katheten 1

2p und a und vermindern die Hypotenuse um 1

2p (Bild a). Im Fall 2

wird ein rechtwinkliges Dreieck aus der Hypotenuse 12p und einer Kathete a konstruiert

a)

p

a

x

/2

b)

pa

x1 x2

/2

c)

p

a

x/2

(Bild b). Die Lange der anderen Kathete wird dann einmal zu 12p addiert bzw. einmal

von 12p subtrahiert. Der Fall 3 ist dem ersten ahnlich, nur daß die Hypotenuse um 1

2p

verlangert wird (Bild c).

A.21 (Bild) Wir erinnern uns hierzu an den Peripheriewinkel-satz, nach dem alle Peripheriewinkel uber gleichem Bogen gleichgroß sind. Den Mittelpunkt O des zu diesem Bogen gehoren-den Kreises finden wir am einfachsten mit Hilfe des Sehnen-Tangentenwinkel-Satzes (vgl. Aufgabe K.4): Wir tragen an AB

nach unten den Winkel α ab und errichten auf dem freien Schenkelin A das Lot. Letzteres liefert mit der Mittelsenkrechten von AB

den Mittelpunkt O. Ebenso ist der spiegelbildlich zu AB gelege-ne Punkt O′ Mittelpunkt eines Kreisbogens, fur den �APB ≡ α

gilt. Der gesuchte geometrische Ort besteht somit aus zwei zu ABsymmetrischen Kreisbogen.

A B

O

O′

P

α

α

Geometrische Orter 101

A.22 (Bild) Angenommen, P sei ein Punkt, der die Bedingung PA/PB ≡ q = consterfullt. Wir bemuhen nun einen Satz, der besagt, daß in jedem �ABP sowohl die Winkel-halbierende eines Innenwinkels als auch die des zugehorigen Außenwinkels die gegenuber-liegende Seite im Verhaltnis der dem Winkel anliegenden Seiten teilt (zum Beweis diesesSatzes s. Aufgabe D.8). Demnach gibt es auf der Geraden g(A,B) zwei Punkte D, E, furdie gerade

q =AD

DB=

∣∣∣∣AEEB∣∣∣∣

gilt, und durch die die Winkelhalbierenden PDund PE gehen. Nun stehen die Winkelhalbieren-den zweier Nebenwinkel stets senkrecht aufeinander(vgl. Aufgabe D.7). Daraus folgt, daß die gesuch-

A BDE

P

g

ten Punkte P diejenigen sind, fur die �DPE ein Rechter ist. Diese Bedingung erfullenbekanntlich alle Punkte auf dem Thales-Kreis uber dem Durchmesser DE. Der gesuchtegeometrische Ort ist damit der sog. Kreis des Apollonius.

A.23 (Bild) Fur einen Punkt P sei AP 2 + BP 2 ≡ e2 = const erfullt. Wir ziehen dieGerade durch A, B und fallen darauf das Lot PQ. Dann gilt nach dem Satz des Pytha-goras mit den Abkurzungen AB ≡ c, BQ ≡ x und PQ ≡ y:

e2 = (x+ c)2 + y2 + x2 + y2 bzw.

e2

2= x2 + cx+

c2

2+ y2 =

(x+

c

2

)2

+ y2 +c2

4

MitO als Mittelpunkt der StreckeAB und r ≡ 12

√2e2 − c2

folgt aus der letzten Gleichung nach der Umkehrung des A BO

P

Qx

y

des Satzes des Pythagoras, daß �OQP stets ein rechtwinkliges Dreieck bei Q mit denKatheten OQ = x + 1

2c und PQ = y sowie der Hypotenuse OP = r = const ist. Da O

und r unabhangig von der konkreten Lage von P sind, ist der gesuchte geometrische Ortder Kreis Or.

A.24 Zunachst sei der Begriff erklart: Ein Isoscelizer eines Winkels mit dem ScheitelO ist eine Strecke AB mit Punkt A auf dem einen, Punkt B auf dem anderen Schenkeldes Winkels und der Eigenschaft, daß OA = OB gilt und somit �AOB gleichschenkligist. Wir behaupten nun, daß der Isoscelizer durch P die geforderte Bedingung erfullt. Umdas einzusehen, spiegeln wir den Scheitelpunkt O an AB und erhalten O′ (Bild a). DasViereck AOBO′ ist dann offensichtlich ein Rhombus mit

AO ‖ BO′ und BO ‖ AO′. (A.103)

Nehmen wir einen beliebigen Punkt P auf AB und betrachten dessen Lotfußpunkte E,F , E ′ und F ′ auf den vier Seiten des Rhombus. Die vorgegebene Summe s der Abstandeist dann

s ≡ PE + PF = PE + PF ′ = EF ′,

also gleich dem Abstand gegenuberliegender Seiten des Rhombus, und dieser ist fur jedenPunkt auf AB wegen (A.103) konstant. Die Idee besteht also darin, den Streckenzug EPF

102 LOSUNGEN: KONSTRUKTIONEN

a)

A

B

E E ′

F F ′

OO′

P

b)

A

A′B

B ′

O

P

(mit einem Knick bei P ) in den”geraden“ Streckenzug EPF ′ zu verwandeln. Damit sind

wir aber noch nicht fertig. Da zwei sich schneidende Geraden die Ebene in vier Gebietezerlegen (Bild b), gibt es insgesamt immer vier von O ausgehende Strahlenpaare, dieeinen Winkel kleiner als 180◦ einschließen, und damit vier Isoscelizer, die sich zu einemRechteck zusammenfugen und von denen einer durch P geht. Der gesuchte geometrischeOrt ist demnach ein aus vier Isoscelizern gebildetes Rechteck.

A.25 Beweis: (Bild) AB sei eine beliebige Sehne des Kreises k, AC ein Durchmesservon k. Nach Voraussetzung sei D der Mittelpunkt von AB sowie M der Mittelpunkt vonAC und somit von k. In den Dreiecken ADM und ABC gilt also

q ≡ AD

AB=AM

AC=

1

2, woraus folgt DM ‖ BC.

Da sie außerdem noch den Winkel bei A gemeinsam haben,sind sie ahnlich. Nach dem Satz des Thales ist �ABC fernerein rechtwinkliges Dreieck (da AC Durchmesser von k ist). Furjeden beliebigen Punkt B auf der Peripherie ist �ADM alsoebenfalls ein rechtwinkliges Dreieck mit dem rechten Winkel

A

B

C

D

M

k

bei D. Der geometrische Ort aller Halbierungspunkte D ist damit der Thales-Kreisuber AM . � — Bemerkung: Nach diesem Prinzip konnen wir jede beliebige ebene Figurmit Hilfe eines sog. Storchenschnabels oder Pantographen um einen Faktor q verkleinern(q < 1) oder vergroßern (q > 1).

A.26 (Bild) Die Positionen des Schmuggler- und Zoll-Bootes seien im Moment der Ortung A bzw. B, ihreGeschwindigkeiten v1 bzw. v2. Da sich bei konstantenGeschwindigkeiten die zuruckgelegten Wege proportio-nal zu den Geschwindigkeiten verhalten (s = vt), gilts1/s2 = v1/v2 ≡ q = const. Damit ist der geometrischeOrt des Rendezvous ein Kreis des Apollonius k in be- A

A′

BC

g

k

s1

s2

zug auf die Strecke AB (vgl. Aufgabe A.22). Trifft der Kreis den Fluchtweg AA′ vor demUfer g im Punkt C, sollte das Manover gelingen†.

†Falls nicht, sieht es gut fur die Schmuggler aus, denn sie konnen meist schneller laufen.

Kreiskonstruktionen 103

A.27 a) Angenommen, ein Punkt P erfulle im Falle g ∦ h die Bedingung PA/PB ≡ q =const mit A, B als Lotfußpunkte von P auf g bzw. h (Bild a). Dann ist leicht einzusehen,daß jeder Punkt auf der Geraden OP , wobei O der Schnittpunkt von g und h sei, ebenfalls

a)

AB

CD

O

P

Q

g

g ′

h

h ′

b)

A′

B ′O

P′

g

g ′′

hh ′

c)

A B

g h

diese Bedingung erfullt (außer O selbst). Denn nach dem 2. Strahlensatz gilt fur jedenPunkt Q ∈ OP

QC

PA=OQ

OP=QD

PB, bzw.

PA

PB= q =

QC

QD.

Ein derartigen Punkt P finden wir, indem wir z. B. eine Gerade g′ ‖ g im Abstand q miteiner Geraden h′ ‖ h im Abstand 1 zum Schnitt bringen.b) Selbstverstandlich konnen wir auch die Parallele g′′ auf der anderen Seite von g mit h′

schneiden und erhalten so einen Punkt P ′, fur den P ′A′/P ′B′ = q = const gilt (Bild b).Jeder Punkt auf OP ′ hat das gleiche Verhaltnis der Abstande zu den Geraden g und h.Der gesuchte geometrische Ort besteht demnach aus zwei sich in O treffenden Geraden.c) Im Falle g ‖ h wahlen wir zwei gegenuberliegende Punkte A ∈ g und B ∈ h und teilendie Strecke AB innerlich bzw. außerlich im Verhaltnis q; es entstehen die Punkte C bzw.D. Die beiden Geraden parallel zu g, die durch C und D gehen, sind hier der gesuchteOrt.

A.31 (Bild) Bilden die drei Punkte P1, P2 und P3 ein Dreieck, soist der Umkreis dieses Dreiecks die einzige Losung. Das Konzept

”geo-

metrischer Ort“ sagt uns, wie dessen Mittelpunkt zu finden ist: DieMittelsenkrechten der Strecken P1P2, P2P3 bzw. P3P1 sind die Ortegleicher Entfernungen zu den Endpunkten, also ist deren Schnittpunktgleich weit von allen drei Punkten entfernt (vgl. Aufgabe A.8). Liegen

P1

P2

P3

dagegen die drei Punkte auf einer Geraden, gibt es keinen derartigen Kreis.

A.32 Angenommen, k sei einer der gesuchten Kreise. Da k die Gerade g in einem Punktberuhren soll, liegen alle Punkte auf der Peripherie von k in derselben von g geteiltenHalbebene. Demzufolge mussen P1 und P2 in einer Halbebene liegen. Weiterhin sei ange-nommen, daß k die Gerade g in T beruhrt und die Gerade h(P1, P2) g in S schneidet (Bilda). Nach dem Sekanten-Tangentensatz (s. Aufgabe K.13) gilt dann SP1 · SP2 = ST 2, d.h., der Tangentenabschnitt ST ist die mittlere Proportionale aus SP1 und SP2 und damitkonstruierbar. Dazu nehmen wir z. B. einen beliebigen Kreis k′′, der durch P1 und P2 geht(etwa den Thales-Kreis uber P1P2 wie im Bild) und konstruieren von S aus die Tangente

104 LOSUNGEN: KONSTRUKTIONEN

a)P1

P2

R

ST T ′g

k

k ′

k ′′

b)

P1 P2

Tg

k

c)

OP1

P2g

k

an diesen. Wir erhalten die Lange SR = ST und damit Punkt T auf g. Somit ist dasBeruhrungsproblem PPG auf das Problem PPP zuruckgefuhrt (s. Aufgabe A.31). Tragenwir ST in die andere Richtung auf g ab, finden wir einen weiteren Kreis k′; es existierenalso zwei Losungen. Ist andererseits h ‖ g, laßt sich die angegebene Konstruktion nichtdurchfuhren. Dann liegt T aber auf der Mittelsenkrechten von P1P2, und k ist eindeutigbestimmt (Bild b).Schließlich gibt es noch den Fall, daß einer der gegebenen Punkte (Bild c) oder beideauf g liegen. Ist etwa P2 = T , so liegt der Mittelpunkt O des gesuchten Kreises auf derSenkrechten in P2 zu g und auf der Mittelsenkrechten von P1P2. Hier gibt es genau eineLosung. Liegen beide Punkte P1, P2 auf g, existiert keine Losung.Bemerkung: Geometrische Orte fuhren im Fall h ∦ g nicht zu einer Euklidischen Kon-struktion, da der g. O. aller derjenigen Punkte, die von einem gegebenen Punkt und einerGerade denselben Abstand haben, eine Parabel ist und diese nicht mit Zirkel und Linealgezogen werden kann.

A.33 Zunachst sei g1 ∦ g2 vorausgesetzt. Dann ist ein geometrischer Ort fur den Mit-telpunkt des gesuchten Kreises k offenbar die Winkelhalbierende w desjenigen Winkels,der von g1 und g2 gebildet wird und P im Innern einschließt. Liegt nun P nicht auf dieserWinkelhalbierenden (Bild a), gibt es aus Symmetriegrunden stets einen Punkt P ′, der als

a)

P

P′g1

g2

w

b)

Pg1

g2

w

c)

P

P′

g1

g2

w

Spiegelbild zu P bezuglich w konstruierbar ist und durch den der gesuchte Kreis ebenfallsverlauft. Damit ist diese Aufgabe auf das Problem PPG zuruckgefuhrt. Liegt P hingegenauf w (also P ≡ P ′, Bild b), so beruhrt k die Senkrechte zu w in P ; der Kreis ist alsoder Inkreis des von g1, g2 und dieser Senkrechten gebildeten Dreiecks (Problem GGG,vgl. Aufgabe A.34). Im Fall g1 ‖ g2 bleibt alles wie bisher, nur daß w(g1, g2) nunmehrdurch die Mittelparallele zu ersetzen ist (Bild c). Losungen existieren dann nur, wenn P

zwischen den Parallelen oder auf einer der Geraden liegt.

Mohr-Mascheronische Konstruktionen 105

A.34 Schneiden sich die drei gegebenen Geraden g1, g2, g3 in drei Punkten (Bild a),bilden diese ein Dreieck. Dann gibt es vier Kreise, die die (verlangerten) Seiten des Dreiecksberuhren: den Inkreis und die drei Ankreise. Die Mittelpunkte Ia, Ib, Ic der letzterenfinden wir, indem wir die Winkelhalbierenden der Außenwinkel des Dreiecks paarweise

a)

Ia

Ib

Ic

g1g2

g3

b)

g1

g2

g3

zum Schnitt bringen. Treffen sich die Geraden dagegen lediglich in zwei Punkten (Bildb), gibt es nur zwei Kreise, die die Geraden beruhren. Deren Mittelpunkte sind ebenfallsdie Schnittpunkte von Winkelhalbierenden. Im Fall, daß sich die Geraden in einem Punktschneiden oder gar parallel verlaufen, gibt es keinen Kreis, der alle drei Geraden beruhrt.

A.41 (Bild) Es ist nur das Parallelogramm ABCD zu bilden,indem wir den Punkt D als Schnittpunkt der beiden KreiseABC und CAB bestimmen. Die Gerade AD ist dann wie gefor-dert parallel zu BC.

A

B C

D

A.42 (Bild) Von einem der beiden Endpunkte aus (etwa von O) beschreiben wir denKreis OA und bestimmen daraufhin auf diesem die Punkte B, C, D, so daß

AB = BC = CD = OA.

Dann wird der Punkt D dem Punkt A diametral entgegengesetzt,und daher die Strecke AD das Doppelte von OA sein. Wenden wirdiese Konstruktion wiederholt an, so konnen wir die Strecke OAverdreifachen, vervierfachen, . . . allgemein mit n (n ganze Zahl)multiplizieren. Mit diesem Verfahren – ausgedehnt in alle Rich-tungen – laßt sich ausgehend von einer gegebenen Strecke die ge-

A

B C

DO

samte Ebene mit einem trigonalen Punktgitter (dessen Elementarzelle ein gleichseitigesDreieck ist) uberdecken.

A.43 Die Kreise AC und BC schneiden sich das zweite Mal im gesuchten Punkt.

A.44 (Bild) Es sei AB der gegebene Bogen, der zu einem Kreis mit dem MittelpunktO gehort. Mit Hilfe der Kreise AO, BO und OAB bestimmen wir die Punkte C und

106 LOSUNGEN: KONSTRUKTIONEN

A B

C D

E

F

MO

D, so daß wir die beiden ParallelogrammeABOC und ABDO erhalten, die einanderkongruent sind. Nun beschreiben wir von denMittelpunkten C und D aus zwei Kreise mitdem Radius CB = DA und betrachten einenvon ihren Schnittpunkten, etwa E. Nennenwir dann F irgendeinen der beiden Schnitt-punkte der Kreise COE und DOE, so wird derPunkt F auf dem gegebenen Kreis liegen undeinen der beiden Bogen AB halbieren.

Beweis: Tatsachlich liegen infolge der ausgefuhrten Konstruktion die Punkte C, O, D inderselben Parallelen zu AB. Da außerdem in dem Parallelogramm ABOC die DiagonaleAO den Seiten AC und BO gleich ist, ergibt sich fur den Radius CB = CE:

CB2 =BM2 + CM2 =BO2 −OM2 +(CO +OM

)2=AO2 + 2CO2, (A.104)

wobei M der Mittelpunkt von OD, also OM = 12OD = 1

2CO ist. Außerdem sind die

Winkel �COE und �DOE Rechte, so daß

CE2 = CB2 = CO2 +OE2,

und da OE = CF , folgt daraus

CB2 = CO2 + CF 2. (A.105)

Aus dem Vergleich von (A.104) und (A.105) erhalten wir CF 2 = AO2 + CO2. Da nunferner der Winkel �COF offenbar ein Rechter ist, ergibt sich CF 2 = CO2 + OF 2. Ausdieser und der vorhergehenden Gleichung folgt OA = OF , und dies besagt, daß der PunktF auf dem Kreis OA liegt. Wenn wir ferner von den gleichen Winkeln �COF und �DOFdie ebenfalls gleichen Winkel �COA und �DOB subtrahieren, so bleibt

�AOF = �BOFubrig, was beweist, daß der Punkt F den Bogen AFB halbiert. �

A.45 (Bild) Wir beschreiben, wenn O irgendein Punkt der Ebene ist, den Kreis Om

und nehmen auf diesem zwei Punkte A und B so an, daß die Sehne AB gleich p ist.Nun bringen wir den Kreis On zum Schnitt mit zwei weiteren Kreisen, die A bzw. B alsMittelpunkte und einen willkurlich gewahlten, in beiden Fallen gleichen Radius haben.Der Abstand zwischen diesen beiden Schnittpunkten C und D ist die gesuchte vierteProportionale.Beweis: Die beiden Dreiecke OAC undOBD sind nach der obigen Konstruktion kongruent(SSS), also ist

�AOC = �BODund, wenn wir zu diesen Winkeln den gemeinsamen Win-kel �AOD hinzufugen (oder ihn ggf. fortnehmen): �COD =�AOB. Daraus folgt, daß die Dreiecke AOB und COD zu-einander ahnlich sind; also besteht zwischen ihren Seiten dieProportion

OA

OC=AB

CD.

A

BC

D

O

Om

On

Mohr-Mascheronische Konstruktionen 107

Wenn wir uns erinnern, daß OA = m, OC = n und AB = p ist, so folgt daraus, daßCD = q tatsachlich die vierte Proportionale zu m, n und p ist. �Bemerkung: Wenn p > 2m ist, so ist es unmoglich, in den ersten Kreis die Sehne AB = p

einzuzeichnen. Die angegebene Konstruktion kann in diesem Fall dennoch angewendetwerden, wenn anstelle der Strecken m und n die jeweils doppelten Strecken oder, fallsdas nicht genugt, die dreifachen Strecken usw. genommen werden, da in jedem Fall dieProportion km : kn = m : n = p : q bestehen bleibt.

A.46 (Bild) Es sei O der Mittelpunkt des Kreises mitdem Radius r und AB die gegebene Gerade. Wir spie-geln zunachst O an der Geraden AB und nennen diesenPunkt P . Sind dann C und D die (als existierend vor-ausgesetzten) gemeinsamen Punkte des Kreises und derGeraden AB, so sind die Seiten des Vierecks OCPD of-fenbar samtlich einander gleich. Wir mussen daher, umC und D zu erhalten, lediglich den gegebenen Kreis mitdem Kreis Pr schneiden.

A BC D

O

P

A.47 (Bild) Ist A ein Punkt der Geraden, der vom Mittelpunkt O des Kreises ver-

AC D

P

Q

O

schieden ist, so beschreiben wir von A aus mit einembeliebigen Radius einen Kreis, der den gegebenen Kreisin zwei Punkten P und Q schneidet. Nun halbieren wirdie beiden durch die Punkte P und Q auf dem Kreisbestimmten Bogen (s. Aufgabe A.44). Die Halbierungs-punkte C und D sind dann die gesuchten Schnittpunkte.

A.48 (Bild) Es seien AB und CD die bei-den gegebenen Geraden. Wir konstruierendie Punkte C ′ und D′, die zu C bzw. Dspiegelbildlich bezuglich AB liegen, und be-stimmen auf CC ′ den Punkt E so, daß dasViereck C ′D′DE ein Parallelogramm ist (s.Aufgaben A.41 und A.43). Damit ist klar,daß durch den gemeinsamen Punkt H von

A

B

C

D

C ′

D ′

E

H

AB und CD auch die Gerade C ′D′ hindurchgeht. Nach dem Strahlensatz ergibt sich nunfolgende Proportion: CE : CC ′ = CD : CH, wobei von den darin auftretenden Streckendie ersten drei eine bekannte Lange haben. Wir konnen also mit Hilfe der Konstruktion inAufgabe A.45 die Lange CH und damit die Lage des PunktesH als einer der gemeinsamenPunkte der beiden Kreise CH und C ′

CH bestimmen.

A.49 (Bild) Wir konstruieren den Punkt C ′, der bezuglich der Geraden AB spiegelbild-lich zu C liegt, als zweiten Schnittpunkt der Kreise AC und BC (vgl. Aufgabe A.43). Derverlangte Schnittpunkt M beider Geraden ist nun offenbar der Mittelpunkt der StreckeCC ′. Wird weiterhin mit C ′′ der bezuglich C ′ symmetrische Punkt zu C bezeichnet, so

108 LOSUNGEN: KONSTRUKTIONEN

konstruieren wir diejenige Strecke, deren Lange CM = m die mittlere Proportionale zuCC ′′ und CC ′ ist, d. h. der Gleichung

CC ′′

CC ′ =CC ′

m

genugt (vgl. Aufgabe A.45). Dann be-schreiben wir den Kreis Cm und bestim-men seinen zwischen C und C ′ auf derGeraden CC ′ liegenden Punkt nach der

A

B

C C ′ C ′′DM

Losung von Aufgabe A.47. Dieser wird dann der Mittelpunkt der Strecke CC ′ sein.Bemerkung: In dieser Konstruktion wird von dem Punkt D kein Gebrauch gemacht. Dasist naturlich, da die Linie CD bereits durch die Bedingungen, durch C zu gehen und aufAB senkrecht zu stehen, festgelegt ist. Damit ist auch sofort ersichtlich, daß der gesuchtePunkt gleichzeitig der Fußpunkt des Lotes von C auf AB ist. Fur eine elegantere Methodezur Konstruktion des Mittelpunktes einer Strecke siehe Aufgabe A.50.

A.50 Konstruktion: (Bild) Es sei AB die gegebene Strecke. Wir zeichnen zunachst mitHilfe der Punkte C, D den zu A bezuglich B symmetrischen Punkt E und dann denKreis AE; auf diesem bestimmen wir die Punkte P undQ derart, daß EP = EQ = EC; anschließend beschrei-ben wir die Kreise PEC und QEC . Diese schneiden sichaußer in E noch im Punkt M , der der Mittelpunkt vonAB ist.Beweis: Mit AB = a ist nach obiger Konstruktion

AE = AP = 2a, CE = PE =√

3a, (A.106)

da CE die doppelte Hohe der beiden gegenseitig anliegen-den, gleichseitigen Dreiecke BDC und BDE ist. Ist wei-terhin F der Fußpunkt des Lotes von P auf AE, so giltnach dem Satz des Pythagoras

AP 2 = AF 2 + PF 2, PE2 = FE2 + PF 2,

A B

C D

EFM

P

Q

woraus nach Subtraktion und mit (A.106) folgt:

AP 2 − PE2 = AF 2 − FE2 =(AF + FE

) · (AF − FE)

4a2 − 3a2 = a2 = 2a(AF − FE

),

oder AF − FE = AF −MF = AM = 12a. �

Verschiedene Konstruktionen 109

A.61 (Bild) Mit g1 und g2 seien diejenigen Geraden bezeichnet, auf denen die beidenTeile der Schenkel des Winkels liegen; der von ihnen eingeschlossene Winkel sei α.

A

B

CD

g1

g2

w

Konstruktion: Wir legen einen beliebigen Punkt A auf g1 fest,fallen das Lot auf g2 (Punkt B) und errichten die Senkrechte zug1 in A; diese schneidet g2 im Punkt C. Bringen wir die Halbie-rende des Winkels �BAC mit g2 zum Schnitt, erhalten wir denPunkt D. Die Mittelsenkrechte von AD ist dann die gesuchteWinkelhalbierende w.Beweis: Die Dreiecke ABO und ACO sind rechtwinklige Drei-ecke (mit O als Scheitel des Winkels). Demzufolge ist �BAO =�ACO = 90◦ − α und daher �BAC = α. Nach Konstruktionbetragt der Winkel �DAB = 1

2α, somit ist �DAO = 90◦− 1

2α;

wegen der Innenwinkelsumme im �AOD gilt ebenfalls �ADO = 90◦ − 12α. �AOD hat

also gleiche Basiswinkel bezuglich der Seite AD und ist demnach gleichschenklig. DieMittelsenkrechte von AD in diesem gleichschenkligen Dreieck ist gleichzeitig Winkelhal-bierende von �AOD und somit diejenige von g1 und g2. �

A.62 (Bild) AOB sei der Kreissektor, M der Mittelpunkt des Bogens AB. Da dereinzubeschreibende Kreis k die Radien OA und OB beruhrt, muß sein Mittelpunkt Iauf der Winkelhalbierenden OM liegen. Demzufolge beruhrtk den Bogen AB in M . Ohne an dem Beruhrungsproblem et-was zu andern, konnen wir nun den Sektor AOB durch dasDreieck COD ersetzen, wobei C und D die Schnittpunkte derverlangerten Radien mit der durch M gehenden, zu OM senk-rechten Geraden sind. I ist somit der Inkreismittelpunkt von�COD, d. h. Schnittpunkt von OM mit der Winkelhalbieren-den von �OCM bzw. �ODM . — Bemerkung: Wir haben hier

A

B

C

D

IM

O k

das Beruhrungsproblem des Apollonius (GGK) vorliegen, welches jedoch aufgrund derSymmetrie in das einfachere Problem (GGG) ubergeht (vgl. Abschnitt A.3).

A.63 (Bild) Der Mittelpunkt E des gesuchten Kreises und sein Beruhrungspunkt Dmit dem Halbkreis um C liegen offenbar auf der Symmetrieachse der Figur, die gleich derMittelsenkrechten von AB ist. Bezeichnen wir ferner die Mitte von AC mit F , so gehtFE durch den Beruhrungspunkt der beiden Kreise mit den Mittelpunkten F und E. Nunsetzen wir AF ≡ r und ED ≡ x. In dem rechtwinkligen �EFC gilt

(r + x)2 = r2 + (2r − x)2 oder x =2

3r.

Damit genugt es, r = 14AB im Verhaltnis 2 : 1 zu tei-

len (vgl. Aufgabe A.12) und das so konstruierte x aufDC in Richtung C abzutragen, womit Mittelpunkt undRadius des gesuchten Kreises gefunden sind. A BC

D

E

F

r

x

110 LOSUNGEN: KONSTRUKTIONEN

A.64 Analysis: (Bild) Es sei EF die gesuchte Halbierungslinie. Wir verlangern AD undBC bis zum Schnittpunkt G und nennen die Strecken GD ≡ d, GE ≡ x und GA ≡ a.Die Dreiecke GDC, GEF und GAB sind einander ahnlich; ihre Flacheninhalte verhal-ten sich daher wie die Quadrate ihrer Abmessungen:[GDC] : [GEF ] : [GAB] = d2 : x2 : a2. Folglich verhaltsich auch

[GAB] − [GEF ]

[GEF ] − [GDC]=

[ABFE]

[EFCD]=a2 − x2

x2 − d2.

Da EF das Trapez halbieren soll, folgt daraus

A B

CD

E F

G

H

K

k

a2 − x2 = x2 − d2 oder x =

√a2 + d2

2.

Konstruktion: Wir errichten das Lot AH auf AG der Lange GD. Die VerbindungsstreckeHG hat dann die Lange HG =

√a2 + d2. Nun beschreiben wir uber HG als Durchmesser

den Halbkreis und bringen ihn mit dem Mittellot von HG zum Schnitt; beide Linientreffen sich in K. �HKG ist dann gleichschenklig rechtwinklig, und nach dem Satz desPythagoras ist GK =

√a2 + d2/

√2 = x. Wir bringen den Kreis GK zum Schnitt mit

AD in E und ziehen durch E die Parallele zu AB, welche BC in F schneidet. EF istsomit die verlangte Gerade.

A.65 Dieses Problem ist zugegebenermaßen ziemlich hart. Angenommen, wir haben einQuadrat, daß den Bedingungen der Aufgabe genugt (Bild a). Nehmen wir z. B. die StreckeAC und drehen sie um 90◦ in die Lage AC ′ und verschieben diese anschließend so parallel,daß A in B und C ′ in C ′′ zu liegen kommt. Nun ziehen wir die Gerade g(D,C ′′), die Par-allele dazu durch B sowie die Senkrechten zu g durch A und C. Die vier Schnittpunkte

C ′ C ′′

EF

a)

A

B

C

D

b)

AB

C

D

dieser Geraden bilden dann das gesuchte Quadrat. Um einzusehen, daß dem so ist, be-trachten wir die rechtwinkligen Dreiecke ACE und AC ′F , die offenbar kongruent sind, dasie sich durch eine Drehverschiebung (d. i. die oben beschriebene Drehung mit anschlie-ßender Parallelverschiebung) ineinander uberfuhren lassen. Der Abstand der konstruiertenGeradenpaare ist dann EA = FC.Da wir in der Auswahl der Anfangsstrecke und in der Richtung der Parallelverschiebungfrei sind, gibt es mehrere Losungsmoglichkeiten (Bild b), bei denen naturlich die gegebe-nen Punkte haufig nur auf den Verlangerungen der Seiten des Quadrats liegen.

Verschiedene Konstruktionen 111

A.66 (Bild) Ein geometrischer Ort fur den Mittelpunkt O des gesuchten Kreises istdie Mittelsenkrechte von AB. Im Fall, daß die durch A, B gehende Gerade parallel zu gverlauft (Bild a), errichten wir uber der Basis AB ein gleichschenkliges Dreieck, dessenSpitze C auf g liegt. C ist dann offenbar Mittelpunkt aller von g herausgeschnittenen

a)

A B

CEF

O

g

b)

A BD

E

F

G H

K

P

g

Sehnen, so daß wir die Endpunkte E und F der Sehne einfach dadurch finden, indem wir12s in beide Richtungen von C aus auf g abtragen. Damit liegt der Kreis fest; ein zweiter

geometrischer Ort fur O ist etwa die Mittelsenkrechte von BE.Schneiden sich dagegen AB und g in einem Punkt P , so liefert der Sekantensatz (s.Aufgabe K.11) die Gleichung PA ·PB = PE ·PF , die sich mit PA ≡ a, PB ≡ b, PE ≡ xund PF = x+ s als quadratische Gleichung fur die unbekannte Lange x herausstellt:

ab = x(x+ s) bzw. x2 + sx− ab = 0. (A.107)

Als Losung von (A.107) kommt wegen x > 0 nur

x = −s

2+

√(s2

)2

+ ab = −s

2+

√(s2

)2

+ (√ab)2 (A.108)

in Frage. — Konstruktion: Wir konstruieren den Schnittpunkt P und tragen die Langea auf dem Strahl BA ab; der Endpunkt sei D. Der Thales-Kreis uber PD schneidedas Lot durch B im Punkt G. Nach dem Hohensatz im rechtwinkligen Dreieck PGD giltBG =

√ab. (A.108) verlangt nun, ein weiteres rechtwinkliges Dreieck aus den Katheten

BG =√ab und HG = 1

2s zu errichten und von dessen Hypotenuse HK = HG = 1

2s

zu subtrahieren. Die Lange von BK ist somit gleich x = PE. Die Endpunkte E und F

finden wir schließlich durch Abtragen von x und x+ s auf g.

A.67 (Bild) ABC sei das gleichseitige Dreieck mit denEckpunkten auf den Geraden g, h und j. Da �ABC langsder Parallelen beliebig verschoben werden kann, durfen wireine Ecke, etwa A auf g, nach Gutdunken auswahlen. Drehenwir nun �ABC um A um 60◦ und nimmt bei dieser DrehungEcke B die Gerade h mit, so gelangt AB in die Lage AC undh nach h′. Eckpunkt C wird demnach durch j und die um60◦ gedrehte Gerade h bestimmt. Der dritte Eckpunkt Bliegt auf h und zugleich dem Kreis AC .

A

B

Cj

g

h

h ′

112 LOSUNGEN: KONSTRUKTIONEN

A.68 Im Fall AB ∦ g konnen wir AB bis zum Schnitt mit g in Punkt S verlangern (Bilda). CB ist dann Winkelhalbierende im Dreieck ACS. Erinnern wir uns an den guten,

a) A

B

C C ′S g

k

b)

A B

C C ′D D ′ g

alten Satz, daß die Winkelhalbierende in einem Dreieck die gegenuberliegende Seite imVerhaltnis der anliegenden Seiten teilt (s. Aufgabe D.8), so folgt

CA

CS=AB

BS≡ q = const.

Dies ruft sofort den Kreis des Apollonius auf den Plan, da er bekanntlich der geometri-sche Ort aller Punkte ist, deren Quotient der Abstande CA/CS konstant ist (vgl. AufgabeA.22). Wir haben den inneren Teilungspunkt B von AS, somit bereitet es keine Schwie-rigkeiten, auch den außeren Teilungspunkt B′ (im Bild nicht gezeigt) nach Aufgabe A.12

zu finden. Der Thales-Kreis k uber BB′ ist damit der Kreis des Apollonius, dessenSchnittpunkte C bzw. C ′ mit g mithin die geforderten Eigenschaften haben.Im Fall AB ‖ g konstruieren wir einfach die Rhomben ABDC bzw. ABD′C ′, indem wirdie Kreise AB bzw. BA mit g zum Schnitt bringen; dies liefert die Punkte C bzw. D′ (Bildb). Die Mittelsenkrechten von CB und AD′ schneiden g dann in den Punkten D bzw. C ′.Die geforderte Eigenschaft ergibt sich aus der Tatsache, daß die Diagonalen CB und C ′Bgleichzeitig Winkelhalbierende sind.

A.69 (Bild) Mitunter sind Probleme einfacher zu losen, wenn sie”umgedreht“ werden.

Nehmen wir hier einmal an, wir haben bereits das gesuchte gleichseitige Dreieck ABCund suchen statt dessen den Mittelpunkt O der drei konzentrischen Kreise, deren Radienwir o. B. d. A. mit 0 < r1 < r2 < r3 bezeichnen. Von O wissen wir, daß seine Entfernun-gen von den Eckpunkten die gegebenen Langen OA = r1, OB = r2 bzw. OC = r3 sind.Er liegt also auf einem Kreis des Apollonius, der zur Strecke AB gehort und fur denOA/OB = r1/r2 gilt (vgl. Aufgabe A.22). Ebenso liegt O auf einem Kreis des Apollo-nius uber AC mit OA/OC = r1/r3. Somit ware O als Schnittpunkt dieser beiden Kreisesofort konstruierbar, wenn wir �ABC in der richtigen Große vorzuliegen hatten. Diesist jedoch keine unuberwindbare Hurde. Da die obigen Streckenverhaltnisse unverandertbleiben, wenn wir die drei Radien mit einer beliebigen reellen Zahl λ strecken oder stau-chen (d. i. eine zentrische Streckung), erhalten wir stets denselben Mittelpunkt O.Konstruktion: Wir zeichnen ein beliebiges gleichseitiges Dreieck A′B′C ′ und konstruierenO als Schnittpunkt zweier Apollonius-Kreise, indem wir beispielsweise A′B′ und A′C ′

innerlich und außerlich im Verhaltnis r1/r2 bzw. r1/r3 teilen und uber diesen Teilungs-punkten jeweils den Thales-Kreis schlagen. Das gesuchte Dreieck erhalten wir schließlichdurch die Schnittpunkte der Strahlen OA′, OB′ und OC ′ mit den drei Kreisen.

Verschiedene Konstruktionen 113

a)

A

A′

B

B ′

C

C ′

O

b)

A

B

C

O

Bemerkung: Da es i. a. zwei Schnittpunkte der Bestimmungskreise fur O gibt, existierenauch zwei verschiedene Losungen (Bild a und b).

A.70 (Bild) Konstruktion: B′ sei das Spiegelbild vom Zielpunkt B. Wir ziehen denKreisbogen k um B′ mit dem Radius B′A; dieser schneide die Gerade g in einem PunktC. Die Mittelsenkrechte von AC schneidet g im gesuchten Punkt X. — Beweis, daß wiegefordert �AXC = 2�BXD gilt, wobei D der Mittelpunkt von BB′ ist: Die Mittelsenk-rechte der Sehne AC, auf der nach Konstruktion X liegt, geht durch den Mittelpunkt B′

des Kreises. Demzufolge sind sowohl �AB′C als auch�AXC gleichschenklige Dreiecke uber der Basis AC,deren Winkelhalbierenden EX bzw. EB′ ubereinan-der fallen. Gleichzeitig sind aber �CXE = �DXB′

kongruente Scheitelwinkel, so daß tatsachlich

A

B

B ′

C D

E

X g

k

�BXD = �DXB′ = �CXE =1

2�AXC

folgt. �Bemerkung: Der zweite Schnittpunkt C ′ des Kreises k mit der Geraden g kommt nur dannin Frage, wenn er zwischen den Lotfußpunkten von A, B auf g liegt (ansonsten wurdeder Strahl auch in Langsrichtung zuruckreflektiert werden, was physikalisch gesehen demImpulserhaltungssatz widersprache).

A.71 (Bild) Ist ABCD das gesuchte Quadrat, so liegen o. B. d. A. die Punkte B und Dauf g, die beiden anderen gegenuberliegenden Punkte A und C auf k1 bzw. k2. Da in einem

A

BC

D

S

g k1k2

k2′Quadrat die Diagonalen AC und BD senkrecht aufeinander ste-hen und die Eckpunkte von deren Schnittpunkt S jeweils gleicheAbstande haben, ist klar, daß A und C symmetrisch zur Geradeng sind. Wir benutzen daher g als Symmetrieachse und spiegelnden Kreis k2 an ihr; es entsteht der Kreis k′2. Die Schnittpunktevon k1 und k′2 erfullen dann genau diese Symmetriebedingung.Die anderen Punkte des Quadrats ergeben sich anschließend aufeinfache Weise.

114 LOSUNGEN: KONSTRUKTIONEN

A.72 (Bild) Konstruktion: Wir ziehen einen Kreisbogen k ≡ As und spiegeln B an g,wodurch wir B′ erhalten. Ein beliebiger, durch B und B′ gehender Kreis k′ schneide kin den Punkten C und C ′. Der Schnittpunkt der Geraden BB′ und CC ′ sei P . Legenwir von P aus die Tangenten an k, so erhalten wir die Punkte F bzw. F ′ (im Bild nichtdargestellt). Die Geraden AF bzw. AF ′ schneiden g in den gesuchten Punkten S bzw. S ′.

A

B

B ′

C

C ′

FP

S Tg

k

k ′

Beweis: Es ist zu zeigen, daß tatsachlich AS+SB = s gilt. PF ist Tangentenabschnitt ank, dessen Quadrat nach dem Sekanten-Tangentensatz gleich dem Produkt der Sekanten-abschnitte PC · PC ′ ist. Letztere Strecken sind zugleich Sekantenabschnitte in k′; derenProdukt ist damit nach dem Sekantensatz ebenfalls gleich PB · PB′. Mit T als Schnitt-punkt von BB′ und g erhalten wir somit folgende Gleichungskette:

PF 2 = PC · PC ′ = PB · PB′ = (PT + TB) · (PT − TB) = PT 2 − TB2

(wegen TB = TB′). Die Dreiecke SFP und STP sind nun nach obiger Konstruktion beiF bzw. T rechtwinklig und es gilt weiter:

SF 2 = SP 2 − PF 2 = (ST 2 + PT 2) − PF 2 = ST 2 + PT 2 − PT 2 + TB2 = SB2,

also SF = SB = SB′, d. h., B, B′ und F liegen auf einem Kreis um S. Da F auf demKreis k liegt, ist schließlich s = AF = AS + SF = AS + SB. �Bemerkung: Werden A und B als Brennpunkte einer Ellipse aufgefaßt, fur welche s dieSumme der Leitstrahlen bildet, haben wir hiermit die Aufgabe gelost, die Schnittpunkteeiner beliebig gegebenen Geraden mit einer gegebenen Ellipse zu bestimmen.

Die Grundaufgaben 115

LOSUNGEN: DREIECKSKONSTRUKTIONEN

B.1 (Bild) Wir tragen von dem Scheitel C des Winkels γ aus auf dem einen Schenkel dieStrecke b bis zum Punkt A ab, auf dem anderen a bis B und verbinden A mit B; dann ent-spricht �ABC den Forderungen der Aufgabe. Wir konnten noch ein zweites Dreieck erhal-ten, wenn wir umgekehrt auf dem ersten Schenkel a bisA′, auf dem zweiten b bis B′ abtragen und dann A′ mitB′ verbunden hatten. �A′B′C entspricht ebenso denForderungen der Aufgabe; es sind aber die Ecken A′, B′

symmetrische Punkte zu den Ecken B, A des �ABCbezuglich der Winkelhalbierenden von �ACB ≡ γ. Esist daher �A′B′C ∼= �ABC, und �A′B′C liefert keineandere Losung der Aufgabe. — Bemerkung: Die Auf-gabe ist nur fur γ = 180◦ nicht losbar.

A B

C

A′

B ′

abγ

B.2 (Bild) Es gibt bei dieser Konstruktion zwei Moglichkeiten. Im ersten Fall liegtder fehlende Winkel der gegebenen Seite gegenuber (z. B. c, α, β gegeben), im zwei-

A B

C

C ′

cα β

γ

ten Fall der Seite an (z. B. c, α, γ). Fall 1 : Wir zeichnendie Strecke AB ≡ c und tragen an diese im Punkt A denWinkel α und im Punkt B nach der gleichen Seite hin (hierim Bild also nach oben) den Winkel β an und nennen denSchnittpunkt ihrer freien Schenkel C. Damit ist das gefor-derte Dreieck konstruiert. Wir konnen die Winkel auch zuranderen Seite von AB abtragen und erhalten so einen PunktC ′, der dann bezuglich AB symmetrisch zu C liegt. BeideDreiecke ABC und ABC ′ sind jedoch kongruent. Im Fall 2wird aus den beiden gegebenen Winkeln zunachst der dritteWinkel bestimmt und anschließend wie in Fall 1 verfahren.

Es ergibt sich wieder nur ein Dreieck. — Bemerkung: Da die Summe der Innenwinkeleines Dreiecks gleich zwei Rechten ist, wird die Aufgabe unlosbar, wenn schon die beidengegebenen Winkel zusammen zwei Rechte betragen oder gar uberschreiten.

B.3 (Bild) Wir ziehen die Strecke AB ≡ c und beschrei-ben die Kreise Ab und Ba. Die Schnittpunkte dieser Kreiseseien C und C ′. Verbinden wir C und C ′ mit A bzw. B,entstehen die Dreiecke ABC und ABC ′, die den Forderun-gen der Aufgabe genugen. Da C bzw. C ′ jeweils gleich weitvon A und B entfernt sind, muß AB Symmetrieachse fur die-se Punkte sein. �ABC und �ABC ′ sind daher kongruent,und die Aufgabe hat nur eine Losung. — Bemerkung: DieAufgabe ist nur losbar, wenn die Summe der Langen zweiergegebener Seiten großer ist als die Lange der dritten Seite(Dreiecksungleichung, vgl. Aufgabe U.3).

A B

C

C ′

ab

c

B.4 Wir tragen auf dem einen Schenkel des gegebenen Winkels α vom Scheitel aus dieStrecke AB ≡ c ab und beschreiben dann mit dem Radius a um B einen Kreis. Dieser

116 LOSUNGEN: DREIECKSKONSTRUKTIONEN

Kreis schneidet den freien Schenkel des Winkels α nur einmal, wenn a > c ist (Bild a).In diesem Fall ist also das Dreieck durch die gegebenen Stucke eindeutig bestimmt, und

a)

A B

C

a > c

b)

A B

C

a = c

c)

A B

C

C ′ a < c

c

h

α

wir finden einen neuen Kongruenzsatz. Ist insbesondere a = c, so schneidet der KreisBa den freien Schenkel in A und noch in einem zweiten Punkt C. Das Dreieck wirdgleichschenklig, und die Kongruenz folgt schon aus dem zweiten Kongruenzsatz WSW(Bild b). Ist aber a < c, so muß, solange a großer als das von B auf den freien Schenkelgefallte Lot h ist, Ba den Schenkel zweimal schneiden, und es entstehen zwei voneinanderverschiedene Dreiecke (Bild c). Wird a gleich dem Lot h, so folgt wieder die Kongruenznach dem zweiten Kongruenzsatz WSW, da dann der c gegenuberliegende Winkel gleicheinem Rechten ist. Ist schließlich a kleiner als h, so gibt es uberhaupt keine Losung.

B.11 Analysis: In dem beliebig gezeichneten �ABC werden zunachst die drei als gege-ben zu betrachtenden Stucke c, b und hc kraftig hervorgehoben (Bild a). Wir bemerken,daß das bei F rechtwinklige Teildreieck AFC aus hc und b konstruierbar ist (dieses istdas vorn erwahnte Hilfsdreieck). Von dem verlangten Dreieck kennen wir nun die EckenA und C sowie die Richtung der Seite AB. Den dritten Eckpunkt B finden wir, indemwir c auf dem Strahl AF abtragen.

a)

A B

C

F

b

c

hc

b)

A BB ′

C

F

b

c c g

hc

c)

A BB ′

C

b = hc

c c

Konstruktion: Wir zeichnen eine Gerade g und errichten auf dieser in einem beliebigenPunkt F die Senkrechte CF ≡ hc (Bild b). Anschließend beschreiben wir Cb, der g in A

schneidet, und Ac, der die Gerade in B trifft. Nun verbinden wir C mit A und B.Behauptung: �ABC ist das verlangte, d. h. es ist 1. AB = c; 2. AC = b; 3. CF = hc undCF ⊥ AB.Beweis: 1. AB = c als Radius des um A beschriebenen Kreises; 2. AC = b als Radius desum C beschriebenen Kreises; 3. CF = hc und CF ⊥ AB nach obiger Konstruktion. �Determination: Die Aufgabe ist stets losbar, wenn b > hc ist. Der Kreis Cb schneidet dieGerade g eigentlich in zwei Punkten A und A′. Diese Punkte liegen aber symmetrischzu CF , und die beiden dadurch entstehenden Dreiecke CFA und CFA′ sind kongruent;wir brauchen also nur einen Schnittpunkt zu berucksichtigen. Der Kreis Ac schneidet gebenfalls in zwei Punkten B und B′ (Bild b). Die Dreiecke ABC und AB′C sind hier

Auffinden von Hilfsdreiecken 117

jedoch voneinander verschieden; das eine Dreieck besitzt bei A einen spitzen Winkel, dasandere bei A einen stumpfen Winkel. Es gibt also im Fall b > hc zwei Losungen.Die Aufgabe ist auch losbar, wenn b = hc ist (Bild c). Dann fallt aber Punkt A mit Fzusammen, und die Punkte B und B′ liegen symmetrisch zu CA. Beide Dreiecke ABCund AB′C sind daher kongruent, und wir brauchen nur den einen Schnittpunkt B zuberucksichtigen. Es gibt also fur b = hc nur eine Losung, und zwar ein rechtwinkligesDreieck. Ist dagegen b < hc, so gibt es keine Losung, da dann der Kreis Cb die Gerade gnicht schneidet.

B.12 (Bild) Analysis: a) Angenommen, �ABC entspreche den Bedingungen der Auf-gabe. Dann gibt es genau einen Punkt P , fur den ABPC ein Parallelogramm ist, und dieDiagonalen AP und BC werden durch ihren Schnittpunkt D halbiert. Ferner sind �DBPund �DCA Rechte, also liegt nach der Umkehrung des Satzes des Thales B auf einemHalbkreis uber DP . Ist weiter G der Schnittpunkt der Seitenhalbierenden von �ABC, sogilt AG = 2

3ma und BG = 2

3mb (vgl. Aufgabe D.10). b) Daher genugt ein Dreieck ABC

nur dann den Bedingungen der Aufgabe, wenn es durch folgende Konstruktion erhaltenwerden kann:

• Wir konstruieren auf einer Geraden die Punkte A, G, D, P in dieser Anordnung so,daß AD = DP = ma und AG = 2

3ma gilt.

• Wir schlagen einen Halbkreis k1 uber DP .

• Wir schlagen den Kreis k2 um G mit 23mb. Schneidet er k1 in einem Punkt, so sei

dieser mit B bezeichnet.

• Wir verlangern die Strecke BD uber D hinaus um ihre eigene Lange und erhaltendamit den Punkt C.

c) Beweis, daß jedes so konstruierte Dreieck ABC

den Bedingungen der Aufgabe genugt: Nach Kon-struktion ist AD Seitenhalbierende der Lange ma in�ABC. Wegen AG = 2

3ma ist G der Schwerpunkt

von �ABC, also enthalt die Verlangerung von BGdie Seitenhalbierende durch B. Wegen BG = 2

3mb

A B

C

DG

P

k1

k2

hat sie die Lange mb. Schließlich ist nach dem Satz des Thales �PBD = 90◦ und wegen�ADC ∼= �PDB daher auch �ACB = 90◦.d) Die ersten drei Konstruktionsschritte ergeben bis auf Kongruenz eindeutig A, G, D,P , k1, k2. Dabei haben k1, k2 die Radien r1 = 1

2ma = 3 cm bzw. r2 = 2

3mb = 16

3cm

und den Mittelpunktsabstand d = 13ma + 1

2ma = 5

6ma = 5 cm. Diese Langen erfullen die

Dreiecksungleichungen (vgl. Losung zu Aufgabe U.3)

r2 − r1 =7

3cm < d =

15

3cm < r2 + r1 =

25

3cm.

Daher existiert genau ein Schnittpunkt B von k1 mit k2, und hiernach ist auch C durchden letzten Konstruktionsschritt bestimmt. Somit existiert �ABC mit den gefordertenEigenschaften und ist durch die gegebenen Langen bis auf Kongruenz eindeutig bestimmt.

118 LOSUNGEN: DREIECKSKONSTRUKTIONEN

B.13 Analysis: (Bild) k sei der Inkreis des gesuchten �ABC; sein Mittelpunkt I liegtauf der Winkelhalbierenden CZ. Weiterhin seien D, E und F die Beruhrungspunkte vonk mit den Seiten BC, CA und AB.Konstruktion: Das rechtwinklige HilfsdreieckIEC wird nach dem Kongruenzsatz WSWaus den Stucken �IEC = 90◦, EI = rund �CIE = 90◦ − γ/2 konstruiert. Um Ials Mittelpunkt wird mit dem Radius r derInkreis k gezeichnet und vom Punkt C ausdie zweite Tangente an k konstruiert; derenBeruhrungspunkt sei D. Nun kann auf der

A B

C

D

E

F

F ′

I

Z

k

Verlangerung von CI uber I hinaus die Lange CZ = wc abgetragen werden. Dies liefertden Punkt Z. Anschließend wird von Z aus die Tangente an k konstruiert. Hierfur gibt eszwei Moglichkeiten (die Beruhrungspunkte F und F ′), falls Z außerhalb von k liegt. DerSchnittpunkt dieser Tangente mit der Geraden durch CE liefert den Eckpunkt A, der mitder Geraden durch CD den Eckpunkt B. Im allgemeinen lassen sich also zwei Dreieckeaus den gegebenen Großen finden, die jedoch kongruent sind.

B.14 Analysis: (Bild) Die Seite AB ist eine Sehne des Umkreises k mit dessen Mit-telpunkt O. Nach dem Peripherie-Zentriwinkel-Satz ist der Zentriwinkel �AOB doppeltso groß wie der Peripheriewinkel �ACB = γ. Daher ergibt sich folgende Konstruktion:Das gleichschenklige Hilfsdreieck ABO laßt sich nach dem Kongruenzsatz SWS aus denSchenkeln AO = BO = R und dem eingeschlossenen Winkel 2γ konstruieren. Um Owird mit dem Radius R der Umkreis k gezeichnet, auf dem der dritte Eckpunkt C desgesuchten Dreiecks liegt. E sei die Mitte der Seite AC, d. h.EB = mb ist die gegebene Lange der Seitenhalbierenden mb.Den Punkt E finden wir als Schnittpunkt zweier geometri-scher Orter, der beiden Kreise k1 und k2. Der erste Kreis k1

hat die Strecke AO als Durchmesser, kann also unmittelbarbeschrieben werden. Er ist der geometrische Ort derjenigenPunkte, die alle von A ausgehenden Sehnen in k halbieren(vgl. Aufgabe A.25). Der zweite Kreis k2 hat den Radiusmb mit B als Mittelpunkt. Haben wir E auf diese Weise ge-funden, so ist der Schnittpunkt der Geraden AE mit dem

A B

C

E

E ′

O

k

k1k2

R2

γ

γ

Umkreis k der dritte Eckpunkt C.Determination: Auch hier erhalten wir im allgemeinen zwei Schnittpunkte E und E ′ derbeiden Kreise k1 und k2. Liegen jedoch E ′ und O in unterschiedlichen durch g(A,B) be-stimmten Halbebenen, so ware der daraus folgende Peripheriewinkel γ′ = 180◦ − γ �= γ(vgl. Aufgabe K.2, Fall 4). E ′ erfullt also nicht die Voraussetzungen. Ergeben sich dage-gen zwei Schnittpunkte, die zusammen mit O in der gleichen Halbebene bezuglich g(A,B)liegen, erhalten wir i. a. zwei verschiedene Dreiecke als Losungen.

Auffinden von Hilfsdreiecken 119

B.15 Analysis: (Bild) �ABC sei das gesuchte Dreieck, CD = mc die Seitenhalbierende,CE = wc die Winkelhalbierende und CF = hc die Hohe, wobei letztere senkrecht auf derGeraden g(A,B) steht. Ferner schneide die Mittelsenkrechte von AB die Verlangerung

A B

C

DEF

K

O

g

h

k

der Winkelhalbierenden CE in einem Punkt K, der aufdem Umkreis k von �ABC liegt (vgl. Aufgabe D.3).Konstruktion: Vom rechtwinkligen Hilfsdreieck CFDsind die Kathete hc und die Hypotenuse mc gegeben.Es laßt sich also aus den gegebenen Stucken unmittel-bar nach Kongruenzsatz SSW konstruieren. Den PunktE finden wir als Schnittpunkt eines Kreises mit dem Ra-dius wc um C mit der Geraden g, die bereits durch Fund D bestimmt ist. Nun ergibt sich der Punkt K alsSchnittpunkt der in D senkrecht zu g errichteten Gera-den h mit der Verlangerung von CE uber E hinaus. DerMittelpunkt O des Umkreises muß nun seinerseits auf

der Geraden h (die zugleich Mittelsenkrechte der noch unbekannten Seite AB ist) undauf der Mittelsenkrechten der Sehne CK liegen; er ist somit ihr gemeinsamer Schnitt-punkt. Die beiden Eckpunkte A und B erhalten wir schließlich als Schnittpunkte von gmit dem Umkreis OC .Bemerkung: Im allgemeinen gibt es noch einen zweiten Schnittpunkt E ′, der von C denAbstand wc hat. Dieser liegt jedoch nicht zwischen dem Hohenfußpunkt F und der Sei-tenmitte D und fuhrt somit nicht zum geforderten Dreieck.

B.16 (Bild) Dreieckskonstruktionen, bei denen die Lange einer Seitenhalbierenden ge-geben ist, lassen sich oft dadurch losen, indem das Dreieck ABC am Mittelpunkt derhalbierten Seite um 180◦ gedreht wird. Wir erreichen dies, indem die Strecke CD = mc

uber den Seitenmittelpunkt D hinaus bis zum Punkt E so verlangert wird, daß CE = 2mc

gilt. Das entstehende Viereck ACBE ist dann ein Parallelogramm. Wenn F der Fußpunktdes Lotes von E auf die Gerade CA ist, dann ist wegen BE ‖ CF die Lange von EF = hb.

A B

C

D

E

F

g

k

hb

mc

mc

γ

Konstruktion: Das �CEF ist aus den gegebenen Stuckenkonstruierbar: Wir zeichnen die Strecke CE der Lange 2mc

mit dem Mittelpunkt D und schlagen um D den Thales-Kreis k. Der Hohenfußpunkt F ist dann der Schnittpunktvon k mit einem Kreis vom Radius hb um E. Nun kann dieGerade CF gezogen werden. In C wird an g der Winkel γso abgetragen, daß CE zwischen beiden Schenkeln zu liegenkommt. Der Eckpunkt B ist dann der Schnittpunkt des frei-en Schenkels mit der Parallelen von g durch E. Schließlichfinden wir A als Schnittpunkt der Geraden BD mit g.

120 LOSUNGEN: DREIECKSKONSTRUKTIONEN

B.17 (Bild) Eine weitere Methode bei der Konstruktion von Dreiecken mit gegebe-ner Lange von Seitenhalbierenden besteht darin, das gesuchte �ABC so zu erweitern,daß der Strahlensatz angewendet werden kann. Hier sei F der Mittelpunkt von AB, so

A B

C

D

F

ab

b

mc

mc2

daß CB = a, CA = b und CF = mc die gegebenenStucke sind. Wir verlangern die Seite AC um ihre Langeb uber C hinaus und erhalten Punkt D. Da nun offen-bar

AC

AD=AF

AB=

1

2

gilt, folgt aus der Umkehrung des 1. Strahlensatzes: CF ‖ DB und ferner DB = 2mc.Damit ist fur diese Aufgabe ein unmittelbar herstellbares Hilfsdreieck gefunden (�DCBaus a, b und 2mc). Eckpunkt A erhalten wir durch Verlangerung von DC uber C hinaus.

B.18 (Bild) Die Mittelpunkte der Seiten BC, CA seien D bzw. E; Punkt F sei derLotfußpunkt von C auf AB. Ziehen wir die Parallele zu CF durch D, schneidet die-se AB in K, und es gilt nach dem zweiten Strahlensatz:BD : BC = DK : CF = 1 : 2. Somit ist �ADK ein ausAD = ma, DK = 1

2hc und �AKD = 90◦ konstruierbares

Hilfsdreieck. Ferner teilt der Schwerpunkt G die Seitenhal-bierende AD im Verhaltnis 1 : 2 und ist damit ebenfallsbekannt. Eckpunkt B finden wir nun auf der Verlangerungvon AK im Abstand 2

3mb von G; den dritten Eckpunkt C

durch Verdopplung von BD.A B

C

DE

F

G

K

B.19 (Bild) G sei wie vereinbart der Schwerpunkt von�ABC. Mit Hilfe des bekannten Satzes uber die Teilungs-verhaltnisse auf den Seitenhalbierenden durch G (vgl. Auf-gabe D.10) und einer Drehung von �GBC in �KCB wirdsofort klar, daß z. B. das Hilfsdreieck �GKB aus den dreiSeiten GK = GA = 2

3ma, GB = 2

3mb und BK = GC =

23mc konstruiert werden kann. Die beiden anderen Eck-

punkte A, C finden wir einfach, indem wir KG verdoppelnund �GKB zum Parallelogramm CGBK erweitern.

A B

C

F

G

K

B.20 Analysis: (Bild) Von dem gesuchten �ABC klappen wir die Seiten BC = a

und AC = b so nach außen, daß die dadurch entstehenden Punkte E und F auf derGeraden g(A,B) zu liegen kommen und somit EF = a + b + c die Lange des Umfangswird. Die Dreiecke CAE und CBF sind dann gleichschenklig, und wir konnen uns leichtuberzeugen, daß die Winkel �CEA = �ECA = 1

2α und �CFB = �FCB = 1

2β betragen

(Außenwinkelsatz). Weiterhin ist der Winkel

�ECF =α

2+β

2+ γ = 90◦ +

γ

2.

Er ist ein Peripheriewinkel uber der Sehne EF innerhalb des Umkreises k von �ECF

(Mittelpunkt M). Der gegenuberliegende Peripheriewinkel betragt demnach 90◦ − 12γ,

Auffinden von Hilfsdreiecken 121

der zugehorige Zentriwinkel �EMF = 180◦ − γ. — Konstruktion: Das gleichschenk-lige �EMF mit den Basiswinkeln �FEM = �EFM = 1

2γ kann nach dem Kon-

A B

C C ′

E F

M

a

a

b

bc g

h

k

/2 /2/2 /2

α β

γ

α βγ γ

γπ

gruenzsatz WSW aus den gegebenenStucken unmittelbar gezeichnet wer-den. Fur den Punkt C liegt als geo-metrischer Ort der Kreis k = ME

fest; ein zweiter Ort ist die Paral-lele h zu g(E,F ) im Abstand hcauf der M gegenuberliegenden Sei-te von EF . Damit ergeben sich alsSchnittmenge beider geometrischerOrter zwei Punkte C bzw. C ′, die je-doch symmetrisch bezuglich der Mit-

telsenkrechten von EF liegen, so daß es genugt, allein C zu betrachten. Die beiden nochfehlenden Eckpunkte A und B erhalten wir schließlich, indem wir die Mittelsenkrechtenvon CE und CF mit g(E,F ) zum Schnitt bringen.

B.21 Analysis: (Bild) Die Losungsidee bei dieser Konstruktion besteht darin, die Win-keldifferenz α − β in dem gesuchten Dreieck ABC ausfindig zu machen. Wir errei-chen dies, indem wir �ACF , wobei F der Fußpunkt des Lotes von C auf AB sei, ander Hohe CF = hc nach innen umklappen (dies ist wegen 0 < α − β < 90◦ immermoglich). Dadurch entsteht das gleichschenklige �ACE

mit den Basiswinkeln α. Nun ist leicht zu uberprufen,daß der Winkel �ECB = α − β betragt (Außenwin-kelsatz im �ECB). Weiter sei K derjenige Punkt, furden CEKB ein Parallelogramm ist. Dann werden dieDiagonalen EB und CK durch deren Schnittpunkt Dhalbiert und es gilt �CBK = 180◦ − (α − β). Somitwird

AB = AF + FE + ED +DB = c,

und aus AF = FE und ED = DB folgt FD = 12c.

A B

C

DEF

KM

g

k

Konstruktion: Das Hilfsdreieck CFD ist aus den Großen hc,12c und �CFD = 90◦ her-

stellbar. Den Punkt K finden wir durch Verlangerung von CD uber D hinaus. EckpunktB liegt nun einerseits auf der Geraden g(F,D), andererseits auf einem Kreisbogen k, furden der Peripheriewinkel �CBK uber der Sehne CK gleich 180◦− (α−β) ist. k ist somitebenfalls konstruierbar (indem z. B. der Winkel 90◦ − (α − β) in K an CK abgetragenund der freie Schenkel mit der Mittelsenkrechten von CK zum Schnitt gebracht wird;dies liefert den Mittelpunkt M von k). Der Rest ist nun schnell erledigt: Punkt E ist derSchnittpunkt von g mit der Parallelen von CB durch K; wird dieser noch an F gespiegelt,erhalten wir Eckpunkt A und das verlangte Dreieck ist konstruiert.

122 LOSUNGEN: DREIECKSKONSTRUKTIONEN

B.22 (Bild) Die geforderte Konstruktion wird nahezu trivial, wenn uns Aufgabe D.25

gelaufig ist. Bezeichnen wir den Hohenfußpunkt von C auf AB mit F , so wird schnell

A B

C

C ′

F

Ohc

k

g

klar, daß das Hilfsdreieck CFC ′ nach Kongruenzsatz SWSaus CF = hc, �FCC ′ = α − β und CC ′ = 2R konstruier-bar ist. Dann haben wir auch den Umkreismittelpunkt O alsMittelpunkt der Strecke CC ′, somit gleichfalls den Umkreisk. Die beiden verbleibenden Eckpunkte A und B ergeben sichnun einfach als Schnittpunkte der Gerade g ⊥ CF durch F

mit dem Umkreis. Bei deren Bezeichnung ist lediglich daraufzu achten, daß fur α− β > 0 bei Eckpunkt A tatsachlich dergroßere Innenwinkel zu liegen kommt.

B.23 (Bild) Die Losung wird besonders einfach, falls uns Trape-ze vertraut sind, bei denen die Lange einer Grundseite gleich derSumme aus der Lange eines Schenkels und der gegenuberliegen-den Grundseite ist (s. Aufgabe M.12). Haben wir also ein TrapezDBCA mit BC = a, CA = b und �ACB = �DBC = 90◦, dannmuß der Winkel �ADB = 45◦ sein. Das Hilfsdreieck ADB ist so-mit nach Kongruenzsatz SSW aus c, a + b sowie dem Winkel 45◦

konstruierbar. Den dritten Eckpunkt C finden wir als Schnittpunktdes Thales-Kreises uber AB mit der Parallelen zu DB durch A.

A B

C

D

a+b

ab

c

B.41 Analysis: (Bild) Die Parallele zur Seite AC durch B schneide die Verlangerungder Winkelhalbierenden CF = wc im Punkt K. Dann ist �CBK offenbar ein gleich-schenkliges Dreieck mit den Basiswinkeln �BKC = �ACK (Wechselwinkel) = �BCK(Winkelhalbierende). Mit Hilfe des 2. Strahlensatzes laßt sich jetzt die Proportion

wc

b=FK

a=

CK

b+ a

ablesen, in der alle gegebenen Großen enthalten sind.Konstruktion: Wir konstruieren nach obiger Gleichung dievierte Proportionale CK (s. Aufgabe A.11) und anschließend�CBK nach Kongruenzsatz SSS aus CK und a. Punkt F er-halten wir auf CK von C aus im Abstand wc; A schließlich alsSchnittpunkt des Kreises Cb mit der Geraden BF .

A B

C

F

K

a

a

bwc

Rekonstruktion aus gegebenen Punkten 123

B.51 Analysis: (Bild) Jede Seite AB eines Dreiecks erscheint vom Inkreismittelpunkt Iaus unter einem Winkel 90◦+ 1

2γ (s. Aufgabe D.9), hier mit γ = 90◦ also �AIB = 135◦. k1

sei der Kreis mit dem Durchmesser AB. Wir betrachten nun AB als Sehne eines anderen

A B

C

DE

F

I

M

k

k1

k2

Kreises k2 mit dem Mittelpunkt M und I auf sei-ner Peripherie liegend. Dann ist fur dieselbe Seh-ne jeder Winkel �APB = 45◦ Supplementwinkel(mit irgendeinem P auf k2, aber auf dem ande-ren Bogen als I) und der zugehorige Zentriwinkel�AMB ein Rechter. Folglich ist M der Schnitt-punkt von k1 mit der Mittelsenkrechten von AB.Konstruktion: Wir haben die Seite AB sowie dar-auf den Beruhrungspunkt F gegeben und konnensomit den Kreis k1 und weiter M als dessenSchnittpunkt mit der Mittelsenkrechten von ABzeichnen. I ist der Schnittpunkt von MA und der

Senkrechten von AB in F . Jetzt spiegeln wir lediglich F an BI und AI und erhalten sodie beiden anderen Beruhrungspunkte D und E. Schließlich finden wir den verlangtendritten Eckpunkt C als Schnittpunkt der verlangerten Tangentenabschnitte AE und BD.

B.52 (Bild) Die Aufgabenstellung gibt uns einen eindeutigen Hinweis auf das Problemvon Fermat (s. Aufgabe D.53). Danach ist der Fermat-Punkt F derjenige Punkt imInnern eines Dreiecks ABC, fur den gerade �AFB = �BFC = �CFA = 120◦ gilt.Gleichzeitig ist er Fermat-Punkt des Dreiecks A′B′C ′ (s. Aufgabe D.54). Wiederholenwir den Prozeß mit den aufgesetzten gleichseitigen Dreiecken nochmals, indem wir die

A

A′

A′′B

B ′

B ′′

C

C ′

C ′′

F

Dreiecke B′C ′A′′, C ′A′B′′ und A′B′C ′′ errichten, so ist F ebenfalls der Fermat-Punktvon �A′′B′′C ′′. Außerdem ist klar, daß der gesuchte Eckpunkt A auf der Strecke A′A′′, Bauf B′B′′ und C auf C ′C ′′ liegt. Anstatt einen Schritt ruckwarts zu gehen (von �A′B′C ′

zu �ABC), gehen wir also einen Schritt vorwarts und anschließend zwei zuruck.

124 LOSUNGEN: DREIECKSKONSTRUKTIONEN

Um nun noch die Lagen zu berechnen, gehen wir von den Gleichungen

s ≡ AA′ = BB′ = CC ′ = FA+ FB + FC und

A′A′′ = B′B′′ = C ′C ′′ = FA′ + FB′ + FC ′ (B.101)

aus. Wegen FA′ = AA′ − FA = s − FA und den analogen Gleichungen FB′ = s − FBund FC ′ = s− FC folgt aus (B.101)

A′A′′ = B′B′′ = C ′C ′′ = 3s− s = 2s = 2AA′ = 2BB′ = 2CC ′,

d. h., die gesuchten Eckpunkte A, B, C sind die Mittelpunkte der Strecken A′A′′, B′B′′

bzw. C ′C ′′.

Winkel und Langen 125

LOSUNGEN: KREISE

K.1 Beweis: (Bild) Die Innenwinkel des SehnenvierecksABCDsind Peripheriewinkel uber den Sehnen AC und BD. Bewegen wireine Ecke des Sehnenvierecks, etwa D, auf dem Kreise, wahrenddie Ecken A, B und C in Ruhe bleiben, so nimmt dabei der Peri-pheriewinkel �ADC nacheinander die Lage samtlicher Peripherie-winkel an, die uber dem Bogen ABC moglich sind. Wir erkennenaber, daß er dabei seine Große γ + δ nicht andern kann, da erstets den unverandert bleibenden Viereckswinkel �ABC = α+βzu zwei Rechten erganzen muß (vgl. Aufgabe V.21). �

A B

C

D

O

α αβ

βγ

γδ

δ

Bemerkung: Eigentlich fallt der Peripheriewinkelsatz als Nebenprodukt des Peripherie-Zentriwinkel-Satzes (s. folgende Aufgabe K.2) mit ab.

K.2 Beweis: (Bild) C sei ein beliebiger Punkt auf der Peripherie des Kreises; die demZentriwinkel �AOB mit O als Mittelpunkt des Kreises und dem Peripheriewinkel �ACBgemeinsame Sehne sei AB. Wir konnen nun vier verschiedene Falle fur die gegenseitigeLage der Punkte C und O bezuglich AB annehmen:

1. O liegt im Innern des �ABC (Bild a),

2. O liegt auf einer der beiden Seiten AC oder BC (Bild b),

3. O liegt außerhalb von �ABC, jedoch mit C auf derselben Seite bezuglich derGeraden g(A,B) (Bild c) und

4. C und O liegen bezuglich g(A,B) auf verschiedenen Seiten (etwa mit C = E,Bild a).

a)

A B

C

E

Or

rr

α

α β

β

b)

A B

C

O2

β

β

β

c) A B

CO

α

α

β

β

Im Fall 1 sind die Dreiecke AOC und BOC wegen r ≡ AO = BO = CO gleichschenkligeDreiecke mit jeweils gleich großen Basiswinkeln α ≡ �ACO = �CAO und β ≡ �BCO =�CBO. Verlangern wir den Radius CO bis zum Schnittpunkt E mit dem Kreis, so sinddie Winkel �AOE und �EOB Außenwinkel dieser gleichschenkligen Dreiecke. Da derAußenwinkel in einem Dreieck gleich der Summe der beiden nicht anliegenden Innenwinkelist, folgt

�AOE = �ACO + �CAO = 2α und �EOB = �BCO + �CBO = 2β.

Durch Addition beider Gleichungen erhalten wir fur den Zentriwinkel

�AOB = �AOE + �EOB = 2(α+ β) = 2 �ACB.

126 LOSUNGEN: KREISE

Fur den Fall 2 ist der Zentriwinkel �AOB gerade Außenwinkel im gleichschenkligen�BOC (AC ist hierbei Durchmesser des Kreises) und damit doppelt so groß wie dessenBasiswinkel, der gleich dem Peripheriewinkel �ACB ist. Im Fall 3 folgt die Behauptungaus

�AOB = �COB − �COA = (180◦ − 2β) − (180◦ − 2α) = 2(α− β) = 2 �ACB.

Liegen dagegen im Fall 4 C und O auf verschiedenen Seiten der Sehne AB (mit C = E

in Bild a), so sind die Basiswinkel der gleichschenkligen Dreiecke �AOE und �EOB

�EAO = �AEO = 90◦ − α und �BEO = �EBO = 90◦ − β

und somit

�AEB = �AEO + �BEO = 180◦ − α− β = 180◦ − �ACB,

d. h., die beiden Peripheriewinkel �ACB und �AEB sind Supplementwinkel. �

K.3 Beweis: (Bild) Verbinden wir den Punkt C mit demMittelpunkt O des Kreises, so entstehen zwei gleichschenkli-ge Dreiecke AOC und BOC. Bezeichnen wir die Basiswinkeldieser Dreiecke mit α bzw. β, dann ist in dem Dreieck ABC2α+ 2β = 180◦, also α+ β = 90◦. Es ist daher CA ⊥ CB. �Bemerkung: Der Satz des Thales ist somit ein Spezialfall desPeripherie-Zentriwinkel-Satzes (vgl. Aufgabe K.2, Fall 2 ).

A B

C

α

β

β

K.4 (Bilder) Bewegen wir den Scheitel C des Peripheriewinkels �ACB = γ langs desKreises, so andern zwar AC und BC ihre Lage, aber der Winkel bei C bleibt nach demPeripheriewinkelsatz konstant. Wird diese Bewegung so lange fortgesetzt, bis C mit Azusammenfallt, so nimmt schließlich CA die Lage der Tangente AD im Punkt A an, undBC fallt in die Richtung der Sehne BA. Der Winkel �DAB ist also gleich dem Winkelbei C; er heißt Sehnen-Tangentenwinkel.

a)

A B

C

M

O

D

γ

γ

γ

b)

A B

C

M

OD

γ

γ

γ

Zum Beweis fallen wir das Lot M vom Mittelpunkt O des Kreises auf die Sehne AB.Dann ist OM die Winkelhalbierende im gleichschenkligen Dreieck AOB. Im Fall 1 sei γein spitzer Winkel (Bild a), so daß nach dem Peripherie-Zentriwinkel-Satz �AOM = γgilt. Daher ist im rechtwinkligen Dreieck AOM �OAM Komplementwinkel zu γ, undwegen �OAD = 90◦ schließlich �BAD = γ = �ACB. Im Fall 2 mit γ als stumpfemWinkel (Bild b) fuhrt eine analoge Betrachtung zum gleichen Ergebnis. �

Winkel und Langen 127

K.11 Beweis: (Bild) Wir fuhren den Beweis uber die Ahnlich-keit der Dreiecke PAD und PCB. Dies ist offensichtlich der Fall,da beide in den Winkeln �ADC = �ABC (Peripheriewinkel uberder Sehne AC) und im Scheitelwinkel bei P ubereinstimmen. Wirkonnen daher sofort die Proportion PA : PD = PC : PB auf-stellen, die umgestellt die behauptete Konstanz des Produktesder Abschnittslangen ergibt: PA · PB = PC · PD. �Insbesondere gilt dies auch fur den Durchmesser EF des Kreises;

A

B

C

D

E

F

OP

dr

mit dem Abstand OP ≡ d und dem Radius OE = OF ≡ r sowie der Vereinbarung, daß— falls die gerichteten Strecken PE und PF den gleichen Richtungssinn haben — derenProdukt positiv sei, anderenfalls negativ (vgl. auch Losung zu Aufgabe A.12), folgt:

PA · PB = PC · PD = PE · PF = −(r − d)(r + d) = d2 − r2 = P(P ). (K.101)

K.12 Beweis: (Bild) Auch im vorliegenden Fall, daß derPunkt P nun außerhalb des Kreises liegt, erkennen wir, daß�PAD ∼ �PCB gilt (ein Paar kongruenter Peripheriewin-kel und ein gemeinsamer Winkel bei P , vgl. Aufgabe K.11).Mithin ist wiederum PA : PD = PC : PB. Die ZentralePF schneidet den Kreis ein zweites Mal in E, so daß hiermit dem Abstand PO ≡ d und dem Radius OE = OF ≡ r

gilt:

A

B

C

D

EF

OP

PA · PB = PC · PD = PE · PF = (d− r)(d+ r) = d2 − r2 = P(P ). � (K.102)

K.13 Beweis: (Bild) Hier liegt der Grenzfall vor, daß eineSekante zur Tangente wird. Wir bemuhen daher anstelledes Peripheriewinkelsatzes den Sehnen-Tangentenwinkel-Satz, der uns zeigt, daß �PAT ∼ �PTB gilt. Dies fuhrtzur Proportion PA : PT = PT : PB. Der Tangentenab-schnitt PT ist somit die mittlere Proportionale der Sekan-tenabschnitte: PT =

√PA · PB. �

A

B

P

T

Bemerkung: Mit (K.102) ergibt sich hieraus eine anschauliche Interpretation der Potenz

P(P ) = PA · PB = PT 2, oder PT =√

P(P ),

d. h., die Potenz eines außerhalb des Kreises liegenden Punktes ist gleich dem Quadratdes zugehorigen Tangentenabschnitts.

128 LOSUNGEN: KREISE

K.14 Beweis: (Bild) A und B seien Beruhrungspunkte der Tangenten an beide KreiseO1a bzw. O2b; die Strecken XY = 1

2x und UV = 1

2y gerade die Halften der in der Be-

hauptung auftretenden Sehnen. Da nun einige rechtwinklige Dreiecke, wie z. B. O1Y Xund O1AO2 zu erkennen sind, die daruber hinaus noch einen Winkel gemeinsam haben(hier �XO1Y = �AO1O2), ist al-les weitere fast zwangslaufig: 1.)Ahnlichkeit beider Dreiecke fest-stellen und 2.) Proportionen auf-stellen. Dies fuhrt auf

XY

O1X=

O2A

O1O2

,x/2

b=a

s.

AB

O1 O2

U

V

X

Y

aa b

bsx y

Analog folgt (vom anderen”Auge“ aus gesehen): �O2V U ∼ �O2BO1 und daraus y/(2a)

= b/s. Aus beiden Gleichungen folgt die Bildgleichheit auf der”Netzhaut“: x = y. �

K.15 Beweis: (Bild) Eine kurze Rechnung fuhrt auf:

AP · AQ = AP (AP + PQ) = AP 2 + AP · PQ= AP 2 + CP · PD= AP 2 + (CE − PE) · (CE + PE)

= AP 2 + CE2 − PE2 = AE2 + CE2 = AC2.

A

B

C DEP

QZweifellos ist AC unabhangig von der Lage von P . �

K.16 Beweis: (Bild) Zweifellos ist OT ⊥ TQ. Darausfolgt mit dem Satz des Pythagoras:

QT 2 = OQ2 −OT 2 = OU2 −OT 2

= (OP 2 − PU2) − (OP 2 − PT 2)

= PT 2 − PU2. �

OP

QT

U

Inversion am Kreis 129

K.21 Beweis: Die Behauptung a) ist selbstverstandlich, da nach der Definition derInversion jeder Punkt auf der Geraden durch O in einen anderen Punkt derselben Geradenabgebildet wird (Bild a). Auch wenn die Punkte ausgetauscht werden, bleibt die Geradeals Ganzes erhalten. Um b) zu beweisen, fallen wir das Lot von O auf die Gerade g; dessenFußpunkt sei A und der inverse Punkt A′ (Bild b). Wir wahlen ferner einen beliebigenPunkt P auf g, und es sei P ′ der zugehorige inverse Punkt. Wegen OA ·OA′ = OP ·OP ′ =r2 folgt

OA′

OP ′ =OP

OA,

die beiden Dreiecke OP ′A′ ∼ OAP sind also ahnlich. Da �OAP = 90◦, ist auch �OP ′A′ =90◦, und P ′ liegt demzufolge auf demThales-Kreis k′ mit dem Durchmesser OA′. Hiermitist b) bewiesen. Die Behauptung c) folgt aus der Tatsache, daß, wenn k′ invers zu g ist,auch g invers zu k′ sein muß.

a)

O

g = g ′Γ

b)

AA′O

PP′

gk ′

Γ

c)

A

A′

B

B ′M

M ′O

T

k

g

t

R

Γ

Es bleibt noch die Behauptung d). Sei k ≡ MR irgendein Kreis, der nicht durch O geht(Bild c). Um sein Bild zu finden, ziehen wir eine Gerade g durch O, die k in A und B

schneidet, und untersuchen, wie sich die Bilder A′, B′ andern, wenn g den Kreis k in allenmoglichen Lagen schneidet. Es gilt nun OA · OA′ = OB · OB′ = r2 nach der Definitionder Inversion sowie OA · OB = t2 nach dem Sekanten-Tangentensatz, wenn t ≡ OT dieLange des Tangentenabschnitts von O an k ist. Dividieren wir die erste Beziehung durchdie zweite, erhalten wir

OA′

OB=OB′

OA=r2

t2≡ λ, (K.103)

wobei λ eine Konstante ist, die nur von r und t abhangt und fur alle Lagen A und B

denselben Wert hat. Wir ziehen nun durch A′ eine Parallele zu BM , die OM in M ′

schneidet. Dann ist nach den Strahlensatzen

OM ′

OM=OA′

OB=A′M ′

R, OM ′ =

OM ·OA′

OB= OM · λ, A′M ′ = R

OA′

OB= R · λ.

Das bedeutet, daß fur jede Lage von A und B der Punkt M ′ stets derselbe Punkt aufOM ist und der Abstand A′M ′ immer denselben Wert hat. Ebenso ist wegen (K.103)auch B′M ′ = A′M ′, d. h., die Bilder aller Punkte A, B auf k sind lauter Punkte, derenAbstand von M ′ konstant ist; mithin ist das Bild von k ein Kreis. �

130 LOSUNGEN: KREISE

K.22 Beweis: (Bild) Da P , P ′ und Q, Q′ invers zueinander sind, gilt

OP ·OP ′ = OQ ·OQ′ oderOP

OQ=OQ′

OP ′ .

Die Dreiecke OPQ und OQ′P ′ sind somit ahnlich (da sieden Winkel bei O gemeinsam haben). Daraus folgt furdie Innenwinkel �OPQ = �OQ′P ′, und (da letztererNebenwinkel zu �QQ′P ′ ist):

�P ′PQ+ �QQ′P ′ = 180◦.

P

P′

QQ ′O

Γ

Nach Aufgabe V.21 ist P ′PQQ′ also ein Sehnenviereck. �

K.23 Beweis: (Bild) Wir beweisen zunachst, daß der Winkel, den eine Kurve C

mit einer Geraden durch das Inversionszentrum bildet, kongruent zu dem Winkelist, unter dem die inverse Kurve C ′ dieselbe Geradeschneidet. P , P ′ und Q, Q′ seien daher zwei inversePunktepaare auf C bzw. C ′. Die vier Punkte P , P ′,Q, Q′ liegen allesamt auf einem Kreis (vgl. AufgabeK.22), so daß gegenuberliegende Winkel Supplementwin-kel sind: P

P′Q

Q ′

O

C

C ′

t

t ′

�OPQ = �OQ′P ′.

Rucken wir mit Punkt Q immer naher an Punkt P auf der Kurve C heran, so wird ausder Sekante PQ die Tangente t und auf der inversen Kurve aus der Sekante P ′Q′ dieTangente t′. Der Winkel �OPQ geht im Grenzfall gegen den eingezeichneten Winkel,dessen Supplementwinkel gegen den Grenzwinkel von �OP ′Q′ geht. Schneiden sich nunzwei beliebige Kurven, so ist der Schnittwinkel zwischen den Kurven naturlich der einge-schlossene Winkel der beiden Tangenten an die Kurven im Schnittpunkt. Damit laßt sichdieser allgemeinere Fall auf den Schnitt zweier Geraden zuruckfuhren. Auch hier bleibtder Winkel bei der Inversion erhalten. �Bemerkung: Wahrend der Betrag des Schnittwinkels erhalten bleibt, kehrt sich jedoch dieRichtung des Winkels um.

K.24 Beweis: (Bild) Der Radius OT des Inversionskreises Γ ≡ Or beruhre den Kreisk tangential, d. h., beide Kreise mogen sich orthogonal schneiden. Eine beliebige Geradedurch das Inversionszentrum O schneide k in den Punkten P bzw. P ′. Dann gilt nachdem Sekanten-Tangentensatz (s. Aufgabe K.13) stets

OP ·OP ′ = OT 2 = r2,

so daß P und P ′ nach Definition inverse Punkte sind. Fur je-den Punkt auf dem Bogen SPT gibt es demnach einen inversenPunkt auf dem Bogen SP ′T , wobei S, T invariant sind. k gehtalso bei einer Inversion in sich selbst uber. �

O

P

P′

S

T

Inversion am Kreis 131

K.25 (Bild) Wie wir aus Aufgabe K.24 wissen, mußder gesuchte Inversionskreis Γ den gegebenen Kreisk in zwei Punkten T und T ′ orthogonal schneiden.Dies ist genau dann der Fall, wenn PT einerseits einTangentenabschnitt bzgl. k und andererseitsOT Tan-gentenabschnitt bzgl. Γ ist. Der Winkel �OTP mußalso ein Rechter sein. Wir finden T bzw. T ′, indemwir den Thales-Kreis uber der Strecke OP mit kzum Schnitt bringen.

O P

T

T ′

K.31 Da im Rhombus APBP ′ die Diagonalen senkrecht aufeinander stehen, und siesich in deren Schnittpunkt C halbieren (Bild a), liegen wegen OA = OB die vier PunkteO, P , C, P ′ auf einer Geraden, die zugleich Mittelsenkrechte von AB ist. Dann gilt:

OP ·OP ′ = (OC − PC)(OC + CP ′) = OC2 − PC2

= (OA2 − AC2) − (PA2 − AC2) = OA2 − PA2 ≡ r2.

Der letzte Term ist von der Konstruktion durch die Langen der Stabe OA und PA vor-gegeben und somit konstant. Nach der Definition der Inversion (K.1) ist der Kreis Or derzugehorige Inversionskreis.

a) A

B

CO P P′

b) A

KK ′

LL′

M

O P P′

Fugt man einen siebenten Stab MP hinzu, der in M so festgehalten wird, daß OM = PM

ist (Bild b), beschreibt P einen Kreisbogen um M . Der inverse Punkt P ′ bewegt sich dannwie gefordert geradlinig.Bemerkung: Konstruktionstechnisch sind zwei Nebenbedingungen zu beachten, wenn derZeichenweg moglichst groß sein soll: Damit der Rhombus sich nicht an den Knien P , P ′

beruhrt oder”uberschlagt“, sollte der Fall wie in Bild K.2b dargestellt vorliegen (d. h.,

der von P durchfahrene Kreisbogen sollte die von P ′ durchlaufene Strecke nicht beruhrenoder schneiden). Dies ist gleichbedeutend mit OK = 2MP < r. Außerdem kann manihn nur so weit aufklappen, bis sich A und B beruhren; P kommt dann in Punkt L zuliegen: OL = OA− PA. Der erreichbare Kreisbogen ist also KL sowie dessen Spiegelbildbezuglich des Durchmessers OK; die durchfahrbare Strecke K ′L′ plus Spiegelbild (ausPlatzgrunden ist jeweils nur die Halfte gezeichnet).

132 LOSUNGEN: KREISE

K.33 Das Problem besteht aus zwei Teilen: 1. die drei Kreise kA, kB, kC zu bestimmenund 2. den Beruhrungskreis k zu finden. Bezeichnen wir im Teil 1 die Radien der dreiKreise mit rA, rB, rC , so lassen sich nach Bild a) unmittelbar die Gleichungen

rB + rC = a, rC + rA = b, rA + rB = c

aufstellen, deren Losung

rA =1

2(b+ c− a), rB =

1

2(c+ a− b), rC =

1

2(a+ b− c)

ist (vgl. auch Aufgabe D.63 und Abschnitt G.1.1). Die Beruhrungspunkte D, E undF sind mithin leicht zu konstruieren; es zeigt sich, daß sie isotomische Punkte zu denHalbumfangspunkten (s. Abschnitt D.2, insbesondere Aufgabe D.39) sind.

a)

A B

C

D

E

F

kA

kB

kC

k ′ k ′

k ′

A B

C

Γ

b)

kA

kB

kC

k ′ k ′

k ′

k

k ′

A B

C

Γ

Im Teil 2 gilt es, einen geeigneten Inversionskreis zu finden, der das BeruhrungsproblemKKK auf ein Problem GGK vereinfacht (s. Abschnitt A.3). Dies erreichen wir jedoch nur,wenn zwei der ursprunglichen Kreise durch den Mittelpunkt des Inversionskreises Γ gehen(vgl. Bild K.2 b,c). Damit ist etwa F eine gute Wahl fur den Mittelpunkt von Γ ; seinRadius kann beliebig gewahlt werden (Bild a). Die nunmehr bekannten Original-Kreise kA,kB, kC gehen bei der Inversion in die parallelen Bild-Geraden k′A, k′B sowie den Bild-Kreisk′C uber. Letzterer muß nach dem Satz uber die Invarianz der Schnittwinkel (vgl. AufgabeK.23) die beiden Bild-Geraden tangieren (da sich die Original-Kreise ja beruhren). Ausgleichem Grunde muß, da der gesuchte Kreis k die drei Original-Kreise beruhren soll, seinBild-Kreis k′ die Bild-Geraden k′A, k′B und den Bild-Kreis k′C tangieren (Bild b). k′ zufinden ist nicht schwer; wir brauchen offensichtlich k′C auf der Mittelparallelen von k′A undk′B nur so zu verschieben, daß sich beide beruhren. Da es hierbei zwei Moglichkeiten gibt,existieren auch zwei Losungen: einer der gesuchten Kreise tangiert die Original-Kreise voninnen, der andere von außen.

Inversion am Kreis 133

K.35 Beweis: (Bild) P und Q seien die Beruhrungspunkte von k1 mit den Seiten CA

bzw. CB. Wegen

CP = CA+ AR = CB +BR = CQ = s

(vgl. Aufgabe D.35) liegen die vier Punkte E, P , Q und F auf einem Kreis Γ ≡ Cs.Nehmen wir diesen Kreis als Inversionskreis und bezeichnen mit i eine Inversion an Γ .

A B

C

E F

P

Q

Rg

kk1

Γ

Dann ist, da i(P ) = P und i(Q) = Q, der Kreis k1 orthogonal zu Γ und somit i(k1) = k1.Andererseits ist i(E) = E und i(F ) = F . Also ist i(k) = g(A,B). Der Kreis k1 beruhrtjedoch AB, mithin muß der Umkreis k den Kreis k1 ebenfalls beruhren. �

K.36 Beweis: (Bild) Betrachten wir zuerst den Fall, daß der gegebene Punkt P außer-halb von Γ liegt. Wir beschreiben den Kreisbogen PO, der Γ in A und B schneidet. Umdiese beiden Punkte schlagen wir Kreisbogen mit dem Radius r, welche sich (außer in O)in einem Punkte P ′ auf der Geraden OP schneiden.Fur die gleichschenkligen Dreiecke OAP und OP ′Agilt

�OAP = �POA = �OP ′A,

so daß die Dreiecke ahnlich sind und daher

OP

OA=

OA

OP ′ , oder OP ·OP ′ = r2

A

B

PP′O

Γ

ist. Somit ist P ′ der gesuchte inverse Punkt zu P .Liegt dagegen P im Innern von Γ , behelfen wir uns mit folgendem Trick: Die Strecke OPwird vom Inversionszentrum ausgehend mit dem Zirkel so oft hintereinander abgetragen(wie das geht, vgl. Aufgabe A.42), bis wir zu einem Punkt Q außerhalb von Γ gelangen.Nach obiger Methode wird dessen inverser Punkt Q′ konstruiert, und es gilt mit n ∈ N:

r2 = OQ′ ·OQ = OQ′ · (n ·OP ) = (n ·OQ′) ·OP.

Mithin ist der Punkt P ′, fur den OP ′ = n ·OQ′ ist, der gesuchte inverse Punkt. �

134 LOSUNGEN: KREISE

K.37 (Bild) Die Figur bestehtaus vier Halbkreisbogen,die einem kleineren Qua-drat im Innern aufgesetztsind.

K.61 Beweis: (Bild) Wir fallen von X und Y die Lote x1 und y1 auf AB sowie x2 undy2 auf CD. Dadurch entstehen mehrere Paare ahnlicher rechtwinkliger Dreiecke:

�MEX ∼ �MFY, �MGX ∼ �MHY,

�AEX ∼ �CHY, �DGX ∼ �BFY.

Mit den Abkurzungen MP = MQ ≡ a, MX ≡ x undMY ≡ y konnen wir nun folgende Gleichungen aufstel-len:

x

y=x1

y1

,x

y=x2

y2

,x1

y2

=AX

CY,

x2

y1

=XD

YB,

A

B

C

D

E

FG

H

M

P

Q

X Yxy

x1

x2

y1

y2

aus denen mit Hilfe des Sehnensatzes

x2

y2=x1

y1

· x2

y2

=x1

y2

· x2

y1

=AX ·XDCY · YB =

PX ·XQPY · Y Q =

(a− x)(a+ x)

(a+ y)(a− y)=a2 − x2

a2 − y2

folgt. Betrachten wir in dieser Gleichungskette den ersten und letzten Term, folgt wiebehauptet x = y. �

K.62 (Bild) AP ≡ u und BQ ≡ v seien die Tangentenabschnitte von A bzw. B an denKreis Γ . Nach dem Sehnensatz gelten die Gleichungen u2 = AC ·AD und v2 = BC ·BE,deren Division

u2

v2=AC

BC· ADBE

ergibt. Da DE ‖ AB gefordert ist, giltaußerdem AC/BC = AD/BE und da-her mit obiger Gleichung

A B

C

C ′D E

P

Q

X Y

Γ

Γ′

AC

BC=u

v≡ q.

Nun sind A, B und Γ fest, so daß das Verhaltnis q eine bekannte Konstante ist. Dergeometrische Ort aller Punkte C, dessen Entfernungen von zwei Punkten A und B einkonstantes Verhaltnis haben, ist der Kreis des Apollonius (vgl. Aufgabe A.22). Somithaben wir lediglich die Strecke AB innerlich und außerlich im Verhaltnis q zu teilen (s.Bemerkung 2 zur Losung von Aufgabe A.12), um die Punkte X und Y als Durchmesserdes Apollonius-Kreises Γ ′ zu bestimmen. Die Schnittpunkte von Γ und Γ ′ sind also diegesuchten Punkte C (bzw. C ′), wobei es sein kann, daß der zweite Schnittpunkt (im BildC ′) herausfallt, da er keine weiteren Schnittpunkte D und E mit Γ zulaßt.

Klassische Transversalen 135

LOSUNGEN: DREIECKE

D.1 Beweis: (Bild) Angenommen, O sei der Schnittpunkt der Mittelsenkrechten derSeiten AB und BC. Dann bleibt zu zeigen, daß die dritte Mittelsenkrechte ebenfalls durch

A B

C

O

O geht. Nun ist O gleich weit von den Punkten A und B entfernt:OA = OB. Wir nutzen dabei den geometrischen Ort

”Mittelsenk-

rechte“, indem wir sagen: Wenn O auf der Mittelsenkrechten vonAB liegt, dann sind seine Abstande zu A und B untereinandergleich (vgl. Punkt 2 aus Abschnitt A.2). Ebenso gilt OB = OC.Aus beiden Gleichungen folgt OA = OC. Jetzt nehmen wir dieUmkehrung: Wenn die Abstande von O zu A bzw. C gleich sind,muß O auf der Mittelsenkrechten von AC liegen, und sind damitschon fertig. �

D.2 Beweis: (Bild) Da F auf der Mittelsenkrechten von AB

liegt, ist �AFB ein gleichschenkliges Dreieck. Daraus folgt, daßdessen Basiswinkel �ABF und �BAF einander gleich sind. Die-se sind aber wegen AF = BF gleichzeitig Peripheriewinkel uberden gleichen Bogen AF und BF . Nach dem Peripheriewinkelsatzist somit

�ACF = �ABF =γ

2, �BCF = �BAF =

γ

2,

A B

C

F

F ′

d. h., die Strecke CF ist tatsachlich Winkelhalbierende von �ACB ≡ γ. Da ferner dieMittelsenkrechte einer Sehne stets durch den Mittelpunkt des Kreises geht, ist FF ′ einDurchmesser und somit �FCF ′ nach dem Satz des Thales ein rechtwinkliges Dreieck.CF ′ steht also senkrecht auf der Winkelhalbierenden CF und ist damit Winkelhalbierendedes Außenwinkels von γ (s. Aufgabe D.7). �

D.3 Beweis: (Bild) Jetzt seien F und F ′ die Schnittpunk-te der Mittelsenkrechten von AB mit der inneren und außerenWinkelhalbierenden von �ACB ≡ γ. Dann muß der Umkreis-mittelpunkt von �ABC auf der Geraden g(F, F ′) liegen undirgendeine Strecke auf g gleich dem Durchmesser des Umkreisessein. Nun ist nach Aufgabe D.7 �FCF ′ = 90◦; somit geht derThales-Kreis k uber FF ′ durch Eckpunkt C. Gleichzeitig gehtk durch A (und wegen der Symmetrie auch durch B) und ist so-mit gerade der Umkreis. Mithin liegen die Schnittpunkte F und

A B

C

F

F ′

gk

F ′ auf dem Umkreis. �

D.4 Auf losung: Wie wir sicherlich schon erkannt haben, liegt der Schnittpunkt D nichtim Innern des Dreiecks, sondern auf dem Umkreis außerhalb von �ABC.

136 LOSUNGEN: DREIECKE

D.5 Beweis: Wir betrachten sowohl ein spitzwinkliges Dreieck (Bild a) als auch einstumpfwinkliges Dreieck (Bild b) und zeichnen jeweils den Durchmesser BD und die Sehne

A B

C

D

O

k

c

γ

γ

a)

A

B

C

D O

k

c

γ

b)

AD des Umkreises k in unsere Planfigur ein. In beiden Fallen bemerken wir, daß �BADein Rechter ist, da er einem Halbkreis einbeschrieben ist. Es gilt somit

sin(�BDA) ≡ sin γ =c

BD=

c

2R. (D.101)

Dabei ist in Bild a) nach dem Peripheriewinkelsatz �BDA = �BCA = γ; in Bild b)�BDA = 180◦ − �BCA = 180◦ − γ, da sich gegenuberliegende Winkel im Sehnenvier-eck ADBC zu 180◦ erganzen. Wegen sin γ = sin(180◦ − γ) ist also in beiden Fallensin(�BDA) = sin(�BCA). (D.101) umgestellt ergibt mithin

c

sin γ= 2R.

Die gleiche Prozedur auf die beiden anderen Winkel des �ABC angewandt, ergibt dieBehauptung. �Bemerkung: Wir sollten uns beim Sinussatz also nicht nur die Tatsache merken, daß ineinem Dreieck der Quotient aus Seitenlange und Sinus des gegenuberliegenden Winkelskonstant ist, sondern auch die Große dieser Konstante, namlich 2R, den Durchmesser desUmkreises des Dreiecks.

D.6 Beweis: (Bild) ABPC ist ein Sehnenviereck, dessen Dia-gonalen sich in Q schneiden und die das Viereck in vier Drei-ecke zerlegen. Dann sind die jeweils gegenuberliegenden Drei-ecke ahnlich (vgl. Aufgabe V.23): �BPQ ∼ �ACQ, �CPQ ∼�ABQ. a) Daher konnen wir mit AB = BC = CA ≡ a fol-gende Verhaltnisgleichungen aufstellen: PQ/PB = CQ/a undPQ/PC = BQ/a, deren Addition

A B

C

PQ

PQ

PB+PQ

PC=CQ+BQ

a=a

a= 1

ergibt. Anschließende Division durch PQ fuhrt auf die Behauptung. b) Der zweite Teillaßt sich am einfachsten mit dem Satz des Ptolemaus (s. Aufgabe V.26) zeigen: AusAC ·PB+AB ·PC = BC ·PA folgt nach Division durch a die behauptete Gleichung. �

Klassische Transversalen 137

D.7 Beweis: (Bild) Die Winkelhalbierende wc teilt den Innen-winkel 2 δ zwischen den Seiten CA und CB, w′

c den zugehorigenAußenwinkel 2 ε in zwei Halften. Da beides Nebenwinkel unddaher Supplementwinkel sind, also 2(δ+ε) = 180◦ gilt, folgt dar-aus

δ + ε = 90◦.

Dies ist genau der Winkel zwischen den beiden Winkelhalbieren-den wc und w′

c. � A B

C

wc

wc′

δ δε

ε

Bemerkung: Diese Winkelbeziehung besteht naturlich ganz allgemein zwischen zwei sichschneidenden Geraden; wir hatten also eigentlich kein Dreieck benotigt.

D.8 Beweis: Die Winkelhalbierende wc teilt die Seite AB = c innerlich im Punkt D indie Abschnitte ca und cb (Bild a). Tragen wir nun die Lange der Seite CB = a auf g(A,C)uber C hinaus ab, so erhalten wir Punkt E. Im gleichschenkligen �BCE ist dann derInnenwinkel an dessen Eckpunkt C Nebenwinkel zu �ACB = γ; betragt also 180◦−γ. Mitder Winkelsumme 180◦ folgt damit fur die Basiswinkel �EBC = �BEC = 1

2γ. Mithin

a)

A B

C

D

E

a

ab

cb ca

wc

b)

A B

C

D

Ea

a

sind �ACD = �AEB gleiche Stufenwinkel, d. h., EB ist parallel zu CD. Die Behauptungfolgt nun direkt aus der Anwendung des 1. Strahlensatzes:

cacb

=DB

DA=CE

CA=a

b.

Auch im Fall der Winkelhalbierenden des Außenwinkels CD (Bild b) finden wir mitCE = a, daß EB ‖ CD ist und damit

DB

DA=CE

CA=a

b. �

D.9 Beweis: (Bild) Der Inkreismittelpunkt ist der Schnittpunkt der Winkelhalbieren-den, so daß im �AIB gilt: �IAB = 1

2α und �IBA = 1

2β. Fur den betrachteten Winkel

�AIB gilt unter Beachtung der Innenwinkelsumme in den Dreiecken AIB und ABC:

�AIB = 180◦ − 1

2(α+ β) = 180◦ − 1

2(180◦ − γ)

= 90◦ +γ

2.

Die gleichen Betrachtungen lassen sich naturlich auch fur dieanderen Seiten anstellen, woraus �AIC = 90◦ + 1

2β bzw.

�BIC = 90◦ + 12α folgt. � A B

C

I

α β/2 /2

138 LOSUNGEN: DREIECKE

D.10 Beweis: (Bild) G sei der Schnittpunkt der bei-den Seitenhalbierenden AD und BE. Die DreieckeACB und ECD sind ahnlich, da sie den Winkel beiC gemeinsam haben und

A B

C

DE

F

G

KLCE

CA=CD

CB=

1

2bzw.

CE

CD=CA

CB

gilt. Mithin sind auch die Winkel �CED und �CAB gleich (Stufenwinkel), d. h. EDist parallel zu AB und nach dem 2. Strahlensatz halb so groß wie die Grundseite AB.Weiterhin seien L und K die Mittelpunkte der Strecken AG und BG. Dann folgt ausden vorangegangenen Uberlegungen, jetzt bezogen auf das �ABG, daß LK ‖ AB undLK = 1

2AB = ED ist. Das Viereck EDKL ist somit ein Parallelogramm. Da sich die

Diagonalen in einem Parallelogramm stets halbieren, gilt

DG = GL = LA, EG = GK = KB bzw. DG =1

3AD, EG =

1

3BE.

Somit teilt G die beiden Seitenhalbierenden AD und BE im Verhaltnis 2 : 1, wobei daslangere Teilstuck die Entfernung zum jeweiligen Eckpunkt ist. Ebenso laßt sich zeigen, daßder gleiche Punkt G Schnittpunkt eines der beiden anderen Paare von Seitenhalbierenden,etwa von BE und CF ist. Damit ist der Schwerpunkt G als gemeinsamer Schnittpunktaller drei Seitenhalbierenden nachgewiesen. �

D.11 Beweis: (Bild) Die Teildreiecke haben die Flachen-inhalte ∆i, i = 1, 2, 3. Ganz offensichtlich ist dann

[AFG] = [BFG] = ∆1, [BDG] = [CDG] = ∆2,

[CEG] = [AEG] = ∆3,

da paarweise jeweils gleiche Grundseiten und gleicheHohen vorliegen und daher unsere Bezeichnung gerecht-

A B

C

DE

F

G

∆1 ∆1

∆2

∆2∆3

∆3

fertigt ist. Außerdem gilt

[AFC] = ∆1 + 2∆3 = [BFC] = ∆1 + 2∆2,

woraus ∆2 = ∆3 folgt. Vollig analog beweisen wir ∆1 = ∆2. �Bemerkung: Dies ist schon ein Vorgeschmack auf das Flachenprinzip (s. Abschnitt M.4).

D.12 Beweis: (Bild) Wir erinnern uns daran, daß G die Sei-tenhalbierende CM im Verhaltnis 2 : 1 teilt und dritteln daherdie Seite BC durch die Punkte U , V so, daß BU = UV = V C

gilt. Dann ist wegen CG : CM = CU : CB = 2 : 3 nachder Umkehrung des 1. Strahlensatzes GU ‖ AB und eben-so GV ‖ AC. Weiterhin gilt nach dem 1. Strahlensatz mitUV = V C = −UB:

A B

C

G

M

U

V

X

Y

Z

GX

(1

GX+

1

GY+

1

GZ

)= 1 +

V X

V C+UX

UB= 1 +

V X − UX

V C= 1 +

V U

UV= 0,

also nach Division durch GX die behauptete Gleichung. �

Klassische Transversalen 139

D.13 Beweis: (Bild) Die Lotfußpunkte der Ecken A, B, C auf der Geraden g bezeichnenwir mit A′, B′ bzw. C ′; die Seitenmitte von AB mit F . Dann ist AA′B′B offensichtlichein Trapez und FF ′ dessen Mittellinie. Deren Langebetragt FF ′ = 1

2(AA′ + BB′). Desweiteren sind die

rechtwinkligen Dreiecke CC ′G und FF ′G einanderahnlich; ferner teilt der Schwerpunkt G die Seiten-halbierende CF im Verhaltnis 2 : 1. Somit gilt auchCC ′ = 2FF ′ = AA′ +BB′. �Bemerkung: Im Falle, daß g durch einen Eckpunkt A

A′

B

B ′

C

C ′

F

F ′G

g

(etwa A) geht, gilt: BB′ = CC ′ (vgl. Aufgabe D.11).

D.21 Beweis: (Bild) Die Punkte D, E, F seien die Fußpunkte der Hohen im �ABC,und �DEF ist damit das Hohenfußpunktdreieck. Da die Winkel �AEB = �ADB Rech-te sind, liegen die Punkte D, E auf dem Thales-Kreis uber dem Durchmesser AB, d. h.,das Viereck AEDB ist ein Sehnenviereck. Bezeichnen wir �ADE = ε1 und �ADF = ε2,so gilt, da sich gegenuberliegende Winkel in einem Sehnenviereck zu 180◦ erganzen (vgl.Aufgabe V.21):

�EAB + �EDB = �EAB + ε1 + 90◦ = 180◦.

Ebenso ist CDFA ein Sehnenviereck mit

�CAF + �CDF = �CAF + ε2 + 90◦ = 180◦.

Wegen �EAB = �CAF folgt aus beiden Gleichun-gen ε1 = ε2, d. h., die Hohe AD ist Winkelhalbierendeim Hohenfußpunktdreieck DEF . Ebenso beweisen wir�BEF = �BED und �CFD = �CFE. Mithin ist H A B

C

D

E

F

H

ε1ε2

als Schnittpunkt der drei Winkelhalbierenden in �DEF dessen Inkreismittelpunkt. �

D.22 Beweis: (Bild) Wie wir in Aufgabe D.21 gesehen ha-ben, sind die Vierecke AEDB, BFEC und CDFA Sehnen-vierecke, in denen die Hohen AD, BE bzw. CF paarweiseSehnen sind und die sich daruber hinaus im Hohenschnitt-punkt H schneiden. Die behaupteten Gleichungen

HA ·HD = HB ·HE = HC ·HFfolgen dann direkt aus der Anwendung des Sehnensatzes (vgl.Aufgabe K.11). A B

C

D

E

F

H

140 LOSUNGEN: DREIECKE

D.23 Beweis: (Bild) Die Hohenfußpunkte seien D, E, F , die gespiegelten Punkte ent-sprechend D′, E ′, F ′. Es genugt nachzuweisen, daß

�ACB + �AF ′B = 180◦

gilt. Offenbar sind �AFH und �CDH ahnliche Dreiecke(zwei gleiche Winkel), ebenso ist �BFH ∼ �CEH. Au-ßerdem ist wegen HF = FF ′ und AB ⊥ HF ′ das ViereckAF ′BH ein Drachenviereck. Somit ergibt sich

�ACB = �ECH + �DCH = �FBH + �FAH

= �FBF ′ + �FAF ′ = 180◦ − �AF ′B.

A B

C

D

D ′

EE ′

F

F ′

H

Damit ist gezeigt, daß F ′ auf dem Umkreis von �ABC liegt; gleiches folgt fur die beidenanderen Punkte D′ und E ′. �

D.24 Beweis: (Bild) Wir haben zwei Falle zu unterscheiden; a) �ABC ist spitzwink-lig und b) �ABC hat einen stumpfen Innenwinkel, so daß der Hohenschnittpunkt Hbekanntlich außerhalb des Dreiecks liegt. Bezeichnen wir �HAB = �DAB = ε, sokonnen wir im ersten Fall wie folgt schließen (Bild a): �BDA ist rechtwinklig, also ist�ABD = �ABC = 90◦−ε. Letzterer ist Peripheriewinkel uber der Sehne AC des Umkrei-ses, somit ist der zugehorige Zentriwinkel doppelt so groß: �AOC = 2�ABC = 180◦−2ε.Da �AOC gleichschenklig ist, sind die Basiswinkel dieses Dreiecks nach dem Innenwinkel-

a)

A B

C

D

F

HO

ε

ε

ε

b)

A B

C

D

F

H

O

satz �OAC = �OCA = 12[180◦−(180◦−2ε)] = ε. Im Falle eines stumpfwinkligen Dreiecks

sei �HAB = �DAB = ε (Bild b). Der Außenwinkel �ABC des rechtwinkligen �BDAbetragt somit 90◦+ε und ist gleichzeitig Peripheriewinkel von AC, der nicht auf derselbenSeite wie der Zentriwinkel �AOC liegt. Daher ist �AOC = 360◦− 2�ABC = 180◦− 2ε.Wie in a) ist �AOC gleichschenklig, damit gilt �OAC = ε = �HAB. �

Ceva & Menelaus 141

D.25 Beweis: (Bild) Wir bezeichnen den Hohenfußpunkt von C auf der Seite AB mitF ; CC ′ sei ein Durchmesser des Umkreises. Dann ist es nicht schwer zu bemerken, daßdie Dreiecke CAF und CC ′B ahnlich sind: Beide sind rechtwinklig (letzteres da CC ′ einDurchmesser ist) und haben gleiche Peripheriewinkel uber derSehne BC; somit gilt fur den dritten Winkel

ε ≡ �ACH = 90◦ − α = �C ′CB.

Wegen �ACB = γ = 180◦−α−β errechnen wir fur den betrach-teten Winkel

�HCO = γ − 2ε = 180◦ − α− β − 180◦ + 2α = α− β.

A B

C

C ′

F

H O

ε ε

Damit der Beweis auch im Fall α < β gultig bleibt, schreiben wir �HCO = |α− β|. �

D.31 Beweis: (Bild) Der gemeinsame Schnittpunkt der Ecktransversalen sei K. Wirzeichnen die Parallele zu g(A,B) durch C und bringen diese mit den Verlangerungen derTransversalen AX und BY zum Schnitt; es ent-stehen die Punkte D und E. Damit erhalten wirfolgende Paare ahnlicher Dreiecke:

�ABY ∼ �CEY, �BAX ∼ �CDX,

�CEK ∼ �ZBK, �CDK ∼ �ZAK.

Mit Hilfe der Strahlensatze konnen wir nun dar-aus in dieser Reihenfolge die Proportionen

A B

C DE

K

XY

Z

CY

YA=CE

AB,

BX

XC=AB

CD,

ZB

CE=ZK

CK=

AZ

CDoder

AZ

ZB=CD

CE

ablesen. Die Behauptung folgt schließlich, wenn wir diese Gleichungen miteinander mul-tiplizieren:

AZ

ZB· BXXC

· CYYA

=CD

CE· ABCD

· CEAB

= 1. �

D.32 (Bild) Bezeichnen wir die Winkel α1, . . . , γ2 wie im Bild gezeigt, so finden wir furAZ und ZB mit Hilfe des Sinussatzes

AZ = CZsin γ1

sinα, ZB = CZ

sin γ2

sin β,

somit

AZ

ZB=

sin γ1

sin γ2

· sin β

sinα.

Analog erhalten wir (durch zyklische Vertauschung)

A B

C

XY

Zα 1

α 2 β1β2

γ1γ2

BX

XC=

sinα1

sinα2

· sin γ

sin β,

CY

YA=

sin β1

sin β2

· sinα

sin γ.

142 LOSUNGEN: DREIECKE

Die Cevasche Gleichung (D.2) schreibt sich dann als

sinα1 sin β1 sin γ1

sinα2 sin β2 sin γ2

= 1. (D.102)

D.33 Beweis: (Bild) Wir fuhren den Beweis indirekt, indem wir annehmen, daß

AZ

ZB· BXXC

· CYYA

= 1

erfullt ist, wenn eine der Ecktransversalen CZ nicht durchden Schnittpunkt K der beiden anderen geht. Dann ziehenwir die Ecktransversale, die durch K geht, und die AB inZ ′ schneiden moge. Dann sagt uns der Satz von Ceva, daß

A B

C

K

XY

ZZ ′AZ ′

Z ′B· BXXC

· CYYA

= 1

gilt. Beide Gleichungen lassen sich jedoch wegen AZ+ZB = AZ ′+Z ′B nur fur AZ/ZB =AZ ′/Z ′B, also AZ = AZ ′ und ZB = Z ′B bzw. Z = Z ′ erfullen, was im Widerspruch zuunserer Annahme Z �= Z ′ steht. �

D.34 Beweis: a) Sind die Punkte X, Y , Z die Seitenmitten von BC, CA, AB (Bild a),so ist jeder Teilungsfaktor

AZ

ZB=BX

XC=CY

YA= 1, somit auch

AZ

ZB· BXXC

· CYYA

= 1.

Nach der Umkehrung des Satzes von Ceva schneiden sich die drei Seitenhalbierenden ineinem Punkt. � b) Bezeichnen jetzt X, Y , Z die Schnittpunkte der Winkelhalbierenden

a)

A B

C

GXY

Z

b)

A B

C

I XY

Z

c)

A B

C

H X

Y

Z

mit deren gegenuberliegenden Seiten und wie ublich BC = a, CA = b, AB = c (Bild b),so gilt nach dem Satz aus Aufgabe D.8

AZ

ZB=b

a,

BX

XC=c

b,

CY

YA=a

c, also

AZ

ZB· BXXC

· CYYA

=bca

abc= 1, (D.103)

so daß sich die drei Winkelhalbierenden ebenfalls in einem Punkt schneiden. � c) Hierseien X, Y , Z die Hohenfußpunkte auf BC, CA, AB (Bild c). Dann sind die rechtwink-ligen Dreiecke CZB und AXB ahnlich, da sie außer dem Rechten den Winkel �ABCgemeinsam haben, also in allen drei Winkeln ubereinstimmen. Somit gilt

ZB

CB=BX

ABoder

BX

ZB=AB

CB=c

a.

Ceva & Menelaus 143

Analog folgt �AXC ∼ �BYC bzw. �BYA ∼ �CZA und daraus CY/XC = a/b

sowie AZ/YA = b/c. Die Multiplikation dieser drei Verhaltnisgleichungen liefert wiederum(D.103); damit schneiden sich auch die Hohen in einem gemeinsamen Punkt. �

D.35 Beweis: (Bild) Wir zeigen die Behauptung an einem Ankreis: Sei A′ derBeruhrungspunkt des Ankreises (Mittelpunkt Ia) mit der Seite BC; K, L diejenigen mitden (verlangerten) Seiten AC bzw. AB. Dadurch, daß wir drei Tangenten an den Ankreishaben, liegen mehrere gleiche Tangentenabschnitte vor:

AK = AL, CK = CA′, BL = BA′.

Somit ist

AK = AC + CK = AC + CA′

= AL = AB +BL = AB +BA′,

also AB + BA′ = AC + CA′. Vollig ahnlich sind dieLangenverhaltnisse bezuglich der anderen Ankreise. � A

A′

B

CIa

K

L

D.36 Beweis: (Bild) Die im Satz von Ceva auftretende Gleichung ist so universell, daßsie auch bestehen bleibt, wenn wir auf beiden Seiten die Kehrwerte bilden:

ZB

AZ· XCBX

· YACY

= 1.

Fur die isotomischen PunkteX ′, Y ′, Z ′ gilt nun bekannter-maßen ZB = AZ ′, AZ = ZB′, usw. so daß wir daraus

AZ ′

Z ′B· BX

X ′C· CY

Y ′A= 1

A B

C

P

P′

X

X ′

Y

Y ′

ZZ ′

erhalten. Dies ist nach der Umkehrung des Satzes von Ceva genau die Bedingung dafur,daß sich die isotomischen Geraden ebenfalls in einem Punkt treffen. �

D.37 Beweis: (Bild) Betrachten wir anstelle der Ecktransversalen AX, BY , CZ derenisogonale Gegenstucke AX ′, BY ′, CZ ′, so gilt nach Definition

�BAX = �X ′AC,

�CBY ′ = �Y BA,�ACZ ′ = �ZCB.

Fur die trigonometrische Form (D.102) der CevaschenGleichung (vgl. Aufgabe D.32),

sinα1 sin β1 sin γ1 = sinα2 sin β2 sin γ2, A B

C

P

P′X

X ′

Y

Y ′

ZZ ′

bedeutet das eine Vertauschung der Winkel α1, β1, γ1 mit den Winkeln α2, β2, γ2, welchedie Bedingung naturlich unverandert laßt. �

144 LOSUNGEN: DREIECKE

D.38 Beweis: (Bild) Zunachst gilt offensichtlich stetsAZ · BX · CY = BX · CY · AZ, unabhangig davon,welche Bedeutung diese Strecken haben. Nun sind X,Y , Z die Beruhrungspunkte des Inkreises mit den Sei-ten, folglich sind AZ und YA gleich lange Tangenten-abschnitte; ebenso gilt BX = ZB und CY = XC.Setzen wir diese Beziehungen in obige Gleichung ein,erhalten wir

A B

C

GeI

X

Y

ZAZ

ZB· BXXC

· CYYA

= 1 · 1 · 1 = 1.

Nach der Umkehrung des Satzes von Ceva folgt somit die Behauptung. �

D.39 Beweis: (Bild) Da die Beruhrungspunkte der Ankreise und die Halbumfangs-punkte zusammenfallen, genugt es, hier nur letztere zu betrachten. Die Bedingung dafur,daß z. B. Punkt X auf der Seite BC gerade auf halbem Umfange liegt, ist c + BX =XC + b; andererseits ist BX + XC = a. Dieses einfachelineare Gleichungssystem hat die Losung

BX =1

2(a+ b− c) = s− c ≡ z,

XC =1

2(c+ a− b) = s− b ≡ y.

Analog gilt fur die anderen Halbumfangspunkte Y , Z: A B

C

Na

X

Y

Z x

x

y

y

zz

CY =1

2(b+ c− a) = s− a ≡ x, YA = z, AZ = y, ZB = x.

Fugen wir diese Ergebnisse zusammen, ergibt sich

AZ

ZB· BXXC

· CYYA

=y

x· zy· xz

= 1,

womit nach der Umkehrung des Satzes von Ceva die Behauptung folgt. �Bemerkung: Bezuglich der Großen x, y, z vgl. auch Aufgabe D.63 sowie Abschnitt G.1.1.

D.40 Beweis: Wie aus beiden Bildern zu erkennen ist, sind die Abschnitte x und y aufder Seite AB gerade vertauscht; gleiches gilt auch auf den anderen Dreieckseiten. Dies

a)

A B

C

Ge

X

Y

Zx

x

y

y

zz

b)

A B

C

Na

X

Y

Z x

x

y

y

zz

sind aber genau die Bedingungen dafur, daß die zugehorigen Ecktransversalen isotomischkonjugiert sind. �

Ceva & Menelaus 145

D.41 Beweis: Die Symmedianen sind die isogonalen Geraden zu den Seitenhalbierenden.Da sich letztere im Schwerpunkt schneiden, mussen sich die Symmedianen nach AufgabeD.37 auch in einem Punkt treffen. �Bemerkung: Dieser Satz konnte auch so formuliert werden:

In einem Dreieck sind der Schwerpunkt und der Lemoine-Punkt isogonal konjugiertePunkte.

D.42 Aus Aufgabe D.24 ist uns bekannt, daß z. B. der Winkel �HAB gleich dem Winkel�OAC ist; die Geraden AH und AO somit isogonale Geraden sind. Gleiches gilt auch furdie Paare BH, BO und CH, CO. Der Hohenschnittpunkt und der Umkreismittelpunktsind demnach isogonal konjugierte Punkte.

D.43 Beweis: Wenn eine Gerade die Seiten eines Dreiecks (bzw. deren Verlangerungen)schneidet, so kann dieses nur geschehen, indem entweder keine Seite direkt geschnittenwird (Fall 1, Bild a) oder zwei Seiten innere Schnittpunkte mit der Geraden haben (Fall 2,Bild b). Anders ausgedruckt, mussen entweder alle drei Seiten oder nur eine Dreieckseite

a)

A

B

C

XY Z

h1 h2h3

b)

A B

C

X

Y

Z

h1

h2

h3

verlangert werden. In beiden Fallen bezeichnen wir mit h1, h2, h3 die Langen der Lotevon den Eckpunkten A, B, C des Dreiecks auf die Gerade g(Y, Z). Die Strahlensatzeliefern uns nun unter Berucksichtigung des Vorzeichens (positives bei innerer, negativesbei außerer Teilung) folgende Gleichungen

Fall 1 :AZ

ZB= −h1

h2

,BX

XC= −h2

h3

,CY

YA= −h3

h1

,

Fall 2 :AZ

ZB= −h1

h2

,BX

XC=h2

h3

,CY

YA=h3

h1

,

die jeweils miteinander multipliziert direkt die behauptete Gleichung ergeben:

AZ

ZB· BXXC

· CYYA

= −h1 · h2 · h3

h2 · h3 · h1

= −1. �

D.44 Beweis: (Bild) X, Y und Z seien drei Punkte,jeder auf einer Seite des Dreiecks ABC, fur die das Pro-dukt der Verhaltnisse gleich −1 sei. Außerdem schneideXY die Seite AB in Z ′. Zu zeigen ist also Z ′ = Z. DerSatz von Menelaus fordert nun

A B

C

X

Y

ZZ ′

146 LOSUNGEN: DREIECKE

AZ ′

Z ′B· BXXC

· CYYA

= −1,

was jedoch

AZ ′

Z ′B· BXXC

· CYYA

=AZ

ZB· BXXC

· CYYA

,AZ ′

Z ′B=AZ

ZB,

AZ ′ + Z ′BZ ′B

=AZ + ZB

ZB

bedeutet. Wegen AB = AZ ′ + Z ′B = AZ + ZB muß aber Z ′B = ZB bzw. Z ′ = Z sein,d. h., X, Y und Z sind kollinear. �

D.45 Beweis: (Bild) Nach Voraussetzung sind PX, PY die Lote auf AB, BC (bzw. aufderen Verlangerungen), also gilt �BXP = �BY P = 90◦, und XBY P ist somit ein Seh-nenviereck. Ebenso folgt aus �CZP = �CY P = 90◦, daßCY PZ ein Sehnenviereck ist. Schließlich ist auch ABCP einSehnenviereck. Nun kann unter Verwendung des Peripherie-winkelsatzes folgende Gleichungskette aufgestellt werden:

A B

CP

X

Y

Z�XY P = �XBP (Peripheriewinkel uber XP )

= �ABP (da X auf AB)

= �ACP (Peripheriewinkel uber AP )

= �ZCP (da Z auf AC)

= �ZY P (Peripheriewinkel uber ZP ).

Es ist also �XY P − �ZY P = �XY Z = 0, was beweist, daß X, Y und Z auf einerGeraden liegen. �

D.46 Beweis: (Bild) Wir bezeichnen die Schnittpunkte der Tangenten in A, B, C mitden jeweils gegenuberliegenden Seiten mit X, Y , Z. Der Satz von Menelaus verlangt

A B

C

X

Y

Z

nun die Berechnung der VerhaltnisseAZ/ZB, BX/XC und CY/YA. Diedarin auftretenden sechs Langen sindSeiten von Dreiecken, die in dieser Figurzahlreich vorhanden sind. Wenn wir al-so ahnliche Dreiecke finden, konnen wirauf eine Losung hoffen. Tatsachlich istz. B. im Dreieck ZCA der Winkel beiC Sehnen-Tangentenwinkel und damitgleich dem Peripheriewinkel �ABC =�ZBC. Die beiden Dreiecke ZCA undZBC haben außerdem noch den Win-

kel bei Z gemeinsam, sind somit ahnlich. Analog schließen wir aus �Y BA = �Y CB =�ACB = �XCA = �XAB: �XAB ∼ �XCA und �Y BC ∼ �YAB. Aus diesenAhnlichkeiten lesen wir nun folgende Proportionen ab:

AZ

CZ=CZ

ZB=b

a,

BX

AX=AX

XC=c

b,

CY

BY=BY

YA=a

c.

Ceva & Menelaus 147

Die Seiten AB, BC und CA werden also außerlich in den Verhaltnissen

AZ

ZB=AZ

CZ· CZZB

= − b2

a2,

BX

XC=BX

AX· AXXC

= −c2

b2,

CY

YA=CY

BY· BYYA

= −a2

c2

geteilt. Multiplikation aller drei Gleichungen ergibt (AZ/ZB)·(BX/XC)·(CY/YA) = −1;nach der Umkehrung des Satzes von Menelaus liegen X, Y , Z auf einer Geraden. �

D.47 Beweis: (Bild) Durch die beschriebene Konstruktion entstehen drei Aufsatzdrei-ecke BDC, CEA und AFB, die durch die Gera-den AD, BE bzw. CF in insgesamt sechs klei-nere Dreiecke zerlegt werden. Fuhren wir nochdie im Bild bezeichneten Winkel α1, α2, β1, β2,γ1, γ2 ein, konnen wir mit Hilfe des Sinussatzesdie benotigten Abschnitte auf den Dreieckseitenberechnen:

A B

C

D

E

F

K X

Y

Z

δ

δ εε

ηη

α 1

α 2

β1β2

γ1

γ2

AZ = FZsin γ1

sin δ, ZB = FZ

sin γ2

sin ε,

BX = DXsinα1

sin ε, XC = DX

sinα2

sin η,

CY = EYsin β1

sin η, YA = EY

sin β2

sin δ.

Unser Zielausdruck (D.2) nimmt damit zunachst folgende Form an:

AZ

ZB· BXXC

· CYYA

=sin γ1

sin γ2

· sinα1

sinα2

· sin β1

sin β2

. (D.104)

Die Hilfswinkel α1, . . . , γ2 werden wir wieder los, indem wir die Dreiecke ABD, CAD,BCE, ABE, CAF , BCF heranziehen: Diese haben paarweise die Winkel β + ε, γ + ηund α+ δ sowie die Seiten AD, BE, CF gemeinsam. Der Sinussatz liefert hier:

sinα1 = sin(β + ε)c

AD, sinα2 = sin(γ + η)

c

AD,

sin β1 = sin(γ + η)a

BE, sin β2 = sin(α+ δ)

a

BE,

sin γ1 = sin(α+ δ)b

CF, sin γ2 = sin(β + ε)

b

CF.

Setzen wir diese Gleichungen in (D.104) ein, kurzt sich auf wundersame Weise alles heraus,so daß tatsachlich 1 ubrig bleibt und damit nach der Umkehrung des Satzes von Ceva

die Behauptung bewiesen ist. �

148 LOSUNGEN: DREIECKE

D.51 (Bild) Bleiben wir kurz bei dem physikalischen Problem der Reflexion der Billard-kugel an der Bande. Dies ist in guter Naherung ein elastischer Stoß, und dabei gilt dasReflexionsgesetz, welches besagt, daß der Einfallswinkelα gleich dem Reflexionswinkel β ist. Dies konnte uns aufdie Idee bringen, Zielpunkt B an der Bande (dargestelltdurch die Gerade g) zu spiegeln; wir erhalten so PunktB′. Dann ist �BCB′ ein gleichschenkliges Dreieck, indem g Winkelhalbierende ist. Damit nun α = β wird,muß der gesuchte Punkt C auf g gerade der Schnitt-punkt der Geraden AB′ mit g sein.Es bleibt jetzt noch zu zeigen, daß AC+CB tatsachlich

A

B

B ′

CC ′gα

ββ

ein Minimum ist. Dies bedeutet, es ist AC + CB < AC ′ + C ′B fur jeden anderen PunktC ′ auf g. Aufgrund der Spiegelung ist aber

AC + CB = AC + CB′ = AB′ und AC ′ + C ′B = AC ′ + C ′B′.

Nach der Dreiecksungleichung ist AB′ < AC ′ +C ′B′, also auch AC +CB < AC ′ +C ′B,wie behauptet. �

D.53 (Bild) Wir betrachten zunachst einen willkurlich gewahlten Punkt F innerhalbdes Dreiecks und verbinden ihn mit A, B und C. Wenn es uns gelingt, die drei StreckenAF , BF , CF so anzuordnen, daß sie sich aneinanderreihen und außerdem noch auf einerGeraden zu liegen kommen, ware die Minimaleigenschaft ihrer Gesamtlange offensicht-lich und der gesuchte Punkt F gefunden.Wir wahlen als Gerade diejenige, die durchdie Punkte F und B geht, und drehen das�AFC um 60◦ um A nach außen. Dadurchentsteht das �ADE. Die Dreiecke ACE

und AFD sind somit gleichseitig, und esgilt fur die Summe der Abstande

A B

C

DE

F

d = CF + AF +BF = ED +DF + FB.

Im allgemeinen ist der Weg von E nach B gebrochen mit Winkeln ungleich 180◦ bei Dund F . Die Summe d wird minimal, wenn alle drei Teilstrecken auf einer Geraden liegen,d. h. �EDF = �DFB = 180◦ ist. Dies ist genau dann der Fall, wenn

�AFB = 180◦ − �AFD = 120◦ und �AFC = �ADE = 180◦ − �ADF = 120◦

gilt. Der gesuchte sog. Fermat-Punkt F ist somit derjenige, von dem aus jede der dreiSeiten des Dreiecks unter einem Winkel von 120◦ gesehen wird. Die vier Punkte A, C, Eund F bilden ein Sehnenviereck, d. h. F liegt auf dem Umkreis des gleichseitigen �ACE.Dies wird ersichtlich, wenn wir die Winkel �ECA = �EFA = 60◦ betrachten. Beide sindPeripheriewinkel uber der Sehne EA. Zur Konstruktion von F errichten wir z. B. auf derSeite AC das gleichseitige �ACE und bringen dessen Umkreis mit der Strecke EB zumSchnitt. Der zweite, von E verschiedene Schnittpunkt ist dann der Fermat-Punkt F .

Extremalaufgaben 149

D.54 Beweis: (Bild) a) Die beiden Dreiecke ACUund V CB sind nach Kongruenzsatz SWS einanderkongruent, da sie in ihren Seiten CU = CB = a,AC = V C = b sowie dem eingeschlossenen Win-kel �ACU = γ + 60◦ = �V CB ubereinstimmen.Daraus folgt sofort AU = BV . Ebenso kann manaus der Kongruenz der Dreiecke �V AB ∼= �CAWschließen, daß BV = CW gelten muß, mithinalso die geforderte Gleichung. Die drei StreckenAU , BV , CW schneiden sich im Fermat-PunktF und fur die Abstande gilt: AU = BV = CW =AF +BF + CF (vgl. Aufgabe D.53). �b) Um zu zeigen, daß F ebenfalls der Fermat-Punkt von �UVW ist, genugt es

A B

C

F

U

V

W

a

ab

b

�UFV = �V FW = �WFU = 120◦ (D.105)

nachzuweisen. Betrachten wir dazu das Sehnenviereck AFBW mit �AFB = 120◦. Dadie Bogen bzw. Sehnen AW und BW gleich sind, gilt nach dem Peripheriewinkelsatz�AFW = �BFW = 60◦. Ebenso sind FU und FV Winkelhalbierende von �BFC und�CFA, woraus

�AFW = �WFB = �BFU = �UFC = �CFV = �V FA = 60◦

und damit (D.105) folgt. �

D.55 (Bild) Der Knoten befindet sich genau dann in Ruhe, wenn die Summe der an ihmangreifenden Gewichtskrafte gleich null ist. Da Krafte entlang ihrer Wirkungslinie, hieralso entlang der Faden, verschoben werden konnen, muß fur die drei in die Tischebeneverschobenen Krafte F 1, F 2 und F 3 im gesuchten Punkt

F 1 + F 2 + F 3 = 0 mit |F 1| = |F 2| = |F 3|gelten. Die Vektorsumme dreier gleich langer Vektorenverschwindet genau dann, wenn sie paarweise Winkelvon 120◦ einschließen. Somit kommt der Knoten genauim Fermat-Punkt (vgl. Aufgabe D.53) des Dreieckszur Ruhe. A B

C

F1

F2

F3

D.56 (Bild) Wir betrachten zunachst ein beliebiges Dreieck UVW mit U auf BC, Vauf CA und W auf AB. Seien W ′ und W ′′ die Spiegelbilder von W an BC bzw. CA.Dann gilt

WU + UV + VW = W ′U + UV + VW ′′.

Dies ist im allgemeinen ein gebrochener Weg vonW ′ nach W ′′ mit Winkeln ungleich 180◦ bei U undV . Ein solcher Weg ist dann so kurz wie moglich,wenn er gerade wie im Bild ist. Unter allen ein-geschriebenen Dreiecken mit fester Ecke W aufAB besitzt daher dasjenige den kleinsten Umfang,

A B

C

U

V

W

W ′

W ′′

150 LOSUNGEN: DREIECKE

fur welche U und V auf der Geraden W ′W ′′ liegen. Es ist also W auf AB so zu wahlen,daß der Umfang W ′W ′′ minimal wird. Da CW ′ und CW ′′ Bilder von CW bei Spiegelungan BC bzw. CA sind, sind diese Strecken gleich und somit ist

�W ′CW ′′ = 2�BCA.

Somit ist im gleichschenkligen �CW ′W ′′ der Winkel bei C unabhangig von der Wahl vonW . Die Basis ist am kurzesten, wenn die Schenkel moglichst klein sind, d. h., wenn CW

minimal ist. CW ist daher die von C auf AB gefallte Hohe. Dies fuhrt zu dem Ergebnis:

Das einem spitzwinkligen Dreieck ABC eingeschriebene Dreieck kleinsten Umfangs ist dasHohenfußpunktdreieck.

D.57 (Bild) Mit PA ≡ x, PB ≡ y und PC ≡ z lautet unsere Extremwertaufgabex2 + y2 + z2 ⇒ Min. Die auftretenden Quadrate lassen hier die Verwendung von Vektorenals geeignet erscheinen: Schreiben wir nach Abschnitt M.1 fur die Ortsvektoren

−→OA ≡ a,−−→

OB ≡ b,−→OC ≡ c,

−→OP ≡ p (wobei O ein beliebiger Punkt ist), so finden wir mit der

Abkurzung 3g ≡ a + b + c fur den zu minimierenden Ausdruck

x2 + y2 + z2 = (p − a)2 + (p − b)2 + (p − c)2

= 3p2 − 2p(a + b + c) + a2 + b2 + c2

= 3p2 − 6pg + 3g2 − 3g2 + a2 + b2 + c2

= 3(p − g)2 − 3g2 + a2 + b2 + c2.

Dieser Ausdruck nimmt offensichtlich ein Minimum an, wenndie runde Klammer verschwindet, also fur p = g. Nach Aufgabe

A B

C

Px

y

z

M.1 ist g leicht als Ortsvektor zum Schwerpunkt des Dreiecks zu erkennen; demnach istder gesuchte Punkt der Schwerpunkt G von �ABC.

D.58 Wir lassen zunachst einmal jedwede Geometrie außer Acht und nehmen dieCauchy-Schwarzsche Ungleichung (vgl. Aufgabe U.11) mit den beiden Tripeln (a, b, c)und (u, v, w). Gleichung (U.13) lautet dann

(a2 + b2 + c2)(u2 + v2 + w2) ≥ (au+ bv + cw)2.

Der rechts stehende Ausdruck ist aber gerade das Quadrat des doppelten Flacheninhaltsdes Dreiecks, so daß daraus

u2 + v2 + w2 ≥ 4∆2

a2 + b2 + c2(D.106)

folgt. Nun sagt uns entweder eine geometrische Interpretation des rechts stehenden Aus-drucks in (D.106) oder (U.13), wann Gleichheit vorliegt: Beide Tripel (a, b, c) und (u, v, w)mussen proportional sein, d. h.

u

a=v

b=w

c= λ ∈ R.

Diese Bedingung ist identisch mit derjenigen aus Aufgabe D.91; es handelt sich demnachbei dem gesuchten Punkt um den Lemoine-Punkt des Dreiecks ABC.

Einige Formeln 151

Bemerkung: Aus (D.106) wird unter Beachtung von Aufgabe D.73 mit dem Brocard-Winkel ω

u2 + v2 + w2 ≥ ∆

cotω.

D.59 Wenn es darum geht, Summen von Streckenlangen zu vergleichen, die nicht aufeiner Geraden liegen (hier QP + PR), ist es immer eine gute Idee, diese auf eine Geradezu bringen. Wir erreichen dies hier, indem wir anstelle von R ∈ g denjenigen Punkt R′

auf der Verlangerung von QP betrachten, fur den PR′ = PR gilt (Bild a). In dem so

a)

P

Q

R

R ′

U

V

g

hb)

P

Q

R

R ′

U

V

g

entstehenden �RR′Q ziehen wir noch die Parallelen zu RQ und RR′ jeweils durch P , sodaß PURV ein Parallelogramm wird. Nach dem ersten Strahlensatz finden wir somit

QP + PR

QR=QR′

QR=QP

QV⇒ Max.

Wegen QP = const lautet die Forderung QV ⇒ Min. Das Dreieck R′PR ist aber furjede beliebige Lage von R gleichschenklig mit gleichen Basiswinkeln, weshalb die Geradeh(P, V ) als Parallele zu R′R ebenfalls unabhangig von R ist. Unsere obige Minimalfor-derung wird also genau dann erfullt, wenn V der Lotfußpunkt von Q auf h ist (Bild b).Aus �PR′R = �R′RP = �RPV = �QPV folgt: �PV Q ∼= �PV R; also ist �QPRgleichschenklig mit PR = PQ.

D.61 Beweis: (Bild) Drehen wir das gegebene �ABC

um den Mittelpunkt M der Seite BC um 180◦, so gehtA in A′, B in B′ = C und C in C ′ = B uber, undABA′B′ ist wegen �ABC ∼= �A′B′C ′ offensichtlichein Parallelogramm. D und E seien die Lotfußpunktevon A bzw. B auf der Geraden g(A′, B′). Dann ist nachKongruenzsatz SWS �C ′A′E ∼= �ACD und somit

A

A′

B=C ′

C=B ′D E

M

2∆ = [ABA′B′] = [C ′A′E] + [ABEB′] = [ABED] = AB ·BE = chc.

Durch Drehung an den anderen beiden Seiten erhalten wir analog 2∆ = aha = bhb. �

152 LOSUNGEN: DREIECKE

D.62 Beweis: (Bild) Wir verwandeln das gegebene Dreieck ABC in ein flachengleichesrechtwinkliges Dreieck ABD, indem wir z. B. den Punkt D als Lotfußpunkt von A aufder Parallelen zu AB durch C bestimmen (s. Aufgabe D.61). Dann benotigen wir nurnoch die Lange der Strecke AD, die sich aber im rechtwinkligenDreieck ADC aus der Definition des Sinus zu AD = b sinαergibt (�BAC = �DCA sind Wechselwinkel). Somit ist:

A B

CD

b

α

∆ = [ABC] = [ABD] =1

2c · AD =

1

2bc sinα.

Vollig analog erhalten wir die anderen Gleichungen. �

D.63 Beweis: (Bild) Der Mittelpunkt I des Inkreises zerlegt die Dreiecksflache in dreiTeildreiecke ABI, BCI und CAI. Bezeichnen wir die Beruhrungspunkte des Inkreisesmit den Seiten BC, CA, AB mit D, E, F , so sind

ID = IE = IF = r

offenbar gerade die Hohen dieser Teildreiecke, da dieBeruhrungsradien stets senkrecht auf den tangieren-den Seiten stehen. Daruber hinaus sind die StreckenAE = AF = x, BF = BD = y sowie CD = CE = zTangentenabschnitte von paarweise gleicher Lange, sodaß mit s = 1

2(a+ b+ c) = x+ y + z gilt: A B

C

I

DE

F

x

x y

y

zz

r

∆ = [ABI] + [BCI] + [CAI] =1

2

[(x+ y)r + (y + z)r + (z + x)r

]=

1

2r · 2(x+ y + z) = rs. �

Bemerkung: Die drei Gleichungen x + y = c, y + z = a, z + x = b ergeben — aufgelostnach den x, y, z — die oft nutzlichen Beziehungen (vgl. Abschnitt G.1.1)

x =1

2(b+ c− a), y =

1

2(c+ a− b), z =

1

2(a+ b− c).

D.64 Beweis: (Bild) Wir zeichnen außer dem Umkreis von �ABC mit seinem Mittel-punkt O noch den Durchmesser CM sowie die Hohe CF = hc ein. Dann sind die beidenDreiecke AFC und MBC ahnlich, da sie einerseits rechtwinkligsind und außerdem nach dem Peripheriewinkelsatz noch in ihrenWinkeln �CAF = �CAB = �CMB ubereinstimmen. Mithingilt die Proportion

hcb

=a

2Roder hc =

ab

2R,

welche in ∆ = 12chc eingesetzt ∆ = abc/(4R) ergibt. �

A B

C

FM

O

ab

c

hc

Einige Formeln 153

D.65 Beweis: Das Bild zeigt die drei Ankreise mit den Mittelpunkten Ia, Ib, Ic bzw.Radien ra, rb, rc sowie den Inkreis des Dreiecks ABC. Betrachten wir z. B. das ViereckAIcBC, so finden wir fur dessen Flacheninhalt [AIcBC] = [CBIc]+ [CAIc] = [ABIc]+∆.

A B

C

I

Ia

Ib

Ic

ra

rb

rc

Nun hat jedes der Dreiecke CBIc, CAIc und ABIc gerade rc als Hohe auf den GrundseitenBC = a, CA = b bzw. AB = c, so daß gilt:

∆ = [CBIc] + [CAIc] − [ABIc] =1

2(a+ b− c)rc = (s− c)rc.

Vollig ahnliche Ausdrucke fur ∆ erhalten wir, wenn wir die obigen Betrachtungen auf dieVierecke BIaCA und CIbAB anwenden: ∆ = (s− a)ra = (s− b)rb. �

D.66 Beweis: Um die Richtigkeit dieser Formel zu zeigen, benotigen wir nur ein wenigTrigonometrie. Nach dem Kosinussatz ist

cosα =b2 + c2 − a2

2bc

und damit

sinα =√

1 − cos2 α =

√−a4 − b4 − c4 + 2b2c2 + 2c2a2 + 2a2b2

2bc.

Diesen Ausdruck setzen wir in ∆ = 12bc sinα ein:

∆ =1

4

√−a4 − b4 − c4 + 2b2c2 + 2c2a2 + 2a2b2

=1

4

√(a+ b+ c)(b+ c− a)(c+ a− b)(a+ b− c) =

√s(s− a)(s− b)(s− c). �

D.67 Beweis: Durch Gleichsetzen der Ausdrucke (D.6) und (D.7) fur den Flacheninhaltin den Aufgaben D.63 und D.64 erhalten wir die angegebene Gleichung. �

154 LOSUNGEN: DREIECKE

D.68 Beweis: Aus Aufgabe D.65 und D.63 folgt:

1

ra=s− a

∆,

1

rb=s− b

∆,

1

rc=s− c

∆,

1

ra+

1

rb+

1

rc=

3s− 2s

∆=

s

∆=

1

r. �

D.69 Beweis: (Bild) Die bei Z auftretenden Winkel �AZC = z und �BZC = π − z

sind Supplementwinkel, so daß wir die Gleichung cos z = − cos(π− z) ausnutzen konnen.Nach dem Kosinussatz gilt nun in den Dreiecken AZC und BZC:

cos z =m2 + t2 − b2

2mt, cos(π − z) =

n2 + t2 − a2

2nt.

Diese Ausdrucke in obige Gleichung eingesetzt, ergibt

A B

C

Z

abt

m nz zπ

t2 =ma2

c+nb2

c−mn =

ma2 + nb2

m+ n−mn. �

D.70 (Bild) Wir wenden den Satz von Stewart an und mussen lediglich geeigneteAusdrucke fur die Langen der Abschnitte m und n finden:

t =

√ma2 + nb2

m+ n−mn.

a) Auf der Seite c ist m = n = 12c und damit

A B

C

Z

abt

m nmc =

√1

2(a2 + b2) − 1

4c2 (D.107)

b) Dank Aufgabe D.8 kennen wir hier das Teilungsverhaltnis m : n = b : a. Mit c = m+nergibt sich m = bc/(a+ b) und n = ac/(a+ b), somit

wc =

√ab

[1 − c2

(a+ b)2

]. (D.108)

c) Hier ist es am einfachsten, (D.5) und (D.9) zu kombinieren:

hc =2

c

√s(s− a)(s− b)(s− c). (D.109)

Analoge Formeln fur ha und hb ergeben sich durch zyklische Vertauschung von (a, b, c).

D.71 Beweis: Nach Aufgabe D.70 b und der Voraussetzung wa = wb gilt:

w2a = bc

[1 − a2

(b+ c)2

]= ca

[1 − b2

(c+ a)2

]= w2

b

und damit

0 = (a− b) +

[a2b

(b+ c)2− ab2

(c+ a)2

]= (a− b) +

a2b(c+ a)2 − ab2(b+ c)2

(b+ c)2(c+ a)2

= (a− b)

[1 + ab

c2 + 2c(a+ b) + a2 + b2

(b+ c)2(c+ a)2

].

Da der zweite Faktor nicht verschwinden kann, folgt zwangslaufig a = b. �

Einige Formeln 155

D.72 (Bild) �ABC wird durch die Hohe CF in zwei rechtwinklige Dreiecke zerlegt, indenen wir den Satz des Pythagoras anwenden konnen:

CF 2 = b2 − AF 2 = a2 −BF 2, oder

b2 − a2 = AF 2 −BF 2 = c(AF −BF ).

Dieses Ergebnis nochmals mit c = AF +BF kombiniert,ergibt

A B

C

F

ab

AF =c2 + b2 − a2

2cund BF =

c2 + a2 − b2

2c.

Bemerkung: Durch zyklische Vertauschung erhalten wir die Formeln fur die Abschnitteauf den anderen Seiten.

D.73 Beweis: (Bild) Es gilt cotα = AF/hc und mit dem Resultat von Aufgabe D.72

fur den Abschnitt AF sowie 4∆ = 2chc:

cotα =b2 + c2 − a2

2chc=b2 + c2 − a2

4∆.

Durch zyklische Vertauschung erhalten wir

A B

C

F

abhc

α βcot β =

c2 + a2 − b2

4∆, cot γ =

a2 + b2 − c2

4∆.

Addition der Gleichungen liefert die Behauptung. �Bemerkung: Es gilt cotω = cotα+ cot β + cot γ mit dem Brocard-Winkel ω.

D.79 Beweis: (Bild) Aus Aufgabe D.3 wissen wir, daß die verlangerte Winkelhalbie-rende von �ACB, auf der I liegt, die Mittelsenkrechte von AB, auf der O liegt, gera-de im Punkt F auf dem Umkreis schneidet. FM sei der Durchmesser des Umkreisessenkrecht zu AB. Schreiben wir zur Abkurzung ε = 1

2�ACB und η = 1

2�CAB, so

ist aus dem Bild leicht �AMF = �ACF = ε und �FAB = �FCB = ε abzulesen.Der Außenwinkel von �CAI bei I ist �AIF = ε+ η =�FAI, �FAI ist demzufolge gleichschenklig: FA = FI.Nun konnen wir den Sehnensatz aus Aufgabe K.11 vor-teilhaft auf die Sehne FC mit ihrem Teilungspunkt Ianwenden:

A B

C

F

I

M

O

Y

d

r

ε ε ε

εη

ηR2 − d2 = FI · IC = FA · IC= FM

FA/FM

IY/ICIY

= FMsin ε

sin εIY = FM · IY = 2Rr.

Damit wird d2 = R2 − 2rR. �

156 LOSUNGEN: DREIECKE

D.81 Beweis: Mit D, E als Fußpunkte der Lote von P auf die Seiten BC, CA gilt furdie Winkel �PDC = �PEC = 90◦, d. h., die Punkte P , D, C, E liegen samtlich aufeinem Kreis mit CP ≡ z als Durchmesser (Bild a). Nun konnen wir den Sinussatz inseiner erweiterten Form (vgl. Aufgabe D.5) bezuglich des Winkels �BCA = γ einmal auf

a)

A B

C

DE

F

P

b)

A B

C

D

E

F

P

c)

A B

C

D

E

F

P

das �EDC, zum anderen auf das �ABC anwenden:

DE

sin γ= z bzw.

c

sin γ= 2R.

Eliminieren wir hieraus sin γ, so folgt die Seitenlange DE = z sin γ = cz/(2R). Analogverfahren wir mit den beiden anderen Sehnenvierecken PEAF und PFBD und erhalten

EF = x sinα =by

2Rbzw. FD = y sin β =

ax

2R. �

Bemerkung: Auch im Falle, daß einige der Lotfußpunkte auf den Verlangerungen derSeiten liegen (Bild b,c) bleibt das Ergebnis dasselbe; wir haben im �EDC lediglich denSupplementwinkel �BCE = 180◦ − γ zu nehmen mit sin(180◦ − γ) = sin γ. In Bild bentartet das Lotfußpunktdreieck gerade zur Simson-Geraden (vgl. Aufgabe D.45).

D.82 (Bild) Wir haben hier zwar nicht so viel rechte Winkel wie beim Hohenfuß-punktdreieck, aber diejenigen an den Lotfußpunkten D, E und F reichen schon, umdrei Sehnenvierecke zu erkennen: PEAF , PFBDund PDCE. In ihnen sind die Strecken PA, PB,PC jeweils Durchmesser. Und wo Sehnenviereckesind, gibt es auch (gleiche) Peripheriewinkel. Neh-men wir o. B. d. A. an, daß das �DEF seinen rech-ten Winkel bei F hat und bezeichnen �PAF ≡ δund �PBF ≡ ε, so gilt nach dem Peripheriewin-kelsatz

�PEF = δ, �PDF = ε.

Aus der Innenwinkelsumme im Viereck CEFD

folgt nun δ + ε = 90◦ − γ. Fur �EFD = 90◦ ist A B

C

D

E

F

P

γ

δ ε

also �APB = 180◦ − (δ + ε) = 90◦ + γ notwendig.

P liegt somit auf einem Kreisbogen, den wir nach Aufgabe A.21 konstruieren konnen.Ferner ist aufgrund der geforderten Gleichschenkligkeit des Lotfußpunktdreiecks �DEF =

Lotfußpunktdreiecke 157

�EDF = 45◦ vorgegeben. Mit demselben Argument finden wir als zweiten geometrischenOrt fur P einen Kreisbogen mit �BPC = 45◦ + α.

D.83 Beweis: (Bild) Seien D1 und D2 die Lotfußpunkte von F und E auf BC; ent-sprechend seien die Punkte E1, E2 und F1, F2 auf den anderen Seiten erklart. Aus Auf-gabe D.82 wissen wir, daß z. B. PFBDein Sehnenviereck ist. Daher ist �FDB =�FDD1 = �FPB und die rechtwinkligenDreiecke FDD1 und BPF sind ahnlich.Daraus folgt fur die Lange

D1D =PF

PB· FD.

Dieses Argument konnen wir insgesamtsechsmal verwenden, was wir an dieserStelle auch einmal ausfuhrlich hinschrei-

A B

C

DD1

D2

E

E1

E2

FF1 F2

P

ben:�DFB = �DFF2 = �DPB ⇒ �DFF2 ∼ �BPD ⇒ FF2 = PD

PB· FD,

�DEC = �DEE1 = �DPC ⇒ �DEE1 ∼ �CPD ⇒ E1E = PDPC

·DE,�EDC = �EDD2 = �EPC ⇒ �EDD2 ∼ �CPE ⇒ DD2 = PE

PC·DE,

�EFA = �EFF1 = �EPA ⇒ �EFF1 ∼ �APE ⇒ F1F = PEPA

· EF,�FEA = �FEE2 = �FPA ⇒ �FEE2 ∼ �APF ⇒ EE2 = PF

PA· EF.

Jetzt nehmen wir unseren Zielausdruck und schatzen ihn mit einem Trick nach unten ab:

PA+ PB + PC ≥ PA · D1D +DD2

EF+ PB · E1E + EE2

FD+ PC · F1F + FF2

DE.

Dies ist offenbar richtig, da z. B. D1D2EF ein Trapez mit benachbarten rechten Winkelnbei D1 bzw. D2 ist und die gegenuberliegende Seite EF nur die minimale Lange D1D2

haben kann. Analog gilt FD ≥ E1E2 und DE ≥ F1F2. Setzen wir nun die gefundenenStreckenlangen fur D1D, DD2 usw. in die bisherige Ungleichung ein, erhalten wir

PA+ PB + PC ≥(PB ·DEPC · FD +

PC · FDPB ·DE

)PD +

(PC · EFPA ·DE +

PA ·DEPC · EF

)PE

+

(PA · FDPB · EF +

PB · EFPA · FD

)PF.

Der Rest ist wieder einfach: die Klammerausdrucke haben die Form x+ 1x

und sind damitallesamt großer oder gleich 2:

PA+ PB + PC ≥ 2(PD + PE + PF ). �

D.84 In der Losung der vorherigen Aufgabe D.83 betraf die erste Abschatzung dieUngleichungen EF ≥ D1D2, FD ≥ E1E2 bzw. DE ≥ F1F2. Diese werden zu Gleichungen,wenn die dort genannten Trapeze zu Rechtecken werden. Dies ist genau dann der Fall,wenn P der Umkreismittelpunkt von �ABC ist (D, E, F sind dann die Seitenmitten

158 LOSUNGEN: DREIECKE

und EF ‖ D1D2 folgt aus der Umkehrung des 2. Strahlensatzes). Gleichzeitig mussenbei der zweiten Abschatzung die Summanden in den Klammerausdrucken gleich sein. Daaber schon PA = PB = PC ist, muß somit auch DE = EF = FD gelten, d. h. dasLotfußpunktdreieck gleichseitig sein. Beide Forderungen erfullt nur der SymmetriepunktP (Schwerpunkt, Umkreismittelpunkt usw.) in einem gleichseitigen Dreieck ABC.

D.91 (Bild) Ein Punkt P ′ auf der Ecktransversalen CK erfulle zunachst die Bedingung

P ′DBC

=P ′ECA

bzw.P ′DP ′E

=BC

CA=a

b. (D.110)

Mit Hilfe des Sinussatzes finden wir mit den im Bild bezeichneten Winkeln γ1 ≡ �BCK,γ2 ≡ �ACK und ε ≡ �BKC

P ′D = CP ′ sin γ1, P ′E = CP ′ sin γ2, BC = KBsin ε

sin γ1

, CA = AKsin(π − ε)

sin γ2

.

Dann wird aus (D.109) mit sin ε = sin(π − ε):

P ′DP ′E

=sin γ1

sin γ2

=KB

AK· sin γ2

sin γ1

=a

b. (D.111)

Nun sei CK ′ die isogonale Gerade zu CK. Mithin ist�ACK ′ = γ1, �BCK ′ = γ2, und der Sinussatz liefert unshier

K ′BAK ′ ·

sin γ1

sin γ2

=a

b. (D.112)

A B

C

D

E

KK ′

P′

ε

γ1γ2

(D.111) und (D.112) konnen nur fur AK ′ = K ′B erfullt werden, d. h., CK ′ ist die Seiten-halbierende von AB, CK demzufolge die zugehorige Symmediane. P ′ liegt also auf derSymmediane zwischen den Seiten CB und CA. Fur die anderen Seitenpaare erhalten wiranaloge Ergebnisse, so daß unser gesuchter Punkt P der Lemoine-Punkt des DreiecksABC ist (vgl. Aufgabe D.41).Bemerkung: Daß der gesuchte Punkt entfernt mit dem Schwerpunkt verwandt ist, ließesich dadurch vermuten, daß fur diesen gerade die Produkte PD ·BC = PE ·CA = PF ·ABuntereinander gleich sind (vgl. Aufgabe W.22).

Allgemeine Vierecke 159

LOSUNGEN: VIERECKE

V.1 Beweis: Erinnern wir uns daran, daß in einem Dreieck die Strecke, die die Mittel-punkte zweier Seiten verbindet, nach der Umkehrung des zweiten Strahlensatzes parallelund halb so lang wie die dritte Seite ist (s. Aufgabe D.10). In unserem Viereck ABCD

seien die Mittelpunkte der Seiten nun mit P , Q, R und S bezeichnet (Bild a). Betrachten

a)

A

B

C

D

P Q

RS

b)

A

B

C

D

P Q

RS

wir die Dreiecke ABC und ADC, so konnen wir schließen, daß beide Strecken PQ undRS parallel zur Diagonalen AC verlaufen und PQ = RS = 1

2AC gilt. Damit ist PQRS

das sog. Varignon-Parallelogramm. Dies ist auch der Fall, wenn wir ein uberschlagenesViereck zugrunde legen (Bild b). Fur den Flacheninhalt des Varignon-Parallelogrammsfinden wir:

[PQRS] = [ABCD] − [SAP ] − [PBQ] − [QCR] − [RDS]

= [ABCD] − 1

4[ABD] − 1

4[ABC] − 1

4[BCD] − 1

4[DAC]

= [ABCD] − 1

4[ABCD] − 1

4[ABCD]

=1

2[ABCD]. �

Bemerkung: PQRS bleibt ein Parallelogramm, wenn A, B, C und D vier beliebige Punkteim Raum sind, die nicht in einer Ebene liegen (vgl. Aufgabe M.9).

V.2 Beweis: (Bild) Angenommen, AC teilt ABCD inzwei flachengleiche Dreiecke ABC und CDA. Da beide diegleiche Basis AC haben, mussen ihre Hohen BE und DF

ebenfalls gleich sein. Aus der Kongruenz (WSW) �BES ∼=�DFS schließen wir BS = DS. Umgekehrt, wenn wirBS = DS voraussetzen, sind diese Dreiecke kongruent mitBE = DF (auch WSW), und es folgt [ABC] = [CDA]. �

A

B

C

D

EFS

160 LOSUNGEN: VIERECKE

V.3 Beweis: (Bild) Hier besteht die Losungsidee darin, das Dreieck XWY in Teil-dreiecke zu zerlegen, um deren Flacheninhalte besser in Relation zum Viereck ABCD zubringen als dies mit XWY als Ganzesmoglich ist. Dazu zeichnen wir die Mit-telpunkte P und Q der Seiten BC bzw.DA in unsere Figur mit ein. Dann istXPY Q das Varignon-Parallelogrammdes uberschlagenen VierecksACBD. Au-ßerdem ziehen wir noch die Verlange-rungen BW und CW sowie PW . DieStrecke XP , die die Mittelpunkte der

A B

C

D

PQ

W

X

Y

Seiten BC und CA des Dreiecks ABC verbindet, ist parallel zur Seite AB und hal-biert damit die

”andere“ Diagonale CW des Vierecks CXWP . Nach der Umkehrung des

Satzes aus Aufgabe V.2 gilt somit

[PXW ] = [XPC] =1

4[ABC]. (V.101)

Analog folgt

[PWY ] = [Y BP ] =1

4[BCD]. (V.102)

Damit fehlt an [XWY ] nur noch [PY X], welcher jedoch mittels Aufgabe V.1 schnellermittelt ist:

[PY X] =1

2[XPY Q] =

1

4[ACBD]

=1

4[BDA] +

1

4[ACB]

=1

4[DAB] − 1

4[ABC]. (V.103)

Addieren wir (V.103), (V.101) und (V.102), so erhalten wir

[XWY ] = [PY X] + [PXW ] + [PWY ]

=1

4[DAB] − 1

4[ABC] +

1

4[ABC] +

1

4[BCD]

=1

4[DAB] +

1

4[BCD] =

1

4[ABCD]. �

Allgemeine Vierecke 161

V.8 (Bild) Die Eckpunkte des Vierecks seien wie ublichmit A, B, C,D bezeichnet; die Innenwinkel bei den Eckpunk-ten in dieser Reihenfolge mit α, β, γ, δ. Betrachten wir nunbeispielsweise die gegenuberliegenden Seiten BC und AD, soschneiden sich ihre Verlangerungen in einem Punkt S, vor-ausgesetzt, sie verlaufen nicht parallel zueinander. Fur dengesuchten Winkel �ASB ≡ ε erhalten wir dann aus denSatzen uber die Innenwinkelsumme im �ABS gleich 180◦

sowie der im Viereck ABCD gleich 360◦: A B

CD

S

α β

γδ

ε

ε = |180◦ − (α+ β)| = |180◦ − [360◦ − (γ + δ)]| = |γ + δ − 180◦| = |180◦ − γ − δ|.

Die Betragsstriche sorgen dafur, daß ε auch im Fall α + β > 180◦ positiv wird (S wurdedann im Bild unterhalb von AB liegen). Im Falle, daß die gegenuberliegenden Seiten diezueinander parallelen Seiten AB, CD eines Trapezes sind, liefert obige Beziehung

�(AB,CD) ≡ η = |180◦ − (β + γ)| = 0,

in Ubereinstimmung damit, daß sich parallele Geraden im Unendlichen schneiden.

V.9 Beweis: (Bild) In der Figur gibt es mehrere rechtwinklige Dreiecke, von deneneinige zueinander ahnlich sind. Dies ist der Fall, wenn sie außer in dem rechten Winkelnoch in einem weiteren Winkel ubereinstimmen. Bezeichnen wir den Schnittpunkt vonAD mit CE mit G, finden wir somit:

�BAD ∼ �GSD (gemeinsamer Winkel �GDS)

∼ �GAE (Scheitelwinkel bei G)

∼ �CBE (gemeinsamer Winkel �GEA).

Mit den Abkurzungen AB ≡ a, BC ≡ b und EA ≡ x konnenwir aus �BAD ∼ �CBE die Proportion A B

CD

E

F

G

S

x a

b

b

a=a+ x

bbzw. b2 − a2 = x · a (V.104)

ablesen. Nun ist nach Voraussetzung BF = BC = b, und da �BAF ebenfalls rechtwink-lig ist, gilt nach dem Satz des Pythagoras AF =

√b2 − a2. Damit sowie mit (V.104)

folgt aus der Umkehrung des Hohensatzes fur rechtwinklige Dreiecke, daß x und a geradedie Hypotenusenabschnitte eines rechtwinkligen �BFE sein mussen; also EF ⊥ FB. �

V.10 Beweis: (Bild) Wir schlagen um den Punkt A einenKreis mit dem Radius AB = AD. Dann ist DB eine Seh-ne dieses Kreises und �DAB = 60◦ der zugehorige Zentri-winkel. Alle Peripheriewinkel in dieser (durch g(D,B) geteil-ten) Halbebene betragen nach dem Peripherie-Zentriwinkel-Satz 30◦, die Peripheriewinkel in der anderen Halbebene dem-zufolge 180◦ − 30◦ = 150◦. Dies ist genau der Winkel �DCB.Also liegt C auch auf dem beschriebenen Kreis und es giltAC = AB. �

A B

C

D

162 LOSUNGEN: VIERECKE

V.11 Direkte Folge der Gleichung aus Aufgabe M.4.

V.19 Beweis: (Bild) Es sei K ≡ DE ∩ AB sowie J ≡ DG ∩ AH. Die Strahlensatzeliefern die Proportionen:

fur �HAK:AG

AB=

GJ

BH,

fur �GKD:KB

KG=KH

KD=BH

GD,

fur �AKD:KB

KA=KE

KD.

A B

CD

E

F

G

H

J

K

Außerdem gilt: AG = KA−KG und AB = KA−KB. Nun ist

DJ

DG=DG−GJ

DG= 1 − GJ

DG= 1 − AG

AB· HBDG

= 1 − AG

AB· KB

KG

=

(KA−KB

) ·KG− (KA−KG

) ·KB

AB ·KG=KA · (KG−KB

)AB ·KG

=1 −KB/KG

1 −KB/KA=

1 −KH/KD

1 −KE/KD=KD −KH

KD −KE=DH

DE;

damit ist nach der Umkehrung des 1. Strahlensatzes AH ‖ GE. �

V.21 Beweis: (Bild) In einem Sehnenviereck liegen dessen Eckpunkte A, B, C, Dsamtlich auf dem Umkreis k. Ziehen wir von dessen Mittelpunkt O Verbindungslini-en zu den Eckpunkten, so entstehen vier gleichschenklige Dreiecke AOB, BOC, CODund DOA, deren gleich lange Schenkel die Lange R (Radi-us des Umkreises von ABCD) haben. Daraus folgt, daß diezugehorigen Basiswinkel in jedem dieser Dreiecke gleich großsind:

�OAB = �ABO ≡ α, �OBC = �BCO ≡ β,

�OCD = �CDO ≡ γ, �ODA = �DAO ≡ δ.

Aus dem Bild lesen wir ab, daß die Summe der Innenwinkel inABCD nun 2(α+β+γ+δ) = 360◦ betragt. Daher ist α+β+γ+δ

A B

C

D

O

k

R

α αβ

βγ

γδ

δ

= 180◦, und dies ist gerade die Summe der Großen gegenuberliegender Innenwinkel. �

V.22 Beweis: (Bild) Aus der Eigenschaft der Mittelsenk-rechten gleichschenklige Dreiecke zu erzeugen wissen wir, daßfur O als Schnittpunkt der Mittelsenkrechten g (von AB) undh (von CD) die Gleichungen AO = BO und CO = DO gel-ten. Es bleibt also, BO = CO oder DO = AO zu zeigen.Dies gelingt uns, wenn wir eine dritte Mittelsenkrechte, et-wa j (von BC), hinzuziehen. Dann sei O1 ≡ g ∩ j der Um-kreismittelpunkt von �ABC und O2 ≡ j ∩ h derjenige von�BCD. Da aber beide Dreiecke denselben Umkreis haben,

A B

C

D

O

g

h

j

muß O1 = O2 = O und damit auch AO = BO = CO = DO sein. �

Sehnenvierecke 163

V.23 Beweis: (Bild) Betrachten wir etwa die Dreiecke ABSund DCS, wobei S der Schnittpunkt beider Diagonalen ist. Esgenugt dann zu zeigen, daß beide Dreiecke in allen drei Winkelnubereinstimmen. Fur die Winkel �ASB = �DSC ist dies derFall, da sie Scheitelwinkel sind. Ebenso gilt �BAC = �BDCbzw. �ABD = �ACD, die jeweils gleiche Peripheriewinkel uberden Sehnen BC bzw. AD sind. � A

B

C

D

S

V.24 Beweis: (Bild) Wir nennen den Schnittpunkt der Dia-gonalen des Sehnenvierecks S. Dann ist �AOB als Zentriwin-kel uber der Sehne AB doppelt so groß wie der Peripherie-winkel �ACB = �SCB. Ebenso gilt �COD = 2�CBD =2�CBS. Nach Voraussetzung ist das Dreieck BCS rechtwink-lig, somit folgt

90◦ = �SCB + �CBS =1

2(�AOB + �COD)

und daraus unmittelbar die Behauptung. �

A B

C

D

O

S

V.25 Beweis: (Bild) AC und BD seien die sich senkrecht in Pschneidenden Diagonalen des Sehnenvierecks ABCD; die Geradedurch H, P (senkrecht zu AB) schneide CD in X. Dann sind�APH und �ABP ahnliche Dreiecke und es gilt: �CPX =�APH = �ABP = �ABD = �ACD = �PCX. Somit ist�PXC gleichschenklig. Analog zeigen wir, daß auch �PXDgleichschenklig ist, also XC = XP = XD. �

A

B

CD

H

P

X

V.26 Beweis: (Bild) Wir nehmen ein Dreieck ABC so-wie einen weiteren Punkt D auf dessen Umkreis, so daßABCD tatsachlich ein Sehnenviereck ist. Nach Aufgabe D.45

liegen die Fußpunkte X, Y , Z der Lote von D auf derSimson-Geraden XY Z, fur die als zur Strecke entartetesLotfußpunktdreieck gerade die Dreiecksungleichung zur Glei-chung wird. Die Strecken XY , Y Z, ZX sind nach AufgabeD.81: A B

C

DX

Y

Z

XY =AB · CD

2R, Y Z =

BC · AD2R

, XZ =AC ·BD

2R,

wobei R der Radius des umschriebenen Kreises ist. Addition aller drei Gleichungen undMultiplikation mit 2R ergibt die Behauptung. �

164 LOSUNGEN: VIERECKE

V.27 Beweis: (Bild) Wir bezeichnen die Winkel �CAB ≡ α, �CBA ≡ β, �DAB ≡ γ

und �DBA ≡ δ; der Inkreismittelpunkt von �ABC sei I1, der von �ABD entsprechendI2. Damit ist

�I1AB =α

2, �I1BA =

β

2, �I2AB =

γ

2, �I2BA =

δ

2.

Nach dem Peripheriewinkelsatz ist �ACB = �ADB ≡ εund demnach

�AI1B = 180◦ − α+ β

2= 90◦ +

ε

2

= 180◦ − γ + δ

2= �AI2B (V.105)

A B

CD

I1

I2

Sw

εε

(vgl. Aufgabe D.9); somit ist auch ABI1I2 ein Sehnenviereck. Wir berechnen nun denWinkel η zwischen den Geraden I1I2 und AB (vgl. Aufgabe V.8):

η ≡ |180◦ − �I1I2A− �I2AB| =∣∣∣180◦ − (�I1I2B + �AI2B) − γ

2

∣∣∣ =|β − γ|

2.

Dabei wurde (V.105) und �I1I2B = �I1AB = 12α (Peripheriewinkel uber BI1) benutzt.

Andererseits ist der Winkel zwischen w und SB 12(α+δ) (d. i. die Halfte des Außenwinkels

�BSC im �ABS), der Winkel zwischen w und der Geraden AB schließlich∣∣∣∣180◦ −(

180◦ − α− δ +α+ δ

2

)− α

∣∣∣∣ =|δ − α|

2= η.

Die beiden Geraden w und I1I2 verlaufen daher parallel. �

V.31 Beweis: (Bild) Wir bezeichnen die Beruhrungspunkte der Seiten des Tangenten-vierecks ABCD mit dem Kreis k mit K, L, M , N . Im �AIK steht der Beruhrungs-

A B

CD

I

K

L

M

N

k

a

a

b

b

c

cd

d

radius IK stets senkrecht auf dem Tangentenabschnitt AK; esist somit rechtwinklig. Das gleiche gilt fur �AIN . Beide Drei-ecke sind daruber hinaus kongruent nach Kongruenzsatz SSW,da sie die Seite AI gemeinsam haben, die Seiten IK = IN gleichlang sind und in den Winkeln �IKA = �INA = 90◦ uberein-stimmen. Somit sind insbesondere ihre Seiten AK = AN ≡ a

gleich lang. Analog zeigen wir BK = BL ≡ b, CL = CM ≡ c,DM = DN ≡ d. Die Summe der Langen gegenuberliegenderSeiten ist daher AB + CD = a+ b+ c+ d = BC +DA. �

Tangentenvierecke 165

V.33 Beweis: (Bild) Wenn wir die Beruhrungssehnendes Tangentenvierecks ABCD mit KM und LN be-zeichnen, lautet die Behauptung

�BKM = �CMK und �CLN = �DNL.

Dies wird sofort offensichtlich, wenn wir erkennen, daßdie Dreiecke IKM und ILN wegen IK = IL = IM =IN ≡ r (Beruhrungsradius) gleichschenklig sind und dieBeruhrungsradien senkrecht auf den Seiten des Tangen-tenvierecks stehen. Damit ist A B

C

D

I

K

L

M

N

�BKM = 90◦ + �IKM = 90◦ + �IMK = �CMK;

analog folgt die zweite behauptete Winkelbeziehung. Gleiches gilt naturlich auch fur dieanderen beiden Vierecke AKMD und BLNA, wenn die Basiswinkel der gleichschenkligenDreiecke jeweils von 90◦ subtrahiert werden. �Bemerkung: Dieser Satz kann somit auch folgendermaßen formuliert werden:

Die an den entgegengesetzten Seiten einer (diagonalen) Beruhrungssehne gelegenen Win-kel in einem Tangentenviereck sind stets Supplementwinkel.

V.34 Beweis: (Bild) S sei der Schnittpunkt der DiagonaleAC mit der Sehne KM . Die Dreiecke AKS und CMS ha-ben den Winkel bei S gemeinsam (Scheitelwinkel); daruberhinaus sind �AKS und �CMS Supplementwinkel (s. Auf-gabe V.33). Mit Hilfe des Sinussatzes folgt daraus wegensin�AKS = sin(180◦ − �CMS) = sin�CMS

AK

AS=CM

CS.

A B

C

D

K

L

M

N

S

Weiterhin sind AK = AN und CL = CM jeweils gleiche Tangentenabschnitte, so daßwir obige Gleichung zu

AK

AS=CM

CS=CL

CS=AN

AS(V.106)

erweitern konnen. Somit sind wir in den Dreiecken CLS und ANS angelangt, die ebenfallsSupplementwinkel enthalten: �CLS = 180◦− �ANS. Mit dem gleichen Argument unterVerwendung des Sinussatzes folgt nun aus (V.106): �ASN und �CSL sind entwedergleich oder supplementar. Letzteres scheidet aus. Da S auf AC liegt, muß daher auch Sauf der Sehne LN liegen. Analog konnen wir (beginnend mit den Dreiecken BLS undDNS) zeigen, daß auch die andere Diagonale durch S geht. �

166 LOSUNGEN: VIERECKE

V.41 Beweis: (Bild) Die Beruhrungspunkte auf den Seiten AB, BC, CD, DA mit demeinbeschriebenen Kreis seien in dieser Reihenfolge K, L, M , N ; der Schnittpunkt derBeruhrungssehnen KM und LN heiße S. Um zu zeigen, daßKM ⊥ LN eine notwendige Bedingung fur die Existenz ei-nes Sehnentangentenvierecks ist, wenden wir auf die beidenVierecke AKSN und CMSL den Satz von der Winkelsum-me im Viereck an, wobei wir die Winkel an den Viereckseckenausnahmsweise durch Uberstreichen der Eckbuchstaben be-zeichnen:

A+K + S +N = 360◦, C +M + S + L = 360◦ AB

C

D

K

L

M

NS

Da die an den entgegengesetzten Seiten der Beruhrungssehnen KM und LN gelegenenWinkel K + M = 180◦ bzw. L + N = 180◦ nach Aufgabe V.23 Supplementwinkel sind,folgt durch Addition der beiden obigen Gleichungen

A+ C + 2S = 360◦. (V.107)

Schließlich muß noch eine Eigenschaft des Sehnenvierecks ABCD ins Spiel kommen, unddiese ist naturlich A+ C = 180◦. Damit und aus (V.107) folgt S = 90◦. �Bemerkung: Daß KM ⊥ LN auch eine hinreichende Bedingung dafur, daß ein Tangenten-viereck ABCD auch ein Sehnentangentenviereck ist, folgt aus (V.107) jetzt mit S = 90◦:A+ C = 180◦. Dies ist nur fur ein Sehnenviereck ABCD erfullt.

Vektorrechnung 167

LOSUNGEN: METHODEN

M.1 (Bild) Wir bezeichnen die Ortsvektoren zu den dreiEckpunkten A, B, C in einem beliebig festgelegten Koordi-natensystem mit dem Ursprung O mit

−→OA ≡ a,

−−→OB ≡ b

und−→OC ≡ c. Vom Schwerpunkt G eines Dreiecks ist bekannt,

daß er jede Seitenhalbierende im Verhaltnis 2 : 1 teilt, wobeider großere Teil der Abstand zum Eckpunkt ist (vgl. AufgabeD.10). Wahlen wir zur Berechnung beispielsweise die Seiten-halbierende von AB mit deren Mittelpunkt M , so erhalten wirfur den Ortsvektor von M :

A B

C

G

M

O

a b

c

−→AB = b − a,

−−→AM =

1

2(b − a),

−−→OM =

−→OA+

−−→AM =

1

2(a + b).

Der Vektor entlang der Seitenhalbierenden MC ist dann

−−→MC =

−→OC −−−→

OM = c − 1

2(a + b) und somit

−−→MS =

1

3

−−→MC =

1

3c − 1

6(a + b).

Fur den Ortsvektor von G finden wir schließlich

−→OG =

−−→OM +

−−→MG =

1

3(a + b + c).

Die Lage des Schwerpunkts eines Dreiecks ist somit durch das arithmetische Mittel derOrtsvektoren der Eckpunkte bestimmt.

M.2 Beweis: (Bild) Bezeichnen wir die Seiten des unregelmaßigen Sechsecks ABCDEFnacheinander mit den Vektoren a, b, c, d, e, f und behalten dabei den mathematischpositiven Umlaufsinn bei, so verschwindet (da der Polygonzug geschlossen ist) deren Vek-torsumme:

a + b + c + d + e + f = 0. (M.101)

Wird der Eckpunkt A als Ursprung O eines Koordina-tensystems gewahlt, so erhalten wir den Schwerpunkt G1

des von den Vektoren a und b aufgespannten �ABCals arithmetisches Mittel der drei Ortsvektoren 0, a unda + b (vgl. Aufgabe M.1):

O=A B

C

D

E

F

ab

c

de

f G1

G2

G3G4

G5

G6

−−→OG1 =

2a + b

3.

Analog erhalten wir fur die Schwerpunkte der anderen Dreiecke

−−→OG2 = a +

2b + c

3,

−−→OG3 = a + b +

2c + d

3,

−−→OG4 = a + b + c +

2d + e

3,

−−→OG5 = a + b + c + d +

2e + f

3,

−−→OG6 =

a − f

3.

Daraus folgen durch Differenzbildung und unter Beachtung von (M.101) die Vektoren derSeiten des inneren Sechsecks:

168 LOSUNGEN: METHODEN

−−−→G1G2 =

a + b + c

3,

−−−→G2G3 =

b + c + d

3,

−−−→G3G4 =

c + d + e

3,

−−−→G4G5 =

d + e + f

3,

−−−→G5G6 =

e + f + a

3,

−−−→G6G1 =

f + a + b

3,

wobei die zu gegenuberliegenden Seiten gehorigen Ausdrucke jeweils untereinander stehen.Wegen (M.101) ist also

−−−→G1G2 +

−−−→G4G5 =

−−−→G2G3 +

−−−→G5G6 =

−−−→G3G4 +

−−−→G6G1 = 0. Dies sind

genau die Bedingungen dafur, daß gegenuberliegende Seiten parallel sind und die gleicheLange haben. �

M.3 Beweis: Die Bedingung, daß die Diagonalen senkrecht aufeinander stehen, lautetin Vektorschreibweise

(a − c) · (b − d) = ab + cd − bc − ad = 0. (M.102)

a) Fur die andere Bedingung finden wir entsprechend

(a − b)2 + (c − d)2 = (b − c)2 + (a − d)2,

die ausmultipliziert auf

a2 + b2 + c2 + d2 − 2 (ab + cd) = b2 + c2 + a2 + d2 − 2 (bc + ad) oder

ab + cd = bc + ad.

fuhrt. Dies ist identisch mit (M.102); beide Gleichungen lassen sich also durch aquivalenteUmformungen ineinander uberfuhren. b) Ahnlich wie unter a) laßt sich hier mit Hilfe vonAufgabe M.1 die Gleichung∣∣∣∣a + b

2− c + d

2

∣∣∣∣ =

∣∣∣∣b + c

2− a + d

2

∣∣∣∣ bzw. |a + b − c − d| = |b + c − a − d|

aufstellen. Wenn die Langen gleich sein sollen, mussen auch deren Quadrate gleich sein:

(a + b − c − d)2 = (b + c − a − d)2.

Weiteres Ausmultiplizieren und Vereinfachen fuhrt ebenfalls auf (M.102). �

M.4 Beweis: Es ist

BC2 + AD2 − AB2 − CD2 = (b − c)2 + (a − d)2 − (a − b)2 − (c − d)2

= 2 (cd + ab − bc − ad)

= 2 (c − a) · (d − b)

= 2−→AC · −−→BD. �

Vektorrechnung 169

M.8 Beweis: (Bild) Das Parallelogramm sei OABC; wir bezeichnen die Vektoren,die das Parallelogramm aufspannen mit

−→OA ≡ a und−→

OC ≡ b, die dazu senkrechten Vektoren mit c und d.Fur die Mittelpunkte der Aufsatzquadrate, die gleichzei-tig deren Schwerpunkte sind, konnen wir aus dem Bilddie Gleichungen

−−→OG1 =

1

2(b + d),

−−→OG2 = b +

1

2(a + c),

−−→OG3 = a +

1

2(b − d),

−−→OG4 =

1

2(a − c)

O A

BC

D

E

G1

G2

G3

G4

a

b

c

d

ablesen. Daraus folgt durch Differenzbildung

−−−→G1G2 =

1

2(a + b + c − d) =

−−−→G4G3 und

−−−→G1G4 =

1

2(a − b − c − d) =

−−−→G2G3.

Damit ist schon nachgewiesen, daß G1G2G3G4 ein Parallelogramm ist. Um zu zeigen, daßes sogar ein Quadrat ist, genugt es,

−−−→G1G2·−−−→G1G4 = 0 (rechter Winkel) und |−−−→G1G2| = |−−−→G1G4|

(gleiche Lange) zu uberprufen. Fur das Skalarprodukt erhalten wir:

−−−→G1G2 · −−−→G1G4 =

1

4

[(a − d) + (b + c)

]·[(a − d) − (b + c)]=

(a − d)2− (b + c)2

4.

Nun gilt offenbar �OAD ∼= OCE nach Kongruenzsatz SWS wegen |a| = |c|, |b| = |d|und �OAD = �OCE; also OD = |a − d| = OE = |b + c| und daher

−−−→G1G2 · −−−→

G1G4 = 0.Schließlich ist

|−−−→G1G2|2 =1

4

[(a − d) + (b + c)

]2=

1

4

[(a − d)2 + (b + c)2 + 2(a − d) · (b + c)

],

|−−−→G1G4|2 =1

4

[(a − d) − (b + c)

]2=

1

4

[(a − d)2 + (b + c)2 − 2(a − d) · (b + c)

].

Beide Ausdrucke sind gleich, wenn

(a − d) · (b + c) = ab + ac − bd − cd = 0,

welches tatsachlich wegen ac = 0 und bd = 0 (da a ⊥ c, b ⊥ d) sowie ab = cd (da|a| = |c|, |b| = |d| und �(a, b) = �(c,d)) der Fall ist. �

170 LOSUNGEN: METHODEN

M.9 Beweis: (Bild) Die Mittelpunkte P , Q, R, S der Verbindungsstrecken werdendurch die Vektoren 1

2(a + b), 1

2(b + c), 1

2(c + d) und 1

2(d + a) beschrieben, wenn wir die

vier Punkte A, B, C, D durch die Ortsvektoren a, b, c, d bezeichnen. Die Beweisideeist nun folgende: PQRS ist genau dann planar, falls die Dreiecke PQR und PQS inderselben Ebene liegen. Dies ist der Fall, wenn die Flachenvektoren, ausgedruckt durchdie Vektorprodukte

−→PQ×−→

PR und−→PQ×−→

PS, parallel sind. Rechnen wir also die Vektorenaus:

−→PQ =

1

2[(b + c) − (a + b)] =

1

2(c − a),

−→PR =

1

2[(c + d) − (a + b)] =

1

2(c + d − a − b),

−→PS =

1

2[(d + a) − (a + b)] =

1

2(d − b),

woraus fur das erste Vektorprodukt mit den Regeln der Vektoralgebra

AB

CD

P

Q

R

S

−→PQ×−→

PR =1

4[(c − a) × (c + d − a − b)]

=1

4[c × d − c × a − c × b − a × c − a × d + a × b]

=1

4[c × d − c × b − a × d + a × b] =

1

4[c × (d − b) − a × (d − b)]

=1

4[(c − a) × (d − b)] .

folgt. Dies ist mit den obigen Gleichungen genau−→PQ× −→

PS. Beide Flachenvektoren sindalso nicht nur parallel, sondern sogar gleich, was beweist, daß sich die Diagonalen halbierenund PQRS demzufolge tatsachlich ein ebenes Parallelogramm ist. �

M.10 Beweis: (Bild) Wir zerlegen die WSZ in drei Paare kongruenter Dreiecke undfuhren neben den Basisvektoren a, b, die das Punktgitter aufspannen, noch die Hilfsvek-toren c, d und e wie im Bild gezeigt ein. Nun konnen wir die Flacheninhalte der Dreieckeals Betrage der Vektorprodukte A1 ≡ 1

2a × c, A2 ≡ 1

2b × d und A3 ≡ [1

2(b − a)] × e

betrachten. Fur die gesamte Flache A ≡ 2(A1 + A2 + A3) der WSZ erhalten wir somit

A = 2

[a × c

2+

b × d

2+

(b − a) × e

2

]= (a × c) + (b × d) + (b × e) − (a × e)

= a × (c − e) + b × (d + e).

Aus dem Bild ist zu erkennen, daß c − e = 12b und

d + e = −12a gilt, so daß

a

b

c

de

b a

A = a × b

2+ b × −a

2=

1

2(a × b + a × b) = a × b

wird. Dies ist genau der Flachenvektor, der von der Elementarzelle aufgespannt wird. �

”Winkel jagen“ 171

M.11 (Bild) Die Dreiecke ADG und AFE sind gleichschenklig, haben also gleich großeBasiswinkel �DGA = �FEA = α. Die Winkel �BDG und �CFE sind Außenwinkeldieser gleichschenkligen Dreiecke und somit von derGroße 2α. Ebenso sind �DGB und �FEC gleich-schenklig, woraus �DBG = �FCE = 2α folgt. Da dieInnenwinkelsumme dieser Dreiecke 180◦ = π ist, fin-den wir weiter: �FEC = �DGB = π−4α und daraus�CEB = �BGC = 3α. Schließlich sind auch �ECB

und �GBC gleichschenklig, also �ECB = �GBC =3α−2α = α. Jetzt konnen wir die Innenwinkelsumme

A

B

C

DE

FG

α

von �ABC berechnen: 3α+ 3α+ α = 7α = π oder α = 17π = 1

7· 180◦ = 25,7◦.

Bemerkung: Mit der abgebildeten Figur laßt sich somit auf einfache Weise ein regelmaßiges14-Eck (und damit auch ein regelmaßiges Siebeneck) zusammensetzen.

M.12 (Bild) Das Trapez zerlegen wir in ein ParallelogrammFBCD und ein Dreieck AFD. In dem Parallelogramm istBC + CD = DF + FB. Damit diese Summe gleich AB =AF+FB wird, muß DF = AF gelten, d. h., �AFD ist gleich-schenklig mit der Basis AD. Bezeichnen wir �DAF ≡ α und�ABC ≡ β, so ist �DFB Supplementwinkel von β und gleich-zeitig Außenwinkel von �AFD. Daraus folgt: 2α+ β = 180◦.

A B

CD

α

β

Bemerkung: Selbstverstandlich konnen wir die Zerlegung statt mit DF ‖ CB auch volliganalog mit CF ‖ DA durchfuhren.

M.13 Beweis: (Bild) Wegen �BCD = �BED = 90◦ ist BCDE ein Sehnenviereck.Somit kann der Peripheriewinkelsatz fur die Sehne DE angewendet werden:

�EBD = �ECD = �SCD = 90◦ − �BCS,

�BCE = �BCS = 90◦ − �EBD = 90◦ − �ABD.

Nach Voraussetzung ist das �CBS gleichschenklig bei B;also �BCS = �BSC und daher

A B

C

D

E

S

�CBD = �CBS = 180◦ − 2�BCS = 2�ABD. �

172 LOSUNGEN: METHODEN

M.14 Beweis: (Bild) Die Verlangerung von AB treffe die Tangente t durch T in Q; dieWinkel α, β, γ und δ seien die im Bild angegebenen. Dann sind QT und QP Tangentenab-schnitte des inneren Kreises mit gleicher Lange, woraus folgt, daß �TQP gleichschenkligist und damit gleich große Basiswinkel hat:

β + γ = δ. (M.103)

Weiterhin sind bezuglich des außeren Kreises TBSehne und TQ ein Tangentenabschnitt. Nach demSehnen-Tangentenwinkel-Satz ist also

�TAB = �BTQ = γ.

Nun ist �TPQ = δ Außenwinkel im �ATP und so-mit gleich der Summe der beiden nicht anliegendenInnenwinkel:

A

B

P

QTt

α βγ

δ

γ

α+ γ = δ. (M.104)

Aus (M.103) und (M.104) folgt unmittelbar, wie behauptet, α = β. �

M.15 Beweis: (Bild) Kurz und bundig:

A

B CK L

M

N

�BCA+ �BNM= �BNA+ �BNM (Peripheriewinkel)

= �MNL (Innenwinkel im Sehnenviereck)

= 180◦ − �MKL (lt. Voraussetzung)

= 180◦ − �BKA (Scheitelwinkel)

= �ABK + �BAK (Winkelsumme)

= �ABC + �BAM (Verlangerung)

= �ABC + �BNM (Peripheriewinkel).

Aus der ersten und letzten Gleichung folgen gleiche Basiswinkel �BCA = �ABC, d. h.,�ABC ist tatsachlich gleichschenklig. �

M.21 Wir wahlen die Gerade g durch die Seite AB des regelmaßigen Funfecks alsBasislinie unserer ersten Verwandlung (Bild a). Die Eckpunkte C und E bringen wir zumVerschwinden, indem die Parallelen zu AD durch E und BD durch C mit g zum Schnittgebracht werden. Offensichtlich ist dann [DFA] = [DEA] = [DCB] = [DGB], womit wirdas Funfeck bereits in das (gleichschenklige) Dreieck DFG verwandelt haben.

a)

A B

C

D

E

F GM g

b)

A BC

D

E

F

Das Flachenprinzip 173

Im zweiten Teil ist letzteres in ein gleichseitiges Dreieck umzuwandeln. Dazu mussen wiretwas rechnen: �DFG habe Schenkel der Lange DF = DG = b, die Hohe DM = h sowiedie Basislange FG = c, unser gleichseitiges Dreieck die Seitenlange a. Dann gilt mit denbekannten Flachenformeln und dem Satz des Pythagoras

2∆ = ch =

√3

2a2 und b2 = h2 +

c2

4.

Eliminieren wir h aus beiden Gleichungen, erhalten wir nach kurzer Rechnung:

a2 = c

√(2b+ c)(2b− c)

3bzw.

(a2

)2

=c

2

√(2b+ c

4

)(2b− c

3

). (M.105)

Mit Hilfe von Aufgabe A.14 finden wir problemlos eine Konstruktion von 12a (Bild b†).

Erklarung: Die Wurzel in (M.105) ist die mittlere Proportionale CD aus den Faktoren ACund BC; das gesuchte 1

2a = CF ebenso mittlere Proportionale aus CD und CE = 1

2c.

†Der einzige Grund, warum wir a/2 anstelle a konstruieren, ist, daß das maßstabgerechte Bild b sonstzu groß geworden ware.

M.22 (Bild) Die Aufgabe vereinfacht sich, wenn das Funfeck ABCDE in ein flachen-gleiches Viereck umgewandelt wird. Das dabei entstehende Viereck kann dann in zweiDreiecke zerlegt werden, die sich einfach in ihrem Flacheninhalt halbieren lassen.a) Man gelangt damit zu folgender Konstruktion:1. Es wird die Parallele q zu AD durch E konstruiert und mitder Verlangerung von CD im Punkt F zum Schnitt gebracht.2. Man konstruiert den Mittelpunkt M der Strecke BF .3. Es wird die Parallele p zu AC durch M konstruiert. Sie schnei-det die Flache des Funfecks ABCDE in einer Strecke XY .Als Ergebnis der Konstruktion erhalt man auf dieser Strecke zwi-schen X und Y die Menge aller Punkte P .b) Beweis: Nach 3. ist PM ‖ AC, daher hat das �ACP den glei-chen Flacheninhalt wie �ACM . Also sind die Vierecke ABCPund ABCM flachengleich. Die Flache von ABCM ist gleich der A B

C

D

E

F

M

X

Y

pq

Summe der Flacheninhalte der Dreiecke �ABM und �BCM . Nach 2. haben diese Drei-ecke jeweils einen halb so großen Flacheninhalt wie �ABF und �BCF . Also ist dasViereck ABCP halb so groß wie ABCF . Nach 1., also EF ‖ AD, hat das �ADF dengleichen Flacheninhalt wie �ADE und folglich ABCF den gleichen Flacheninhalt wiedas Funfeck ABCDE. Somit ist der Flacheninhalt von ABCP halb so groß wie der vonABCDE und daher so groß wie der von CDEAP . �

M.31 Beweis: Da beide Dreiecke PAB und PCD die gleiche Hohe haben, folgt die Be-hauptung aus der bekannten Formel ∆ = 1

2chc fur den Flacheninhalt eines Dreiecks. �

Bemerkung: Naturlich gilt der Satz auch, wenn zwei Punkte zusammenfallen, also insge-samt drei Punkte auf einer Geraden liegen.

174 LOSUNGEN: METHODEN

M.32 Beweis: (Bild) Wir wenden den Satz der gemeinsamen Hohen (s. Aufgabe M.31)auf die beiden Paare �PAM , �QAM bzw. �PBM , �QBM an:

[PAM ]

[QAM ]=PM

QM=

[PBM ]

[QBM ]=

[PBM ]

[QBM ]

(1 + [PAM ]

[PBM ]

)(1 + [QAM ]

[QBM ]

)

=[PBM ] + [PAM ]

[QBM ] + [QAM ]=

[PAB]

[QAB]. �

A

B

M

P

Q

Bemerkung: Haufig wird dieser Satz auch in der Form [PAM ] · [QBM ] = [PBM ] · [QAM ]benutzt (”Fliegen-Satz“, da die gegenuberliegenden Dreiecke wie eine Fliege aussehen).Der Satz gilt auch, wenn P und Q auf derselben Seite von AB liegen, wobei dann M derSchnittpunkt der Verlangerung von PQ mit AB ist.

M.33 Beweis: (Bild) a) Wer-den die beiden Winkel �ABCund �A′B′C ′ so gezeichnet, daßsie Wechselwinkel bei B sind, sokonnen wir den Satz der gemein-samen Hohen (s. Aufgabe M.31)sowohl auf die Figur AA′C als a)

A

A′

B = B ′

C

C ′b)

A

A′

B = B ′

C

C ′

auch auf CC ′A′ anwenden:

[ABC]

[A′BC]=

AB

A′B′ bzw.[A′BC]

[A′B′C ′]=

BC

B′C ′ . (M.106)

Multiplikation beider Gleichungen liefert die Behauptung. b) Im Fall von Supplementwin-keln werden beide als Nebenwinkel angeordnet und die zweimalige Anwendung des Satzesder gemeinsamen Seite fuhrt wie in a) auf die behauptete Gleichung (M.106). �

M.34 Beweis: (Bild) Die drei Ecktransversalen mogen sich in K schneiden und zerlegendie Dreiecksflache in sechs Teilflachen, die wir mit ∆i, i = 1, 2, . . . , 6 bezeichnen. Nachdem Satz der gemeinsamen Hohen gelten nun folgende Gleichungen:

AZ

ZB=∆1

∆2

=∆5 +∆6 +∆1

∆3 +∆4 +∆2

=∆5 +∆6

∆3 +∆4

,

BX

XC=∆3

∆4

=∆1 +∆2 +∆3

∆5 +∆6 +∆4

=∆1 +∆2

∆5 +∆6

,

CY

YA=∆5

∆6

=∆3 +∆4 +∆5

∆1 +∆2 +∆6

=∆3 +∆4

∆1 +∆2

.

Multiplikation aller drei Gleichungen ergibt 1. �

A B

C

KX

Y

Z

∆1 ∆2

∆3

∆4∆5

∆6

Das Flachenprinzip 175

M.35 Beweis: In beiden Fallen, in denen die Gerade durch X, Y , Z die Dreieckseitenentweder zweimal (Bild a) oder gar nicht (Bild b) direkt (d. h. innerlich) teilt, konnen wirdie Figur zu einem großeren unterteilten Dreieck erweitern. Wir erhalten im Fall a) —analog zum Beweis des Satzes von Ceva — wieder sechs Teilflachen ∆i, i = 1, 2, . . . , 6.

a)

A B

C

KX

Y

Z∆1 ∆2

∆3

∆4∆5

∆6

b)

A

B

C

XY Z

∆1 ∆2

∆3∆4

∆5

Die Satze der gemeinsamen Seite bzw. Hohen liefern uns nun folgende drei Gleichungen:

AZ

ZB= −∆1 +∆2

∆2

,BX

XC=

∆2

∆3 +∆4

,CY

YA=∆5

∆6

=∆3 +∆4 +∆5

∆1 +∆2 +∆6

=∆3 +∆4

∆1 +∆2

,

die gemeinsam multipliziert −1 ergeben. Im Fall b) kommen wir mit funf Teilflachen ∆i,i = 1, 2, . . . , 5 aus, wobei zur Abkurzung ∆ =

∑∆i gesetzt wird:

AZ

ZB= −∆1 +∆2 +∆5

∆1 +∆2

,BX

XC= − ∆1

∆1 +∆4 +∆5

= − ∆2

∆2 +∆3

= −∆1 +∆2

∆,

CY

YA= − ∆

∆1 +∆2 +∆5

, so daß wiederumAZ

ZB· BXXC

· CYYA

= −1. �

M.36 Beweis: (Bild) Die zu beweisende Gleichung diktiert uns, wie wir vorzugehenhaben: Fur die Teilungsverhaltnisse u, v, w lesen wir folgende Flachenverhaltnisse ab:

u =AK

KX=∆1 +∆2

∆3

=∆5 +∆6

∆4

, v =BK

KY=∆3 +∆4

∆5

=∆1 +∆2

∆6

,

w =CK

KZ=∆5 +∆6

∆1

=∆3 +∆4

∆2

.

Somit folgt mit ∆ =∑

i∆i

A B

C

KXY

Z∆1 ∆2

∆3

∆4∆5

∆6

1

1 + u=

∆3

∆1 +∆2 +∆3

=∆4

∆4 +∆5 +∆6

=∆3 +∆4

∆,

1

1 + v=

∆5

∆3 +∆4 +∆5

=∆6

∆6 +∆1 +∆2

=∆5 +∆6

∆,

1

1 + w=

∆1

∆1 +∆5 +∆6

=∆2

∆2 +∆3 +∆4

=∆1 +∆2

∆.

Die Summe der rechts stehenden Terme ist — wie behauptet — eins. �Bemerkung: Die angegebene Gleichung laßt sich auch als uvw = u+ v+w+2 schreiben.

176 LOSUNGEN: METHODEN

M.37 Beweis: Da wir die Bezeichnungen der Flacheninhalte ∆i, i = 1, 2, . . . , 6, in denAufgaben M.34 und M.36 gleich gewahlt haben (s. obiges Bild zur Losung von AufgabeM.36), konnen wir von dort folgende Gleichungen direkt ubernehmen:

u =AK

KX=∆1 +∆2

∆3

=∆5 +∆6

∆4

=∆1 +∆2 +∆5 +∆6

∆3 +∆4

,

1

y=AY

Y C=∆6

∆5

=∆6 +∆1 +∆2

∆3 +∆4 +∆5

=∆1 +∆2

∆3 +∆4

,

z =AZ

ZB=∆1

∆2

=∆5 +∆6 +∆1

∆3 +∆4 +∆2

=∆5 +∆6

∆3 +∆4

.

Die Gleichung u = 1/y+ z folgt offensichtlich. Durch zyklisches Vertauschen erhalten wirauch die beiden anderen Beziehungen. �Bemerkung: Ist K der Schwerpunkt des Dreiecks, so gilt x = y = z = 1. Folglich istu = v = w = 2, d. h., die Seitenhalbierenden werden durch K im Verhaltnis 2 : 1 geteilt(vgl. Aufgabe D.10).

M.38 Beweis: (Bild) Um das Flachenprinzip vorteilhaftanwenden zu konnen, mussen wir eine geeignete Zerle-gung des Dreiecks ABC finden. Je weniger Teilflachenwir benotigen, desto ubersichtlicher wird der Beweis. Eszeigt sich, daß wir außer [RST ] = ∆4 lediglich 3 weite-re Dreiecke betrachten mussen, deren Flacheninhalte wirmit [ARB] = ∆1, [BSC] = ∆2 und [CTA] = ∆3 bezeich-nen. Außerdem sei AT = k, BR = l, CS = m, TR = r,RS = s und ST = t. Jetzt wenden wir den Satz des ge- A B

C

R

S

T

X

Y

Z

r

st

kl

m

∆1

∆2

∆3∆4

meinsamen (Erganzungs-) Winkels an und lesen folgende Gleichungen aus dem Bild ab:

∆1

∆4

=(r + k)l

rs,

∆2

∆4

=(s+ l)m

st,

∆3

∆4

=(t+m)k

tr. (M.107)

Um die gegebenen Verhaltnisse x, y, z ins Spiel zu bringen, bemuhen wir den Satz vonMenelaus: S, T , Z liegen auf einer Geraden und gleichzeitig auf den Dreieckseiten von�ABR. Somit gilt

AZ

ZB· BSSR

· RTTA

= −1 oder z =AZ

ZB=RS

BS· TART

=sk

(s+ l)r. (M.108a)

Durch zyklische Vertauschung von (x, y, z), (r, s, t) bzw. (k, l,m) finden wir weiterhin

x =tl

(t+m)s, y =

rm

(r + k)t. (M.108b,c)

Die Gleichungen (M.107) stellen ein lineares Gleichungssystem fur k, l, m dar, dessenLosung sich leicht errechnen laßt:

k =(xy + x+ 1)zr

1 − xyz, l =

(yz + y + 1)xs

1 − xyz, m =

(zx+ z + 1)yt

1 − xyz. (M.109)

Das Flachenprinzip 177

Nun werden erst (M.108) und anschließend (M.109) in die Ausdrucke (M.107) substituiert:

∆1

∆4

=lm

yst=

(yz + y + 1)(zx+ z + 1)x

(1 − xyz)2,

∆2

∆4

=mk

ztr=

(zx+ z + 1)(xy + x+ 1)y

(1 − xyz)2,

∆3

∆4

=kl

xrs=

(xy + x+ 1)(yz + y + 1)z

(1 − xyz)2.

Aus der Addition aller drei Gleichungen folgt nach Ausmultiplizieren und erneutem Zu-sammenfassen

[ABC]

[RST ]= 1 +

∆1 +∆2 +∆3

∆4

=(xy + x+ 1)(yz + y + 1)(zx+ z + 1)

(1 − xyz)2. �

Bemerkung: Fur x = y = z = n = 2, 3, 4, 5, 6, . . . wird das Verhaltnis der Flacheninhalte

[RST ]

[ABC]=

(n− 1)2

n2 + n+ 1=

1

7,

4

13,

3

7,

16

31,

25

43, . . . .

M.39 (Bild) Wir schreiben zur Abkurzung [ABE] = ∆1 = 72und [CDE] = ∆2 = 50. Nun gilt (da ABCD ein Trapezist): [ABC] = [ABD] = ∆1 + [BEC] = ∆1 + [AED], also[BEC] = [AED] = ∆. Der Satz der gemeinsamen Hohen lie-fert, angewandt auf die Diagonale AC:

AE

EC=

∆2

=∆1

∆, A B

CD

E

∆1

∆2

∆∆

oder ∆2 = ∆1∆2 = 3600 bzw. ∆ = 60. Somit ist [ABCD] = ∆1 +∆2 + 2∆ = 242.

M.40 Beweis: (Bild) Schreiben wir fur die funf Teildreiecke wie im Bild angegeben ∆i

mit i = 1, 2, . . . , 5, so ist zu zeigen:

[∆1 +∆2 +∆3 +∆4 +∆5]∆5 = ∆1∆3. (M.110)

Der Satz der gemeinsamen Hohen, angewandt auf die DreieckeCED, EAD bzw. CEB, EAB, ergibt

A B

C

DE

P∆1

∆2

∆3

∆4∆5

CE

EA=

∆3

∆4 +∆5

=∆2 +∆3 +∆5

∆1 +∆4

bzw.

∆2∆4 +∆2∆5 +∆3∆4 + [∆3 +∆4 +∆5]∆5 = ∆1∆3 +∆3∆4. (M.111)

Aus dem Viereck ABDE lesen wir mit dem Fliegen-Satz (vgl. Bemerkung in der Losung zuAufgabe M.32) unmittelbar die Relation ∆2∆4 = ∆1∆5 ab, welche in (M.111) eingesetztund nach Subtraktion von ∆3∆4 die Behauptung (M.110) liefert. �

178 LOSUNGEN: METHODEN

M.41 Beweis: Mit den bisher ublichen Abkurzungen u = PA/PX, v = PB/PY , w =PC/PZ lautet die zu beweisende Ungleichung uv + vw + wu ≥ 12. Nun gilt nach demSatz von Euler-Gergonne (s. Aufgabe M.36)

1

1 + u+

1

1 + v+

1

1 + w= 1 oder uvw = u+ v + w + 2; (M.112)

die darin auftretende Summe von Reziproken laßt sich mit der AM-HM Ungleichung (vgl.Aufgabe U.7) abschatzen:

(1 + u+ 1 + v + 1 + w)

(1

1 + u+

1

1 + v+

1

1 + w

)≥ 9. (M.113)

Einsetzen von (M.113) in (M.112) liefert zunachst

u+ v + w ≥ 6, und uvw ≥ 8. (M.114)

Fur den letzten Schritt wenden wir die AM-GM Ungleichung (vgl. Aufgabe U.5) in Ver-bindung mit dem Zwischenergebnis (M.114) an:

uv + vw + wu ≥ 3(uvw)23 ≥ 3 · 8 2

3 = 12. �

M.42 Beweis: (Bild) Wir beweisen den allgemeineren Fall, daß die angegebene Glei-chung fur jeden beliebigen Schnittpunkt K dreier Ecktransversalen AX, BY und CZ gilt.In der Notation von Aufgabe M.36 ist dann

AK

AX=

u

1 + u,

BK

BY=

v

1 + v,

CK

CZ=

w

1 + w.

Nach dem Satz von Euler-Gergonne gilt aber (s. AufgabeM.36)

1

1 + u+

1

1 + v+

1

1 + w= 1, A B

C

K XY

Z

so daß

3 − 1 =

(1 + u

1 + u+

1 + v

1 + v+

1 + w

1 + w

)−

(1

1 + u+

1

1 + v+

1

1 + w

)

=u

1 + u+

v

1 + v+

w

1 + w=AK

AX+BK

BY+CK

CZ= 2. �

Deutsche Mathematik-Olympiade 179

LOSUNGEN: WETTBEWERBSAUFGABEN

W.1 Beweis: (Bild) Wir konnen unsere Ausfuhrungen mit dem Hinweis auf AufgabeV.1 betrachtlich verkurzen, indem wir feststellen, daßM1M2M3M4 gerade dasVarignon-Parallelogramm des Vierecks ABCD ist. Gleiches gilt furM4M5M2M6, welches das Varignon-Parallelogramm des

”uberschlagenen“ Vierecks DACB ist. Nun ist in beiden

Parallelogrammen M2M4 gemeinsame Diagonale. Da sichParallelogramm-Diagonalen in ihrem Schnittpunkt S stetshalbieren, fallen somit die Schnittpunkte von M2M4 undM1M3 bzw. M2M4 und M5M6 in S zusammen. �

A B

C

D

M1

M2

M3

M4

M5

M6

S

W.2 Beweis: (Bild) Wir bezeichnen die Seitenlangen des Funfecks wie im Bild ange-geben mit a, b, c, d, e sowie die Schnittpunkte der Diagonalen untereinander mit K, L,M , N , P . Es ist klar, daß uns nur die Dreiecksungleichungen schnell ans Ziel bringen.a) Um die rechte Ungleichung 2u > s zu zeigen, mussen wir Teildreiecke finden, die ausjeweils zwei Seiten (zu u gehorend) und einer Diagonalen(zu s gehorend) bestehen:

a+ b > AC, b+ c > BD, c+ d > CE,

d+ e > DA, e+ a > EB.

Durch Addition dieser Ungleichungen und mit s ≡ AC +BD + CE +DA+ EB folgt unmittelbar 2u > s.b) Bei der zweiten Ungleichung u < s ist genau umge-kehrt vorzugehen: Jetzt durfen nur jeweils eine Seite aufder

”Kleiner“seite der Ungleichung und Diagonalenlangen

auf der”Großer“seite stehen:

A B

C

D

E

KL

MN

P

a

b

cd

e

a < AK +KB, b < BL+ LC, c < CM +MD,

d < DN +NE, e < EP + PA.

Die Summe der rechten Seiten s′ ≡ AK +KB +BL+ LC + CM +MD +DN +NE +EP +PA ist nun offenbar kleiner als s, da an s der Umfang des kleinen eingeschlossenenFunfecks KLMNP fehlt. Mithin haben wir u < s′ < s. �

W.3 (Bild) Der erste Schritt ist es herauszufinden,wo der Mittelpunkt M des Kreises liegen konnte, fallses ihn gibt. Nehmen wir also an, daß DEFGHJ ein ei-nem Kreis einbeschriebenes Sechseck ist. Dessen Seitensind dann Sehnen dieses Kreises. Die Mittelsenkrech-ten aller dieser Sehnen mußten sich folglich in M tref-fen. Aufgrund der Symmetrie der Figur vermuten wir,daß M der Mittelpunkt der Strecke AB ist. Beweis:Dann sind die Trapeze MBFE, MCGK, MCHL undMAJL offensichtlich kongruent, und wir konnen denRadius r ≡ MF = MG = MH = MJ nach dem Satzdes Pythagoras berechnen (AC = BC ≡ a):

A B

C

D E

F

GH

J

KL

M

180 LOSUNGEN: WETTBEWERBSAUFGABEN

r =

√(3

2a

)2

+

(1

2a

)2

=

√10

2a.

Ebenso erhalten wir

MD = ME =

√(1√2a

)2

+(√

2a)2

=

√5

2a =

√10

2a = r,

d. h., der Kreis existiert. � Mit a = 2 betragt der Radius somit r =√

10.

W.4 Beweis: (Bild) In jedem spitzwinkligen �ABC gelten mit den ublichen Bezeich-nungen a, b, c fur die Seitenlangen und den weiteren Bezeichnungen im Bild nach demSatz des Pythagoras die Gleichungen

b2 = c 21 + h 2

c = a 22 + h 2

a , (W.101)

c2 = a 21 + h 2

a = b 22 + h 2

b , (W.102)

a2 = b 21 + h 2

b = c 22 + h 2

c . (W.103)

Nach Addition der rechts stehenden Gleichungen und Sub-traktion von h 2

a + h 2b + h 2

c folgt die behauptete GleichungA B

C

DE

F

a1

a2b1

b2

c1 c2

ha hb

hc

c 21 + a 2

1 + b 21 = c 2

2 + a 22 + b 2

2 .

Ist fur rechtwinklige Dreiecke der rechte Winkel etwa bei C, so gelten die zweite Gleichungin (W.101) wegen a2 = 0, ha = b und die erste Gleichung in (W.103) wegen b1 = 0, hb = aebenfalls (und die Gleichungen (W.102) sind die gleiche Aussage wie c2 = a2 + b2). �Bemerkung: Die behauptete Gleichung gilt, mit gleicher Herleitung wie fur spitzwinkligeDreiecke, auch fur stumpfwinklige Dreiecke.

W.5 Beweis: (Bild) Wir verlangern die Lote von P auf die Radien uber die FußpunkteC, D hinaus bis zum Schnitt mit dem Kreis in den Punkten E, F . Nach dem Satz, daßjeder Radius die zu ihm senkrechten Sehnen halbiert, gilt

PC

PE=PD

PF=

1

2.

Nach der Umkehrung des 1. Strahlensatzes folgt hierausCD ‖ EF und nach dem 2. Strahlensatz CD/EF = 1 : 2.Wegen �MCP = �MDP = 90◦ gilt nach dem Satz von

A B

CD

E

F

M

P

der Winkelsumme im Viereck �EPF = �CPD = 180◦ − �AMB. Also ist die Großedes Winkels �EPF von der Wahl des Punktes P unabhangig. Nach der Umkehrung desPeripheriewinkelsatzes folgt daraus, daß die Lange EF ebenso von der Wahl des PunktesP unabhangig ist. Damit ist auch CD unabhangig von der Wahl von P . �

W.6 Bei dieser Olympiade-Aufgabe sind wir fein raus, wenn wir den Beweis des Sat-zes aus Aufgabe D.21 parat haben: Der Hohenschnittpunkt H in einem spitzwinkligenDreieck ist gleichzeitig Mittelpunkt des Inkreises des Hohenfußpunktdreiecks DEF . Dieerwahnten Abstande sind dann die Langen der Beruhrungsradien von H zu den Seitendes �DEF , und diese sind fur den genannten Inkreis, dessen Mittelpunkt stets der ge-meinsame Schnittpunkt der Winkelhalbierenden in �DEF ist, naturlich gleich.

Deutsche Mathematik-Olympiade 181

W.7 a) Beweis: Aus Aufgabe D.63 wissen wir, daß der Flacheninhalt ∆ eines beliebigenDreiecks gleich dem Produkt aus Inkreisradius r und halbem Umfang s ≡ 1

2(a + b + c)

des Dreiecks ist. Somit gilt

aha = 2∆ = (a+ b+ c)r bzw. ha =

(1 +

b

a+c

a

)r

und entsprechend auch

hb =(ab

+ 1 +c

b

)r, hc =

(a

c+b

c+ 1

)r.

Eine Addition dieser drei Gleichungen ergibt

ha + hb + hc =

[3 +

(a

b+b

a

)+

(b

c+c

b

)+

( ca

+a

c

)]r. (W.104)

Nun gilt fur je zwei positive Zahlen x, y stets die Ungleichung xy

+ yx≥ 2. (Davon uber-

zeugen wir uns leicht, wenn wir zu (x − y)2 ≥ 0 auf beiden Seiten 2xy addieren undanschließend durch xy dividieren, s. Abschnitt U.2.1.) Aus (W.104) folgt daher die Be-hauptung

ha + hb + hc ≥ 9r. � (W.105)

b) In einem gleichseitigen Dreieck gilt in (W.105) das Gleichheitszeichen. Beweis: Imgleichseitigen Dreieck sind die Hohen zugleich Seitenhalbierende und Winkelhalbierende.Ihr Abschnitt vom gemeinsamen Schnittpunkt, der zugleich Schwerpunkt und Inkreismit-telpunkt ist, bis zur zugehorigen Seite ist somit gleich r und betragt nach Aufgabe D.10

ein Drittel der Hohen ha = hb = hc. Daher gilt hier ha + hb + hc = 9r. �

W.8 Beweis: (Bild) Das”verdachtige“ Produkt CP · PD

erinnert uns naturlich sofort an den Sehnensatz (vgl. AufgabeK.11), nur brauchen wir, um ihn anwenden zu konnen, einezweite Sehne, die ebenfalls durch P geht. Dazu klappen wirden gegebenen Halbkreis um, so daß ein Vollkreis entsteht.Dieser schneidet nun aus der Geraden g(P,Q) unsere zweite,gesuchte Sehne EF heraus. Somit folgt

A B

C

DE

F

P

Q

CP · PD = EP · PF. (W.106)

Da Q der Lotfußpunkt von P bzw. E auf dem Durchmesser AB ist, halbiert Q die SehneEF , also EQ = QF . Damit wird

EP · PF = (EQ− PQ) · (PQ+ EQ) = EQ2 − PQ2. (W.107)

Weiterhin ist nach dem Thales-Satz �AEB ein rechtwinkliges Dreieck mit dem rechtenWinkel bei E, fur welches wir den Hohensatz hinschreiben konnen:

EQ2 = AQ ·QB. (W.108)

Aus (W.106), (W.107), (W.108) folgt CP · PD = AQ ·QB − PQ2. �

182 LOSUNGEN: WETTBEWERBSAUFGABEN

W.9 (Bild) Es ist nicht schwer zu erkennen, daß die beiden betrachteten Dreiecke APCund BPD zueinander ahnlich sind. Außer im rechten Winkel stimmen sie noch in denWinkeln �CAP = �CAB = �CDB = �PDB uberein, die Peripheriewinkel uber derSehne BC des Umkreises sind. Fur den Ahnlichkeitsfaktor giltsomit:

λ ≡ CP

PB=AP

PD=AC

DB(W.109)

und daher nach Voraussetzung auchA B

C

D

P

λ =CP + AP + AC

PB + PD +DB= 2. (W.110)

Da AB Durchmesser des Kreises ist, gilt ferner CP = PD und somit nach (W.109) und(W.110): AP = λ · PD, PB = CP/λ, also AP/PB = λ2 = 4.

W.10 Beweis: (Bild) Spiegeln wir die drei Quadrate an der unteren Kante, so er-kennen wir, daß die rechtwinkligen Dreiecke AHF und EFN kongruent sind (z. B.SSS). Daher ist �AFN gleichschenklig mit den Basiswin-keln

�FNA = �FAN = �FAD + �DAN.

β = �HFA = �FAD und α = �DAN = �ANK sindjeweils Wechselwinkel an geschnittenen Parallelen, so daßaus obiger Gleichung

A D

EFGH

K L M N

α

α

α

β

β

β

�FNA = 90◦ − α− β = �FAN = α+ β

folgt. Daraus ergibt sich unmittelbar die Behauptung α+ β = 45◦. �

W.11 Beweis: (Bild) Wir betrachten im �ABC zunachst eines dieser Lote, bei-spielsweise den Lotfußpunkt S, der vom Mittelpunkt U von AB auf die an C ge-legte Tangente t gefallt wird. Weiterhin seien R, T dieLotfußpunkte von A, B auf t sowie F der Fußpunktdes von C auf g(A,B) gefallten Lotes. Die beiden Drei-ecke CFA und BTC sind ahnlich, da sie einerseits recht-winklig sind, und andererseits die Winkel �BAC und�BCT Peripherie- bzw. Sehnen-Tangentenwinkel uberder Sehne CB des Umkreises k und damit gleich sind.Daher gilt mit CF ≡ hc die Proportion

A B

C

F

R

S

T

U

ab

k

t

BT

BC=CF

CAoder BT = hc

a

b.

Analog folgt aus �ARC ∼ �CFB die Lange AR = hc(b/a). Nun ist ABTR offenbarein Trapez, in dem US gerade die Mittellinie ist (wegen AU = UB). Daraus folgt mit∆ = 1

2chc:

US =1

2(AR +BT ) =

hc2

· a2 + b2

ab= ∆ · a

2 + b2

abc. (W.111)

Deutsche Mathematik-Olympiade 183

Fur die beiden anderen Entfernungen erhalten wir ganz analoge Ausdrucke, in denen a, b, cjeweils zyklisch vertauschbar sind, also ∆(b2 + c2)/(bca) und ∆(c2 + a2)/(cab), so daß alsSumme der Ausdruck 2∆(a2 + b2 + c2)/(abc), wie behauptet, folgt. �Bemerkung: Fur �BAC = 90◦ wird A = F und C = T und (W.111) folgt wie oben direktaus �ARC ∼ �CAB.

W.12 (Bild) a) Die Punkte H, E, G seien die Mittelpunkte der drei Kreise k1, k2, k3 mitdem Radius r1 sowie M der Mittelpunkt des Quadrats ABCD. Der Schnittpunkt von GHmit CM sei F . Aus den Beruhrungen von Kreisen miteinander, mit Seiten des Quadratssowie mit der Diagonale BD ergibt sich: Das Dreieck EJC ist gleichschenklig-rechtwinkligmit der Kathetenlange r1, ebenso die Dreiecke EFG und EFH mit der Hypotenusenlange2r1, ferner gilt FM = r1. Damit folgt

a

2

√2 = CM = CE + EF + FM = r1

√2 + r1

√2 + r1,

r1 =a√

2

2(2√

2 + 1)=

a

4 +√

2=

a

14(4 −

√2).

a)A B

CD

E

F

G

HJ

K

M

k1

k2

k3

b)A B

CD

M

P

QRS

k4

k5

b) Hier sind P und S die Mittelpunkte der Kreise k4 bzw. k5 mit dem Radius r2. DerMittelpunkt von BC sei Q, das Lot von P auf MQ habe den Fußpunkt R. Wie untera) folgt MS = RQ = r2, PS = 2r2 und PR = 1

2a − r2. Der Satz des Pythagoras,

angewandt auf das �SRP , ergibt(a2− 2r2

)2

+(a

2− r2

)2

= 4r 22 , r 2

2 − 3ar2 +a2

2= 0.

Diese quadratische Gleichung hat die beiden Losungen

r21,2 =3

2a±

√9

4a2 − 1

2a2 =

a

2(3 ±

√7),

von denen wegen 12a(3 +

√7) > 1

2a(3 +

√4) = 5

2a > a aber nur

r2 =a

2(3 −

√7)

in Betracht kommt. Nun beweisen wir r1 > r2, also

a

14(4 −

√2) >

a

2(3 −

√7).

184 LOSUNGEN: WETTBEWERBSAUFGABEN

Dies folgt der Reihe nach aus

4 −√

2 > 7(3 −√

7),

7√

7 > 17 +√

2,

343 > 289 + 34√

2 + 2,

26 > 17√

2,

676 > 578.

In Bild a) sind also die großeren Kreise gezeichnet. — Bemerkung: Man beachte die Un-gleichungskette am Ende der Losung, deren Text im wesentlichen original ubernommenwurde. Es genugt zur vollstandigen und richtigen Losung dieser Olympiade-Aufgabe nicht,die erste Ungleichung etwa unter Verwendung von Naherungswerten (z. B. mit dem Ta-schenrechner erhaltenen) nachzuweisen. Mit anderen Worten: Der Taschenrechner ist furOlympiade-Aufgaben selten geeignet.

W.13 Beweis: Der einfachste Teil bei der Losung dieser Olympiadeaufgabe besteht darin,die Formel fur das Volumen eines Tetraeders hinzuschreiben: V = 1

3R∆, wobei ∆ der

Flacheninhalt der Grundflache und h = R die Hohe des Tetraeders senkrecht zu dieserGrundflache ist. Wenn uns nun die Gleichung aus Aufgabe D.64 gelaufig ist, namlich∆ = abc/(4R) mit a, b, c als Seitenlangen von �ABC sowie R als dessen Umkreisradius,haben wir auch schon den schwierigsten Teil geschafft:

V =abc

12. (W.112)

Die Aufgabe verlangt nun, das Produkt abc durch die Summe u ≡ a+ b+ c nach oben ab-zuschatzen. Dies erreichen wir durch Anwendung der AM-GM Ungleichung (vgl. AufgabeU.5):

3√abc ≤ a+ b+ c

3bzw. abc ≤ (a+ b+ c)3

27=u3

27. (W.113)

(W.113) in (W.112) eingesetzt liefert die Behauptung. �Gleichheit ist genau dann erfullt, wenn in (U.6) a = b = c gilt, die Grundflache also eingleichseitiges Dreieck ist.

W.14 Wir betrachten zunachst die vier Mittelpunkte E, F , G, H der Strecke PQ bei denspeziellen Lagen PQ ≡ {AD,BD,BC,AC}. Sind nun AB und DC nicht parallel zuein-ander (Bild a), so ist EFGH gerade das Varignon-Parallelogramm des uberschlagenenVierecks ADBC; im Fall AB ‖ DC dagegen liegen F und H auf der Strecke EG (Bild b);das Parallelogramm EFGH ist zu dieser Strecke entartet. Nun wird gezeigt: Die Mengeder Mittelpunkte aller Strecken PQ ist im Fall a) (AB ∦ DC) die ParallelogrammflacheM ≡ EFGH bzw. im Fall b) (AB ‖ DC) die Strecke M ≡ EG. Angenommen, X sei derMittelpunkt einer Strecke PQ. Dann gilt fur den Schnittpunkt U von PD mit EF und denSchnittpunkt V von PC mit HG nach dem 1. Strahlensatz PU : PD = PV : PC = 1 : 2und nach dessen Umkehrung UV ‖ DC. Also schneidet UV die Strecke PQ in derenMittelpunkt, d. h. in X. Daher liegt X auf UV und somit in M . �

Deutsche Mathematik-Olympiade 185

a)

A B

C

D

EF

GH

P

Q

U

VX

b)

A B

CD

E F GH

P

Q

U VX

W.15 Beweis: (Bild) Die Verlangerungen von XA, XB und XC schneiden k mit dessenMittelpunkt O zum zweiten Mal in D, E bzw. F . Die Sehnen XD, XE, XF werdendabei durch die senkrecht auf ihnen stehenden Durch-messer P1P4, P2P5, P3P6 gerade in den Punkten A, B,C halbiert. Es gilt demnach

XA

XD=XB

XE=XC

XF=

1

2,

und �ABC geht somit aus �DEF durch Streckung umden Faktor 1

2hervor. Weiterhin stehen beide Schenkel

XB, XC des Winkels �BXC senkrecht auf den Schen-keln OP2, OP3 des Winkels �P2OP3 = 60◦, woraus mitdem Peripheriewinkelsatz

AB

C

D

E

F

O

P1 P2

P3

P4P5

P6

X k

�P2OP3 = �BXC = �EXF = �EDF = 60◦

folgt. Entsprechend ergibt sich �DFE = �FED = 60◦. Das �DEF hat somit als eindem Kreis einbeschriebenes gleichseitiges Dreieck einen Flacheninhalt, der nicht von derLage des Punktes X auf k abhangt. Dasselbe gilt folglich auch fur das Dreieck ABC. �

W.16 Beweis: (Bild) E, F seien die Lotfußpunkte von O auf die Seiten AB und CD,P der Schnittpunkt beider Diagonalen. Nun sind die Dreiecke AOB und COD gleich-schenklig und werden somit durch ihre Hohen OE bzw. OF in zwei Paare kongruenterrechtwinkliger Dreiecke zerlegt:

�AOE ∼= �BOE, �COF ∼= �DOF. (W.114)

Wenn es uns noch gelingt, die Kongruenz aller vier obigenDreiecke nachzuweisen, sind wir schon fertig. Bezeichnen wirdie Winkel �DBC ≡ α und �ACB ≡ β, so ist nach Voraus-setzung α + β = 90◦. Aus dem Peripherie-Zentriwinkel-Satzfolgt �DOC = 2α, somit wegen (W.114) �DOF = α undaus der Innenwinkelsumme in �ODF : �ODF = β. Ana-log erhalten wir �AOE = β und �OAE = α. Schließlichstimmen beide Dreiecke �AOE und �ODF in ihren Hypo-tenusen AO = OD uberein, sind daher in der Tat kongruent.Daraus folgt unmittelbar CD = 2DF = 2OE. �

A

B

C

D

E

FO

P

k

αβ

186 LOSUNGEN: WETTBEWERBSAUFGABEN

W.17 (Bild) Aus dem Innenwinkelsatz fur die Dreiecke ABC und ABD folgt mit dengegebenen Winkelgroßen �ACB = 80◦ und �ADB = 70◦. Letztere beiden Dreiecke sinddaher gleichschenklig mit CA = CB bzw. BD = BA. Es seinun P derjenige Punkt, der auf derselben Seite der Geradendurch A, B liegt wie C und fur den �ABP ein gleichseiti-ges Dreieck ist. Die Punkte P , C sowie der Mittelpunkt Mvon AB liegen auf der Mittelsenkrechten von AB, und esgilt

�APM = �BPM = 30◦,

�ACM = �BCM = 40◦. A B

CD

M

P

S

Nach Kongruenzsatz SWS ist �BCD ∼= �BCP wegen der gemeinsamen Seite BC,�CBD = �CBP = 10◦ sowie BD = BA = BP ; daher gilt �BDC = �BPC = 30◦.Damit und nach dem Satz uber die Winkelsumme im Viereck ergeben sich die beidengesuchten Innenwinkelgroßen:

�ADC = �ADB + �BDC = 70◦ + 30◦ = 100◦,

�BCD = 360◦ − 50◦ − 70◦ − 100◦ = 140◦.

W.21 (Bild) Zwei der durch P vorgegebenen Teilungsverhaltnisse u ≡ AP/PD = 1 undv ≡ BP/PE = 3 sind uns bekannt; mit Hilfe des Satzesvon Euler-Gergonne (s. Aufgabe M.36) konnen wirdas dritte Verhaltnis sofort berechnen:

A

B CD

EF

P1569

5

63

w ≡ CP

PF=u+ v + 2

uv − 1= 3.

Daraus folgt CP = 15 und PF = 5. Weiterhin erlaubt uns die Tabelle M.1, die Strecken-verhaltnisse auf den Seiten des Dreiecks ABC zu bestimmen:

x ≡ BD

DC=

1 + v

1 + w= 1, y ≡ CE

EA=

1 + w

1 + u= 2, z ≡ AF

FB=

1 + u

1 + v=

1

2.

Nun kennen wir in den Dreiecken PBC, PCA und PAB jeweils die Langen zweier Sei-ten, die Lange einer zugehorigen Ecktransversalen sowie das Teilungsverhaltnis auf dergegenuberliegenden Seite. Mit Hilfe des Satzes von Stewart (s. Aufgabe D.69) konnenwir somit die Langen BC ≡ a, CA ≡ b und AB ≡ c berechnen:

PD2 = pPC2 + q PB2 − pqa2, mit p = q = 12: a2 = 468,

PE2 = pPA2 + q PC2 − pqb2, mit p = 23, q = 1

3: b2 = 405,

PF 2 = pPB2 + q PA2 − pqc2, mit p = 13, q = 2

3: c2 = 117.

Den gesuchten Flacheninhalt liefert uns schließlich Herons Formel (s. Losung zu AufgabeD.66):

16 [ABC]2 = 2(a2b2 + b2c2 + c2a2) − (a4 + b4 + c4) = 186 624, also [ABC] = 108.

Nationale Wettbewerbe 187

W.22 (Bild) Schreiben wir zur Abkurzung PD ≡ u, PE ≡ v und PF ≡ w, so lautet dieForderung uvw ⇒ Max. Wir brauchen noch irgendeine Nebenbedingung in den Langenu, v, w, um eine der Standard-Ungleichungen anwenden zukonnen. Das ist naturlich der Flacheninhalt, fur den offen-bar

2∆ = au+ bv + cw = const

gilt. Nun genugt bereits eine einmalige Anwendung derAM-GM-Ungleichung, um die Losung zu finden: A B

C

DE

F

P

uv

w

2∆ = au+ bv + cw ≥ 3(abc)13 (uvw)

13 , oder uvw ≤ 8∆3

27abc=

2

27

∆2

R.

In der letzten Umformung wurde das Ergebnis von Aufgabe D.64 abc = 4R∆ benutzt. DieAM-GM-Ungleichung sagt uns auch, wann Gleichheit auftritt, das Produkt also seinenMaximalwert annimmt: au = bv = cw. Da diese Terme gerade die doppelten Flachenin-halte der Teildreiecke BCP , CAP und ABP sind, folgt daraus, daß der gesuchte PunktP der Schwerpunkt von �ABC ist (vgl. Aufgabe D.11).

W.23 Beweis: (Bild) Die beiden Tangenten lassen vermuten, daß bei dieser Aufgabe derSehnen-Tangentenwinkel-Satz eine Rolle spielt. Außerdem wimmelt es nur so an Peri-pheriewinkeln. Wir versuchen es also wieder mitder

”Winkel-Jagd“, schreiben �ABC als Differenz

zweier anderer Winkel und wenden daraufhin diegenannten Satze an:

A

B

C

P

Q

R

kk ′

�ABC = �ABP − �CBP= �ARP − �PBC= �RBC − �PBC= �RBP= �PAR. �

W.24 Beweis: (Bild) Da die Strecken AO und PQ sich nicht schneiden, verlangern wirAO uber O hinaus und bezeichnen den Schnittpunkt mit X. Wir haben also �AXQ = 90◦

zu zeigen. Nun ist �AXQ Außenwinkel in �AXP und als solcher gleich der Summe derInnenwinkel �PAX = �BAO und �APX. Letzterer istNebenwinkel zu �BPX, der wiederum Supplementwin-kel zu �BCQ im Sehnenviereck BPQC ist. Somit habenwir

�AXQ = �BAO + �BCQ = �BAO + �BCA

und sind mit unseren Winkeln wieder vollstandig in�ABC. Nach dem Peripherie-Zentriwinkel-Satz gilt wei-ter �BCA = 1

2�BOA, und da �BOA gleichschenklig

ist: �BCA = 90◦ − �BAO. Diese Beziehung in obigeGleichung eingesetzt, ergibt die Behauptung. �

A B

C

O

P

Q

X

k

188 LOSUNGEN: WETTBEWERBSAUFGABEN

W.25 (Bild) Rechtwinklige Lotfußpunktdreiecke haben wir bereits in Aufgabe D.82 be-trachtet, so daß wir uns hier kurz fassen konnen. Demnach ist fur �FDE = 90◦ die Win-

A B

C

D

E

F

O

P

kelbeziehung �BPC = 90◦ + �BAC = 150◦

notwendig. Dieser Winkel ist somit unabhangigvon der Lage von P konstant, woraus folgt,daß P auf einem Kreisbogen mit BC als Sehneliegt. Den zugehorigen Mittelpunkt O des Krei-ses konnen wir ermitteln, wenn wir den Spe-zialfall nehmen, daß das �BPC gleichschenk-lig ist. Dann ist �BOP = �COP = 30◦,�OBP = �OCP = 75◦ und nach dem Innen-winkelsatz auch �OPB = �OPC = 75◦, beide

Dreiecke BOP und COP damit gleichschenklig. Hieraus folgt OP = OB = OC; dergesuchte Mittelpunkt O ist also gerade der gespiegelte Punkt von A an der Seite BC.

W.26 Beweis: (Bild) Der Satz von Ceva ist doch ein guter Freund! Bisher hatten wir esjedoch stets mit gleichartigen Ecktransversalen zu tun; hier treten zur Abwechslung maleine Hohe, eine Winkel- sowie eine Seitenhalbierende auf. Wir zeigen zuerst: Wenn sichalle drei Ecktransversalen in einem Punkt schneiden, dann gilt die angegebene Gleichung.Nach Ceva gilt nun:

AF

FB· BDDC

· CEEA

= 1. (W.115)

Wir haben AF = FB, CE/EA = a/c nach Aufgabe D.8

sowie

BD =a2 + c2 − b2

2a, DC =

a2 + b2 − c2

2a A B

C

DE

F

nach Aufgabe D.72. Damit wird aus (W.115) zunachst a ·BD = c ·DC und weiter

a(a2 + c2 − b2) = c(a2 + b2 − c2), aa2 − a(b2 − c2) = ca2 + c(b2 − c2),

a2(a− c) = (a+ c)(b2 − c2). (W.116)

Die Umkehrung gilt ebenfalls: Von (W.116) gelangen wir durch aquivalente Umformungenund mit eindeutigen Ausdrucken zu (W.115). Nach der Umkehrung des Satzes von Cevaschneiden sich mithin die drei unterschiedlichen Ecktransversalen in einem Punkt. �

Nationale Wettbewerbe 189

W.27 (Bild) Wir betrachten die Strecke OA als Sehne eines zweiten Kreises k′. Fur allePunkte P auf der Peripherie dieses Kreises, die bezuglich OA auf derselben Seite liegen,

A

M

O

P

P′

kk ′

g

sind die Winkel �OPA nach dem Peripheriewinkelsatz gleichgroß. Der Winkel �OPA wird nun um so großer, je großerder zugehorige Zentriwinkel �OMA = 2�OPA wird, d. h.,je dichter der Mittelpunkt M dieses Kreises, der stets auf derMittelsenkrechten g von OA liegt, an OA heranruckt. Damitwird auch der Radius von k′ kleiner. Da P jedoch auf k liegensoll, ist der Grenzfall genau derjenige, bei dem k′ den Kreis kvon innen in P beruhrt. Somit ist OP ein Durchmesser vonk′, und �OAP nach dem Satz des Thales ein Rechter. Wirfinden folglich zwei Punkte P und P ′, fur die �OPA maximal

wird, indem wir die Senkrechte zu OA in A mit k zum Schnitt bringen.

W.28 (Bild) Wir zeichnen eine Parallele zu BC durchD; diese schneide die Verlangerungvon AB in E. Mit den Bezeichnungen AC ≡ x und BE ≡ y konnen wir dann mit Hilfedes Strahlensatzes unmittelbar die Beziehung x : 1 = 1 : y oder xy = 1 ablesen. Nun ist�BED rechtwinklig mit �EDB = �CBD = 30◦; dessen Hypotenuse wegen sin 30◦ = 1

2

somit BD = 2y und nach dem Satz des Pythagoras ED =√

3y. Die Lange der StreckeBC erhalten wir, indem wir den 2. Strahlensatz anwenden:

BC

AB=ED

AE, oder BC =

√3y

y + 1.

Der Kosinussatz fur das �CBD lautet jetzt:

12 =3y2

(y + 1)2+ 4y2 − 2

√3y

y + 1(2y) cos 30◦,

A

B

C

D

E

x

y

1 1

woraus nach Multiplikation mit (y + 1)2 und mit cos 30◦ =√

32

(y + 1)2 = 3y2 + 4y2(y + 1)2 − 6y2(y + 1) bzw. 4y4 + 2y3 − 2y − 1 = 0

folgt. Das geubte Auge erkennt im letzten Polynom in y den Faktor (2y + 1), also

4y4 + 2y3 − 2y − 1 = (2y + 1)(2y3 − 1) = 0.

Nullsetzen des ersten Faktors liefert y = −12

(ungultige Losung), somit bleibt y3 = 12.

Diese Gleichung dritten Grades hat eine reelle Losung y = 13√2

(die anderen beiden sind

konjugiert komplex). Wegen xy = 1 betragt die gesuchte Lange daher AC = x = 3√

2.

190 LOSUNGEN: WETTBEWERBSAUFGABEN

W.29 Beweis: (Bild) Der konstante Teil in der Behauptungbesteht aus der Geraden OE, auf der X liegt. Es genugtalso, EX = const oder XO = const nachzuweisen. Aus�OEM = �OBM = 90◦ folgt, daß A, B und E auf einemKreis k′ mit dem Durchmesser OM liegen. Bezeichnen wirnoch die Schnittpunkte von OE mit dem Kreis k mit C,D, so haben wir genugend Sehnen in beiden Kreisen, daßuns der Rest mit Hilfe des Sehnensatzes nicht schwer fallendurfte. Tatsachlich ist AB Sehne in beiden Kreisen, und esgilt mit x ≡ XO und r ≡ AO = CO = DO:

A

B

C

D

E

M

O

X

k

k ′t

EX ·XO = AX ·XB = CX ·XD,

(EO − x)x = (r − x)(r + x), oder x =r2

OE= const. �

W.30 Beweis: (Bild) Wir sehen unmittelbar �BAP ∼ �BCA und �BAQ ∼ �BDA,da samtliche Dreiecke rechtwinklig sind und jeweils in den Winkeln �ABP bzw. �ABQubereinstimmen. Daraus folgen die Proportionen

AP

AB=CA

CBund

AQ

AB=DA

DB.

Multiplikation beider Gleichungen liefert

AP · AQAB2

=CA

CB· DADB

. (W.117)

Da ADBC ein Sehnenviereck ist, bestehen außerdemdie Ahnlichkeiten �ACK ∼ �DBK und �DAK ∼�BCK, somit

A

B

CDK

M

P Q

k

t

CA

BD=CK

BK,

DA

BC=AK

CK,

CA

BD· DABC

=CK

BK· AKCK

=AK

BK. (W.118)

Nun folgt aus (W.117) und (W.118)

AP · AQAB2

=AK

BK, oder AP · AQ = AB2 · AK

BK= const,

unabhangig von der Wahl von CD.

W.31 Beweis: (Bild) Um die Flachengleichheit eines Dreiecks und eines Vierecks zuzeigen, bietet es sich an, das Viereck in ein flachengleiches Dreieck umzuwandeln, umdann zwei Dreiecke vergleichen zu konnen. Dazu zerlegen wir das Viereck ADPC durchseine Diagonale AP zunachst in die Dreiecke ADP und APC. Letzteres wandeln wirnun in ein flachengleiches Dreieck um, indemwir die Parallele zur Grundseite AP durch Cziehen und mit der Verlangerung von AD zumSchnitt bringen; wir erhalten Punkt E. Mithinist [APC] = [APE] und somit

A B

C

DE

PM

[ADPC] = [EDP ]. (W.119)

Nationale Wettbewerbe 191

Aus dem 1. Strahlensatz folgt wegen EC ‖ AP ‖ DM und BC = 2BM : BE = 2BD bzw.ED = DB. Damit haben �EDP und �DBP gleiche Grundseiten und gleiche Hohen,sind also flachengleich, woraus mit (W.119) die Behauptung folgt. �

W.32 Beweis: (Bild) Da I der Inkreismittelpunkt von �ABC ist, halbiert AI denInnenwinkel �CAB. Die Umkreismittelpunkte O und O1 liegen weiterhin auf der Mit-telsenkrechten von AB; O1 und O3 auf der Mittelsenkrechten von AI. Bezeichnen wirdie Schnittpunkte von O1O und AB mit E sowie von O1O3 und AB bzw. AI mit Dbzw. F , so sind die beiden Dreiecke �ADF und �O1DE offenbar ahnlich, da sie au-

A B

C

D E

F I

O

O1

O2

O3

ßer in den rechten Winkeln noch in den Scheitelwinkeln�ADF = �O1DE, also in allen drei Winkeln ubereinstim-men. Somit ist

�IAB = �O3O1O.

Genauso konnen wir zeigen, daß �IAC = �O1O3O ist.�O1OO3 hat daher gleiche Basiswinkel und ist folglichgleichschenklig. Vollig analog zeigen wir OO1 = OO2, al-so OO1 = OO2 = OO3. Daraus folgt, daß der Umkreisdes Dreiecks O1O2O3 mit demjenigen von �ABC zusam-menfallt. �

W.33 Beweis: (Bild) Die zu AB, BC, CD, DA spiegelbildlichen Punkte von S sei-en K, L, M bzw. N . Um nun zu zeigen, daß KLMN ein Sehnenviereck ist, genugt esnachzuweisen, daß in ihm gegenuberliegende Winkel Supplementwinkel sind: �KLM =�MNK. Bezeichnen wir dazu weiter �BAS ≡ α

und �DCS ≡ γ. Nun ist ASBK nach Vorausset-zung AS ⊥ SB ein Drachenviereck, gleiches gilt furBSCL, CSDM und DSAN . Somit gilt BK = BS =BL, d. h., Eckpunkt B ist Mittelpunkt des Umkreisesvon �KLS. Analog folgt: C ist Umkreismittelpunktvon �LMS,D von �MNS bzw. A von �NKS. Die-se Kreise erlauben es, die Winkel α und γ aus ABCDwie folgt in das Viereck KLMN zu transportieren:�SBK = 180◦ − 2α ist Zentriwinkel uber der SehneSK, also ist �SLK = 90◦−α. Mit dem gleichen Ar-gument erhalten wir �SLM = γ, �SNM = 90◦− γ,�SNK = α. Durch Addition† folgt:

A

B

C

D

K

L

M

N

S

α

α

γ

γ

�KLM = �SLK + �SLM = (90◦ − α) + γ und

�MNK = �SNM + �SNK = (90◦ − γ) + α,

schließlich, wie behauptet, �KLM + �MNK = 180◦. �†Hier setzen wir ohne weitere Begrundung voraus, daß S im Innern des konvexen Vierecks KLMN

liegt.

192 LOSUNGEN: WETTBEWERBSAUFGABEN

W.51 Beweis: (Bild) Wegen AC = BC ist unsere Figur symmetrisch bezuglich desDurchmessers CM des Umkreises, wobei M der Beruhrungspunkt von k mit dem Um-kreis ist. Nun halbiert CM den Scheitelwinkel �ACB, den Winkel �PMQ sowie dieStrecke PQ, welche wegen PC = QC und der daraus folgenden Ahnlichkeit der Dreiecke

A B

C

I

M

P Q

k

ACB und PCQ parallel zu AB ist und deren Mittelpunkt wirmit I bezeichnen. Fur die Basiswinkel in beiden gleichschenk-ligen Dreiecken schreiben wir �CPQ = �CAB ≡ 2β. Nachdem Sehnen-Tangentenwinkel-Satz ist daher auch �PMQ =�CPQ = 2β, also �PMI = 1

2�PMQ = β. Weiterhin ist

AMIP wegen �CAM = �PAM = 90◦ und �PIM = 90◦ einSehnenviereck, in dem �PMI = �PAI = β gleich große Peri-pheriewinkel sind. Somit halbiert AI den Innenwinkel �CAB.I ist folglich als Schnittpunkt zweier Winkelhalbierenden derMittelpunkt des Inkreises von �ABC. �

W.52 (Bild) Die Seitenlangen des Dreiecks seien BC ≡ a, CA ≡ b und AB ≡ c; dieLangen der Lote PD ≡ u, PE ≡ v, PF ≡ w. Dann gilt fur den Flacheninhalt ∆ vonDreieck ABC:

2∆ = au+ bv + cw.

Die Aufgabe verlangt,

a

u+b

v+

c

w A B

C

D

E

F

Puv

w

zu minimieren. Die einfachste Moglichkeit dies zu erreichen, ist die Ausnutzung derCauchy-Schwarzschen Ungleichung (vgl. Aufgabe U.11) unter Verwendung der Tri-pel (

√au,

√bv,

√cw) und (

√a/u,

√b/v,

√c/w). Somit ist

(a+ b+ c)2 ≤ (au+ bv + cw)

(a

u+b

v+

c

w

)= 2∆

(a

u+b

v+

c

w

),

odera

u+b

v+

c

w≥ (a+ b+ c)2

2∆. (W.120)

Gleichheit gilt genau dann, wenn die Tripel (√au,

√bv,

√cw) und (

√a/u,

√b/v,

√c/w)

zueinander proportional sind, d. h., genau dann, wenn u = v = w ist. Der Ausdruck(W.120) wird folglich minimal, wenn P der Inkreismittelpunkt von �ABC ist.

Internationale Wettbewerbe 193

W.53 (Bild) Angenommen, B, M und N sind kollinear undjede Seite des Sechsecks habe die Lange eins. Es bietet sichan, r mit Hilfe des Satzes von Menelaus (s. Aufgabe D.43)zu bestimmen. Dazu sei X der Schnittpunkt von AC mit BE.Die Punkte B, M und N mussen nun auf den (gegebenenfallsverlangerten) Seiten eines Dreiecks liegen; wir finden das �CEXals ein solches und schreiben den Satz von Menelaus hierfurhin:

CN

NE· EBBX

· XMMC

= −1. (W.121)

A

B

C

D

E

F

M

N

X

Nun versuchen wir, jede dieser Langen durch r auszudrucken. Mit AC = CE =√

3,AX =

√3

2, EB = 2 und BX = −1

2(das Minuszeichen ruhrt vom entgegengesetzten

Richtungssinn zu EB) errechnen wir:

CN = AM =√

3 r, NE = CE − CN =√

3 (1 − r),

XM = AM − AX =√

3

(r − 1

2

), MC = AC − AM =

√3 (1 − r).

Dies in (W.121) eingesetzt, ergibt

√3 r√

3 (1 − r)· 2

−12

·√

3 (r − 12)√

3 (1 − r)= −1 oder 4r

(r − 1

2

)= (1 − r)2,

woraus schließlich r = 1√3

folgt.

W.54 Beweis: (Bild) O sei der Mittelpunkt des Kreises auf der Seite AB; die Beruhrungs-punkte mit den anderen Seiten seien X, Y bzw. Z. Wir drehen nun das rechtwinklige�OYC so um O, daß es in �OZE uber-geht, wobei E auf der Verlangerung vonAD liegt. Bezeichnen wir ε ≡ �OCY =�OEZ, dann ist auch �OCX = ε. DaABCD ein Sehnenviereck ist, gilt �OAE =180◦− 2ε und weiter �AOE = 180◦− (ε+180◦− 2ε) = ε = �AEO. �OAE ist somitgleichschenklig und wir erhalten: A B

C

DE F

O

XY

Z

AO = AE = AZ + YC = AZ + CX. (W.122)

Mit derselben Begrundung, d. h. durch Drehung von �OYD um O in �OXF usw., folgt

BO = BF = BX + YD = BX +DZ. (W.123)

Durch Addition von (W.122) und (W.123) folgt die Behauptung: AB = AD +BC. �

194 LOSUNGEN: WETTBEWERBSAUFGABEN

W.61 Beweis: (Bild) Aufmerksame Leser haben vielleicht sofort eine Idee: Aus Auf-gabe D.42 wissen wir, daß der Hohenschnittpunkt und der Umkreismittelpunkt isogonalkonjugierte Punkte sind. Daraus folgt, daß die Winkelhalbierendewc gleichzeitig die Winkelhalbierende der Geraden CF (identischmit der Geraden hc) und des Durchmessers COC ′ des Umkrei-ses von �ABC ist. Nach Voraussetzung soll nun mc isogonalzu hc sein, d. h., O muß auf der Seitenhalbierenden CM liegen.Das ist jedoch nur erfullt, wenn O mit M zusammenfallt. ABist somit Durchmesser und nach dem Satz des Thales �ABCrechtwinklig bei C. �

A B

C

C ′

F M

Ohc mc

W.81 Beweis: (Bild) Die Beruhrungspunkte des Inkreises (Radius r) mit den SeitenAB ≡ c, BC ≡ a, CA ≡ b bezeichnen wir mit D, E bzw. F ; die mit den Tangentenab-schnitten ST ‖ AB, UP ‖ BC und QR ‖ CA mit K, L bzw. M . Wir erkennen nun leicht,daß das Sechseck in drei Paare kongruenter Drachenvierecke

KSEI ∼= DPLI, ERMI ∼= LUFI, MQDI ∼= FTKI

zerfallt. Daß z. B. KSEI ein Drachenviereck ist, folgt aus dem Kongruenzsatz SSS fur�KSI ∼= �ESI (gemeinsame Seite SI, KI = EI = r und gleich lange Tangentenab-schnitte SK = SE); durch eine 180◦-Drehung vonKSEI um I geht es inDPLI uber, alsoKSEI ∼= DPLI. Damit ist gezeigt, daß jeweils gegenuberliegende Seiten des Sechsecksgleich lang sind:

UP = RS ≡ a′, QR = TU ≡ b′, ST = PQ ≡ c′,

wobei s′ ≡ a′+b′+c′ der halbe Umfang des Sechsecks ist. Weiterhin ist �CTS ∼ �CAB,und mit der Hohe hc als Lange des Lotes von C auf AB (und entsprechend h′c in �CTS)erhalten wir:

TS

AB=c′

c=h′chc

=hc − 2r

hc= 1 − 2r

hc.

Daraus folgt mit ∆ = rs fur die Flache eines Dreiecks

A B

C

D

EF

I

K

LM

P Q

R

ST

U1

2chc = rs,

2r

hc=c

s, somit c′ = c− c2

s.

Mit analogen Ausdrucken fur a′ und b′ wird

s′ = a′ + b′ + c′ = (a+ b+ c) − a2 + b2 + c2

s= 2s− a2 + b2 + c2

s. (W.124)

Es bleibt noch, a2 + b2 + c2 durch den Umfang des Dreiecks 2s ≡ a+ b+ c abzuschatzen:

(a− b)2 + (b− c)2 + (c− a)2 ≥ 0,

2(a2 + b2 + c2) ≥ 2ab+ 2bc+ 2ca

3(a2 + b2 + c2) ≥ (a+ b+ c)2 = 4s2

a2 + b2 + c2

s≥ 4s

3, −a2 + b2 + c2

s≤ −4s

3.

Nach (W.124) ist schließlich s′ ≤ 2s− 43s = 2

3s. �

Crux Mathematicorum 195

W.82 (Bild) a) Naturlich ist CFDE in jedem Fall ein Rechteck, und da in diesemdie Diagonalen stets gleich lang sind, konnen wir auch CD anstelle von EF betrachten.Offensichtlich hat CD minimale Lange, wenn D gerade der Lotfußpunkt von C auf ABist. Damit ist dieses einfache Problem bereits gelost. b) Es wird uns zunachst vielleichtuberraschen; die Antwort ist auch im Fall, daß �ABC nicht rechtwinklig ist, dieselbe:

a)

A B

C

D

E

F

b)

A B

C

D

E

F

Wenn D der Fußpunkt des Lotes von C auf AB ist, wird EF minimal. Der entscheidendeFakt ist namlich nicht der, daß CFDE ein Rechteck ist, sondern daß es wegen �CFD =�CED = 90◦ immer ein Sehnenviereck mit CD als Durchmesser ist. Je kleiner also CDund damit der Umkreis von CFDE, desto kleiner wird auch die Sehne EF .

W.83 (Bild) Angenommen, PQ ist eine Sehne, die die Bedingungen der Aufgabenstel-lung erfullt; M sei deren Mittelpunkt. Wenn es uns also gelingt, M zu konstruieren,schneidet die Gerade durch B und M den Kreis k in den gesuchten Punkten P undQ, und das Problem ist gelost. Nun steht jede Sehne PQ senkrecht auf OM , also ist�BMO = 90◦, und M liegt somit auf dem Thales-Kreis uber dem Durchmesser BO.Um einen zweiten geometrischen Ort fur M zu finden, bemerken wir, daß im rechtwink-ligen �PAQ der Mittelpunkt M der Hypotenuse PQ stets gleiche Abstande zu den dreiEckpunkten hat:

MP = MA = MQ.

Nach dem Satz des Pythagoras gilt:

r2 = OM2 +MQ2 = OM2 +MA2.

Damit haben wir eine Gleichung fur die Langen OMund MA im �OMA. Eine zweite finden wir, indemwir den Mittelpunkt N der gegebenen Strecke OA

A

B M N

O

P Q

k

r

betrachten. Dann ist MN die Lange der Seitenhalbierenden, fur die bekanntlich (vgl.Losung zu Aufgabe D.70 a) gilt:

r2 = OM2 +MA2 = 2MN2 +1

2OA2, oder MN =

√r2

2− OA2

4.

Die Lange der Strecke MN kann damit aus den gegebenen Langen r und OA konstruiertwerden (indem z. B. ein rechtwinkliges Dreieck mit der Hypotenusenlange r und einerKathetenlange 1√

2OA gezeichnet wird; die andere Kathete hat dann die Lange

√2MN).

Der zweite geometrische Ort fur M ist ein Kreis um N mit dem Radius MN . Die Aufgabehat also zwei, eine oder gar keine Losung.

196 LOSUNGEN: WETTBEWERBSAUFGABEN

W.84 Beweis: (Bild) Bezeichnen wir die Winkel bei P mit �BPC ≡ x, �CPA ≡ y

und �APB ≡ z, so ist mit Hilfe des Peripheriewinkel-satzes leicht zu erkennen, daß das Dreieck BA′C anden Eckpunkten in dieser Reihenfolge die Innenwinkelπ − y, π − x bzw. π − z hat. Die Langen der SeitenBA′ und A′C dieses Dreiecks lassen sich nun mit demSinussatz berechnen: A

A′

B

C

P

BA′ = asin(π − z)

sin(π − x)= a

sin z

sinx, A′C = a

sin y

sin x, somit

BA′ + A′C = asin y + sin z

sinx= (sinx+ sin y + sin z)

a

sin x− a.

Durch zyklische Vertauschung erhalten wir fur die Langen der anderen Seiten des Sechs-ecks analoge Ausdrucke, so daß fur dessen Umfang folgt:

p = (sinx+ sin y + sin z)

(a

sinx+

b

sin y+

c

sin z

)− (a+ b+ c).

Das Produkt der beiden Klammerausdrucke verlangt formlich danach, durch dieCauchy-Schwarzsche Ungleichung abgeschatzt zu werden (vgl. Aufgabe U.11):

p ≥ (√a+

√b+

√c)2 − (a+ b+ c) = 2(

√ab+

√bc+

√ca). �

W.85 (Bild) �ABC sei das gesuchte Dreieck. Wir spie-geln �ABC an der Seite AC; es entsteht zusatzlich derPunkt B′. Weiterhin wird �ABD an AB umgeklappt,und wir erhalten dadurch F als spiegelbildlichen Punktzu D. Dann ist offensichtlich AB = AB′ und somit dieerste Bedingung

CD = AB + AD = AB′ + AD = B′D

genau dann erfullt, wenn �CDB′ gleichschenklig ist. Fer-ner ist AD = AF ; also die zweite Bedingung

BC = AC + AD = AC + AF = CF

A B

B ′

C

D

F

ββ

γγ

γ

γ

β2

4

2

dann zutreffend, wenn B, F (und B′) auf einem Kreisbogen mit C als Mittelpunkt liegen.Da nun AD nach Voraussetzung die Winkelhalbierende von �CAB ist, also �CAD =�DAB = �BAF gilt, und deren Summe 180◦ betragt, folgt hieraus: �CAB ≡ α = 120◦.Die Summe der beiden anderen Innenwinkel �ABC ≡ β und �BCA ≡ γ ist daher

β + γ = 180◦ − α = 60◦. (W.125)

Aus der Innenwinkelsumme im �CBF folgt

4β + γ = 180◦. (W.126)

Aus (W.125) und (W.126) folgen die gesuchten Winkel β = 40◦ sowie γ = 20◦.

Crux Mathematicorum 197

W.86 Beweis: (Bild) Jede Seite wird vom Inkreismittelpunkt I aus unter einem Winkelgesehen, der gleich dem um 90◦ vermehrten halben Gegenwinkel der Seite ist (vgl. AufgabeD.9). Somit ist in unserem Dreieck gerade nach Voraussetzung �AIC = 90◦+ 1

2β = γ. Ist

A B

C

D

E

FG

H

I

G der Schnittpunkt der inneren Winkelhalbierenden AI mit BC, so folgt aus �ACI =�ICG = 1

2γ und �CIG = 180◦ − �AIC = 180◦ − γ, daß auch �ICG = 1

2γ gilt; �CIG

hat gleiche Basiswinkel und ist daher gleichschenklig. F und H seien die beiden anderenBeruhrungspunkte des Inkreises von �ABC mit BC und AC. Mithin ist

2CF = CG. (W.127)

Die Drachenvierecke AHIE und CHIF werden dann durch ihre Diagonalen AI und CI injeweils kongruente Dreiecke �AHI ∼= �AEI bzw. �CHI ∼= �CFI zerlegt. Insbesondereist also

AE = AH bzw. CH = CF. (W.128)

Den Beweis fuhren wir nun aus der Ahnlichkeit der beiden Dreieckspaare:

a) �ACG ∼ �AIC und b) �ADG ∼ �FIC. (W.129a,b)

Um dieses zu zeigen, genugt es nachzuweisen, daß sie jeweils in zwei Winkeln uberein-stimmen. Im Fall a) ist �ACG = �AIC = γ und �CGA = �ICA = 1

2γ; im Fall b)

�ICF = �DGA = 12γ und nach dem Satz, daß die Winkelhalbierenden der von zwei

Seiten eingeschlossenen Winkel stets senkrecht aufeinander stehen (vgl. Aufgabe D.7):�GAD = �CFI = 90◦. Aus (W.129a) folgt nun mit (W.127)

AG

CG=AC

CIbzw. AG · CI = CG · AC = 2CF · AC; (W.130)

aus (W.127b) mit (W.130)

DG

AG=

CI

CFbzw. DG =

AG · CICF

=2CF · AC

CF= 2AC. (W.131)

Schließlich ist wegen (W.127), (W.128) und (W.131)

CD = DG− CG = DG− 2CF = 2(AC − CF ) = 2(AC − CH) =

= 2AH = 2AE. �

198 LOSUNGEN: WETTBEWERBSAUFGABEN

W.87 (Bild) Die beschriebene Konstruktion laßt sofort erkennen, daß es sich bei Pum den Fermat-Punkt eines spitzwinkligen Dreiecks handelt (vgl. Aufgabe D.54). Vondiesem Punkt aus wird jede Dreieckseite unter einem Winkel von 120◦ gesehen; somit ist�BPC = �FPG = 120◦. Nehmen wir zunachst an, der gesuchte Winkel �BAC betrage60◦ (Bild a). Dann ist die Summe der gegenuberliegenden Winkel �FAG+�FPG gleicheinem Gestreckten, und AFPG ist somit ein Sehnenviereck. �APF = 60◦ = �AGF sindnun Winkel uber derselben Sehne AF , woraus folgt, daß �AFG gleichseitig ist. Wegender Voraussetzung �EAC = �DAB = 60◦ sind die Punkte E, A und D kollinear und

a)

A B

C

D

E

F

GP

b)

A B

C

D

E

F

GP

c)

A B

C

D

E

F

GP

daher gilt FG ‖ DA. Nun konnen wir �FAG in das flachengleiche �FDG verwandelnund erhalten somit [AFPG] = [PDG]. Mit AC ‖ DB und �PCG ∼ �PDB folgt

PC

PD=PG

PB, oder PB · PC = PD · PG. (W.132)

Erinnern wir uns an die Formel ∆ = 12ab sin γ fur den Flacheninhalt eines Dreiecks, ist

mit (W.132) sofort klar, daß beide gerasterte Flachen im Falle �BAC = 60◦ gleich großsind. Gilt dagegen �BAC < 60◦ (Bild b), so ist [FAG] > [FDG] und wir konnen dieUngleichungen

[AFPG] > [PDG] > [PBC]

aufstellen. Andererseits erhalten wir fur �BAC > 60◦ (Bild c) die Ungleichungen

[AFPG] < [PDG] < [PBC].

Je kleiner der Winkel �BAC wird, desto kleiner wird auch das Verhaltnis der Flachen-inhalte q ≡ [PBC]/[AFPG], so daß q eine monoton wachsende Funktion von �BAC ist.Daraus folgt, daß �BAC = 60◦ die einzige Losung des Problems ist.

Beweise fur Chou’s Aufgaben 199

W.88 Beweis: (Bild) Die gegebenen Summen vonQuadraten lassen den Satz des Pythagoras erah-nen, weshalb wir versuchen, rechtwinklige Dreieckemit den Katheten BI, ID bzw. CI, IE zu kon-struieren. An den Inkreis von �ABC wird parallelzur Seite BC eine Tangente gezeichnet, die den In-kreis in R beruhrt sowie AB in P und AC in Qschneidet. Weiterhin seien S, T die Lotfußpunkte vonI auf AB bzw. AC. Mit den ublichen Abkurzun-gen fur die Innenwinkel erhalten wir unmittelbar:β + γ = 90◦. Nun sind RP = PS gleich langeTangentenabschnitte, so daß die rechtwinkligen Drei-ecke RIP und SIP kongruent sind. Da �APQ ∼�ABC ist, errechnen wir fur den Außenwinkel von�APQ:

A B

C

D

E

I

P

Q

R

S

T

�QPB = �RPS = 90◦ + γ = 2�RPI = 2(90◦ − �RIP ) = 2�SPI.

Somit ist

�RIP = 45◦ − γ

2=β

2= �SIP und �PIB = 180◦ − �SPI − β

2= 90◦.

Nach dem Kongruenzsatz WSW folgt daraus

�SIP ∼= �TID, also IP = ID

(gleiche Stucke sind jeweils der rechte Winkel, der Inkreisradius und der Winkel �SIP =�TID = �ABD = 1

2β). Damit ist �PIB das gesuchte rechtwinklige Dreieck. Mit dersel-

ben Argumentation — angewandt auf die andere Kathete AC — konnen wir zeigen, daßebenso �QIC rechtwinklig mit IQ = IE ist. Schließlich folgt aus der oben erwahntenAhnlichkeit

AB

AC=AP + PB

AQ+QC=AP

AQ=PB

QC, und nach Quadrieren

AB2

AC2=PB2

QC2=BI2 + IP 2

CI2 + IQ2=BI2 + ID2

CI2 + IE2. �

200 LOSUNGEN: UNGLEICHUNGEN

LOSUNGEN: UNGLEICHUNGEN

U.1 Beweis: Wir fuhren den Beweis uber die vollstandige Induktion. Mit P(n) be-zeichnen wir dazu die Behauptung: x ≥ −1 ⇒ (1 + x)n ≥ 1 + nx mit Gleichheit genaufur n = 1 oder x = 0. Dann ist P(1) trivialerweise erfullt. Nun sei vorausgesetzt, daßP(n) fur n > 1 erfullt ist. Wir bemerken, daß als Bedingung fur Gleichheit in P(n + 1)wegen n+1 �= 1 nur x = 0 in Frage kommt. Multiplizieren wir die Voraussetzung mit dernichtnegativen Zahl 1 + x, erhalten wir

(1 + x)n+1 ≥ (1 + x)(1 + nx) = 1 + (n+ 1)x+ nx2 ≥ 1 + (n+ 1)x. (U.101)

Das Gleichheitszeichen gilt in (U.101) genau fur nx2 = 0, somit genau fur x = 0. �

U.2 Beweis: Wir entwickeln f(x) = (1 + x)a nach der Taylorschen Formel in eineReihe und brechen diese nach dem linearen Glied ab:

(1 + x)a = 1 + ax+a(a− 1)x2

2(1 + θx)a−2, 0 < θ < 1.

Wegen x > −1 ist 1 + θx > 0 und das Vorzeichen

sgn{(1 + x)a − 1 − ax} = sgn{a(a− 1)} =

{ −1 < 0, fur 0 < a < 1,+1 > 0, fur a > 1 oder a < 0,

beides fur x �= 0. �

U.3 Beweis: Der Fall n = 1 ist trivial. Deshalb nehmen wir n = 2 als Induktionsanfang.Zunachst benotigen wir einige elementare Beziehungen beim Rechnen mit komplexenZahlen. Fur alle z ∈ C gilt:

#[z] = x ≤√x2 + y2 = |z| und analog $[z] = y ≤ |z|. (U.102)

Weiterhin ist

|z| = |z|, |z|2 = zz, #[z] =1

2(z + z), $[z] =

1

2i(z − z), |z1z2| = |z1||z2|.

Diese Beziehungen benutzen wir, um |z1 + z2|2 zu berechnen:

|z1 + z2|2 = (z1 + z2)(z1 + z2) = (z1 + z2)(z1 + z2) = |z1|2 + |z2|2 + 2#[z1z2].

Den letzten Summanden konnen wir nun mit (U.102) abschatzen:

2#[z1z2] ≤ 2|z1z2| = 2|z1||z2|. (U.103)

Damit wird

|z1 + z2|2 ≤ |z1|2 + |z2|2 + 2|z1||z2| = (|z1| + |z2|)2.Nun konnen wir die Wurzel ziehen, und, da beide Seiten positiv sind, bleibt das Rela-tionszeichen erhalten:

|z1 + z2| ≤ |z1| + |z2|. (U.104)

Fundamentale Ungleichungen 201

Allgemein haben wir also als Induktionsbeweis:∣∣∣∣∣n+1∑i=1

zi

∣∣∣∣∣ =

∣∣∣∣∣n∑i=1

zi + zn+1

∣∣∣∣∣ ≤∣∣∣∣∣

n∑i=1

zi

∣∣∣∣∣ + |zn+1| ≤n∑i=1

|zi| + |zn+1| =n+1∑i=1

|zi|. (U.105)

Gleichheit gilt in (U.105) nur, wenn sie fur jedes Paar zi, zj in (U.103) erfullt ist. Dies fuhrtnach kurzer Rechnung auf die Bedingung, daß alle Argumente arg z (Winkel gegenuberder reellen Achse) untereinander gleich sein mussen:

arg z1 = arctany1

x1

= · · · = arg zn = arctanynxn,

gleichbedeutend damit, daß alle zi auf einer durch den Ursprung der komplexen Zahlen-ebene gehenden Geraden liegen. �Bemerkung: Ebenso erhalten wir

|z1 − z2|2 = |z1|2 + |z2|2 − 2#[z1z2] ≥ |z1|2 + |z2|2 − 2|z1||z2| = (|z1| − |z2|)2,also in Kombination mit (U.104)

||z1| − |z2|| ≤ |z1 ± z2| ≤ |z1| + |z2|. (U.106)

Geometrisch interpretiert bedeutet die Ungleichungskette (U.106), daß die Lange jederDreieckseite einerseits immer kurzer als die Summe und andererseits immer langer als dieDifferenz der beiden anderen Seiten ist.

U.4 Beweis: Hier mussen wir eine Anleihe aus der Analysis machen, indem wir diebekannte Ungleichung x − 1 − lnx ≥ 0 fur alle x > 0 verwenden, wobei Gleichheit nurfur x = 1 gilt (Zeichnen wir die Funktion y = lnx in ein kartesisches Koordinatensystem,sehen wir, daß die Gerade y = x − 1

”uber“ der Logarithmusfunktion liegt und diese

gerade im Punkt x = 1 tangiert). Mit der Abkurzung

A ≡n∑

k=1

δkak

liefert die Anwendung dieser Ungleichung ai/A−1−ln(ai/A) ≥ 0 fur jedes i. Multiplizierenwir jeden dieser Terme mit δi and summieren uber i, erhalten wir

n∑i=1

(δiaiA

− δi

)−

n∑i=1

δi ln(aiA

)≥ 0.

Wegen der obigen Definition von A und der Bedingung δ1 + · · · + δn = 1 gilt

n∑i=1

(δiaiA

− δi

)= 0 und somit

n∑i=1

δi ln(aiA

)≤ 0. (U.107)

Jetzt nutzen wir die Eigenschaft der Logarithmus-Funktion, im gesamten Definitionsbe-reich streng monoton zu wachsen, sowie das Exponentialgesetz aus:

exp

[n∑i=1

δi ln(aiA

)]=

n∏i=1

exp[δi ln

(aiA

)]=

n∏i=1

(aiA

)δi

=1

A

n∏i=1

aδii ≤ exp(0) = 1.

202 LOSUNGEN: UNGLEICHUNGEN

Somit ist (aδ11 · · · aδnn )/A ≤ 1 oder

aδ11 · · · aδnn ≤ δ1a1 + · · · + δnan. (U.108)

Gleichheit gilt in (U.108) genau dann, wenn sie auch in (U.107) gilt. Da jeder Summandin (U.107) wegen ai > 0, δi > 0 nichtnegativ ist, kann Gleichheit dort nur vorliegen, wennjeder Summand null ist. Dies ist gleichbedeutend mit ai/A = 1. Mit anderen Worten,Gleichheit gilt nur fur a1 = · · · = an. �

U.5 Beweis: Die Behauptung folgt unmittelbar aus der gewichteten AM-GM Unglei-chung (U.3), wenn wir darin alle Gewichte gleich setzen: δ1 = · · · = δn = 1

n. �

U.6 Beweis: Wir wenden die AM-GM Ungleichung auf die Reziproken 1/ai an. �

U.7 Beweis: Wir kombinieren die AM-GM Ungleichung und die GM-HM Ungleichungund lassen den GM-Teil weg. �

U.8 Beweis: Wie die folgende Rechnung zeigt, sind die Holderschen Ungleichungeneine direkte Folgerung aus der gewichteten AM-GM Ungleichung (U.5), wenn letztere aufjeden, in den eckigen Klammern stehenden Summanden angewendet wird:

n∑j=1

|a1j|p1 · · · |amj|pm

Rp11 · · ·Rpm

m=

n∑j=1

[( |a1j|R1

)p1· · ·

( |amj|Rm

)pm]

≤n∑

j=1

[p1

|a1j|R1

+ · · · + pm|amj|Rm

]= p1 + · · · + pm = 1,

m∑i=1

|ai1|q1 · · · |ain|qn

C q11 · · ·C qn

n=

m∑i=1

[( |ai1|C1

)q1· · ·

( |ain|Cn

)qn]

≤m∑i=1

[q1

|ai1|C1

+ · · · + qn|ain|Cn

]= q1 + · · · + qn = 1.

Nach der AM-GM Ungleichung gilt Gleichheit genau dann in (U.9), wenn fur alle j

|a1j|R1

= · · · =|amj|Rm

= λj = const

bzw. |aij| = λjRi ist. Dies ist gleichbedeutend mit |akj|/|alj| = Rk/Rl = const fur allePaare (k, l) unabhangig von j, welches Proportionalitat zwischen allen Spaltenvektorender Matrix [|aij|] bedeutet. Analog bestatigen wir die Gleichheitsbedingung in (U.10). �

U.9 Beweis: Wir setzen in die verallgemeinerteHoldersche Ungleichung (U.9)m = 2,p1 ≡ 1

p, p2 ≡ 1

qund |a1i| ≡ |ai|p, |a2i| ≡ |bi|q fur alle i = 1, . . . , n ein. Gleichheit gilt

demnach fur |ai|p ∝ |bi|q oder |bi| ∝ |ai|p−1. �

Fundamentale Ungleichungen 203

U.10 Beweis: Fur p = 1 folgt die Behauptung direkt aus der Dreiecksungleichung (U.4).Im Fall p > 1 wahlen wir eine Zahl q > 1 derart, daß die Relation 1

p+ 1

q= 1 erfullt ist.

Nehmen wir die Holdersche Ungleichung (U.11) in der Form

n∑i=1

|αiβi| ≤(

n∑i=1

|αi|p)1

p(

n∑i=1

|βi|q)1

q

und setzen einmal αi ≡ |ai|, βi ≡ |ai + bi|pq und ein weiteres Mal αi ≡ |bi|, βi ≡ |ai + bi|

pq

ein, erhalten wir

n∑i=1

|ai||ai + bi|pq ≤

(n∑i=1

|ai|p)1

p(

n∑i=1

|ai + bi|p)1

q

bzw. (U.109)

n∑i=1

|bi||ai + bi|pq ≤

(n∑i=1

|bi|p)1

p(

n∑i=1

|ai + bi|p)1

q

. (U.110)

Wegen p = 1 + pq

ist weiterhin fur jedes i nach der Dreiecksungleichung

|ai + bi|p = |ai + bi||ai + bi|pq ≤ |ai||ai + bi|

pq + |bi||ai + bi|

pq . (U.111)

Summieren wir uber die Terme (U.111) und benutzen dabei (U.109) und (U.110), folgt

n∑i=1

|ai + bi|p ≤(

n∑i=1

|ai|p)1

p(

n∑i=1

|ai + bi|p)1

q

+

(n∑i=1

|bi|p)1

p(

n∑i=1

|ai + bi|p)1

q

=

(

n∑i=1

|ai|p)1

p

+

(n∑i=1

|bi|p)1

p

(

n∑i=1

|ai + bi|p)1

q

. (U.112)

Der zweite Faktor auf der rechten Seite von (U.112) verschwindet gewiß nicht (sonst waredie Behauptung offensichtlich), so daß nach Division durch diesen (U.12) folgt. �

U.11 Beweis: Die Cauchy-Schwarzsche Ungleichung (U.13) folgt durch Einsetzenvon p = q = 2 in die Holdersche Ungleichung (U.11). �

U.12 Beweis: In beiden Fallen bezuglich der jeweiligen Ordnung der Zahlen ai und bigilt fur jedes Paar (i, j)

(ai − aj)(bi − bj) ≥ 0. (U.113)

Summation uber beide Indizes und Ausmultiplizieren fuhrt auf die Behauptung:

n∑i=1

n∑j=1

(ai − aj)(bi − bj) ≥ 0,

n∑i=1

aibi

n∑j=1

(1) −n∑i=1

ai

n∑j=1

bj −n∑

j=1

aj

n∑i=1

bi +n∑

j=1

ajbj

n∑i=1

(1) ≥ 0 oder

2nn∑i=1

aibi − 2n∑i=1

ai

n∑i=1

bi ≥ 0.

204 LOSUNGEN: UNGLEICHUNGEN

Aus (U.113) erkennen wir, daß das Gleichheitszeichen fur ai = aj oder bi = bj fur allePaare (i, j), also a1 = · · · = an oder b1 = · · · = bn gilt. �

U.13 Beweis: Da die Ungleichung symmetrisch in den Variablen x, y, z ist (d. h. jedePermutation der x, y, z die Ungleichung unverandert laßt), konnen wir o. B. d. A. x ≥y ≥ z annehmen. Schreiben wir die behauptete Ungleichung um als

(x− y)[xλ(x− z) − yλ(y − z)] + zλ(x− z)(y − z) ≥ 0,

so ist jeder Term auf der linken Seite offensichtlich nichtnegativ, also (U.15) wahr. �

U.14 Beweis: Wir fuhren den Beweis mit vollstandiger Induktion. Fur n = 2 folgt(U.17) unmittelbar aus der Konvexitat (U.16) von f(x). Angenommen, die zu beweisendeUngleichung ist wahr fur n = k und die Zahlen c1, . . . , ck+1 addieren sich zu 1:

c1 + · · · + ck+1 = 1 bzw.k∑i=1

ci1 − ck+1

= 1. (U.114)

Dann ist

f

(k+1∑i=1

cixi

)= f

((1 − ck+1)

k∑i=1

ci1 − ck+1

xi + ck+1xk+1

)

≤ (1 − ck+1)f

(k∑i=1

ci1 − ck+1

xi

)+ ck+1f(xk+1)

wegen der Konvexitat von f . Da die Zahlen ci/(1− ck+1) fur 1 ≤ i ≤ k sich nach (U.114)ebenfalls zu 1 addieren, haben wir

f

(k∑i=1

ci1 − ck+1

xi

)≤

k∑i=1

ci1 − ck+1

f(xi) =1

1 − ck+1

k∑i=1

cif(xi),

also gilt (U.17) auch fur n = k + 1 und der Induktionsbeweis ist gefuhrt. �

U.21 Beweis: Wegen des Satzes des Pythagoras fur rechtwinklige Dreiecke a2+b2 = c2

ist die Behauptung gleichwertig mit a+ b ≤ √2√a2 + b2. Dies folgt der Reihe nach aus

0 ≤ (a− b)2,

2ab ≤ a2 + b2,

a2 + 2ab+ b2 ≤ 2(a2 + b2),

a+ b ≤√

2√a2 + b2.

Gleichheit liegt bei a = b vor, also bei einem Quadrat. �Bemerkung: Die obige Reihenfolge fuhrt korrekt von wahren Voraussetzungen zur Behaup-tung. Bei der Losungssuche gehen wir meist den umgekehrten Weg; deshalb ist bei derAbfassung des Beweises daran zu denken, daß die einzelnen Schritte umgekehrt werden.

U.22 Beweis: Zunachst multiplizieren wir die Behauptung mit x4, damit die Brucheverschwinden (wegen x4 > 0 bleibt das Relationszeichen erhalten):

x12 − x9 − x3 + 1 ≥ 0.

Einfache Tips und Tricks 205

Unsere Strategie ist es hier, dieses Polynom so zu faktorisieren, daß alle Faktoren positivsind. Das ist hier nicht besonders schwer, da es nur zwei Faktoren sind:

x12 − x9 − x3 + 1 = (x9 − 1)(x3 − 1). (U.115)

Nun unterscheiden wir1. x ≥ 1: damit auch x3 ≥ 1 und x9 ≥ 1, somit ist (U.115) nichtnegativ;

2. x ≤ 1: damit auch x3 ≤ 1 und x9 ≤ 1, also auch hier (U.115) nichtnegativ. �

U.23 Beweis: a) Wir konnen o. B. d. A. annehmen, daß c die großte Zahl ist. Dann ist

a3 + b3 + c3 − (a2b+ b2c+ c2a) = a3 − a2b+ b3 − b2c+ c3 − c2a

= a2(a− b) + b2(b− c) + c2(c− a)

= a2(a− b) − b2(a− b+ c− a) + c2(c− a)

= (a2 − b2)(a− b) + (c2 − b2)(c− a)

= (a− b)2(a+ b) + (c− b)(c− a)(c+ b) ≥ 0

wegen c ≥ b und c ≥ a. b) Siehe Losung von Aufgabe G.1. �

U.24 Beweis: Die Teile a4+b4+c4 und a2bc sind nicht so offensichtlich in einer einfachenUngleichung unterzubringen, wohl aber

2a4 + b4 + c4 = a4 + a4 + b4 + c4 ≥ 4a2bc. (U.116)

Letztere folgt aus der AM-GM Ungleichung w + x + y + z ≥ 4 4√wxyz, wenn wir dort

w = a4, x = a4, y = b4 und z = c4 einsetzen, oder direkt aus der gewichteten AM-GMUngleichung

1

2a4 +

1

4b4 +

1

4c4 ≥ (a4)

12 · (b4) 1

4 · (c4) 14 = a2bc.

Nun konnen wir die Variablen in (U.116) zyklisch tauschen, anschließend die vier Unglei-chungen addieren und durch 4 dividieren. �

U.25 Beweis: Die Methode der Zerlegung in kurzere Ungleichungen laßt hoffen, daß

a3 + b3 + c3

a+ b+ c≥ a2 + b2 + c2

3(U.117)

gelten konnte, da dann die Behauptung durch zyklische Vertauschung und Addition folgt.Eine Beseitigung der Bruche in (U.117) fuhrt auf

3(a3 + b3 + c3) ≥ (a2 + b2 + c2)(a+ b+ c), (U.118)

die ihrerseits unmittelbar aus der Tschebyscheffschen Ungleichung (U.14) folgt. Fallsuns letztere nicht gelaufig ist, konnen wir noch den beschwerlichen Weg gehen und (U.118)in einen Ausdruck umformen, der offensichtlich großer gleich null ist:

2a3 + 2b3 + 2c3 − a2b− ab2 − a2c− ac2 − b2c− bc2

= (a3 − a2b− ab2 + b3) + (a3 − a2c− ac2 + c3) + (b3 − b2c− bc2 + c3)

= (a− b)2(a+ b) + (a− c)2(a+ c) + (b− c)2(b+ c) ≥ 0. �

206 LOSUNGEN: UNGLEICHUNGEN

U.26 Beweis: Was fur das Aufteilen von Summen gilt, kann ggf. auch mit Produkten— wie in der vorliegenden Ungleichung — probiert werden:

1

a+

1

b≥ 2√

ab

ware die naheliegende Vermutung, die sich auch sofort mittels (U.18), diesmal in der Form(√a−√

b)2 ≥ 0, bestatigt. Zyklische Vertauschung der Variablen und Multiplikation derdrei Ungleichungen liefert die Behauptung. �

U.27 Beweis: Manchmal ist das Vorgehen zu rigoros: Nehmen wir z. B. x2/y2 ≥ y/xheraus, fuhrt das auf x ≥ y bzw. bei den anderen Termen auf y ≥ z und z ≥ x. Zweidieser Ungleichungen lassen sich muhelos erfullen, alle drei aber nicht. Deshalb versuchenwir, etwas mehr zu

”greifen“:

x2

y2+y2

z2≥ 2

x

zwegen

(x

y− y

z

)2

≥ 0.

Nun ist es nicht schwer zu erkennen, daß die Behauptung aquivalent ist zu

(x

y− y

z

)2

+(yz− z

x

)2

+

(z

x− x

y

)2

≥ 0,

was zweifellos wahr ist. Gleichheit liegt bei x = y = z vor. �

U.31 Beweis: Zweimalige Anwendung der AM-GM Ungleichung ergibt mit 1+x4 ≥ 2x2

x4(1 + y4) + y4(1 + z4) + z4(1 + x4) ≥ 3(xyz)43

3√

(1 + x4)(1 + y4)(1 + z4)

≥ 3(xyz)43 · 2(xyz) 2

3 = 6(xyz)2. �

U.32 Beweis: Setzen wir uk ≡ √k ak und vk ≡ bk/

√k, k = 1, 2, . . . , n, dann gilt

u2 =n∑

k=1

u2k =

n∑k=1

ka2k, v2 =

n∑k=1

v2k =

n∑k=1

1

ka2k, u · v =

n∑k=1

ukvk =n∑

k=1

akbk.

Die behauptete Ungleichung folgt direkt aus der Cauchy-Schwarzschen Ungleichungu2 · v2 ≥ (u · v)2. �

U.33 Bringen wir die bekannten Großen a, b, c, d auf die rechten Seiten der beidenGleichungen, erhalten wir

7 − e = a+ b+ c+ d, (U.119)

13 − e2 = a2 + b2 + c2 + d2, (U.120)

zwischen denen nach Cauchy-Schwarz die Ungleichung vom Typ c)

a2 + b2 + c2 + d2 ≥ 1

4(a+ b+ c+ d)2 (U.121)

Einfache Tips und Tricks 207

besteht. Dies wird sofort klar, wenn wir die Vektoren uT ≡ (a, b, c, d) und vT ≡ (1, 1, 1, 1)betrachten. (U.119) und (U.120) in (U.121) eingesetzt, ergibt

4(13 − e2) ≥ (7 − e)2,

52 − 4e2 ≥ 49 − 14e+ e2,

0 ≥ 5e2 − 14e− 3,

0 ≥ e2 − 14

5e− 3

5,

49

25≥

(e− 7

5

)2

− 15

25,

8

5≥

∣∣∣∣e− 7

5

∣∣∣∣ .Nun machen wir die ubliche Fallunterscheidung:1. e ≥ 7

5(und die Betragsstriche konnen wegfallen) liefert e ≤ 3, also zusammen 7

5≤ e ≤ 3.

2. e ≤ 75

(und der Betrag ist 75− e) ergibt e ≥ −1

5, also −1

5≤ e ≤ 7

5.

Beide Falle konnen somit zur Losung e ∈ [−15, 3] zusammengefaßt werden.

U.34 Beweis: Beim Typ d) S1 ≥ S3 ist es manchmal nicht einfach, die Summe S2, durchdie ja dividiert wurde, zu erraten. S1 gibt uns jedoch die Summe u2 und S3 die Summeu · v vor (vgl. Aufgabe U.11):

S1 ≡ u2 = u21 + u2

2 + · · · + u2n =

a21

a1 + b1+

a22

a2 + b2+ · · · + a2

n

an + bn,

S3 ≡ u · v = u1v1 + u2v2 + · · · + unvn =a1

2+a2

2+ · · · + an

2.

Um nun u2 · v2 ≥ (u · v)2 erfullen zu konnen, vermuten wir durch gliedweisen Vergleichvon

u2i =

a2i

ai + biund u2

i v2i =

a2i

4: v2

i =ai + bi

4,

somit

S2 ≡ v2 = v21 + v2

2 + · · · + v2n =

a1 + b14

+a2 + b2

4+ · · · + an + bn

4

=a1

2+a2

2+ · · · + an

2.

Wir erhalten also tatsachlich S2 = S3 und haben damit die Behauptung bewiesen. �

U.35 Beweis: Der Typ d) mit S1 = S2 laßt sich am einfachsten realisieren, wenn dieElemente von Vektor u gleich denen von Vektor v sind, jedoch zyklisch verschoben werden.Bei der vorliegenden Ungleichung ist also

uT ≡ (a1, a2, . . . , an−1, an) und vT ≡ (a2, a3, . . . , an, a1).

Diese zyklische Verschiebung hat keinen Anderung der Summe der Quadrate in u2 zur Fol-ge, so daß u2 = v2 bzw. S1 = S2 gilt und auf beiden Seiten der Cauchy-SchwarzschenUngleichung die Wurzel gezogen werden kann:

u2 ≥ u · v = (a1a2 + a2a3 + · · · + ana1). �

208 LOSUNGEN: UNGLEICHUNGEN

U.36 Beweis: Die AM-HM Ungleichung in den positiven Zahlen ai, i = 1, 2, . . . , n,

(a1 + a2 + · · · + an)

(1

a1

+1

a2

+ · · · + 1

an

)≥ n2 (U.122)

hat die Besonderheit, daß sie außer der Summe∑

ai auch die Summe der Reziproken∑(1/ai) enthalt. Setzen wir die linke Seite der behaupteten Ungleichung gleich

∑ai, also

ai = S/(S − xi), so vereinfacht sich∑

(1/ai) wegen ubereinstimmender Zahler der ai zu

∑ 1

ai=S − x1

S+S − x2

S+ · · · + S − xn

S=nS − S

S= n− 1. (U.123)

Die Behauptung folgt direkt aus (U.122) und (U.123). �Bemerkung: Auch die Cauchy-Schwarzsche Ungleichung (U.13) fuhrt hier zum Ziel.

U.37 Beweis: Obwohl hier die Zahler auf den ersten Blick nicht gleich sind, gelangenwir trotzdem dorthin, wenn einfach drei

”nahrhafte Einsen“ in Form von

b2 + c2

b2 + c2+c2 + a2

c2 + a2+a2 + b2

a2 + b2= 3

auf beiden Seiten addiert werden:

(a2 + b2 + c2)

(1

b2 + c2+

1

c2 + a2+

1

a2 + b2

)≥ 3

2+ 3 =

9

2.

Diese Ungleichung folgt nun direkt aus der AM-HM Ungleichung. �Bemerkung: Der

”AM-HM Trick“ funktioniert also auch, falls bei allen gebrochenen Sum-

manden die Summe aus Zahler und Nenner untereinander gleich ist.

U.41 Beweis: Bei dieser Aufgabe fallt auf, daß die Nebenbedingung symmetrisch in dendrei Variablen ist, der abzuschatzende Ausdruck dagegen nicht. Wir sehen ihm aber an,daß x und z vertauschbar sind und damit die Variable y offensichtlich bevorzugt ist. Wennwir also durch die Nebenbedingung eine Variable eliminieren wollen, dann sollte es y sein:

xyz(x+ y + z) = xzy2 + xz(x+ z)y = 1, oder y2 + (x+ z)y − 1

xz= 0.

Der Zielausdruck expandiert zu

A = (x+ y)(y + z) = xy + y2 + xz + yz = y2 + (x+ z)y + xz.

Ein simpler Vergleich fuhrt auf A = xz + 1xz

≥ 2, letzteres wegen der allseits bekanntenUngleichung (U.18). �

U.42 Beweis: Ein Ausmultiplizieren der Nebenbedingung ergibt

(1 + a)(1 + b)(1 + c) = 1 + (a+ b+ c) + (bc+ ca+ ab) + abc = 8.

Da abc unser Zielausdruck ist, mussen wir die Terme (a+ b+ c) und (bc+ ca+ ab) durchabc abschatzen. Dabei hilft uns naturlich AM-GM:

a+ b+ c ≥ 3(abc)13 , bc+ ca+ ab ≥ 3(abc)

23 ,

Einfache Tips und Tricks 209

mit deren Hilfe aus der Nebenbedingung

8 = (1 + a)(1 + b)(1 + c) ≥ 1 + 3(abc)13 + 3(abc)

23 + abc =

[1 + (abc)

13

]3

wird. Daraus folgt 2 ≥ 1 + 3√abc und schließlich abc ≤ 1. �

U.43 Beweis: Die Kantenlangen des Quaders seien mit a, b, c bezeichnet, sein Volumenist V = abc und seine Oberflache betragt A = 2(bc + ca + ab). Wir suchen eine Unglei-chung zwischen A und V , die wir bereits in der Losung zu Aufgabe U.42 mittels AM-GMgefunden haben:

bc+ ca+ ab ≥ 3(abc)23 , oder A ≥ 6V

23

mit Gleichheit nur bei bc = ca = ab bzw. aquivalent dazu a = b = c. Ist also im Fall a) Afest, wird das großte Volumen V = (A/6)

32 fur a = b = c (der Wurfel) bzw. b) V fest, die

kleinste Oberflache A = 6V23 ebenfalls fur a = b = c angenommen. �

U.44 Beweis: Wir erkennen, daß ein bloßes Abschatzen des gegebenen Ausdrucks Amittels (U.18) ohne Berucksichtigung der Nebenbedingung lediglich auf die schwachereUngleichung A ≥ 8 fuhrt. Um zur Verscharfung die Nebenbedingung a + b = 1 ins Spielzu bringen, multiplizieren wir zunachst aus:

A =

(a+

1

a

)2

+

(b+

1

b

)2

= (a2 + b2) +

(1

a2+

1

b2

)+ 4, (U.124)

und erkennen daran, daß es im weiteren um die beiden Ausdrucke in den runden Klammerngeht. Diese treten auf, wenn wir die Nebenbedingung quadrieren und mittels a2+b2 ≥ 2 abnach unten abschatzen:

1 = (a+ b)2 = a2 + b2 + 2 ab ≥ 4 ab oder ab ≤ 1

4.

Es ist also

a2 + b2 = 1 − 2 ab ≥ 1 − 1

2=

1

2bzw.

1

a2+

1

b2=a2 + b2

(ab)2≥ 1

2(ab)2≥ 8.

Beides in (U.124) eingesetzt, ergibt die Behauptung A ≥ 252. �

U.45 Beweis: Es liegt die Vermutung nahe, daß es sich hierbei um Cauchy-Schwarz

u2 · v2 ≥ (u · v)2 mit

uT ≡(

a√a+ b

,b√b+ c

,c√c+ d

,d√d+ a

)

handelt. Dazu paßt nur eine Wahl von v, namlich

vT ≡(√

a+ b

2,

√b+ c

2,

√c+ d

2,

√d+ a

2

),

und zwar wegen

v2 =a+ b+ c+ d

2=

1

2und u · v =

a+ b+ c+ d

2=

1

2.

210 LOSUNGEN: UNGLEICHUNGEN

Daraus ergibt sich unmittelbar die behauptete Ungleichung. Gleichheit liegt nur dann vor,wenn beide Vektoren proportional sind: u = 2λv. Dies fuhrt auf ein lineares, homogenesGleichungssystem, dessen Determinante fur eine nichttriviale Losung verschwinden muß:∣∣∣∣∣∣∣∣

1 − λ 1 0 00 1 − λ 1 00 0 1 − λ 11 0 0 1 − λ

∣∣∣∣∣∣∣∣=

= (1 − λ)

∣∣∣∣∣∣1 − λ 1 0

0 1 − λ 10 0 1 − λ

∣∣∣∣∣∣ −∣∣∣∣∣∣

0 1 00 1 − λ 11 0 1 − λ

∣∣∣∣∣∣ = (1 − λ)4 − 1 = 0.

Diese charakteristische Gleichung hat die (jeweils zweifachen) Losungen λ1,2 = 0 undλ3,4 = 2, wobei die ersten beiden herausfallen. Also bleibt λ = 2, welches auf die Glei-chungen 2a = a+ b usw., somit a = b(= c = d) fuhrt. Daraus folgt unmittelbar Gleichheitnur fur a = b = c = d = 1

4. � — Bemerkung: Vgl. Aufgabe U.34.

U.53 Beweis: Daß aus {x1 > 0, x2 > 0, . . . , xn > 0} auch {σ1 > 0, σ2 > 0, . . . , σn > 0}folgt, ist unmittelbar einzusehen, da in der Definition der σ’s nur positive Summandenvorkommen. Um die umgekehrte Richtung zu zeigen, schreiben wir die Definition (U.19)nochmals hin:

xn − σ1xn−1 + σ2x

n−2 − · · · + (−1)n−1σn−1x+ (−1)nσn = 0,

und uberprufen, ob diese Gleichung auch negative Wurzeln haben kann, indem wir x durch−x ersetzen:

xn + σ1xn−1 + σ2x

n−2 + · · · + σn−1x+ σn = 0. (U.131)

Wenn nun alle σi positiv sind, kann das Polynom (U.131) nicht fur positive x verschwinden.Da es aber gerade fur x = −xk (k = 1, 2, . . . , n) Nullstellen hat, folgt daraus xk > 0. �

U.54 Nein, wie folgendes Gegenbeispiel zeigt: a > 0, b = c = −a/3 erfullen a+ b+ c =a/3 > 0 und abc = a3/9 > 0, sind aber nicht allesamt positiv.

U.81 Beweis: Aus der Anzahl der Summanden auf der linken Seite der behauptetenUngleichung und dem Faktor 4 auf der rechten Seite konnen wir vermuten, daß das Prinzip

”Teile und (be)herrsche“ (s. Abschnitt U.2.1) zur Anwendung kommt. Tatsachlich folgt

wegen a, b, c, d > 0 aus a + b + c < a + b + c + d ≡ s bzw. den analogen Ungleichungena+ b+ d < s, a+ c+ d < s und b+ c+ d < s:

1

a+ b+ c>

1

s,

1

a+ b+ d>

1

s,

1

a+ c+ d>

1

s,

1

b+ c+ d>

1

s.

Deren Addition liefert die Behauptung. �Bemerkung: Das geubte Auge erkennt die Verscharfung durch AM-HM:

1

a+ b+ c+

1

a+ b+ d+

1

a+ c+ d+

1

b+ c+ d≥ 16

3· 1

a+ b+ c+ d.

Weitere Ungleichungen 211

U.82 Beweis: Wir benotigen zwei Schritte: den ersten nach Cauchy-Schwarz undden zweiten nach AM-GM:

(a2 + b2 + c2 + d2 + e2)(a3 + b3 + c3 + d3 + e3) ≥(a

52 + b

52 + c

52 + d

52 + e

52

)2

≥ 52(a

52 b

52 c

52d

52 e

52

) 25

= 25abcde. �

U.83 Beweis: Die Ungleichung ist eine direkte Konsequenz der AM-GM Ungleichung,da

a1

b1· a2

b2· · · an

bn= 1. �

U.84 Beweis: Nach der AM-GM Ungleichung ist

[(1 + x1)(1 + x2) · · · (1 + xn)]1n ≤ (1 + x1) + (1 + x2) + · · · + (1 + xn)

n=n+ S

n,

(1 + x1)(1 + x2) · · · (1 + xn) ≤(

1 +S

n

)n

.

Fur die Potenz auf der rechten Seite konnen wir nach dem binomischen Satz schreiben:(1 +

S

n

)n

= 1 + S +n(n− 1)

2!· S

2

n2+n(n− 1)(n− 2)

3!· S

3

n3

+ · · · + n(n− 1) · · · (n− n+ 1)

n!· S

n

nn

= 1 + S + 1

(1 − 1

n

)S2

2!+ 1

(1 − 1

n

)(1 − 2

n

)S3

3!

+ · · · + 1

(1 − 1

n

)· · ·

(1 − n− 1

n

)Sn

n!

≤ 1 + S +S2

2!+S3

3!+ · · · + Sn

n!

mit Gleichheit nur fur n = 1. �

U.85 Beweis: Setzen wir m ≡ mink

(ak/bk) und M ≡ maxk

(ak/bk), so ist

m ≤akbk

≤ M,

mbk ≤ak ≤ Mbk,

mn∑

k=1

bk ≤n∑

k=1

ak ≤ Mn∑

k=1

bk,

m ≤

n∑k=1

ak

n∑k=1

bk

≤ M. �

212 LOSUNGEN: UNGLEICHUNGEN

Anwendung: Fur x ≥ 0 gilt

1

n≤ 1 + 2x+ · · · + nxn−1

n+ (n− 1)x+ · · · + xn−1≤ n,

1

n≤ 1 + 2x+ · · · + nxn−1

1 + 22x+ · · · + n2xn−1≤ 1.

U.86 Beweis: Aus (√b−√

c)2 ≥ 0 folgt

2√bc ≤ b+ c, (U.181)

1

b+ c≤ 1

2√bc,

bc

b+ c≤

√bc

2, also

bc

b+ c+

ca

c+ a+

ab

a+ b≤ 1

2(√bc+

√ca+

√ab).

Die rechte Seite nochmals mit (U.181) abgeschatzt, ergibt die Behauptung. �

U.87 Beweis: Folgerung der AM-HM Ungleichung (U.8). �

U.88 Beweis: a) Expandieren des Produkts fuhrt auf

P = 1 +

(1

x+

1

y+

1

z

)+

(1

yz+

1

zx+

1

xy

)+

1

xyz.

Nach AM-GM gilt

3 3√xyz ≤ x+ y + z = 1, somit

13√xyz

≥ 3,

und weiter(1

x+

1

y+

1

z

)≥ 3

(xyz)13

,

(1

yz+

1

zx+

1

xy

)≥ 3

(xyz)23

,

so daß

P ≥ 1 +3

(xyz)13

+3

(xyz)23

+1

xyz=

(1 +

13√xyz

)3

≥ (1 + 3)3 = 64. �

b) Ein Expandieren liefert hier

P =(1 − x)(1 − y)(1 − z)

xyz=

1 − (x+ y + z) + (yz + zx+ xy) − xyz

xyz

=1

x+

1

y+

1

z− 1,

wobei jetzt AM-HM weiterhilft:

P + 1 =1

x+

1

y+

1

z≥ 9

x+ y + z= 9, also P ≥ 8. �

Weitere Ungleichungen 213

U.89 Beweis: Diese Aufgabe ist identisch mit Aufgabe U.88 b, wenn wir dort

x ≡ 1

1 + a, y ≡ 1

1 + b, z ≡ 1

1 + c, a =

1

x− 1, b =

1

y− 1, c =

1

z− 1

setzen. �

U.90 Beweis: Etwas ungewohnlich, aber kurz und knapp:

ad+ bc ≤ (c+ d)max(a, b)

≤ min(a, b)max(a, b)

= ab. �

U.91 Beweis: Multiplikation der beiden Voraussetzungen ergibt

ac+ bd+ ad+ bc ≤ ef.

Damit und wegen AM-GM 2√ac

√bd = 2

√ad

√bc ≤ ad+ bc ist(√

ac+√bd

)2

= ac+ bd+ 2√ac

√bd ≤ ac+ bd+ ad+ bc ≤ ef.

Da c und d vertauschbar sind, folgt auch die andere Ungleichung. �

U.92 Beweis: Folgerung der AM-GM Ungleichung (U.6). �

214 LOSUNGEN: GEOMETRISCHE UNGLEICHUNGEN

LOSUNGEN: GEOMETRISCHE UNGLEICHUNGEN

G.1 Beweis: Mit der Substitution (G.4) wird aus der Behauptung

(y + z)(z + x)(x+ y) ≥ 8xyz,

die unmittelbar aus (√x−√

y)2 ≥ 0 und ihren zyklischen Vertauschungen folgt. �

G.2 Beweis: Die Substitution (G.4) fuhrt hier auf

√2 (

√x+

√y +

√z) ≤ √

x+ y +√y + z +

√z + x. (G.101)

Nach der Methode”Teile und (be)herrsche“ (s. Abschnitt U.2.1) finden wir weiter:

0 ≤ (√x−√

y)2, (G.102)

2√xy ≤ x+ y,

x+ 2√xy + y = (

√x+

√y)2 ≤ 2(x+ y),

√x+

√y ≤

√2√x+ y.

Zyklische Vertauschung und Addition ergibt (G.101). Wegen (G.102) und der entspre-chenden anderen Gleichungen gilt Gleichheit fur x = y = z, d. h. fur alle gleichseitigenDreiecke. �

G.3 Beweis: Mit unserer Substitution a ≡ y + z, b ≡ z + x, c ≡ x+ y uberfuhren wirdie behauptete Ungleichung in

(y + z)2(z + x)(y − x) + (z + x)2(x+ y)(z − y) + (x+ y)2(y + z)(x− z) ≥ 0,

welche nach auswucherndem Ausmultiplizieren glucklicherweise durch gegenseitige Aus-loschung der meisten Terme in

xy3 + yz3 + zx3 ≥ x2yz + xy2z + xyz2 = xyz(x+ y + z) (G.103)

ubergeht. Dieses Resultat ist ein guter Kandidat fur die Cauchy-Schwarzsche Unglei-chung, und zwar vom Typ d) mit S2 = S3 bzw. v2 = u · v (s. Abschnitt U.2.2). Dasie durch zyklische Vertauschung der Variablen unverandert bleibt, genugt es, allein denersten Term auf der linken Seite zu betrachten:

u21 = xy3. (G.104)

Fur u1v1 haben wir nun auf der rechten Seite drei Moglichkeiten:

u1v1 =

x2yzxy2z,

xyz2

denen mit (G.104) v21 =

x3y−1z2

xyz2

xy−1z4

entspricht. Wir erkennen Ubereinstimmung (links unten und rechts Mitte) und sind damitschon fertig. �

Ungleichungen in Vierecken 215

G.4 Beweis: Aus der Behauptung wird mit (G.2) und (G.4)

1

2

(1

x+

1

y+

1

z

)≥ 1

y + z+

1

z + x+

1

x+ y, (G.105)

die ein Kandidat fur ein Aufteilen ist. Versuchen wir es also mit

1

4

(1

x+

1

y

)≥ 1

x+ y(G.106)

und deren zyklischen Vertauschungen, die addiert offenbar wieder (G.105) ergeben. (G.106)ist nach kurzer Rechnung jedoch nichts anderes als (x− y)2 ≥ 0. �

G.42 Beweis: a) Mit den Seitenlangen a ≡ |a| = |−−→BC|, b ≡ |b| = |−→CA| und c ≡ |c| =|−→AB| des Dreiecks ist(

a

a+

b

b+

c

c

)2

= 3 + 2

(b · cbc

+c · aca

+a · bab

)≥ 0.

Mit b · c = −bc cosα, c · a = −ca cos β, a · b = −ab cos γ ergibt sich die Behauptung. �b) Da f(x) = sinx in [0, π] eine streng konkave Funktion ist, folgt mittels der JensenschenUngleichung (s. Aufgabe U.14)

sin

(α+ β + γ

3

)= sin

π

3=

√3

2≥ 1

3(sinα+ sin β + sin γ). �

G.51 Beweis: Nach Aufgabe D.79 gilt stets d2 = R2 − 2rR, wobei d der Abstand vonUmkreis- und Inkreismittelpunkt sind. Naturlich ist d2 ≥ 0, woraus aus obiger Gleichungnach Umstellen und Division durch R > 0 die Behauptung folgt. �

G.61 Beweis: (Bild) Wir wenden die Dreiecksungleichungen auf die Teildreiecke ABG,BCG und CAG an, und berucksichtigen dabei, daß der Schwerpunkt G jede Seitenhal-bierende drittelt:

2

3(ma +mb) > c,

2

3(mb +mc) > a,

2

3(mc +ma) > b.

Addition der Ungleichungen liefert die Abschatzung nachunten 3

2s < m. Fur die Abschatzung nach oben erganzen wir

das Dreieck zu einem Parallelogramm ABA′C mit den Dia-gonalen a und 2ma. Die Dreiecksungleichung fur das Teil-

A

A′

B

C

G

ab

c

dreieck AA′C lautet dann: b + c > 2ma. Analog erhalten wir mit anderen Erganzungendie Ungleichungen c + a > 2mb und a+ b > 2mc, deren Addition 4s > 2m bzw. m < 2sergibt. �Bemerkung: Die Anwendung von Dreiecksungleichungen auf alle anderen moglichen Teil-dreiecke liefert lediglich die schwachere Abschatzung s < m < 3s.

G.81 Beweis: Wir hatten in Aufgabe D.45 gesehen, daß das Lotfußpunktdreieck einesPunktesD auf dem Umkreis von �ABC gerade zur Simson-GeradenXY Z entartet (Bilda). Fur alle anderen Punkte D haben wir tatsachlich ein Dreieck XY Z vorliegen (Bild

216 LOSUNGEN: GEOMETRISCHE UNGLEICHUNGEN

a)

A B

C

DX

Y

Z

b)

A B

C

D X

Y

Z

b), fur welches die Seitenlangen nach Aufgabe V.26 die Dreiecksungleichung erfullen:

XY + Y Z > XZ, oderAB · CD

2R+BC · AD

2R>AC ·BD

2R.

Beide Falle zusammen genommen ergeben nach Multiplikation mit 2R die behaupteteUngleichung. �

G.82 Beweis: (Bild) S sei der Schnittpunkt der Diagonalen AC und BD. Dann gelteneinerseits die Dreiecksungleichungen

a < AS +BS, b < BS + CS,

c < CS +DS, d < DS + AS,

deren Addition und anschließende Division durch zwei s < p+ q

ergibt. Andererseits ist

p < a+ b, q < b+ c, p < c+ d, q < d+ a,

A B

C

D

S

a

b

c

d

aus deren Addition und Division durch zwei p+ q < 2s folgt. �

Sachwortverzeichnis

Ahnlichkeit, von Dreiecken A.25, A.45, A.64,K.11, K.12, K.13, K.14, K.21, K.22, K.36,K.61, D.6, D.10, D.13, D.23, D.25, D.31, D.34,D.46, D.64, D.83, V.9, V.23, V.25, W.9, W.11,W.30, W.32, W.51, W.86, W.88

Abbildung K.21Abstand, eines Punktes von Eckpunkten D.57,

D.81— eines Punktes von einer Geraden A.24, A.27— eines Punktes von einer Seite D.58, D.83,

D.84, V.26, W.16, W.22, W.52— zweier Geraden A.65— zweier Punkte K.14, D.79AIME W.21Analysis A.64, B.11, B.12, B.13, B.14, B.15,

B.20, B.21, B.41, B.51Ankreis A.34, K.35, D.35, D.39, D.65Ankreismittelpunkt A.34, D.35, D.65Ankreisradius D.65, D.68APMO U.34, U.84, G.2Apollonius, Beruhrungsproblem des A.31,

A.32, A.33, A.34, K.33— Kreis des A.22, A.26, A.68, A.69, K.62Außenwinkel A.22, A.34, B.20, K.2, D.2, D.7,

D.24, V.27, M.11, M.12, M.14Aufsatzdreiecke B.52, D.47, D.53, D.54, W.87Aufsatzhalbkreise K.37Aufsatzquadrate M.8, W.3

Basiswinkel A.61, B.20, B.21, B.41, K.2, K.3, D.2,D.8, D.24, D.59, V.21, V.33, M.11, M.14,M.15, W.10, W.32, W.51, W.86

Beruhrungspunkt A.9, A.10, A.32, A.33, A.34,A.62, A.63, A.72, B.13, B.51, K.14, K.24,K.25, K.33, K.35, D.35, D.38, D.39, D.63,V.31, V.32, V.33, V.34, V.41, M.14, W.12,W.51, W.54, W.86, W.88

Beruhrungsradius K.14, K.16, K.24, K.25, D.63Beruhrungssehne V.33, V.34, V.41Bernoullische Ungleichung U.1— verallgemeinerte U.2— indirekter D.33, D.44binomischer Satz U.84Brocard-Winkel D.58, D.73

Cauchy-Schwarzsche Ungleichung D.58,W.52, W.84, U.11, U.32, U.33, U.34, U.35,U.36, U.45, U.82, G.3

Ceva, Satz von D.31, D.32, D.33, M.34, W.26— Umkehrung des Satzes von D.33, D.34,

D.36, D.37, D.38, D.39, D.41, D.47, W.26Crux Mathematicorum B.51, K.62, M.40,

W.81, W.82, W.83, W.84, W.85, W.86, W.87,W.88

Determination B.11, B.14

Diagonale A.44, A.68, A.71, B.12, B.21, D.6,D.10, V.1, V.2, V.3, V.9, V.23, V.24, M.9,M.39, W.1, W.2, W.12, W.15, W.16, W.17,W.31, W.33, W.53, W.82, W.86, G.61, G.81,G.82

Diagonalen, senkrechte A.2, A.3, K.31, V.25, M.3,W.16, W.33

Drachenviereck A.3, D.23, W.33, W.81, W.86Drehung A.7, B.20, D.59, W.54— um 180 Grad B.16, B.19, D.61, W.81, G.61— um 60 Grad A.67, D.53— um 90 Grad A.65Drehverschiebung A.65Dreieck A.8, B.19, B.52, K.11, K.12, K.13, K.33,

K.35, D.1, D.2, D.3, D.4, D.5, D.6, D.7, D.8,D.9, D.10, D.11, D.12, D.13, D.22, D.23, D.24,D.25, D.31, D.32, D.33, D.34, D.35, D.36,D.37, D.38, D.39, D.40, D.41, D.42, D.43,D.44, D.45, D.46, D.47, D.57, D.58, D.61,D.63, D.64, D.65, D.66, D.67, D.68, D.69,D.70, D.72, D.73, D.79, D.81, D.83, D.91, V.1,V.2, V.3, V.23, V.26, V.27, V.32, V.34, M.1,M.2, M.9, M.12, M.31, M.32, M.33, M.34,M.35, M.36, M.37, M.38, M.39, M.40, M.41,M.42, M.43, W.7, W.11, W.13, W.21, W.26,W.28, W.31, W.32, W.52, W.53, W.81, W.84,W.85, W.86, G.1, G.2, G.3, G.4, G.42, G.51,G.61

— gleichschenkliges A.1, A.5, A.8, A.24, A.61,A.66, A.70, B.14, B.20, B.21, B.41, K.2, K.3,K.4, K.36, D.2, D.4, D.51, D.56, D.59, D.71,D.82, V.21, V.22, V.24, V.25, V.33, M.11,M.12, M.13, M.14, M.15, M.21, W.3, W.10,W.16, W.17, W.24, W.25, W.51, W.54, W.85

— gleichseitiges A.42, A.50, A.67, A.69, B.52,D.6, D.53, D.54, D.84, M.21, W.7, W.13,W.15, W.17, W.25, W.87

— rechtwinkliges A.14, A.15, A.23, A.25, A.61,A.65, A.72, B.11, B.13, B.15, B.18, B.23,B.51, K.4, K.14, K.21, K.61, D.25, D.62, D.64,D.82, D.83, V.9, V.24, M.13, W.3, W.4, W.8,W.9, W.10, W.11, W.16, W.54, W.61, W.82,W.88, U.21

— spitzwinkliges D.21, D.53, D.54, D.55, D.56,W.4, W.6, W.24, W.82, W.87

Dreieckskonstruktion A.67, A.69, B.1, B.2,B.3, B.4, B.11, B.12, B.13, B.14, B.15, B.16,B.17, B.18, B.19, B.20, B.21, B.22, B.23,B.41, B.51, B.52

Dreiecksungleichung B.3, B.12, D.51, V.26,W.2, U.3, U.10, G.61, G.82

Drittelung D.12, D.13, M.13, G.61Durchmesser A.63, A.64, K.3, K.11, K.12, K.15,

K.21, D.25, D.64, D.82, W.8, W.9, W.15,

218 Sachwortverzeichnis

W.29, W.30, W.61

Eckpunkt A.67, B.52, D.13, D.53, D.81, M.1,M.2, M.8, W.13, W.32

Ecktransversale D.31, D.32, D.33, D.34, D.36,D.37, D.38, D.39, D.40, D.41, D.47, D.69,D.70, D.91, M.13, M.34, M.35, M.36, M.37,M.38, M.40, M.41, M.42, W.21, W.26, W.84,W.88

elementare symmetrische Funktionen U.51,U.52, U.53, U.54

Elementarzelle A.42, M.10Ellipse A.72Erdos-Mordell, Satz von D.83, D.84Euler-Gergonne, Satz von M.36, M.37, M.41,

M.42, W.21Eulers Abstand D.79Extremalaufgabe, geometrische D.51, D.53,

D.56, D.57, D.58, D.59, W.22, W.27, W.52,W.82

Funfeck, konvexes M.22, W.2— regelmaßiges A.13, M.21Fagnano, Problem von D.56Faktorisierung W.28, U.22Fallunterscheidung A.10, A.15, A.32, A.33, A.34,

A.65, A.66, A.68, A.69, B.2, B.4, K.2, K.4,K.36, D.5, D.24, D.43, V.1, W.12, W.14,W.28, W.82, W.87, U.2, U.12, U.22, U.33

Fermat-Punkt B.52, D.53, D.54, D.55, W.87Flacheninhalt, allgemein A.15— eines Dreiecks D.11, D.58, V.3, M.9, M.22,

M.31, M.32, M.33, M.34, M.35, M.36, M.37,M.38, M.39, M.40, W.7, W.11, W.13, W.15,W.21, W.22, W.31, W.52, W.87

— eines Funfecks M.22— eines Parallelogramms M.10— eines Sechsecks M.10— eines Trapezes A.64— eines Vierecks V.3, M.22, W.31, W.87— Formel D.61, D.62, D.63, D.64, D.65, D.66,

M.31Flachenprinzip M.31, M.32, M.33, M.34, M.35,

M.36, M.37, M.38, M.39, M.40, M.41, M.42,M.43

Flachenumwandlung D.62, M.21, M.22, W.31,W.87

Flachenvektor M.9, M.10Funktion, konvexe U.14— streng konkave G.42— streng konvexe U.14— streng monoton wachsende W.87, U.4

Gelenkmechanismus K.31geometrischer Ort A.21, A.22, A.23, A.24,

A.25, A.26, A.27, A.31, A.32, A.33, A.66,A.68, B.14, B.20, K.62, D.1, D.82, W.25, W.83

Gerade A.1, A.3, A.4, A.6, A.9, A.23, A.24, A.27,

A.32, A.33, A.34, A.41, A.43, A.46, A.47,A.48, A.49, A.66, A.71, A.72, K.21, D.7, D.12,D.13, D.59, D.61, V.25, M.31

— parallele A.27, D.62Geraden, nichtparallele A.24— parallele A.4, A.10, A.11, A.12, A.41, A.64,

A.67, K.33, K.62, V.19, V.27, W.31— zueinander senkrechte K.3, K.24, K.25, D.7,

V.9, V.41, W.24Gergonnes Punkt D.38, D.40Gitter, schiefwinkliges M.10— trigonales A.42Gleichung, kubische W.28— quadratische A.13, A.15, A.66, W.12, W.53Gleichungssystem, lineares K.33, D.39, D.63,

M.38goldener Schnitt A.13

Hohe A.5, B.11, B.15, B.16, B.18, B.20, B.21,B.22, D.21, D.22, D.34, D.42, D.56, D.59,D.61, D.64, D.65, D.70, D.72, D.73, V.2, M.31,W.4, W.6, W.7, W.13, W.16, W.26, W.61

Hohenfußpunkt B.22, D.23, W.6Hohenfußpunktdreieck D.21, D.56, W.6Hohensatz A.14, A.66, V.9, W.8Hohenschnittpunkt D.21, D.22, D.23, D.24,

D.25, D.42, W.6, W.61Holdersche Ungleichung U.9, U.10, U.11Holdersche Ungleichungen, verallgemeinerte

U.8, U.9Halbierung A.25, A.44, A.64, B.12, B.21, D.2,

D.21, V.1, V.2, M.9, M.14, M.22, W.15, W.61Halbkreis A.63, A.64, B.12, W.8Halbkugel W.13Halbumfangspunkt K.33, D.35, D.39Heron, Problem von D.51Herons Formel D.66, W.21Hypotenuse A.14, A.15, A.23, A.66, B.15, B.51,

V.9, M.13, W.12, W.16, W.28, W.82, W.83,U.21

IMO W.51, W.52, W.53, W.54, G.3IMTS M.39Inkreis A.33, A.34, A.62, B.13, B.51, D.38, D.63,

D.65, V.27, W.6, W.81, W.86, W.88Inkreismittelpunkt A.34, A.62, A.62, B.13,

D.9, D.21, D.63, D.63, D.79, V.24, V.27, V.32,M.43, W.6, W.32, W.51, W.52, W.86, W.88

Inkreisradius B.13, D.63, D.67, D.68, D.79, V.32,M.43, W.7, W.81, W.88, G.51

Innenwinkel D.2, D.7, D.8, D.21, M.12, W.17Inversion K.21, K.22, K.23, K.24, K.25, K.31,

K.33, K.35, K.36, K.37isogonal konjugierter Punkt D.37, D.41,

D.42, W.61isogonale Gerade D.37, D.41, D.42, D.91, W.61isogonaler Punkt D.37, D.41Isoscelizer A.24

Sachwortverzeichnis 219

isotomisch konjugierter Punkt D.36, D.40isotomische Gerade D.36, D.40isotomischer Punkt K.33, D.36

Jensensche Ungleichung U.14, G.42

Kathete A.15, A.23, A.66, B.15, M.13, W.3,W.12, W.83, W.88

kollineare Punkte K.31, D.43, D.44, D.45, D.46,W.53, W.87

Komplementwinkel B.13, B.20, B.21, K.2, K.4,D.24, D.25, M.13, W.24, W.33, W.88

komplexe Zahlen U.3— konjugiert W.28Kongruenzsatz SSS A.6, A.45, B.3, B.41,

W.10, W.81Kongruenzsatz SSW B.4, B.15, B.23, V.31,

V.32Kongruenzsatz SWS B.1, B.14, B.22, D.4,

D.54, D.61, M.8, W.17Kongruenzsatz WSW A.21, B.2, B.4, B.13,

B.20, D.4, V.2, W.16, W.88Kongruenzsatz, WSW D.59Konstruktion, Euklidische A.1, A.2, A.3, A.4,

A.5, A.6, A.7, A.8, A.9, A.10, A.11, A.12,A.13, A.14, A.15

— geometrische A.61, A.62, A.63, A.64, A.65,A.66, A.67, A.68, A.69, A.70, A.71, A.72, K.37

— Mohr-Mascheronische A.41, A.42, A.43,A.44, A.45, A.46, A.47, A.48, A.49, A.50, K.36

Kosinussatz D.66, D.69, W.28Kreis A.8, A.9, A.10, A.23, A.25, A.34, A.41,

A.43, A.44, A.46, A.47, A.63, A.66, A.71, K.2,K.4, K.11, K.12, K.13, K.14, K.15, K.21, K.22,K.23, K.24, K.25, K.35, K.37, K.61, K.62, V.9,V.10, V.27, W.3, W.5, W.8, W.9, W.12, W.23,W.24, W.27, W.29, W.30, W.51, W.54, W.83,W.84

Kreisbogen A.21, A.70, A.72, K.1, K.2, K.24,K.31, D.82, W.5

Kreise, beruhrende A.10, A.25, A.33, A.62, A.63,B.14, K.33, K.35, M.14, W.12, W.27, W.51

— konzentrische A.9, A.10, A.45, A.68, A.69,K.16, W.32

— schneidende A.9, A.10, A.32, A.33, K.24, K.25,K.35, K.62, D.54, W.23, W.24

Kreiskonstruktion A.31, A.32, A.33, A.34,A.66, K.33

Kreissektor A.62

Logarithmusfunktion U.4Lot A.3, A.23, A.49, K.21, K.61, D.43, D.45, V.9,

M.13, W.5, W.11, W.82Lotfußpunkt A.3, A.24, A.27, A.49, A.50, B.15,

B.16, B.18, B.21, K.21, D.13, D.34, D.45,D.59, D.61, D.62, D.82, D.83, D.91, V.26,V.32, M.13, M.43, W.4, W.5, W.8, W.11,W.12, W.15, W.16, W.25, W.52, W.82, W.88

Lotfußpunktdreieck D.81, D.82, D.83, V.26,W.22, W.25

Lemoine-Punkt D.41, D.58, D.91

Mathematik-Olympiade B.12, B.21, K.35,D.22, V.3, V.25, M.8, M.22, M.42, M.43, W.1,W.2, W.3, W.4, W.5, W.6, W.7, W.8, W.9,W.10, W.11, W.12, W.13, W.14, W.15, W.16,W.17, W.22, W.23, W.24, W.25, W.26, W.27,W.28, W.29, W.31, W.32, W.33, W.51, W.52,W.53, W.54, U.22, U.23, U.41, U.44, U.45,U.81, G.3, G.4

— Australien M.42— Auswahlwettbewerb W.61— Bulgarien K.35— DDR B.12, D.22, M.8, M.43, W.10, U.44— Deutschland B.21, V.25, M.22, W.1, W.2,

W.3, W.4, W.5, W.6, W.7, W.8, W.9, W.11,W.12, W.13, W.14, W.15, W.16, W.17, U.81

— Großbritannien W.22, W.23, W.24, U.23,U.41

— Internationale W.51, W.52, W.53, W.54, G.3— Irland W.25, W.26, U.22, U.45— Kanada V.3, W.27, W.28— Lettland W.29— Polen G.4— Schweden W.31— USA W.32, W.33Maximum D.59, W.22, W.27, U.85, U.90Menelaus, Satz von D.43, D.44, M.35, M.38,

W.53— Umkehrung des Satzes von D.44, D.46Minimum D.51, D.53, D.56, D.57, D.58, D.59,

W.52, W.82, U.85, U.90Minkowskische Ungleichung U.10Mittel, arithmetisches A.14, M.1, M.2, U.5, U.7— geometrisches A.14, U.5, U.6— harmonisches A.14, U.6, U.7Mittellinie D.13, V.1, V.3, M.3, M.9, W.1, W.11,

W.14, W.31Mittelparallele A.33, K.33Mittelpunkt, einer Diagonalen V.1, V.3, W.1,

W.14— einer Sehne A.25, K.61, W.83— einer Seite B.16, B.17, B.18, D.13, D.61, V.1,

V.3, M.1, M.9, W.1, W.11, W.14, W.17, W.31— einer Strecke A.2, A.9, A.13, A.14, A.49, A.50,

B.52, K.61, D.10, D.11, M.22, W.3, W.14,W.51, W.83

— eines Kreisbogens A.44, A.62— eines Kreises A.1, A.8, A.9, A.10, A.14, A.21,

A.23, A.31, A.32, A.33, A.45, A.46, A.47,A.63, A.66, A.68, A.69, A.70, A.72, B.12,B.20, B.21, K.2, K.3, K.4, D.2, V.21, V.22,W.3, W.12, W.14, W.15, W.25, W.27, W.29,W.30, W.54, W.85

— eines Quadrates M.8

220 Sachwortverzeichnis

Mittelsenkrechte A.1, A.2, A.8, A.31, A.61, A.66,A.68, A.70, K.31, D.1, D.2, D.3, D.4, D.42,M.10, W.3, W.17, W.27

Nagels Punkt D.39, D.40Nebenwinkel A.21, A.22, K.22, D.7, D.8, M.33,

W.24Netzhaut-Satz K.14

Oberflacheninhalt U.43Ornament A.63, K.37, M.11orthogonale Kreise K.24, K.25, K.35

Packung, von Kreisen W.12Parabel A.32Paradoxon D.4Parallelogramm A.41, A.44, A.48, B.12, B.16,

B.19, B.21, D.10, D.59, D.61, V.19, M.8,M.10, M.12, W.1, W.14, G.61

Parallelverschiebung A.10, A.14, A.65, A.67,D.55

Peaucelliers Inversor K.31Peripherie W.13Peripherie-Zentriwinkel-Satz B.14, K.2, K.3,

K.4, V.10, W.16, W.24Peripheriewinkel A.21, B.14, B.20, B.21, K.2, K.4,

K.11, K.12, D.2, D.5, D.24, D.25, D.45, D.53,D.64, D.82, V.10, V.23, V.24, V.27, M.13,M.15, W.5, W.9, W.11, W.15, W.23, W.27,W.51, W.84

Peripheriewinkelsatz K.1, M.15Permutation U.83Polygon, regelmaßiges A.13Polynom U.22, U.23, U.24, U.31, U.32, U.33,

U.35, U.51, U.52, U.53, U.54, U.82, U.84Polynomwurzeln W.28, U.53Potenz, eines Punktes K.11, K.12, K.13Proportionale, mittlere A.13, A.32, A.49, K.13,

K.14, M.21— vierte A.11, A.45, A.48, B.41, W.11Ptolemaus’ Ungleichung G.81Ptolemaus, Satz des D.6, V.26, G.81Punkt A.1, A.3, A.4, A.6, A.8, A.9, A.22, A.23,

A.24, A.31, A.32, A.33, A.41, A.43, A.49,A.66, K.22, K.25, K.36, D.59, D.83, D.91,V.19, M.13, M.22, M.31, M.41, W.5, W.14,W.25, W.27, W.29, W.30, W.82, W.84

— im Raum M.4, M.9Pythagoras, Satz des A.13, A.15, A.23, A.44,

A.50, A.63, A.64, A.72, K.15, K.16, D.72, V.9,M.21, W.3, W.4, W.12, W.28, W.83, W.88,U.21

Quader U.43Quadrat A.14, A.15, A.64, A.65, A.71, M.8, W.10,

W.12, U.21— umschriebenes K.37

Radius K.24

Rechteck A.14, V.9, W.82Relation, dimensionslose M.36, M.37— Flacheninhalt A.14, A.64, D.11, D.61, D.62,

D.63, D.64, D.65, D.66, D.73, V.1, V.2, V.3,M.9, M.10, M.21, M.31, M.32, M.33, M.34,M.35, M.36, M.37, M.38, M.39, M.40, M.43,W.7, W.11, W.21, W.22, W.31, W.87, U.43

— Lange A.12, A.13, A.14, A.22, A.23, A.24,A.25, A.26, A.27, A.44, A.50, A.63, A.64,A.66, A.68, B.21, B.23, B.41, B.52, K.11,K.12, K.13, K.14, K.15, K.16, K.21, K.22,K.24, K.35, K.36, K.61, K.62, D.1, D.5, D.6,D.8, D.10, D.12, D.13, D.22, D.31, D.33, D.34,D.35, D.36, D.37, D.38, D.39, D.43, D.44,D.46, D.47, D.51, D.53, D.54, D.56, D.57,D.58, D.59, D.63, D.64, D.67, D.68, D.69,D.70, D.71, D.72, D.73, D.79, D.81, D.83,D.84, D.91, V.1, V.10, V.11, V.19, V.22, V.25,V.26, V.31, V.34, M.1, M.2, M.3, M.4, M.9,M.12, M.13, M.21, M.31, M.32, M.33, M.34,M.35, M.38, M.41, M.42, M.43, W.2, W.3,W.4, W.5, W.7, W.8, W.9, W.11, W.12, W.16,W.21, W.22, W.26, W.28, W.29, W.30, W.51,W.52, W.53, W.54, W.81, W.82, W.84, W.85,W.86, W.88, U.21, G.1, G.51, G.61, G.81, G.82

— Vektor D.55, D.57, M.1, M.2, M.3, M.4, M.8,M.9, M.10

— Volumen U.43— Winkel A.61, A.70, B.20, B.21, K.1, K.2, D.5,

D.7, D.9, D.21, D.23, D.24, D.25, D.32, D.42,D.45, D.53, D.56, D.59, D.73, D.82, V.8, V.10,V.21, V.27, V.32, V.33, V.34, M.11, M.12,M.13, M.14, M.15, M.43, W.5, W.10, W.17,W.23, W.24, W.33, W.54, W.85, W.86

Rhombus A.2, A.4, A.24, A.68, K.31Routh, Satz von M.38

Scheitelpunkt A.5, A.6, A.24, A.61, B.1, B.4,W.13

Scheitelwinkel A.70, V.9, V.23, V.34, M.15, W.32,W.51

Schenkel A.5, A.6, A.24, A.61, B.1, B.2, B.4,B.14, B.16, D.47, D.56, V.21

Schmetterling-Satz K.61Schnittpunkt V.19— dreier Geraden D.1, D.31, D.33, D.34, D.36,

D.37, D.38, D.39, D.40, D.41, V.22, M.34,M.35, M.36, M.37, M.41

— vierer Geraden V.34— von Diagonalen A.71, B.12, K.31, D.6, V.23,

V.24, V.25, V.34, M.39, W.1, W.2, W.16, W.33— von Ecktransversale und Kreis W.84, W.86— von Ecktransversale und Seite W.87, W.88— von Ecktransversalen D.33, D.34, M.13, M.42,

W.7, W.26— von Gerade und Kreis A.26, A.46, A.47, A.68,

A.70, B.15, B.22, K.16, V.9, W.23, W.24, W.27

Sachwortverzeichnis 221

— von Winkelhalbierenden A.34— zweier Geraden A.8, A.11, A.27, A.48, A.68,

K.23, D.3, D.4, D.43, D.44, D.51, V.22, M.40,W.24, W.31

— zweier Kreise A.41, A.43, A.44, A.45, A.69,B.12, W.23

— zweier Sehnen K.61— zweier senkrechter Geraden A.49, A.65, V.24— zwischen Sekante und Tangente D.46, W.29,

W.30Schnittwinkel, zweier Kurven K.23Schursche Ungleichung U.13Schwerpunkt G.61— eines Dreiecks B.12, B.18, B.19, D.10, D.11,

D.12, D.13, D.57, D.84, M.1, M.2, M.37, W.22— eines Quadrats M.8Sechseck M.2, W.3, W.81, W.84— regelmaßiges W.15, W.53Sehne A.25, A.45, A.66, A.70, B.14, B.20, K.1,

K.2, K.4, K.11, K.14, K.15, K.61, K.62, D.6,V.10, V.24, V.27, M.13, M.14, M.15, W.3,W.8, W.11, W.15, W.29, W.30, W.33, W.83

Sehnen-Tangentenwinkel K.4, K.13, M.14, W.11,W.51

Sehnen-Tangentenwinkel-Satz D.46, M.14,W.23

Sehnensatz K.11, K.15, K.61, K.62, D.22, D.79,W.8, W.29

Sehnentangentenviereck V.41Sehnenviereck K.1, K.22, D.5, D.6, D.21, D.22,

D.23, D.45, D.53, D.81, D.82, D.83, V.21,V.22, V.23, V.24, V.25, V.26, V.27, V.41,M.13, M.15, W.16, W.24, W.29, W.30, W.33,W.51, W.54, W.82, W.87, G.81

Seite A.63, B.1, B.2, B.3, B.4, B.11, B.17, B.21,B.41, D.6, D.8, D.9, D.23, D.38, D.39, D.40,D.45, D.46, D.54, D.58, D.61, D.62, D.63,D.64, D.65, D.67, D.69, D.70, D.72, D.73, V.8,V.9, V.19, V.31, V.32, M.3, M.4, M.8, M.32,W.2, W.4, W.6, W.11, W.14, W.16, W.25,W.51, W.52, W.54, W.82, W.85, W.87, U.21,G.1, G.2, G.3, G.4

Seitenhalbierende B.12, B.14, B.15, B.16, B.17,B.18, B.19, D.10, D.11, D.12, D.13, D.34,D.41, D.70, D.91, M.1, M.37, W.26, W.61,W.83, G.61

Sekante K.12, K.13, K.21, K.23, K.62, W.30Sekanten-Tangentensatz A.32, A.72, K.13,

K.21, K.24Sekantensatz A.66, A.72, K.12Senkrechte A.1, A.61, B.11, W.27Siebeneck M.11Simson-Gerade D.45, D.81, V.26, G.81Sinussatz D.5, D.32, D.47, D.81, D.91, V.34,

W.84Skalarprodukt D.57, M.3, M.4, M.8, U.11,

U.32, U.34, U.35, G.3Soddys Kreise K.33Spiegelung A.24, A.33, A.43, A.70, A.71, A.72,

B.21, D.23, D.51, D.56, W.8, W.10, W.25,W.33, W.85

Steiner-Lehmus, Satz von D.71Stereometrie M.9, W.13Stewart, Satz von D.69, D.70, D.71, W.21Strahlensatz, erster A.11, A.25, B.17, B.41, K.21,

K.62, D.8, D.12, D.13, D.31, D.59, V.19, W.5,W.14, W.31

— zweiter A.12, A.27, B.18, B.41, K.21, D.10,D.31, D.43, V.1, V.19, W.5

Strecke A.2, A.7, A.11, A.12, A.13, A.21, A.24,A.42, A.45, A.50, A.63, A.72, K.61, D.38,D.40, W.5, W.14, W.28, W.30, W.32, W.82,W.87

Streckung, zentrische A.69, W.15Stufenwinkel D.8, D.10Supplementwinkel B.20, B.21, B.51, K.1, K.2,

K.22, K.23, D.5, D.7, D.8, D.21, D.23, D.69,D.81, V.8, V.24, V.27, V.32, V.33, V.34, V.41,M.12, M.33, M.43, W.24, W.33, W.84, W.86

Symmediane D.41, D.91

Tangente A.9, A.10, A.32, B.13, K.4, K.13, K.14,K.16, K.23, K.24, D.46, M.14, W.11, W.23,W.29, W.30, W.51, W.54, W.81, W.88

Tangenten, gemeinsame A.10Tangentenabschnitt A.32, A.72, B.51, K.13, K.14,

K.16, K.21, K.25, K.62, D.35, D.38, D.39,D.46, D.63, V.31, V.32, V.34, M.14, W.81,W.88

Tangentenviereck V.31, V.32, V.33, V.34, V.41,M.43

Teilung, stetige A.13Teilungsverhaltnis A.12, A.13, A.22, A.27, A.63,

A.68, B.19, D.6, D.8, D.10, D.12, D.13, D.22,D.31, D.32, D.33, D.34, D.43, D.44, D.46,D.72, M.1, M.36, M.37, M.41, M.42, W.9,W.21, W.26, W.53

Tetraeder W.13Thales-Kreis A.9, A.14, A.22, A.25, A.66, A.68,

B.12, B.16, B.23, K.3, K.21, K.25, D.2, D.21,W.8, W.27, W.61, W.83

Tournament of Towns M.15Trapez A.64, B.23, D.13, D.83, V.8, V.11, M.12,

M.39, W.3, W.11Tschebyscheffsche Ungleichung U.12, U.25

Umfang, eines Dreiecks B.20, D.56, D.63, D.65,D.67, W.9, W.13, W.81, G.61

— eines Funfecks W.2— eines Sechsecks W.81, W.84— eines Vierecks G.82Umkreis K.35— eines Dreiecks A.8, A.31, B.14, B.15, B.20,

B.22, D.1, D.2, D.3, D.4, D.6, D.23, D.25,

222 Sachwortverzeichnis

D.45, D.46, D.53, D.64, V.22, V.26, M.15,W.11, W.23, W.32, W.33, W.51, G.81

— eines Sechsecks W.15Umkreismittelpunkt B.22, D.1, D.3, D.24,

D.25, D.42, D.64, D.79, V.22, W.24, W.32,W.33, W.61

Umkreisradius B.14, B.22, D.5, D.64, D.67, D.79,D.81, V.26, W.13, G.51

Ungleichung U.22, U.23, U.25, U.26, U.27, U.31,U.32, U.33, U.34, U.35, U.36, U.37, U.42,U.43, U.52, U.53, U.54, U.81, U.82, U.83,U.84, U.85, U.86, U.87, U.88, U.89, U.90,U.91, U.92

— AM-GM M.41, W.13, W.22, U.5, U.6, U.7,U.24, U.31, U.42, U.43, U.82, U.83, U.84,U.88, U.91, U.92

— AM-HM M.41, U.7, U.36, U.37, U.81, U.87,U.88

— geometrische D.58, D.83, D.84, M.41, W.7,W.13, W.22, W.52, W.84, U.3, U.21, U.43,G.1, G.2, G.3, G.4, G.42, G.51, G.81, G.82

— gewichtete AM-GM U.4, U.5, U.8, U.24— GM-HM U.6, U.7— mit Nebenbedingung W.22, U.41, U.42, U.43,

U.44, U.45, U.88, U.89, U.91— unsymmetrische U.41

Varignon-Parallelogramm V.1, V.3, M.9, W.1,W.14

Vektor D.55, D.57, M.1, M.2, M.3, M.4, M.8,M.9, M.10

Vektorprodukt M.9, M.10Verdopplung, einer Strecke A.42, B.19Vervielfachung, einer Strecke A.42, K.36Viereck A.4, A.46, D.82, V.1, V.2, V.3, V.8, V.10,

V.33, V.41, M.3, M.22, W.31, W.87— im Raum M.4, M.9

— konvexes W.1, W.14, W.17, G.81, G.82vollstandige Induktion U.1, U.3, U.14Volumen W.13, U.43

Wurfel U.43Wechselwinkel B.41, D.62, M.33, W.10Wigner-Seitz-Zelle M.10Winkel A.5, A.6, A.7, A.24, A.33, A.61, A.62, B.2,

B.4, B.13, B.14, B.16, B.20, B.22, D.9, D.24,D.32, D.47, D.62, V.21, M.11, M.13, M.15,W.10, W.17, W.27, W.28, W.85, W.86, W.87,G.42

— 120 Grad B.52, D.53, D.54, D.55— 150 Grad V.10, W.25— 36 Grad A.13— 45 Grad B.23— 60 Grad D.54, V.10, W.15— 72 Grad A.13— 75 Grad W.25— eingeschlossener B.1, K.23, V.8— gegenuberliegende V.10— konstanter A.21— rechter B.12, B.51, K.3, W.25, W.83Winkeldifferenz W.23— von Innenwinkeln B.21, B.22, D.25Winkelhalbierende A.1, A.5, A.22, A.33, A.34,

A.61, A.62, A.68, B.1, B.13, B.15, B.41, K.4,D.2, D.3, D.4, D.7, D.8, D.9, D.21, D.34,D.51, D.70, D.71, V.27, M.14, W.6, W.26,W.51, W.61, W.85, W.86

Zehneck, regelmaßiges A.13Zentrale K.12Zentriwinkel B.14, B.20, B.51, K.2, D.24, V.10,

V.24, W.5, W.27, W.33Zerlegung A.8, D.6, D.11, D.63, V.3, V.23, M.12,

M.22, M.34, M.35, M.36, M.37, M.38, M.39,M.40, M.41, W.22, W.32